You are on page 1of 107

TOPNOTCH MEDICAL BOARD PREP PEDIATRICS MAIN HANDOUT BY DRS.

DE VERA AND PUNONGBAYAN


For inquiries visit www.topnotchboardprep.com.ph or https://www.facebook.com/topnotchmedicalboardprep/
This handout is only valid for October 2022 PLE batch. This will be rendered obsolete for the next batch since we update our handouts regularly.
DEFINITION:
• Neonatal Period – birth up to 1st month old
IMPORTANT LEGAL INFORMATION
ESSENTIAL NEWBORN CARE
The handouts, videos and other review materials, provided by Topnotch Medical Board
Preparation Incorporated are duly protected by RA 8293 otherwise known as the • Series of time bound chronologically ordered, standard
Intellectual Property Code of the Philippines, and shall only be for the sole use of the person: procedures that a baby receives at birth
a) whose name appear on the handout or review material, b) person subscribed to Topnotch
Medical Board Preparation Incorporated Program or c) is the recipient of this electronic
• Immediate drying → prevents hypothermia
communication. No part of the handout, video or other review material may be reproduced, • Uninterrupted skin-to-skin contact → prevents hypothermia,
shared, sold and distributed through any printed form, audio or video recording, electronic increases colonization with protective family bacteria and
medium or machine-readable form, in whole or in part without the written consent of
Topnotch Medical Board Preparation Incorporated. Any violation and or infringement, improves breastfeeding initiation and exclusivity.
whether intended or otherwise shall be subject to legal action and prosecution to the full • Delayed cord clamping after 1 to 3 minutes → decreases
extent guaranteed by law.
anemia in 1 out of 3 premature babies and prevents brain
hemorrhage in 1 out of 2; prevents anemia in 1 out of 7 term
DISCLOSURE babies.
The handouts/review materials must be treated with utmost confidentiality. It shall be the
responsibility of the person, whose name appears therein, that the handouts/review • Non-separation of mother and baby. Breastfeeding within
materials are not photocopied or in any way reproduced, shared or lent to any person or first hour of life prevents 19.1% of all neonatal deaths.
disposed in any manner. Any handout/review material found in the possession of another
person whose name does not appear therein shall be prima facie evidence of violation of RA Please take note of the chronological order and importance of each step.
8293. Topnotch review materials are updated every six (6) months based on the current A common mistake of most students is to answer cord clamping as the 2nd
trends and feedback. Please buy all recommended review books and other materials listed
step. Remember this is the 3rd step. Think “delayed”
below. Dr. De Vera
THIS HANDOUT IS NOT FOR SALE!
✔ GUIDE QUESTIONS
After catching a newborn full-term male, and placing him in the
INSTRUCTIONS incubator, you noticed that the air-conditioning system was set to full
To scan QR codes on iPhone and iPad
1. Launch the Camera app on your IOS device
and that the radiant warmer was not working. You are worried that the
2. Point it at the QR code you want to scan baby might develop hypothermia which might lead to following except?
3. Look for the notification banner at the top A. Acidosis
of the screen and tap B. Hypoglycemia
To scan QR codes on Android C. Hypoxemia
1. Install QR code reader from Play Store D. Increased renal excretion of water and solutes
2. Launch QR code app on your device
E. NOTA
3. Point it at the QR code you want to scan
4. Tap browse website The target temperature for newborns is?
A. 36 - 37°C C. 36.5 - 37.5°C
Approach to Topnotch Pediatrics B. 36 - 37.5°C D. 36.7 - 37°C
• Please have the following Topnotch materials at hand: The following are the most common mechanisms of heat loss in the
o (1)Topnotch Main Handout will serve as your main reference immediate newborn period EXCEPT?
material A. Convection
• Please buy the following: B. Conduction
o Nelson Textbook of Pediatrics, 20th / 21st ed C. Heat Radiation
o Pedia Platinum D. Evaporation
E. NOTA
This handout is only valid for the March 2022 PLE batch. THERMOREGULATION
This will be rendered obsolete for the next batch • Newborns are prone to heat loss and hypothermia
since we update our handouts regularly. • body surface area of a newborn infant is approximately 3 times
that of an adult
PEDIATRICS Mechanism
Convection Heat energy to cooler surrounding air
By Adrian Salvador M. De Vera, MD, DPPS Conduction Heat to colder materials touching the infant
Ruby Ann L. Punongbayan, MD, FPPS, MA Radiation Transfer of heat to nearby cooler objects
Contributor: Mary Joeline D. Arada, OTRP, MD Evaporation Losses from skin and lungs (respiration)
• Optimal method for maintaining temperature in a stable
TOPIC PAGE
neonate?
Neonatology 1
o Skin-skin contact
Pediatric Nutrition 13
Preventive Pediatrics 18 Here are some examples of heat loss for you to better remember them.
IMCI 22 Convection: This is the reason why room temperature should be set at 25-28°C.
Conduction: Important to pre-heat radiant warmer.
Growth and Development 23
Radiation: Cold metal cabinets inside the operating room are sources of
Gastroenterology 26 heat radiation.
Nephrology 33 Dr. De Vera

Hematology/Oncology 39 ✔ GUIDE QUESTION


Neurology 46 All of the ff. is an effect of early skin to skin contact in newborn care except?
Pulmonology 57 A. Bonding
Rheumatology 70 B. Prevents hypothermia
Cardiology 76 C. Prevents anemia
D. Increases colonization with protective bacterial flora
Infectious Diseases 86
Endocrinology 96 Classification of Prematurity Based on Birth Weight and
Immunology/ Allergology 102 Gestational Age:
CLASSIFICATION METRIC
Birth weight
NEONATOLOGY • Low birth weight <2,500 grams
✔ GUIDE QUESTIONS
• Very low birth weight <1,500 grams
The significance of delayed cord clamping is to:
• Extremely low birth weight <1,000 grams
A. Prevent hypothermia
B. Decrease the incidence of anemia Gestational age
C. Remove the crawling reflex • Late preterm birth Equal/>34 and <37 weeks
D. To allow good bacteria from the mother to penetrate the newly born • Very preterm birth <32 weeks
The rationale of this intervention in the Essential Newborn Care is to <28 weeks
• Extremely preterm birth
prevent neonatal hypothermia and to increase colonization with
protective family bacteria: Please do not get confused with SGA and low birth weight.
A. Immediate drying Low birth weight pertains to weight alone, regardless of gestational age.
B. Delayed cord clamping SGA compares the birth weight to the gestational age using the
C. Uninterrupted skin-to-skin contact Lubchenco chart.
D. Breastfeeding Dr. De Vera

TOPNOTCH MEDICAL BOARD PREP PEDIATRICS MAIN HANDOUT BY DRS. DE VERA AND PUNONGBAYAN Page 1 of 106
For inquiries visit www.topnotchboardprep.com.ph or email us at topnotchmedicalboardprep@gmail.com
This handout is only valid for October 2022 PLE batch. This will be rendered obsolete for the next batch since we update our handouts regularly.
TOPNOTCH MEDICAL BOARD PREP PEDIATRICS MAIN HANDOUT BY DRS. DE VERA AND PUNONGBAYAN
For inquiries visit www.topnotchboardprep.com.ph or https://www.facebook.com/topnotchmedicalboardprep/
This handout is only valid for October 2022 PLE batch. This will be rendered obsolete for the next batch since we update our handouts regularly.
SMALL FOR GESTATIONAL AGE LARGE FOR GESTATIONAL AGE
• also known as intra-uterine growth retardation (IUGR) • Birth weight of >90th percentile for gestational age
• BW is < 3rd percentile for calculated gestational age • maternal diabetes & obesity are predisposing factors
• growth of the fetus affected by fetal, uterine, placental, and • At risk for hypoglycemia, polycythemia
maternal factors • At increased risk of:
• increased morbidity and mortality o respiratory distress syndrome
IUGR o congenital cardiac defects
o lumbosacral agenesis
ASYMMETRIC SYMMETRIC
o Hyperbilirubinemia
• Associated with poor • Associated with genetic and o birth injuries
maternal nutrition or with metabolic conditions
late onset or exacerbation • Associated with diseases ✔ GUIDE QUESTION
of maternal vascular that seriously affect fetal All of the ff. are components of essential newborn care except?
A. Early skin to skin contact
disease cell number B. Immediate thorough drying
• Weight affected > length • Weight, height, HC equally C. Properly timed clamping and cutting of the cord
• Head continues to grow affected D. Timely vitamin k and eye prophylaxis
• Fetus affected in late • Fetus affected early in
SUCTIONING
gestation gestation <18 weeks
• Postnatal catch-up growth • Poor brain growth pre- and • Routine suctioning of nose and mouth has no proven benefit
is good postnatally • No benefit of routine suction for babies born with meconium
• High morbidity and mortality stained amniotic fluid
• Oropharyngeal suctioning is not indicated for a healthy, active baby

APGAR
APGAR SCORING SYSTEM
0 POINTS 1 POINT 2 POINTS
Arms and Active
Activity Absent
legs flexed movement
IMMEDIATE NEWBORN EINC Below 100
Pulse Absent Over 100 bpm
CARE https://qrs.ly/bjdueho bpm
https://qrs.ly/lyduehe Grimace Active motion
Some flexion
(reflex Flaccid (sneeze, cough,
of extremities
✔ GUIDE QUESTIONS irritability) pull away)
A child was delivered preterm via emergency CS to a 37-year-old Body pink,
primigravida because of fetal distress. Upon delivery, he was noted to Appearance Completely
Blue, pale extremities
be cyanotic on the extremities, HR 120/min, coughed when suctioned, (skin color) pink
blue
with good tone and with regular respiration.
Slow,
What is the APGAR score at 1 minute? Respiration Absent Vigorous cry
Answer: APGAR 9
irregular
Total Score:
The most important component of the APGAR score is the: 0-3 Severely depressed
A. Heart rate 4-6 Moderately depressed
B. Activity
7-10 Excellent Condition
C. Respiration
D. Grimace This should be an easy point but believe me, even residents and
The best description of the APGAR score is that it: consultants get this wrong. Please memorize this table. Remember when
A. Accurately predicts who will develop cerebral palsy given the word “grimace” the score for that is 1.
Dr. De Vera
B. Assesses neonates in need of resuscitation
C. Accurately predicts a low umbilical cord pH
D. Accurately predicts neonates who will die in the neonatal period

TOPNOTCH MEDICAL BOARD PREP PEDIATRICS MAIN HANDOUT BY DRS. DE VERA AND PUNONGBAYAN Page 2 of 106
For inquiries visit www.topnotchboardprep.com.ph or email us at topnotchmedicalboardprep@gmail.com
This handout is only valid for October 2022 PLE batch. This will be rendered obsolete for the next batch since we update our handouts regularly.
TOPNOTCH MEDICAL BOARD PREP PEDIATRICS MAIN HANDOUT BY DRS. DE VERA AND PUNONGBAYAN
For inquiries visit www.topnotchboardprep.com.ph or https://www.facebook.com/topnotchmedicalboardprep/
This handout is only valid for October 2022 PLE batch. This will be rendered obsolete for the next batch since we update our handouts regularly.
APGAR ADDITIONAL FACTS ABOUT APGAR SCORE
• rapidly assess the need to resuscitate • It should NOT be used to determine the need for resuscitation
• some value in predicting neonatal mortality and cerebral palsy, or to guide the steps of resuscitation
it has a poor positive predictive value. • Changes in the score MAY reflect how well the infant is
• Most children with cerebral palsy have had normal APGAR responding to resuscitation
scores • Several factors may affect the APGAR score such as prematurity,
• Neonates with low APGAR scores do not universally get cerebral sedative drugs, congenital anomalies, and neuropathies.
palsy. • There is NO consistent data on the significance of APGAR score
on preterm infants
• It is NOT used to established asphyxia
• It does NOT predict specific neurologic outcomes
• A low 1-minute score does not correlate with outcome.
However, the 5-minute score is a valid predictor of neonatal
mortality.

NEWBORN RESUSCITATION: FROM PHILIPPINE SOCIETY OF NEWBORN MEDICINE, NRPH+

NEWBORN SCREENING EXPANDED NEWBORN SCREENING


• Congenital Hypothyroidism (CH) • Expanded NBS from 6 to 28 diseases
• Congenital Adrenal Hyperplasia • Includes the following
• Galactosemia o Hemoglobinopathies
o Disorder of amino acid and organic acid metabolism
• Phenylketonuria
o Fatty acid oxidation
• Glucose-6-Phosphate Dehydrogenase Deficiency
o Carbohydrate metabolism
• MSUD
o Biotin metabolisms
✔ GUIDE QUESTIONS o Cystic Fibrosis
On what day of life is NBS done?
Answer: 24-48 hours of life
CONGENITAL HYPOTHYROIDISM
NEWBORN SCREENING TEST • Normal birth weight & length
• RA #9288 • Delayed physical, mental & sexual development
• Ideally done at 48 hours of life • Sluggish, feeding difficulties, hypothermia
• If blood was collected <24 hours old, repeat at 2 weeks old. • Edema of scrotum / genitals
• For preterm: ideal time for NBS should be at 5-7 days old • Prolonged physiologic jaundice
• Can be done until 1 month old (for sick babies) • Most common cause is Thyroid Dysgenesis
• Expanded NBS covers 28 diseases

TOPNOTCH MEDICAL BOARD PREP PEDIATRICS MAIN HANDOUT BY DRS. DE VERA AND PUNONGBAYAN Page 3 of 106
For inquiries visit www.topnotchboardprep.com.ph or email us at topnotchmedicalboardprep@gmail.com
This handout is only valid for October 2022 PLE batch. This will be rendered obsolete for the next batch since we update our handouts regularly.
TOPNOTCH MEDICAL BOARD PREP PEDIATRICS MAIN HANDOUT BY DRS. DE VERA AND PUNONGBAYAN
For inquiries visit www.topnotchboardprep.com.ph or https://www.facebook.com/topnotchmedicalboardprep/
This handout is only valid for October 2022 PLE batch. This will be rendered obsolete for the next batch since we update our handouts regularly.

CONGENITAL ADRENAL HYPERPLASIA • Treatment


• Autosomal recessive disorders of cortisol biosynthesis o Glucocorticoid replacement
§ 15-20 mg/m2/24 hr of hydrocortisone daily administered
• Deficiency in cortisol → ↑corticotropin (ACTH) →
orally in 3 divided doses.
adrenocortical hyperplasia and overproduction of
§ Stress dose (2-3x usual dose)
intermediate metabolites
§ Goal is to achieve normal growth and development.
• Depending on pathway
o Mineralocorticoid replacement for salt-wasting type
o Mineralocorticoid deficiency or excess
§ Fludrocortisone
o Incomplete virilization or precocious puberty in males
o Surgical management
o Virilization or sexual infantilism in females
o Prenatal treatment
• Deficiency of 21-hydroxylase enzyme (90%): deficiency of
§ Dexamethasone – suppresses androgens and prevents
cortisol
virilization if given by 6 weeks AOG
• Normal at birth but signs of sexual & somatic precocity appear
within the 1st 6 months of life Take note that prenatal treatment of Dexamethasone is done for females only
Dr. De Vera
• Vomiting, failure to thrive
• Lab findings: Hyponatremia, hyperkalemia, hypoglycemia
(usually 10th-14th day of life)
• Marked elevation of 17-hydroxyprogesterone
o Normally elevated in first 2-3 DOL and in sick pre-term infants

CONGENITAL ADRENAL CONGENITAL ADRENAL


HYPERPLASIA 1 HYPERPLASIA 2
https://qrs.ly/w3duei4 https://qrs.ly/sadueia

© Topnotch Medical Board Prep


ENZYME SEX
MINERALOCORTICOIDS [K+] BP CORTISOL LABS PRESENTATION
DEFICIENCY HORMONES
• XY: atypical genitalia,
17𝝰- undescended testes
⬆ ⬇ ⬆ ⬇ ⬇ ⬇ androstenedione
hydroxylase • XX: lacks 2° sexual
development
• Most common
⬆renin activity • Presents in infant (salt
21-
⬇ ⬆ ⬇ ⬇ ⬆ ⬆17- wasting) or childhood
hydroxylase
hydroxyprogesterone (precocious puberty)
• XX: virilization
• Presents in infancy
⬇aldosterone
(severe hypertension)
11β- ⬆11-
⬇ ⬆ ⬇ ⬆ ⬇renin activity or childhood
hydroxylase deoxycorticosterone
(precocious puberty)
(results in ⬆BP)
• XX: virilization
All congenital adrenal enzyme deficiencies are autosomal recessive disorders and most are characterized by skin hyperpigmentation (due to ↑ MSH production,
which is coproduced and secreted with ACTH) and bilateral adrenal gland enlargement (due to ↑ ACTH stimulation.
If deficient enzyme starts with 1, it causes hypertension; if deficient enzyme ends with 1, it causes virilization in females
Adapted from First Aid USMLE 2022

TOPNOTCH MEDICAL BOARD PREP PEDIATRICS MAIN HANDOUT BY DRS. DE VERA AND PUNONGBAYAN Page 4 of 106
For inquiries visit www.topnotchboardprep.com.ph or email us at topnotchmedicalboardprep@gmail.com
This handout is only valid for October 2022 PLE batch. This will be rendered obsolete for the next batch since we update our handouts regularly.
TOPNOTCH MEDICAL BOARD PREP PEDIATRICS MAIN HANDOUT BY DRS. DE VERA AND PUNONGBAYAN
For inquiries visit www.topnotchboardprep.com.ph or https://www.facebook.com/topnotchmedicalboardprep/
This handout is only valid for October 2022 PLE batch. This will be rendered obsolete for the next batch since we update our handouts regularly.

GALACTOSEMIA ✔ GUIDE QUESTIONS


The cheese-like material that covers the normal term infant in varying
• 3 distinct enzyme deficiencies: amounts:
1. Galactose-1-phosphate uridyltransferase deficiency A. Lanugo
(GALT) – classic form B. Vellus hair
§ Classic galactosemia – complete or near complete deficiency C. Vernix caseosa
§ Partial Transferase Deficiency D. Milia
2. Galactokinase deficiency (GALK) The purplish reticulated pattern noted on the skin of a neonate when
§ presents with cataracts exposed to cold:
3. Galactose-4-epimerase deficiency (GALE) A. Mongolian spots
B. Acrocyanosis
§ Benign form – healthy individuals. Enzyme deficiency limited
C. Cutis marmorata
to leukocytes and erythrocytes D. Harlequin color change
§ Severe form – similar to transferase deficiency
• Without the enzyme, unable to convert galactose to galactose-1- PHYSICAL FINDINGS DIAGNOSIS
phosphate and uridine diphosphate galactose Slate blue, well demarcated areas of
Mongolian Spots
o → accumulation & injury to parenchymal cells of the kidney, pigmentation over the buttocks and back
liver, & brain (may begin in utero) Small, white occasionally
• Classic form may manifest within weeks after birth vesiculopustular papules on an
Erythema toxicum
• Feeding intolerance, vomiting, jaundice, convulsions, lethargy, erythematous base develop after 1-3
hypotonia, mental retardation days. Contains eosinophils.
• Patients with galactosemia are at increased risk for E. coli Vesiculopustular eruption over a dark
Pustular
neonatal sepsis macular base around the chin, neck,
melanosis
• Tx: Lactose-free milk back, and soles. Contains neutrophils.
• Long term sequelae (even with treatment): ovarian failure, Cysts appearing on the hard palate
developmental delay, learning disability which is composed of accumulations of Epstein pearls
epithelial cells.
Pearly white papules seen mostly on
G6PD DEFICIENCY Milia
the chins and around the cheeks
• Episodic or chronic hemolytic anemia Open and closed comedones or
• Episodic: symptoms develop 1-2 days after exposure to a inflammatory pustules and papules on
substance with oxidant properties: Neonatal acne
the cheeks of the baby usually after a
o sulfonamides, nalidixic acid, nitrofurantoin, chloramphenicol, week from delivery
antimalarials, vitamin K analogs, ASA, benzene, naphthalene Pink macular lesions on the nape, glabella,
Nevus simplex
upper eyelids, or nasolabial region
PHENYLKETONURIA Hair that covers the skin of preterm
• What is the enzyme that is deficient in phenylketonuria? infants. Seen on term infants around Lanugo
o Phenylalanine hydroxylase --- hyperphenylalaninemia the shoulders.
• What are the effects of excessive phenylalanine? Thick, white creamy material usually
Vernix caseosa
o Excess phenylalanine is transaminated to phenylpyruvic absent in post term infants
acid or decarboxylated to phenylethylamine & disrupts Mottling of the skin with venous
normal metabolism & causes brain damage prominence. Cobblestone, lacy Cutis marmorata
• Affected infant is normal at birth appearance
• Most common manifestation without treatment is PHYSICAL
developmental delay DIAGNOSIS INTERVENTION
EXAMINATION
• MR develop gradually • Abnormal
• Infant: severe vomiting, hypertonic, hyperactive DTRs, seizures; crepitus
Older: hyperactive with purposeless movements, rhythmic palpated
rocking, & athetosis around the
Clavicular
• unpleasant musty odor clavicle Immobilization
fracture
• Clavicle not
MAPLE SYRUP URINE DISEASE clearly
delineated in
• Genetic metabolic disorder resulting from the defective activity the skin
ABSENT
of the enzyme branched chain alpha-keto acid MORO • Arm abducted
dehydrogenase complex REFLEX • Pronated
• Accumulation of branched chain keto acid is toxic to the brain • Internally
• Irritability, poor feeding, maple syrup odor, encephalopathy, rotated
central respiratory failure • History of Brachial
• Treatment: removal of sources of EAA of leucine, isoleucine, or Observe
excessive plexopathy
valine in the diet traction on the
QUICK SHEET head
• Intact hand
Galactosemia: CNS + kidney + liver
grasp reflex
PKU: CNS + delayed presentation + musty odor
MSUD: CNS + more abrupt presentation + maple syrup odor

SUMMARY TABLE FOR NEWBORN SCREENING-RELATED DISEASES


DISEASE WHAT TO LOOK FOR
Galactosemia Kidney, liver, brain
Phenylketonuria musty odor, neurologic manifestation, gradual
Maple syrup odor, neurologic
MSUD
manifestation, usually earlier in onset
Chronic or acute hemolytic anemia.
G6PD
Usually triggered by oxidative stress
CAH Normal at birth. Ambiguous genitalia.
(21-hydroxylase Vomiting. Sexual and somatic precocity.
deficiency) Hyponatremia, Hyperkalemia, Hypoglycemia
Congenital Delayed physical, mental and sexual
Hypothyroidism development. Macroglossia. GDD http://misc.medscape.com/

TOPNOTCH MEDICAL BOARD PREP PEDIATRICS MAIN HANDOUT BY DRS. DE VERA AND PUNONGBAYAN Page 5 of 106
For inquiries visit www.topnotchboardprep.com.ph or email us at topnotchmedicalboardprep@gmail.com
This handout is only valid for October 2022 PLE batch. This will be rendered obsolete for the next batch since we update our handouts regularly.
TOPNOTCH MEDICAL BOARD PREP PEDIATRICS MAIN HANDOUT BY DRS. DE VERA AND PUNONGBAYAN
For inquiries visit www.topnotchboardprep.com.ph or https://www.facebook.com/topnotchmedicalboardprep/
This handout is only valid for October 2022 PLE batch. This will be rendered obsolete for the next batch since we update our handouts regularly.
Feature CAPUT SUCCEDANEUM CEPHALHEMATOMA SUBGALEAL HEMORRHAGE
At point of contact; can extend Usually over parietal bones; does not Beneath epicranial aponeurosis; may
Location
across sutures cross sutures extend to orbits, nape of neck
Characteristic Vaguely demarcated; pitting Distinct margins; initially firm, more Firm to fluctuant; ill-defined borders;
findings edema that shifts with gravity fluctuant after 48h may have crepitus or fluid waves
Maximal size and firmness at Increases after birth for 12-24 h; Progressive after birth; resolution
Timing
birth; resolves in 48-72h resolution over 2-3 weeks over 2-3 weeks
May be massive, especially if there is
Volume of blood Minimal Rarely severe
an associated coagulopathy
WHY IS IT IMPORTANT TO CLASSIFY?
CAPUT SUCCEDANEUM • To be able to determine an effective treatment regimen for the
vs. CEPHALHEMATOMA disease condition
vs. SUBGALEAL HEMORRHAGE • Example: Early onset neonatal sepsis
https://qrs.ly/awdueig o Organisms: BEL mnemonics
o Empiric antimicrobials: Ampicillin + Gentamicin
✔ GUIDE QUESTIONS
A. Cephalohematoma SURGICAL CONDITIONS IN NEWBORN
B. Caput succedaneum ✔ GUIDE QUESTION
____________1. Cross the midline and suture lines A 12 hour-old newborn born NSD to a G1P1 28-year-old mother was
____________2. Subperiosteal hemorrhage noted to have frothing at the mouth of secretions. When he was given
____________3. Limited to the surface of one cranial bone milk, he regurgitated it. PE: crying, CR 155/min, RR=68/min, harsh
____________4. No discoloration of overlying scalp breath sounds, good cardiac tone, soft abdomen, cyanotic lips and
____________5. Edema, ecchymosis, and swelling apparent right after nailbeds. What is your impression?
birth A. GER
____________6. May cause jaundice Answers: B A A A B A B. GERD
C. EA with TEF
D. Diaphragmatic Hernia
CONGENITAL INFECTIONS
ESOPHAGEAL ATRESIA
CASE KEY CLUES DIAGNOSIS
Vesicular lesions on the • Most common congenital anomaly of the esophagus
HSV • >90% have an associated tracheoesophageal fistula (TEF)
face and mouth
Purpuric hemorrhagic • Most common form of EA
Rubella o the upper esophagus ends in a blind pouch and the TEF is
lesions all over the body
Maculopapular rash connected to the distal esophagus (type C)
A baby with
Imaging: Periostitis of the Syphilis
IUGR born to
bone
a mother
Chorioretinitis
with a
Imaging: Periventricular CMV
history of
calcifications
infection
Chorioretinitis
during
Microcephaly
pregnancy
Hepatosplenomegaly Toxoplasmosis
Imaging: Intracerebral
calcifications
Cutaneous scars
Varicella
Imaging: Cortical atrophy

✔ GUIDE QUESTIONS
Early onset neonatal infection, according to Nelson’s occurs
A. 1st week of life
B. 7-30 days of life
C. >4 weeks of life
D. None of the above

EPIDEMIOLOGY OF NEONATAL INFECTIONS


Infection Description Remarks
Usually by viruses and
Congenital Acquired in bacteria. Associated with
Infection utero injury to developing
organs TEF MANIFESTATIONS:
Acquisition • Frothing & bubbling at the mouth & nose, cough, cyanosis,
Perinatal
around the time -- respiratory distress
Infection
of delivery • Feeding exacerbates the symptoms, causes regurgitation &
Consequence of infection precipitate aspiration
by organism acquired • Diagnosis: inability to pass an NGT or OGT in the newborn is
Early-Onset
1st week of life during the perinatal suggestive
Infection
Period. Maternal flora
(GBS, E.coli, Listeria) MANAGEMENT OF TEF
Late onset • Management: maintain a patent airway
7-30 days of life Postnatally acquired • Prone position
infection
• Esophageal suctioning
Very Late
• Surgical ligation of TEF & primary end-to-end anastomosis of
onset After 30 days Postnatally acquired
the esophagus
Infection
Hospital Typically occur Type C (esophagus ends in blind pouch + distal TEF) is the most common.
Acquired beyond the first -- Do not be confused with this. It’s not type A. It’s type C! Trust me. You can
cross reference with surgical books.
Infection week of life Dr. De Vera

TOPNOTCH MEDICAL BOARD PREP PEDIATRICS MAIN HANDOUT BY DRS. DE VERA AND PUNONGBAYAN Page 6 of 106
For inquiries visit www.topnotchboardprep.com.ph or email us at topnotchmedicalboardprep@gmail.com
This handout is only valid for October 2022 PLE batch. This will be rendered obsolete for the next batch since we update our handouts regularly.
TOPNOTCH MEDICAL BOARD PREP PEDIATRICS MAIN HANDOUT BY DRS. DE VERA AND PUNONGBAYAN
For inquiries visit www.topnotchboardprep.com.ph or https://www.facebook.com/topnotchmedicalboardprep/
This handout is only valid for October 2022 PLE batch. This will be rendered obsolete for the next batch since we update our handouts regularly.
✔ GUIDE QUESTIONS ✔ GUIDE QUESTIONS
A newborn infant develops tachypnea, grunting, and use of accessory The classic story of NEC is a preterm neonate with a stormy clinical
muscles together with cyanosis at the 2nd hour of life. He also has a course given formula milk. The neonate then deteriorates with signs
scaphoid abdomen, (+) decreased breath sounds bilaterally, the point of bowel compromise. This is in line with the triad of the
of maximal impulse is shifted near the sternal side, what is the most pathophysiology of NEC:
likely diagnosis? 1. Intestinal ischemia (GI tract of toxic neonates are ischemic)
A. Bochdalek 2. Enteral nutrition (serves as substrate for organism)
B. Morgagni 3. Pathologic organisms (colonic bacteria like Clostridium, E. coli,
C. Congenital cystic adenomatous malformation Klebsiella)
D. Pulmonary Sequestration Dr. De Vera

A term infant with severe respiratory disease. Scaphoid abdomen on PE. A 1-day old male neonate is being referred for vomiting. The vomitus
Diagnosis: Congenital Diaphragmatic Hernia is greenish yellowish in color according to the mother. PE revealed a
Most common type: Bochdalek hernia non-distended, non-tender abdomen. What is the most likely finding on
workup?
A. Double Bubble Sign
B. Increased pyloric thickness
C. Donut Ring Sign
D. Absence of parasympathetic postganglionic cell bodies
A 1-week-old neonate presents with vomiting characterized as bilious,
last passage of stool was 2 days ago. Ultrasound shows that the
superior mesenteric vein is to the left of the artery, upper GI series
shows absence of the cecum at the RLQ, what is the best next step in the
management of this patient?
A. Detorsion C. Watchful Waiting
B. Barium Enema D. PEG Insertion
A term infant is born with extrusion of the abdominal viscera
Diagnosis? In questions like this, always stay calm. There are several ways of
Omphalocele versus Gastroschisis answering this. First would be to memorize the diagnostic finding. If
you know your anatomy from the ultrasound you will know that the
• What is your immediate intervention? answer is malrotation, hence the management is detorsion. If you are
o Decompression and wrapping not able to memorize the ultrasound finding, the examiner is kind
• Which one is associated with other congenital anomalies? enough to give you another test which is the upper GI series. From
o Omphalocele (OEIS complex) UGIS findings, one can deduce that this is malrotation (cecum did not
• Gastroschisis is more common on left side or right side of umbilicus? migrate to the RLQ).
o Right Dr. De Vera

The following is TRUE regarding this disease A 28-year-old primigravida went into labor at 33 weeks age of
A. The defect is more commonly located gestation. The obstetrician was concerned regarding the fetal lung
at the right side of the umbilicus maturity and requested for amniotic fluid L/S ratio. Which should be?
B. There is a membranous sac covering A. >2 C. >4
the defect B. >3 D. >5
C. Other congenital anomalies are A one-month-old male was brought to your clinic due to scrotal
commonly associated with this defect swelling. Mother noticed this to be present since birth. There were no
D. Bowel and alimentation is normal other associated symptoms. No fever, no tenderness. Baby was well,
with good suck and activity. On physical examination you see this
OMPHALOCELE VS GASTROSCHISIS
OMPHALOCELE GASTROSCHISIS
SAC + -
Below the Lateral to the
LOCATION
umbilicus umbilicus, Right
UMBILICAL Center of
Left of the defect
CORD membrane
ASSOCIATED
60% 10%
DEFECTS
BOWEL Normal Inflamed What is your diagnosis?
ALIMENTATION Normal Delayed
Surgical, TPN, Surgical, TPN,
Answer: Hydrocele
MANAGEMENT
Hydration Hydration
GENITOURINARY ANOMALIES
✔ GUIDE QUESTIONS
• Hydrocele - accumulation of fluid in the tunica vaginalis (1-2%
A preterm baby won’t stop crying. He then developed abdominal
distention with abdominal erythema. The baby cries more when of neonates); majority are noncommunicating; resolves by 12
touched. What is your diagnosis? months old
NEC • Hernias – usually indirect inguinal hernias; presents as a
• Coagulation necrosis – histologic finding reducible scrotal swelling
• Thickened bowel walls and air in the bowel wall: Important points to remember:
o PNEUMATOSIS INTESTINALIS Hydrocoele – may observe up to 1 year of age
What will be your intervention? Undescended testes – may observe up to 3-4 months of age
Supportive Inguinal hernia – needs to be repaired surgically
Dr. De Vera
Two days later, the baby developed pneumoperitoneum. What will be
your intervention?
Surgery

All of the ff. are accepted interventions for Necrotizing enterocolitis


except?
A. Broad Spectrum Antibiotic Therapy
B. Umbilical Catheterization
C. IV Volume Replacement
D. Surgery
TOPNOTCH MEDICAL BOARD PREP PEDIATRICS MAIN HANDOUT BY DRS. DE VERA AND PUNONGBAYAN Page 7 of 106
For inquiries visit www.topnotchboardprep.com.ph or email us at topnotchmedicalboardprep@gmail.com
This handout is only valid for October 2022 PLE batch. This will be rendered obsolete for the next batch since we update our handouts regularly.
TOPNOTCH MEDICAL BOARD PREP PEDIATRICS MAIN HANDOUT BY DRS. DE VERA AND PUNONGBAYAN
For inquiries visit www.topnotchboardprep.com.ph or https://www.facebook.com/topnotchmedicalboardprep/
This handout is only valid for October 2022 PLE batch. This will be rendered obsolete for the next batch since we update our handouts regularly.

COMPOSITION OF SURFACTANT RECOVERED BY ALVEOLAR


• Undescended testes
WASH
o About 4.5% of boys at birth Nelson Textbook of Pediatrics, 20th ed.
o Majority descend simultaneously during the 1st 3 months of ✔ GUIDE QUESTION
life After catching a full-term baby boy, 2600g, AGA, on the 5th minute of life
o By 6 mos old, the incidence decreases to 0.8% you decided to check the oxygen saturation of the patient. A maximum
o If the testes has not descended by 4 months, it will remain saturation of 90% was read. The baby has good APGAR score, is
undescended. comfortable, and there are no other abnormal physical findings. What
is the next step in managing this patient?
o Treated surgically not later than 9-15 months old
A. Provide O2 support C. Observe and monitor
B. Stimulate the patient D. Suction secretions
RESPIRATORY CONDITIONS MINUTE(S) OF LIFE TARGET PRE DUCTAL O2 SATS
SURFACTANT 1 min 60-65%
• Surfactant is present in high concentrations in fetal lung 2 min 65-70%
homogenates by 20 wk of gestation
3 min 70-75%
• It appears in amniotic fluid between 28 and 32 wk of gestation
• Mature levels of pulmonary surfactant are present usually after 4 min 75-80%
35 wk of gestation 5min 80-85%
10min 85-95%

SURFACTANT
https://qrs.ly/iudueiq

This image simply shows the huge gamut of possible differential diagnoses for a neonate that presents with respiratory distress. Remember that the problem may
not be limited to the lungs (e.g., anemia can present with respiratory distress).
Dr. De Vera

✔ GUIDE QUESTION APNEA


A preterm 34-week neonate was referred by the nurse due to cessation
• defined as cessation of breathing for longer than 20 seconds or for
of breathing lasting 10 seconds followed by rapid respiration of 50-
any duration if accompanied by cyanosis and bradycardia
60bpm. Vital signs are stable, no cyanosis. If another episode occurs
what will you advise the nurse? • More common in pre-term infants
A. Stimulate patient • Causes
B. Positive Pressure Ventilation o Most common cause is idiopathic apnea of prematurity
C. CPR o direct depression of the central nervous system’s control of
D. NOTA respiration (hypoglycemia, meningitis, drugs, hemorrhage,
seizures)
o disturbances in oxygen delivery (shock, sepsis, anemia)
o ventilation defects (obstruction of the airway, pneumonia, muscle
weakness).
TOPNOTCH MEDICAL BOARD PREP PEDIATRICS MAIN HANDOUT BY DRS. DE VERA AND PUNONGBAYAN Page 8 of 106
For inquiries visit www.topnotchboardprep.com.ph or email us at topnotchmedicalboardprep@gmail.com
This handout is only valid for October 2022 PLE batch. This will be rendered obsolete for the next batch since we update our handouts regularly.
TOPNOTCH MEDICAL BOARD PREP PEDIATRICS MAIN HANDOUT BY DRS. DE VERA AND PUNONGBAYAN
For inquiries visit www.topnotchboardprep.com.ph or https://www.facebook.com/topnotchmedicalboardprep/
This handout is only valid for October 2022 PLE batch. This will be rendered obsolete for the next batch since we update our handouts regularly.
• Apnea Management ✔ GUIDE QUESTIONS
o immediate management What is the expected chest x-ray finding of the above case?
§ Stimulation + O2 for 30 seconds, if it does not work → A. Fine reticular granularity of the parenchyma and air
§ PPV for 30 seconds, if it does not work → bronchograms
§ Intubate B. Prominent pulmonary vascular markings, fluid in the
§ CPR anytime if heart rate falls <60bpm intralobar fissures, overaeration, flat diaphragms
o Identify and correct causes C. Normal findings
o Methylxanthines (Caffeine or theophylline) D. Patchy infiltrates, coarse streaking of both lung fields,
§ increase central respiratory drive by lowering the threshold of increased anteroposterior diameter, and flattening of the
response to hypercapnia diaphragm
§ enhances contractility of the diaphragm and preventing Which of the following is very useful in evaluating a patient with
diaphragmatic fatigue the above case?
A. Chest x-ray
✔ GUIDE QUESTIONS B. Chest CT with IV contrast
A 23-year-old primigravid gave birth at 33-week AOG to male C. 12 lead ECG
newborn via SVD. Several minutes after birth, the newborn D. Real-time echocardiography with Doppler flow
developed tachypnea, prominent grunting, intercostal and E. AOTA
subcostal retractions, nasal flaring which progressed to cyanosis. A 30-weeker premature baby was treated for severe RDS. The
Upon auscultation, breath sounds were diminished with a harsh patient was intubated on the first day of life, given surfactant
tubular quality and on deep inspiration, fine crackles were heard. therapy, and was on assisted ventilation for two weeks due to
Which of the following is your initial diagnosis? complications. Patient was subsequently extubated; however,
A. Respiratory distress syndrome oxygen support was never completely weaned off. At 36 weeks, he
B. Transient tachypnea of the newborn could tolerate oxygen support at 1 lpm via nasal cannula. What is
C. Bronchopulmonary dysplasia the pathophysiology of this disease condition?
D. Persistent Pulmonary Hypertension A. Chronic lung injury due to hypoxia
What is the primary cause of the above case? B. Generation of free radicals from supplemental oxygenation
A. Persistence of the fetal circulatory pattern of right-to-left C. Inflammation
shunting through the PDA and foramen ovale after birth D. All of the above
B. Slow absorption of fetal lung fluid What is the classification of the case above?
C. Surfactant deficiency A. Mild
D. AOTA B. Moderate
What is the expected chest x-ray finding of the above case? C. Severe
A. Fine reticular granularity of the parenchyma and air D. NOTA
bronchograms What is the expected chest x-ray finding in the case above?
B. Prominent pulmonary vascular markings, fluid in the A. interstitial emphysema, wandering atelectasis with
intralobar fissures, overaeration, flat diaphragms concomitant hyperinflation, and cyst formation
C. Normal findings B. Ground glass opacity with underinflation
D. Patchy infiltrates, coarse streaking of both lung fields, C. Increased pulmonary vascular marking
increased anteroposterior diameter, and flattening of the D. All of the above
diaphragm The following are acceptable management for this patient except?
E. NOTA A. Fluid restriction
A 23-year-old primigravid gave birth at 38 weeks AOG via CS to B. Nutritional Support
male newborn. Several minutes after birth, the newborn C. Treatment of Infection
developed tachypnea, retractions, expiratory grunting. Upon D. NOTA
auscultation, the lungs are clear without crackles or wheeze.
CONDITION CXR FINDINGS
Which of the following is the initial diagnosis?
A. Respiratory distress syndrome Hyaline Membrane Finely granular lungs
B. Transient tachypnea of the newborn Disease Ground glass appearance
C. Bronchopulmonary dysplasia Prominent pulmonary vascular
D. Persistent Pulmonary Hypertension TTN markings
What is the primary cause of the above case? Fluid lines in fissure
A. Slow absorption of fetal lung fluid BPD “bubbly lungs” (cystic lucencies)
B. Surfactant deficiency Meconium Aspiration Coarse streaking granular pattern
C. Persistence of the fetal circulatory pattern of right-to-left Syndrome of both lung fields
shunting through the PDA and foramen ovale after birth Neonatal Pneumonia Perihilar streaking
D. AOTA
What is the expected chest x-ray finding of the above case? DEFINITION OF BPD
Adapted from Nelson Textbook of Pediatrics, 20th ed.
A. Fine reticular granularity of the parenchyma and air
GESTATIONAL AGE
bronchograms
B. Prominent pulmonary vascular markings, fluid in the <32 Weeks ≥32 Weeks
intralobar fissures, overaeration, flat diaphragms • 36 weeks • >28 days but <56
C. Normal findings postmenstrual age or days postnatal age or
D. Patchy infiltrates, coarse streaking of both lung fields, increased Time point discharge home, discharge home,
anteroposterior diameter, and flattening of the diaphragm of whichever comes first whichever comes first
E. NOTA assessment • Treatment with • Treatment with
A 32-year-old G4P3 gave birth to a 42 weeks male neonate via SVD >21% oxygen for at >21% oxygen for at
after 18 hours of labor. The neonate was noted to have aspirated least 28 days plus least 28 days plus
meconium. Twelve hours after birth, the neonate was noted to Breathing room air at 36 Breathing room air by
have grunting, nasal flaring, and intercostal retractions. He was weeks postmenstrual 56 days postnatal age
Mild BPD
also tachycardic and was hypoxemic at 80% O2 saturation. After age or discharge home, or discharge home,
drawing blood from the right radial artery and umbilical artery, a whichever comes first whichever comes first
PaO2 gradient was noted. Which of the following is the initial Need† for <30% oxygen
Need† for <30% at 56
diagnosis? at 36 weeks
Moderate days postnatal age or
A. Respiratory distress syndrome postmenstrual age or
BPD discharge home,
B. Transient tachypnea of the newborn discharge home,
whichever comes first
C. Bronchopulmonary dysplasia whichever comes first
D. Persistent Pulmonary Hypertension Need† for >30% oxygen Need† for >30%
What is the primary cause of the above case? and/or positive oxygen and/or positive
A. Surfactant deficiency pressure (PPV or pressure (PPV or
B. Slow absorption of fetal lung fluid Severe BPD NCPAP) at 36 weeks NCPAP) at 56 days
C. Persistence of the fetal circulatory pattern of right-to-left postmenstrual age or postnatal age or
shunting through the PDA and foramen ovale after birth discharge home, discharge home,
D. AOTA whichever comes first whichever comes first
TOPNOTCH MEDICAL BOARD PREP PEDIATRICS MAIN HANDOUT BY DRS. DE VERA AND PUNONGBAYAN Page 9 of 106
For inquiries visit www.topnotchboardprep.com.ph or email us at topnotchmedicalboardprep@gmail.com
This handout is only valid for October 2022 PLE batch. This will be rendered obsolete for the next batch since we update our handouts regularly.
TOPNOTCH MEDICAL BOARD PREP PEDIATRICS MAIN HANDOUT BY DRS. DE VERA AND PUNONGBAYAN
For inquiries visit www.topnotchboardprep.com.ph or https://www.facebook.com/topnotchmedicalboardprep/
This handout is only valid for October 2022 PLE batch. This will be rendered obsolete for the next batch since we update our handouts regularly.
*BPD usually develops in neonates being treated with oxygen and PPV for PERSISTENT PULMONARY HYPERTENSION (PPHN)
respiratory failure, most commonly respiratory distress syndrome.
Persistence of the clinical features of respiratory disease (tachypnea, • Persistence of the fetal circulatory pattern of right-to-left
retractions, crackles) is considered common to the broad description of BPD shunting through the PDA and foramen ovale after birth is a
and has not been included in the diagnostic criteria describing the severity of result of excessively high PVR
BPD. Infants treated with >21% oxygen and/or PPV for non-respiratory • PVR normally declines rapidly as a consequence of vasodilation
disease (e.g., central apnea or diaphragmatic paralysis) do not have BPD secondary to lung inflation, a rise in postnatal PaO2, a reduction
unless parenchymal lung disease also develops and they have clinical in PaCO2, increased pH, and release of vasoactive substances
features of respiratory distress. A day of treatment with >21% oxygen means
that the infant received >21% oxygen for more than 12 hr on that day.
Treatment with >21% oxygen and/or PPV at 36 weeks postmenstrual age or Pathogenesis
at 56 days postnatal age or discharge should not reflect an “acute” event, but • Maladaptation from acute injury
should rather reflect the infant’s usual daily therapy for several days • the result of increased pulmonary artery medial muscle
preceding and after 36 weeks postmenstrual age, 56 days postnatal age, or thickness and extension of smooth muscle layers into the usually
discharge. non-muscular, more peripheral pulmonary arterioles in
BPD, bronchopulmonary dysplasia; NCPAP, nasal continuous positive airway response to chronic fetal hypoxia
pressure; PPV, positive-pressure ventilation.
• a consequence of pulmonary hypoplasia (diaphragmatic hernia,
GROUND GLASS OPACITY Potter syndrome)
• obstructive (ex TAPVR, Polycythemia)

Other Important Facts About PPHN


• PaO2 or oxygen saturation gradient between a preductal (right
radial artery) and a post-ductal (umbilical artery) site of blood
sampling suggests right-to-left shunting through the ductus
arteriosus.
• 2D echo with Doppler is helpful

http://learningradiology.com/notes/chestnotes/hyalinemembranepage.htm

https://educalingo.com/en/dic-en/ground-glass

DISEASE AERATION Take a look at the flow of blood through a PDA. PDA is generally
Hyaline Membrane Disease Under acyanotic because the flow is left to right. However, in certain instances
Bronchopulmonary Dysplasia Over where PDA is large and/or the pulmonary vascular resistance is high
Transient Tachypnea Over (such as in PPHN), shunt becomes right to left and patient therefore
Meconium Aspiration Over becomes CYANOTIC!
Dr. De Vera
Neonatal Pneumonia Over
RDS TTN PPHN
DEMOGRAPHIC • Pre-Term • Pre-term or term, CS • Term or post term, MAS
• Ground Glass opacities, Under- • Hyper-aerated, Prominent Vascular • May be normal OR depending on
X-RAY
aerated, atelectasis markings co-morbid condition
ONSET OF • Early onset but relieved with • Within first 12 hours of birth.
• Within minutes of birth
SIGNS AND minimal oxygen • Cyanosis
• Grunting
SYMPTOMS supplementation • Oxygen gradient
• Progressive worsening of cyanosis
NATURAL and dyspnea
• Recovers rapidly within 3 days • Unpredictable course
COURSE • Symptoms peak within 3 days
then improves
• Prevent with antenatal steroids
• Supportive
TREATMENT • Surfactant replacement • Supplemental O2
• Treat underlying cause
• PEEP
✔ GUIDE QUESTIONS
A newborn infant born at 38 weeks AOG developed jaundice at the 12th
hour of life. Within the next 8 hours, the jaundice spread up to the mid
abdomen. Careful examination revealed some hepatosplenomegaly,
chorioretinitis, and mild hydrocephalus. Based on the given data, what
is the estimated level of bilirubin in this neonate?
LUNG PATHOLOGY RESPIRATORY DISTRESS A. 5mg/dL
IN NEONATES IN NEWBORNS B. 10mg/dL
C. 15mg/dL
https://qrs.ly/9fduek3 https://qrs.ly/m8duek9
D. 20mg/dL

SUPPLEMENT: Respiratory Distress in the Newborn When you have newborns with chorioretinitis and other features like
hydrocephalus and organomegaly, TORCHes should always be
Five common signs considered.
• Tachypnea (RR >60) Dr. De Vera

• Retractions
• Nasal flaring
• Grunting
• Central cyanosis

TOPNOTCH MEDICAL BOARD PREP PEDIATRICS MAIN HANDOUT BY DRS. DE VERA AND PUNONGBAYAN Page 10 of 106
For inquiries visit www.topnotchboardprep.com.ph or email us at topnotchmedicalboardprep@gmail.com
This handout is only valid for October 2022 PLE batch. This will be rendered obsolete for the next batch since we update our handouts regularly.
TOPNOTCH MEDICAL BOARD PREP PEDIATRICS MAIN HANDOUT BY DRS. DE VERA AND PUNONGBAYAN
For inquiries visit www.topnotchboardprep.com.ph or https://www.facebook.com/topnotchmedicalboardprep/
This handout is only valid for October 2022 PLE batch. This will be rendered obsolete for the next batch since we update our handouts regularly.
✔ GUIDE QUESTIONS PHYSIOLOGIC PATHOLOGIC
In relation to the above case, if a blood test was performed and is Jaundice visible only Appears on the
consistent with the bilirubin levels estimated against the physical on the 2nd-3rd day first 24-36 hours of life
exam, what is this child’s risk classification based on the Bhutani chart?
Serum bilirubin is
A. High Risk Peaks at 5-6 mg/dL
B. High Intermediate Risk
rising at a rate faster than
on the 2nd-4th day
C. Low Intermediate Risk 5 mg/dL/24 hours
D. Low Risk Decrease to below 2 mg/dL Serum bilirubin >12 mg/dL
between 5-7 days of life or 10-14 mg/dL in preterm
TB increases not TB increases
> 5mg/dL/day > 0.5 mg/dL/hour
Decline to adult levels by Jaundice persist after
10-14 days of life 10-14 days
Direct reacting bilirubin is
>2 mg/dL at any time
Physiologic jaundice is a DIAGNOSIS OF EXCLUSION. JUST ONE FEATURE
of pathologic makes it pathologic. Students often fail to remember this.
Dr. De Vera
JAUNDICE WITHIN 24 H
KEY CLUE MOST LIKELY ETIOLOGY
First born child ABO-incompatibility
Second born child Rh-incompatibility
History of prolonged second stage of labor
Sepsis Neonatorum
No prenatal check up
History of maternal infection during
TORCH infection
pregnancy
JAUNDICE AFTER 24 H
ONSET OF MOST LIKELY
Nelson Textbook of Pediatrics, 20th ed. KEY CLUE
In relation to the above case, what is the best step in the management JAUNDICE ETIOLOGY
of the child’s hyperbilirubinemia? 2-3 days Baby otherwise normal Physiologic
A. Evaluate for phototherapy Mother supplements
3-4 days Breastfeeding
B. Administer phenobarbital feeding with sugar water
C. Follow up within 2 days >1 week Baby is purely breastfed Breast Milk
D. Exchange transfusion
In relation to the above case, which diagnostic modality can help COMPARISON OF JAUNDICE RELATED TO BREASTFEEDING
confirm the diagnosis? BREAST FEEDING BREAST MILK
A. CT Scan JAUNDICE JAUNDICE
B. Blood Culture Onset 3-4 DOL End of the 1st week
C. Indirect Coombs Test
D. Newborn Screening
Inadequate nursing, Substance in
In relation to the case above, which among the ff. will most likely be Factors decreased caloric breastmilk:
seen in this patient? intake Glucuronidase
A. Coombs Test Positive Duration Few days 3 weeks – 3 mos
B. Triangular Cord Sign Continue
C. (+) blood culture growth of gram-positive cocci Treatment Stop for 2 days
breastfeeding 10h/day
D. Intracranial Calcifications
The most serious complication of hyperbilirubinemia in the newborn is: An easy way to remember which comes first. It’s alphabetical, F comes
A. Severe anemia before M. BFeeding jaundice occurs earlier than BMilk jaundice.
Dr. De Vera
B. Heart failure
C. Respiratory distress ✔ GUIDE QUESTION
D. Encephalopathy A 3-day old infant is jaundiced from the head down to the upper trunk.
Jaundice appearing between the second and third day after birth in full- His serum bilirubin level is probably between:
terms infants is likely due to: A. 6-8 mg/dL
A. Normal changes B. 9-12 mg/dL
B. Acute hemolysis C. 12-14 mg/dL
C. Sepsis neonatorum D. 15-18 mg/dL
D. Erythroblastosis fetalis
The most common cause of jaundice in neonates is:
A. Physiologic
B. Acute hemolysis
C. Sepsis neonatorum
D. Erythroblastosis fetalis

NEONATAL JAUNDICE
JAUNDICE RISK FACTORS IN NEONATAL
HYPERBILIRUBINEMIA
• Jaundice visible on the 1st day of life
• A sibling with neonatal jaundice or anemia
• Unrecognized hemolysis (ABO, Rh, other blood group,
incompatibility); UDP-glucuronyl transferase deficiency
(Crigler-Najjar, Gilbert disease)
• Non-optimal feeding (formula or breast-feeding)
• Deficiency of glucose-6-phosphate dehydrogenase
KERNICTERUS
• Infection (viral, bacterial). Infant of diabetic mother. Immaturity
(prematurity) • Results from deposition of unconjugated bilirubin in the basal
• Cephalohematoma or bruising. Central hematocrit >65% ganglia and brainstem
(polycythemia) • Kernicterus is rare in healthy infants if the serum level is less
• East Asian, Mediterranean, Native American heritage than 25 mg/dL
• Clinical manifestation
o Phase 1 – poor sucking, stupor, hypotonia, seizure
o Phase 2 – hypertonia, opisthotonos, fever
o Phase 3 – hypertonia
TOPNOTCH MEDICAL BOARD PREP PEDIATRICS MAIN HANDOUT BY DRS. DE VERA AND PUNONGBAYAN Page 11 of 106
For inquiries visit www.topnotchboardprep.com.ph or email us at topnotchmedicalboardprep@gmail.com
This handout is only valid for October 2022 PLE batch. This will be rendered obsolete for the next batch since we update our handouts regularly.
TOPNOTCH MEDICAL BOARD PREP PEDIATRICS MAIN HANDOUT BY DRS. DE VERA AND PUNONGBAYAN
For inquiries visit www.topnotchboardprep.com.ph or https://www.facebook.com/topnotchmedicalboardprep/
This handout is only valid for October 2022 PLE batch. This will be rendered obsolete for the next batch since we update our handouts regularly.

HYPERBILIRUBINEMIA COOMBS TEST NEGATIVE


Coombs test negative with ↑ Hb and ↑B1
DIRECT HYPERBILIRUBINEMIA • polycythemia
(ALWAYS PATHOLOGIC)
• infant of diabetic mother
1. Intrahepatic Cholestasis • SGA
o sepsis / TORCHeS
• delayed cord clamping
o prolonged TPN
• twin transfusion / maternal-fetal transfusion
o hypothyroidism
o galactosemia
Coombs test negative with normal / ↓ Hb and normal
o cystic fibrosis
reticulocyte count
o alpha-1-antitrypsin deficiency
• enclosed hemorrhage
2. Extrahepatic Cholestasis
• increased enterohepatic circulation
o choledochal cyst
o biliary atresia • decreased calories (e.g. breastfeeding jaundice)
§ paucity of bile ducts • disorders of conjugation (e.g. breastmilk jaundice)
Coombs test negative with normal / ↓Hb & ↑reticulocyte count
COOMBS TEST • with characteristic RBC morphology
• Direct Coombs test – used to detect antibodies that are bound o Spherocytosis
to the surface of RBCs o Elliptocytosis
• Indirect Coombs test – detects antibodies against RBCs that are • with non-characteristic RBC morphology
present unbound to the patient’s serum o G6PD deficiency
• Clinical uses: o Pyruvate kinase deficiency
1. Blood transfusion preparation
2. Antenatal antibody screening ✔ GUIDE QUESTIONS
An infant is born premature at 36 weeks to a mother with prolonged
preterm rupture of membranes. He developed jaundice on the 3rd day
INDIRECT HYPERBILIRUBINEMIA of life and has poor suck, is lethargic, and has retractions. You suspect
COOMBS TEST POSITIVE = ISOIMMUNIZATION him as having Gram-negative sepsis. Which antibiotic therapy will
• Rh / ABO incompatibility (before treatment) most likely benefit this child?
A. Ampicillin + amikacin
ABO INCOMPATIBILITY B. Trimethoprim + sulfamethoxazole
• Most common cause of hemolytic disease of the newborn C. Ciprofloxacin
• Occurs in 20-25% of pregnancies but hemolysis develops in only D. Imipenem + Cilastatin
10% of such offspring The most important risk factor that predisposes a neonate to sepsis is:
A. History of maternal infection
• Mother is type O and baby is either A or B
B. Prematurity
• Most cases are mild; jaundice C. Route of delivery
• Mild hepatosplenomegaly D. Unestablished normal flora
• Phototherapy; if severe, IVIG or exchange transfusion
• ABO incompatibility lab test results: EVALUATION OF HYPERBILIRUBINEMIA
1. Weakly to moderately (+) direct Coombs test
2. Spherocytes in blood smear
3. Hemoglobin is usually normal but maybe as low as 10-12
g/dL
4. Increased reticulocyte count in 10-15%
5. Increased B1 (may reach 20 mg/dL in 10-20%)

RH INCOMPATIBILITY
• Rh antigenic determinants are genetically transmitted from each
parent & direct the production of blood group factors
(C, c, D, d, E, e)
• Each factor can elicit a specific antibody response where 90%
are due to D antigen.
• Conditions when Rh incompatibility occurs:
1.When Rh+ blood is infused into a Rh- woman by error, or;
2.When Rh+ fetal blood with D Ag inherited from a Rh+ father
enter the maternal circulation during pregnancy, with
spontaneous or induced abortion, at delivery
• Ab formation against D Ag may be induced in the unsensitized
Rh- recipient mother → rise in IgM initially then rise in IgG
crossing the placenta
• Rarely occurs during the 1st pregnancy because transfusion of Rh+
fetal blood into a Rh- mother occurs near the time of delivery, too
late for the mother to become sensitized & transmit antibody to
her infant before delivery.
• Injection of anti-D gamma globulin (RhoGAM) into the mother CAUSES OF EVALUATION OF
immediately after the delivery of each Rh+ infant reduces Rh HYPERBILIRUBINEMIA HYPERBILIRUBINEMIA
hemolytic disease https://qrs.ly/kxduekh https://qrs.ly/jeduekq
Rh incompatibility lab test results: Please study this flowchart as it is a guide to evaluate jaundice /
• Before treatment: hyperbilirubinemia. Some key points.
1. Direct Coombs test is + 1. Direct hyperbilirubinemia = pathologic = cholestasis
2. Anemia 2. In neonates, it is mostly indirect hyperbilirubinemia
3. Increased reticulocyte count 3. First thing to check is Coombs if positive think RH vs ABO
4. B1 rises rapidly in the 1st 6 hours of life 4. Coombs (-) with increase Hgb = polycythemia
5. Coombs neg with normal Hgb and normal retic = physiologic
5. B2 may also be elevated Dr. De Vera

TOPNOTCH MEDICAL BOARD PREP PEDIATRICS MAIN HANDOUT BY DRS. DE VERA AND PUNONGBAYAN Page 12 of 106
For inquiries visit www.topnotchboardprep.com.ph or email us at topnotchmedicalboardprep@gmail.com
This handout is only valid for October 2022 PLE batch. This will be rendered obsolete for the next batch since we update our handouts regularly.
TOPNOTCH MEDICAL BOARD PREP PEDIATRICS MAIN HANDOUT BY DRS. DE VERA AND PUNONGBAYAN
For inquiries visit www.topnotchboardprep.com.ph or https://www.facebook.com/topnotchmedicalboardprep/
This handout is only valid for October 2022 PLE batch. This will be rendered obsolete for the next batch since we update our handouts regularly.

PEDIATRIC NUTRITION STAGES OF LACTATION


✔ GUIDE QUESTIONS FIRST 0-7 DAYS: COLOSTRUM
The most practical and pertinent guide in evaluating nutritional status • 37-84 mL/day; D1-2 of life
in children is: • Watery (>80% water)
A. Biochemical studies of food and determination of vitamin levels • protein-rich
B. Regular or periodic follow up of weight and height • High in Ig/protective factors: lactoferrin, lysozyme
C. X-ray studies of bone
D. Dietary history
• Na, vit A/K & growth factors
Colostrum is produced over the first 2 days of lactation. The secretion • Low levels: fat & carbohydrates
has higher protein and lower fat and lactose than mature human milk. TRANSITIONAL MILK
Likewise, it is especially rich in:
A. IgM • Between colostrum & mature milk
B. IgG • Rising levels of macronutrients
C. IgD
MATURE MILK
D. IgA
The whey-to-casein ratio in mature human milk is about: • D 10-14 of life
A. 1:1 • Colostrum content + high fat & lactose
B. 1:4
C. 3:2
INVOLUTIONARY MILK
D. 4:1 • produced when breastfeeding frequency decreases
Whey is better than Casein. Think bodybuilders. • reverts to being more like colostrum
Dr. De Vera
Until when can you feed breastmilk solely to a child? BREAST MILK COMPOSITION
A. 3 months • casein to whey ratio - low
B. 6 months o 10:90 in early milk
C. 12 months
o 40:60 in mature milk
D. 2 years
o 50:50 in late lactation
Exclusive up to 6 mos. Then complimentary feeding. Breastfeeding is • fat globules bound by membranes rich in:
best for baby up to 2 years old and BEYOND…
Dr. De Vera
o phospholipids – cell growth & brain development
o cholesterol – facilitates myelination of the CNS
FEEDING: 1ST 6 MONTHS OF LIFE • Breast milk has MORE compared to Formula milk
• Initiated as soon as after birth o Exceptions
o Within 1-4 hrs § Iron
§ Vit D
• Schedule: “Self-regulation” by infant
§ Vit K
o 1st wk 60-90mL/feeding: 6-9 feedings/24hrs
o Middle of the night feedings: from birth to 6-8wks, beyond ✔ GUIDE QUESTION
o By 9-12 mos: 3 meals/day + snacks Overall, the 3-month-old male infant is apparently well but he
o There is no need to feed infants every time they cry. regurgitates 10-20 mL of milk usually after feeding, what is the best
management for this condition?
A. Advise Thick Feeding
BREAST MILK B. Endoscopy
BREAST TIMING OF C. Observe and Reassurance
CONSISTENCY CONTENT D. Upper GI Endoscopy
MILK RELEASE
At the start of High lactose, BREAST MILK STORAGE GUIDELINES PPS
Foremilk Watery TEMPERATURE DURATION
feeding high protein
As feeding Room temperature <25°C 4 hours
High fat (5x Room temperature >25°C 1 hour
progresses
Hind the fat Refrigerator 4°C 8 days
until towards Creamy
milk content of Freezer (1-door) 2 weeks
the end of
foremilk) Freezer (2-door) 3 months
feeding
Deep freezer (-20°C) 6 months
TOPNOTCH MEDICAL BOARD PREP PEDIATRICS MAIN HANDOUT BY DRS. DE VERA AND PUNONGBAYAN Page 13 of 106
For inquiries visit www.topnotchboardprep.com.ph or email us at topnotchmedicalboardprep@gmail.com
This handout is only valid for October 2022 PLE batch. This will be rendered obsolete for the next batch since we update our handouts regularly.
TOPNOTCH MEDICAL BOARD PREP PEDIATRICS MAIN HANDOUT BY DRS. DE VERA AND PUNONGBAYAN
For inquiries visit www.topnotchboardprep.com.ph or https://www.facebook.com/topnotchmedicalboardprep/
This handout is only valid for October 2022 PLE batch. This will be rendered obsolete for the next batch since we update our handouts regularly.

GERD Important points to remember:


TB – after two weeks of treatment, mother no longer considered
• Epidemiology: infants’ condition peak at 4 mos & resolve infectious, may do direct breastfeeding. Prior to this, expressed mother’s
mostly at 12 mos of age & nearly all at 24 mos old milk can be given
• Genetic predisposition: AD (Chromosome 13q, 14 & 19) HIV – not absolutely contraindicated. In times where no safer alternative
• Clinical manifestations: is available, HIV mother can breastfeed provided that:
o infants: postprandial regurgitation, signs of esophagitis 1. Shortest duration possible
(irritability, arching, choking, gagging, feeding aversion), 2. Exclusive breastfeeding (mixed feeding increases risk of AGE which
increases transmission rate because of damaged intestinal villi)
failure to thrive Dr. De Vera
o Older children: regurgitation, abdominal & chest pain; SYMPTOMS ACCORDING TO AGE
respiratory manifestations related to asthma or sinusitis MANIFESTATIONS INFANTS CHILDREN
ADOLESCENTS
AND ADULTS
• Diagnosis: Barium study of esophagus & upper GIT (poor Impaired quality of life +++ +++ +++
sensitivity and specificity) Regurgitation ++++ + +
• Definitive test: esophageal pH probe Excessive crying / irritability +++ + -
• Esophageal pH monitoring of distal esophagus: document acidic Vomiting ++ ++ +
reflux episodes (normal value of distal esophageal acid exposure Food refusal / feeding
++ + +
<5-8% of total monitored time) disturbances / anorexia
Persisting hiccups ++ + +
MANAGEMENT OF GERD Failure to thrive ++ + -
• Dietary measures: thicken the formula; avoid acidic/spicy foods, Abnormal posturing / Sandifer
juices, alcohol, caffeinated & carbonated drinks ++ + -
syndrome
• Awake: prone or upright carried position Esophagitis + ++ +++
• Antacids, H2RAs, PPIs, prokinetic agents Persistent cough / aspiration
+ ++ +
• Fundoplication – for intractable GERD, refractory esophagitis & pneumonia
strictures, with morbidities from chronic pulmonary disease Wheezing / laryngitis / ear
+ ++ +
problems
(recurrence rate of 14%)
Laryngomalacia /stridor /
+ ++ -
croup
Sleeping disturbances + + +
Anemia/melena/hematemesis + + +
Apnea / ALTE / desaturation + - -
Bradycardia + ? ?
Heartburn/pyrosis ? ++ +++
Epigastric pain ? + ++
Chest pain ? + ++
Dysphagia ? + ++
Dental erosions / water brush ? + +
Hoarseness / globus
? + +
pharyngeus
Chronic asthma / sinusitis - ++ +
Laryngostenosis / vocal nodule
- + +
problems
Stenosis - (+) +
Barrett / esophageal
- (+) +
adenocarcinoma
+++, Very common; ++ common; + possible; (+) rare; - absent; ? unknown;
ALTE, apparent life-threatening event.
This table simply shows presentation of GERD on different age group. For
children please take note that GERD should be a differential diagnosis for
chronic cough and/or respiratory symptoms.
Dr. De Vera

MICRONUTRIENT DEFICIENCIES
https://www.bariatricinnovationsatl.com/acid-reflux-surgery
Vesiculobullous, Chronic diarrhea,
eczematous, dry scaly or Stomatitis,
psoriasiform lesions Glossitis, Zinc
symmetric perioral, sacral, Irritability,
and perianal areas superinfection
Alopecia ,
Generalized scaly Essential
Thrombocytopenia,
dermatitis Fatty Acids
Failure to thrive
Edema, erythema and
burning of sun exposed skin Niacin
Diarrhea
on the face neck hands on (Pellagra) /
dementia
butterfly distribution Vit B3
https://med.stanford.edu/ctsurgery/clinical-care/thoracic-surgery-services/benign-esophageal-disease.html
dermatitis around the neck
✔ GUIDE QUESTION
Alopecia with Biotin / Vit
Which of the following is an absolute contraindication to --
neurological symptoms B7
breastfeeding?
A. An infant with galactosemia Xerosis, Night
Vit A
B. A mother with postpartum blues blindness
C. A mother with active pulmonary tuberculosis Perifollicular
D. An infant with breastfeeding jaundice Follicular hyperkeratosis erythema and
hemorrhage Vit C
Bleeding swollen
CONTRAINDICATIONS TO BREASTFEEDING gums
• Galactosemia
Tips on studying micronutrient deficiency. When answering the exam,
• Septicemia think first of micronutrients with disease entities such as Rickets, Scurvy,
• Active TB Zinc (acrodermatitis-enterohepatica), etc. Because as you will see these
• Breast Cancer micronutrient deficiencies have overlapping presentation such as
• Malaria eczema, diarrhea, dermatitis and others.
Dr. De Vera
• Substance Abuse
• Severe neurosis or Psychosis
• HIV

TOPNOTCH MEDICAL BOARD PREP PEDIATRICS MAIN HANDOUT BY DRS. DE VERA AND PUNONGBAYAN Page 14 of 106
For inquiries visit www.topnotchboardprep.com.ph or email us at topnotchmedicalboardprep@gmail.com
This handout is only valid for October 2022 PLE batch. This will be rendered obsolete for the next batch since we update our handouts regularly.
TOPNOTCH MEDICAL BOARD PREP PEDIATRICS MAIN HANDOUT BY DRS. DE VERA AND PUNONGBAYAN
For inquiries visit www.topnotchboardprep.com.ph or https://www.facebook.com/topnotchmedicalboardprep/
This handout is only valid for October 2022 PLE batch. This will be rendered obsolete for the next batch since we update our handouts regularly.

FAT SOLUBLE VITAMINS IN BREAST MILK VKDB OF THE NEWBORN


VITAMIN A • Early VKDB
o called classic hemorrhagic disease
• Bioavailability of Vit A in breast milk is more than 90%
o occurs at 1-14 days of age
• Preferentially transferred to breast milk from maternal stores o low stores of vitamin K at birth
and dietary intake § poor transfer of vitamin K across the placenta
• Premature babies – born with smaller stores of Vit A → more § Inadequate intake during the 1st few days of life
vulnerable to deficiency § In addition, there is no intestinal synthesis of vitamin K2
• Component of retinal pigments; for bone & tooth development; because the newborn gut is sterile
formation & maturation of epithelia • Late VKDB
Vitamin A deficiency o 2-12 weeks of life
• Nyctalopia (reduced ability to see in decreased illumination) o Some up to 6 months of age
• Photophobia o Mostly breastfed infants (low vit K content)
• Xerophthalmia (abnormal dryness of the conjunctiva & cornea) o Malabsorption syndromes
• Keratomalacia (softening & drying of the cornea) • Third form
• Faulty epiphyseal bone formation o At birth or shortly after birth
• Defective tooth enamel o Maternal medications (warfarin, phenobarbital, phenytoin)
• Impaired resistance to infection
• Bitot spot – oval or triangular spot of keratinized epithelium in WATER SOLUBLE VITAMINS IN BREAST MILK
the conjunctiva THIAMINE / B1
VITAMIN D • Coenzyme in ketoacid decarboxylation
• content in human milk depends on maternal vitamin D status • Beri-beri, polyneuropathy, calf tenderness, heart failure
(maternal sunlight exposure)
• The amount of sunlight needed to produce sufficient vitamin D RIBOFLAVIN / B2
depends on skin color, latitude, and season. • FAD coenzyme in oxidation-reduction reactions
• Facial exposure to sunlight of 2 hours at least once a week or 30 • mucositis, cheilosis, anemia, nasolabial seborrhea
minutes with bare arms and legs is sufficient. o Angular cheilosis (perleche) - begins with pallor at the angles
of the mouth; fissures covered by yellow crusts
Rickets
o Glossitis (magenta tongue) - painful tongue inflammation
• Disease of growing bone which occurs in children only before
fusion of the epiphyses Manifestations of B2 deficiency is non-specific. It is associated with
perleche and glossitis but please take note these are also present in
• Due to unmineralized matrix at the growth plates
Scurvy, Zinc deficiency, and other diseases
• Increase in the circumference of the growth plate and the Dr. De Vera
metaphysis → widening of the wrists & ankles NIACIN / B3
• General softening of the bones • NAD coenzyme in
• Craniotabes or softening of the cranial bones oxidation-reduction
• Widening of the costochondral junctions leads to “rachitic reactions
rosary” • Manifestations: pellagra:
• “Harrison groove” is a horizontal depression along the lower dermatitis, diarrhea,
anterior chest due to pulling of the softened ribs by the dementia
diaphragm during inspiration • Sources: meat, fish, liver,
whole grains, green leafy Casal Necklace
vegetables http://mkginbc.blogspot.com/2007/06/

VITAMIN B6
• In well-nourished populations, breast milk provides sufficient
B6 to meet requirements for up to 6 months
• Mothers who have been long term users of high estrogen-
containing contraceptives may have lower levels of B6 and need
• Windswept deformity (valgus deformity of 1 supplements of 20 mg/day
leg with varus deformity of the other leg) • Cofactor in amino acid metabolism
• Treatment of Rickets: • Microcytic anemia, cheilosis, glossitis, dermatitis
• Vitamin D 300,000-600,000 IU orally or IM as Vitamin B6 deficiency is also non-specific and not common. But take note
2-4 doses over 1 day (Stoss therapy) of the mother who takes in estrogen pills and the infant presenting with
• Either strategy should be followed by daily Vit microcytic anemia + other findings stated above.
Dr. De Vera
D intake of 400 IU/day as a multivitamin
BIOTIN (VITAMIN B7)
• Adequate dietary calcium & phosphorus
• Cofactor in carboxylase reactions of amino acids
VITAMIN E • Alopecia, dermatitis, hypotonia
• Antioxidant; protects PUFA in cell membranes from oxidative • Sources: meats, yeast
damage
• constant in mature milk at 0.32 mg tocopherol VITAMIN B12 AND FOLATE
equivalents/100mL • bound to proteins in the whey fraction of breast milk (enhanced
• If deficient: bioavailability)
o RBC hemolysis in premature infants • breastfed infants of strict vegetarians or malnourished mothers
o loss of neural integrity can exhibit clinical Vitamin B12 deficiency leading to severe and
o posterior column and cerebellar dysfunction persisting neurological damage
o pigmentary retinopathy • Coenzyme for 5-methyl-tetrahydrofolate formation
VITAMIN K • Megaloblastic anemia, peripheral neuropathy
• Prothrombin formation, coagulation factors II, VII, IX, X
• The vitamin K content of breast milk is between 0.1 – 0.4 VITAMIN C
µg/100mL and is higher in colostrum than mature milk. • Infants of well-nourished mothers ingest around 20 mg/day of
• deficiency leads to hemorrhagic disease of the newborn or Vitamin C from breast milk compared with 8 mg required to
vitamin K deficiency bleeding (VKDB) prevent scurvy
• VKDB with breastfeeding is due to low overall intakes of vitamin • Integrity & maintenance of intercellular material; facilitates
K in the first week of life, rather than low vitamin K content of absorption of iron & conversion of folic acid to folinic acid;
breast milk. metabolism of tyrosine & phenylalanine
TOPNOTCH MEDICAL BOARD PREP PEDIATRICS MAIN HANDOUT BY DRS. DE VERA AND PUNONGBAYAN Page 15 of 106
For inquiries visit www.topnotchboardprep.com.ph or email us at topnotchmedicalboardprep@gmail.com
This handout is only valid for October 2022 PLE batch. This will be rendered obsolete for the next batch since we update our handouts regularly.
TOPNOTCH MEDICAL BOARD PREP PEDIATRICS MAIN HANDOUT BY DRS. DE VERA AND PUNONGBAYAN
For inquiries visit www.topnotchboardprep.com.ph or https://www.facebook.com/topnotchmedicalboardprep/
This handout is only valid for October 2022 PLE batch. This will be rendered obsolete for the next batch since we update our handouts regularly.
SCURVY B vitamins Clues
• Vitamin C deficiency early B1 Thiamine Beri-beri
symptoms: low-grade fever,
B2 Riboflavin Nonspecific ( cheilosis, glossitis)
irritability, tachypnea,
B3 Niacin Pellagra
anorexia, generalized
Panthothenic
tenderness esp. in the legs B5 Non specific
Acid
• Pseudoparalysis with hips &
B6 Pyridoxine Microcytic anemia + estrogen pills
knees semi-flexed & the feet
B7 Biotin Alopecia + neurologic changes
rotated outward
• “scorbutic rosary” at the B9 Folic Acid Megaloblastic anemia
costochondral junction & Megaloblastic anemia +
B12 Cobalamin
depression of the sternum neurologic deficit
• Angulation of the scorbutic Bowlegs or knock knees
beads is sharper than the Caput quadratum, Periosteal osteoid
rachitic rosary Craniotabes
Vit D
• Bluish, purple spongy swellings of the mucous membranes esp. Ping pong ball sensation
over the upper incisors Rachitic rosary, Pigeon chest
• Other symptoms: swollen joints, purpura and ecchymoses, poor Harrison groove
wound & fracture healing “White line” on end of shafts
Vit C
• perifollicular hemorrhages, hyperkeratosis of hair follicles, Scorbutic rosary/beads
“corkscrew hair” Pellagra
• X ray changes: distal ends of long bones with a ground-glass Casal necklace
Niacin (Vit B3)
appearance Pellagrous glove and boots
Glove and boots lesion
Treatment of scurvy:
Perleche Vit B2
• Daily intake of 3-4 oz of orange or tomato juice
Bitot spots
• Vitamin C supplements of 100-200 mg orally or parenterally are
Xerosis conjunctivae Vit A Deficiency
preferable to ensure more rapid and complete cure.
Xerosis cornea
• Larger doses (>2 grams) may produce abdominal pain and
Hyperostosis
osmotic diarrhea Vit A Intoxication
Absence of metaphyseal changes
✔ GUIDE QUESTIONS
MINERALS IN BREAST MILK
A deficiency of this trace element is associated with skin ulcers, reduced
IRON immune response and hypogonadal dwarfism:
• The iron content of breast milk is unaffected by maternal iron A. Zinc
status, maternal iron deficiency, or supplementation B. Chromium
• Infants’ iron requirements are largely met from body stores built C. Cobalt
up in utero D. Copper
Irritability, pruritus, painful extremities, with brawny swelling, coarse
• Combined with breastfeeding, these stores are usually sufficient hair, dry skin, seborrhea and increased intracranial pressure is seen in:
to meet infants iron needs for 6 – 12 months. A. Hypervitaminosis A
• Lesser in breast milk but more bioavailable B. Hypervitaminosis D
C. Hypovitaminosis A
ZINC D. Hypovitaminosis D
• Infant’s requirement in the first 6 months are largely met by fetal In a Southwestern town in Mindanao, children are fed mostly with corn
as staple. The most common vitamin deficiency encountered in these
stores accumulated in the last trimester of pregnancy →
children is:
thereafter should be met by appropriate complementary foods A. Niacin
B. Folate
IODINE C. Thiamine
• Iodine accumulates in the mammary gland and levels in breast D. Riboflavin
milk reflect maternal diet. In children with malnutrition, the most seriously compromised
immunologic function is:
• In areas where iodine deficiency is common, maternal
A. Antibody production
supplementation is necessary. B. Phagocytosis
Please remember this. Most micronutrients in breastmilk are not affected C. Cell-mediated immunity
by maternal diet/status. Vitamin D and Iodine are affected by maternal D. Complement activation
status. One of the macronutrients severely affected in malnutrition are
For Iron and Zinc, please remember that full-term neonates are born with proteins (think Kwashiorkor). Therefore, problem in proteins →
sufficient stores that are enough for the first 6 months of life. problem in antibodies.
Dr. De Vera
Dr. De Vera
Micronutrient Clinical Clues Remarks A 5-year-old male child is brought to the clinic for being weak and wants
90% bioavailable in her child to be dewormed. It has been going on for the past 6 months
Retinal piments, bone, according to the mother. She says that the child usually is prone to
breast milk
Vitamin A tooth and epithelium having diarrhea and usually he has episodes every month. On
Premature babies
development examination, the child looks apathetic, he is underweight, there is some
more vulnerable
edema of the bilateral lower extremities, the abdomen is protuberant,
Bone formation Content in
the hair is sparse thin with reddish streaks, what is the most likely
Vitamin D Rickets and breastmilk depends
diagnosis in this patient?
Osteomalacia on maternal status A. Marasmus
RBC hemolysis in B. Non-edematous protein energy malnutrition
Vitamin E Anti-oxidant
premature infants C. Kwashiorkor
Prevented by D. Pellagra
Vitamin K VKDB
prophylaxis
Affects bones and
joints
UNDERNUTRITION
Vitamin C Perifollicular Scurvy • Greatest risk occurs in the 1st 1000 days (0-24 months)
hemorrhages • Term malnutrition covers undernutrition to over-weight
Gum swelling • International standards of determining anthropometry using
Stores are adequate for 6 months WHO charts
Iron & Zinc Content in breastmilk NOT affected by o Height for age (length for age if <2)
maternal status § Measures linear growth
Content in breastmilk depends on maternal § Represents cumulative impact of adverse events
Iodine
status
TOPNOTCH MEDICAL BOARD PREP PEDIATRICS MAIN HANDOUT BY DRS. DE VERA AND PUNONGBAYAN Page 16 of 106
For inquiries visit www.topnotchboardprep.com.ph or email us at topnotchmedicalboardprep@gmail.com
This handout is only valid for October 2022 PLE batch. This will be rendered obsolete for the next batch since we update our handouts regularly.
TOPNOTCH MEDICAL BOARD PREP PEDIATRICS MAIN HANDOUT BY DRS. DE VERA AND PUNONGBAYAN
For inquiries visit www.topnotchboardprep.com.ph or https://www.facebook.com/topnotchmedicalboardprep/
This handout is only valid for October 2022 PLE batch. This will be rendered obsolete for the next batch since we update our handouts regularly.
o Weight for height
§ Wasting
§ Acute malnutrition
o Weight for age
§ Low value
§ Limited clinical significance
o Mid upper arm circumference
§ 11.5-12.5 cm = wasting
§ <11.5 cm = severe wasting
For malnutrition, the term we are looking for is WASTING. Only three http://www.differencebetween.info/difference-between-kwashiorkor-and-marasmus
parameters can be used depending on age of patient: Weight for
length/height, MUAC, BMI SEVERE ACUTE MALNUTRITION
Dr. De Vera
• defined as severe wasting and/or bilateral Edema
✔ GUIDE QUESTION • Severe wasting is extreme thinness diagnosed by a weight-for-
The following is more commonly seen in Kwashiorkor than in length (or height) below −3 SD of the WHO Child Growth
Marasmus:
Standards
A. Good appetite
B. Wasting • children ages 6-59 mos, a mid-upper arm circumference
C. Fatty liver infiltration <115mm also denotes extreme thinness
D. Absent subcutaneous fat • Differentiates wasted from stunted
o Stunted not priority in acute clinical care
KWASHIORKOR VS MARASMUS
• Pathophysiology
FEATURE KWASHIORKOR MARASMUS
o When intake cannot meet daily needs → physiologic and
Growth Failure Present Present
metabolic changes in an orderly progression to compensate =
Wasting Present Present, marked
Reductive Adaptation
Present
Edema Absent 1. Fat stores mobilized for energy
(sometimes mild)
2. Protein is mobilized
Hair changes Common Less common
3. Energy conserved by reducing activity and growth, basal
Mental changes Very common Uncommon
metabolism, functional reserve of organs, inflammatory
Dermatosis, flaky- and immune response
Common Does not occur
paint o Liver makes less glucose = hypoglycemia
Appetite Poor Good o Liver makes less albumin, transferrin and other proteins
Reduced but o Heat production is less = hypothermia
Subcutaneous fat Absent
present o Kidneys excrete less = fluid overload
Drawn in, o Heart is smaller and weaker = cardiac failure
Face May be edematous
monkey-like o Sodium builds up due to leaky membranes and inactive pumps
Fatty infiltration = edema
Present Absent
of liver o Potassium leaks out of cells
Having problem differentiating the two? Super easy! Kwashiorkor is o Loss of muscle protein
more complicated (spelling pa lang mahirap na! hehe). If the finding in Remember the words REDUCTIVE ADAPTATION. The body tries to adapt
questions is negative in nature, probably that is Kwashiorkor. Don’t to the lack of nutrients at the expense of forgoing other “less important”
believe me? Look at the table above. Marasmus is more benign among processes. In the long run this becomes counter-productive. This explain
the two. J all the features of severe malnutrition.
Dr. De Vera
Dr. De Vera

IN PATIENT MANAGEMENT OF MALNUTRITION (WHO)


Nelson Textbook of Pediatrics, 20th ed.
✔ GUIDE QUESTION REFEEDING SYNDROME
A 4-year-old malnourished girl was admitted for diarrhea with severe
dehydration. Patient was resuscitated successfully and is completing
• Malnutrition usually has normal serum electrolytes but is
antibiotic treatment. She was feeding well and is able to finish the associated with intracellular electrolyte depletion.
regular serving of food given. On the 4th hospital day, patient was • Excessive carbohydrates → surge of insulin → hypokalemia,
suddenly developed seizure and cardiac arrest. What is the possible hypophosphatemia, hypomagnesemia
cause of the patient’s demise? • Hallmark is severe hypophosphatemia
A. Inadequate antibiotic coverage for CNSI • Usually during the 1st week of refeeding
B. Excessive feeding syndrome
• S/Sx: Serum phosphate levels of ≤0.5 mmol/L can produce
C. Dehydration
D. Refeeding Syndrome weakness, rhabdomyolysis, neutrophil dysfunction,
cardiorespiratory failure, arrhythmias, seizures, altered level of
consciousness, or sudden death
TOPNOTCH MEDICAL BOARD PREP PEDIATRICS MAIN HANDOUT BY DRS. DE VERA AND PUNONGBAYAN Page 17 of 106
For inquiries visit www.topnotchboardprep.com.ph or email us at topnotchmedicalboardprep@gmail.com
This handout is only valid for October 2022 PLE batch. This will be rendered obsolete for the next batch since we update our handouts regularly.
TOPNOTCH MEDICAL BOARD PREP PEDIATRICS MAIN HANDOUT BY DRS. DE VERA AND PUNONGBAYAN
For inquiries visit www.topnotchboardprep.com.ph or https://www.facebook.com/topnotchmedicalboardprep/
This handout is only valid for October 2022 PLE batch. This will be rendered obsolete for the next batch since we update our handouts regularly.

OBESITY TREATMENT OF OBESITY


• First predictors • Aim: not weight loss but to slow or halt weight gain – weight
o High birth weight maintenance
§ Maternal obesity • For every 20% excess of ideal BW, the child needs 1 ½ years of
§ Maternal diabetes weight maintenance to attain ideal BW
o Overweight children – overweight adults • Weight loss – for skeletally mature with severe complications
o Parental obesity o 0.5kg or less/wk to 10% weight reduction
§ Doubles risk for adult obesity o Interventions: lifestyle changes
§ Parental modeling of eating and exercise behaviors o Physical activity, diet management, behavior modification
PATHOGENESIS TRAFFIC LIGHT / STOPLIGHT DIET:
GREEN LIGHT YELLOW LIGHT RED LIGHT
• Dysregulation of caloric intake and energy expenditure
• Low Calorie
• Environmental changes • Nutrient Dense • High Calorie
• High Fiber
o Fast food diets • High Calorie • High Fat
• Low Fat
o Snacking in between meals • High Fat • High Sugar
• Nutrient Dense
o Increased sedentary activities
• Lean Meat
o Lack of exercise • Fatty Meat
• Fruits • Dairy
• Genetics • Sugar
• Veggies • Starches
o Genes determine resting energy expenditure and the weight • Fried Food
• Grains
set point – level of stored fuel that satisfies an individual
• Unlimited • Limited • Infrequent
o Heredity: influences fatness, regional fat distribution,
• Quantities • Quantities • Avoided
response to overfeeding
• BMI – Body Mass Index ROLE OF MEDICATIONS IN CHILDHOOD OBESITY
𝑊𝑒𝑖𝑔ℎ𝑡("#) • Meds for child with severe complications
𝐵𝑀𝐼 = &
𝐻𝑒𝑖𝑔ℎ𝑡(%) o The use of pharmacologic treatment for overweight and obese
BMI PERCENTILE FOR AGE WEIGHT STATUS children is of marginal value with potential and unclear risks.
< 5th percentile Underweight § i.e. Orlistat: intestinal lipase inhibitor (decreases fat
5th – 84th Normal weight absorption) for >12 yrs old
85th –94th At risk for overweight § WOF diarrhea, flatulence, bloating, dyspepsia
≥ 95th Overweight PLEASE READ MORE ON:
OBESITY • Components of breast milk (bioactive, vitamins, etc.)
• Excessive accumulation of • Skinfold of triceps + calf • Macronutrients vs Micronutrients
body fat o boys: N 10-25mm • Essential Fatty Acids
o boys: TBW >25% fat o Girls: N 16-30mm SUPPLEMENT: QUICK SHEET
o girls: TBW>32% fat Important notes on obesity
• Appears most frequently in the 1st year of life, at 5-6 yrs of age & 1. As bad as undernutrition
during adolescence 2. Lots of complications
• Disproportionately fine facial features, adiposity in the mammary 3. Management is mostly dietary and activity modification
regions, pendulous abdomen, disproportionately small external 4. Pharmacologic regimen for severe cases
genitalia for boys 5. Goal most of the time is weight maintenance (as the child grows taller
while maintaining weight, BMI decreases. Remember the formula for BMI)
OBESITY IN CHILDHOOD
• ↑ risk for obesity in adulthood
PREVENTIVE PEDIATRICS
• Leading cause of pediatric hypertension
• Associated with Type 2 DM IMMUNIZATION
• ↑ risk of coronary heart disease • Ultimate goal: eradication of disease
• ↑ stress on weight-bearing joints • Immediate goal: prevention of disease
• Low self-esteem • Two types:
EVALUATION: 1. Active immunization
2. Passive immunization
• Screening includes:
o (+) family history (CV disease, parental elevated total
cholesterol level, DM, parental obesity) ACTIVE IMMUNIZATION
o BP • Efficacy is assessed by the evidence of protection against the
o Total cholesterol level (>200 mg/dL) particular disease.
o Large annual incremental increase in BMI (increase over the o Antibody formation – indirect measure
previous year of 2 BMI units) • Types of antigen for active immunization:
o Concern about weight (assessment of personal concerns, 1. Live attenuated virus or bacteria (weakened)
emotional or psychological, related to overweight or the 2. Inactivated vaccine (killed microorganisms)
perception of overweight) INACTIVATED VACCINES LIVE VACCINES
COMORBIDITIES • Hepatitis B vaccine • BCG vaccine
• Harvard Growth Study: • DPT vaccine • Measles vaccine
o Doubling of cardiovascular disease esp. in male adolescents • H. influenzae b vaccine • MMR vaccine
• Bogalusa Health Study: • Pneumococcal vaccine • Varicella vaccine
o BMI >85th percentile • Hepatitis A vaccine • Rotavirus vaccine
§ Hypercholesterolemia, hypertriglyceridemia, hypertension • Meningococcal vaccine • Influenza attenuated
• Metabolic syndrome • Influenza trivalent vaccine vaccine (intranasal)
o Hypertension, glucose intolerance, hypertriglyceridemia, low • Human papillomavirus • Typhoid fever (oral)
HDL, abdominal central obesity vaccine vaccine
• Type 2 DM • Typhoid fever vaccine (IM) • Oral polio vaccine
• Gallstones • Rabies vaccine • Japanese Encephalitis
• Asthma • Inactivated polio vaccine
• Obstructive sleep apnea - snoring (IPV)
• Polycystic ovary syndrome Memorize all the live vaccines. They are the only ones which will
o hirsutism, male pattern baldness, severe acne complicate scheduling. Very easy to memorize, if it is NOT intramuscular
• Fatty liver disease – non-alcoholic (IM), probably it is live!
• Psychosocial disorders Dr. De Vera

o Anxiety, depression
TOPNOTCH MEDICAL BOARD PREP PEDIATRICS MAIN HANDOUT BY DRS. DE VERA AND PUNONGBAYAN Page 18 of 106
For inquiries visit www.topnotchboardprep.com.ph or email us at topnotchmedicalboardprep@gmail.com
This handout is only valid for October 2022 PLE batch. This will be rendered obsolete for the next batch since we update our handouts regularly.
TOPNOTCH MEDICAL BOARD PREP PEDIATRICS MAIN HANDOUT BY DRS. DE VERA AND PUNONGBAYAN
For inquiries visit www.topnotchboardprep.com.ph or https://www.facebook.com/topnotchmedicalboardprep/
This handout is only valid for October 2022 PLE batch. This will be rendered obsolete for the next batch since we update our handouts regularly.

PRINCIPLES OF IMMUNIZATION: CONTRAINDICATIONS TO VACCINATION


1. Simultaneous administration of multiple vaccines • 2 permanent contraindications:
(for missed doses): 1. Anaphylactic reaction
• most vaccines can be safely and effectively given simultaneously 2. Encephalopathy not due to another identifiable cause
• use separate syringes and sites occurring within 7 days after pertussis vaccination
• 2 temporary contraindications to live vaccines but not with
Vaccine spacing and intervals:
inactivated vaccines:
ANTIGEN RECOMMENDED MINIMUM 1. Pregnancy
COMBINATION INTERVAL BETWEEN DOSES 2. immunosuppression
2 or more inactivated None; can be given simultaneously or
vaccines at any interval between doses
Inactivated & live None; can be given simultaneously or
vaccines at any interval between doses
This is very important. The DEFAULT ANSWER IS YES if asked whether
two vaccines can be given simultaneously on the same visit. There’s is
only one exception: yellow fever and cholera. CHILDHOOD EPI ADDITIONAL TIPS ON
Question: can chickenpox and pneumococcal vaccine be given https://qrs.ly/lhduekv IMMUNIZATION
simultaneously? Bakit nag-isip ka pa? YES NGA! https://qrs.ly/yfduf9p
Dr. De Vera
✔ GUIDE QUESTION
SUPPLEMENT: QUICK SHEET Which of the following is an example of naturally acquired passive
EXCEPTIONS!!! immunity?
1. Yellow fever vaccine (live) and Cholera vaccine A. Immunization with HPV vaccine
(inactivated) – separate by at least 3 weeks because of B. Administration of HRIg
diminished Ab response if given simultaneously C. Immunity from a natural infection
2. Live oral vaccines can be given simultaneously and D. Transplacental transfer of maternal antibodies
regardless of interval from other live vaccines PASSIVE IMMUNIZATION
• For IM administration: peaks at 48-72 hrs not to exceed 5 ml
Vaccine spacing and intervals: • Indications:
ANTIGEN RECOMMENDED MINIMUM o Congenital or acquired B-lymphocyte cell defects alone or in
COMBINATION INTERVAL BETWEEN DOSES combination with other immunodeficiencies
2 parenteral live May be given simultaneously at the o When time does not permit adequate protection by active
vaccines same visit immunization alone (hepatitis B, measles, rabies, varicella)
As said earlier, only live-live poses a problem in scheduling. o When a person susceptible to a disease is exposed to or has a
Can two live vaccines be given together? YES! (sabi ko wag na mag-isip high likelihood of exposure to that infection or has a high risk
diba). of complications from the disease (leukemic child exposed to a
Can live-inactivated be given together? YES! person with varicella or measles)
Can I give a live vaccine today followed by an inactivated tomorrow? YES o When a disease is already present and antibody may aid in
Can I give an inactivated today then inactivated tomorrow? YES
How about inactivated today and live tomorrow? YES
suppressing its toxin effects (tetanus) or the inflammatory
Live today and another live tomorrow? NO response (Kawasaki disease)
If live + live vaccine, either give it SIMULTANEOUSLY, otherwise you ✔ GUIDE QUESTION
have to wait 4 weeks. What is the earliest time you can give the measles vaccine?
Dr. De Vera
A. at birth C. 6 months
2. Lapsed Immunizations B. 3 months D. 9 months
• a lapse does not require reinstitution of the entire series
• subsequent immunizations should be given at the next visit as if
PPS RECOMMENDED VACCINES
the usual interval had elapsed
1. BCG 9. Measles
2. Hep B 10. Japanese Encephalitis
3. Unknown or uncertain immunization status
3. DPT 11. MMR
• Treat as if patient is unimmunized
4. HiB 12. Varicella
• In general, when in doubt, immunizations should be initiated 5. Polio 13. MMRV
without delay on a schedule commensurate with the person’s 6. Pneumococcal 14. Hepatitis A
current age. 7. Rotavirus 15. Td/Tdap
• No evidence that giving vaccines to already immune recipients is 8. Influenza 16. HPV
harmful.

PPS RECOMMENDED VACCINES


VACCINE MINIMUM AGE PRIMARY SERIES BOOSTER NOTES
At birth,
preferably • Intradermal. 0.05 ml < 12 months.
BCG --- ---
within first 2 0.1 ml >12 months
months
At birth w/in 12 depends on
Hep B 6-10-14 weeks ---
hours of life titers
• Interval between 3rd and 4th dose is
Usually at 1
6 months (4 months is acceptable).
DPT 6 weeks 6-10-14 weeks year old and 4-
5th dose is not need if 4th dose given
year-old
at 4 years old or older
• Booster given 6 mo from 3rd dose
Hib 6 weeks 6-10-14 weeks 12-15 months
• Not routinely given >5 years old
• Oral or IM
On or after 4th
Polio 6 weeks 6-10-14 weeks • Booster 6 months from the
birthday
previous dose
• Healthy children 2-5 years old
without previous vaccine may be
6 months after
Pneumococcal 6 weeks 6-10-14 weeks given 1 dose PCV 13 or 2 doses PCV
3 dose
rd
10 8 weeks apart
• Not recommended for >5 years old
TOPNOTCH MEDICAL BOARD PREP PEDIATRICS MAIN HANDOUT BY DRS. DE VERA AND PUNONGBAYAN Page 19 of 106
For inquiries visit www.topnotchboardprep.com.ph or email us at topnotchmedicalboardprep@gmail.com
This handout is only valid for October 2022 PLE batch. This will be rendered obsolete for the next batch since we update our handouts regularly.
TOPNOTCH MEDICAL BOARD PREP PEDIATRICS MAIN HANDOUT BY DRS. DE VERA AND PUNONGBAYAN
For inquiries visit www.topnotchboardprep.com.ph or https://www.facebook.com/topnotchmedicalboardprep/
This handout is only valid for October 2022 PLE batch. This will be rendered obsolete for the next batch since we update our handouts regularly.
VACCINE MINIMUM AGE PRIMARY SERIES BOOSTER NOTES
• Oral
• Monovalent RV1 – 2 doses
• *Pentavalent RV5 – 3 doses
Rotavirus 6 weeks 6-10-*14 weeks
• Do not start in infants >15 weeks
• Not given later than 32 weeks due to increased risk of
intussusception
Initial of 2 doses 4
• 0.25 ml for 6 months-35 months
Influenza 6 months weeks apart for 6 Yearly
• 0.5ml for 36 months-18-year-old
months to 8-year-old
• Subcutaneous
Measles 9 months 9 months ---
• May be given at 6 months if with outbreak
• Subcutaneous
• 9-17 years old give 1 booster 12-24 months from
9 months of age and
Japanese Encephalitis 9 months primary series
above
• 18 years old and above total 1 dose
• <18 years old total of 2 doses
• 2 doses at least 4 • Subcutaneous
weeks apart • Children <12 months given any
MMR 12 months ---
• 2nd dose usually at 4- measles containing vaccine should
6 years old receive additional 2 doses of MMR
• <13 years old – interval between 2
• 2 doses
doses at least 3 months
Varicella 12 months • 2nd dose at 4-6 ---
• (*4 weeks is acceptable)
years old
• 13 and above- 4 weeks interval
2 doses 6 months
Hepatitis A 12 months --- ---
apart
• Fully immunized children, defined as 5 doses of DTP or 4 doses if 4th dose given on or after 4th
Tetanus and diphtheria birthday, Td booster given every 10 years.
toxoid (Td) / Tetanus and • Single Tdap can be given to replace due dose of Td
diphtheria toxoid and • Fully immunized pregnant adolescent give 1 dose Tdap any time after 20 weeks AOG
acellular pertussis (TDaP) • Unimmunized pregnant adolescent give 3 dose td containing vaccine (Td/Tdap) following 0-1-6-
month schedule. Tdap should replace Td preferably after 20 weeks AOG
• Bivalent 0-1-6 • Interval between:
months • 1st and 2nd dose is 1 month
Human Papilloma Virus 9 years old ---
• Quadrivalent 0-2-6 • 2nd and 3rd dose is 3 months
months • 3rd and 1st dose is 6 months
Please do memorize this table. This is arranged according to sequence from birth. Most common values in case you forget to memorize. Schedule usually is 6-
10-14 weeks, boosters usually 1 year old and 4 years old. You need total of 2 doses of chickenpox and MMR.
Dr. De Vera

MEMORY AID! ✔ GUIDE QUESTIONS


VACCINES GIVEN AT BIRTH The above regimen should be given within how many hours after birth?
A. 12 hrs C. 48 hrs
• BCG (within 1st 2 months)
B. 24 hrs D. 72 hrs
• Hep B (within 12 hrs)
MATERNAL HEPATITIS B INFECTION
VACCINES GIVEN AT 6-10-14 WEEKS
• Hep B • HBsAg (+)
o HBV plus HBIg (0.5ml) within 12 hours of life
• DPT
• Unknown status
• Hib
o HBV within 12 hours of life
• Polio
o Determine status
• Pneumococcal
o If HBsAg (+)
• Rotavirus
§ Give HBIg no later than 7 days of life
VACCINES GIVEN WITH MIN AGE OF 9 MONTHS
• Measles VACCINES FOR HIGH RISK/SPECIAL GROUPS
• Japanese encephalitis • Typhoid
Michael Jordan 9 o 2 years old
VACCINES GIVEN AT MIN AGE OF 12 MONTHS o Every 2-3 years
(1+2 = 3; A’s) • Rabies
• MMR (measles, mumps, rubella) o Pre-exposure – IM or ID days 0, 7, 21 or 28 (old guidelines)
• Varicella o WHO 2018
• Hep A § 2 site ID at Day 0 and 7
2nd dose usually at 4-6 years old § 1 site IM at day 0 and 7
2nd dose after 6 mos • Pneumococcal for High Risk (PCV + PPSV)
4TH DOSE OF HEP B VACCINE o IM
o High Risk: Chronic heart, lung, kidney disease, DM, CSF leak, cochlear
• If 3rd dose given at age less than 24 weeks (6 months)
implant, sickle cell disease and other hemoglobinopathies, anatomic
• Pre-term infants less than 2 kgs, whose 1st dose was given at and functional asplenia, HIV and immunodeficiency
birth
o 2-5 years old
o Pre-term, HBsAg (-) mother, clinically stable § One dose PCV 13 if incomplete PCV was given
o 1st dose given at 30 days of life (this can be continued as part
§ 2 doses of PCV 13 (8 weeks apart) if unimmunized
of the 3-dose vaccination regimen)
§ One dose PCV 13 if completely immunized with PCV 7
✔ GUIDE QUESTIONS § PPSV at least 8 weeks after the most recent PCV 13
A male neonate was born from a HbsAg (+) mother, which among the o 6-18 years old
ff. should be given to the neonate? § 1 dose of PCV 13 then PPSV after 8 weeks
A. Hep B Vaccine D. A +B o Single revaccination of PPSV after 5 years
B. Hep B Immunoglobulin E. A+B+C
C. Ribavirin

TOPNOTCH MEDICAL BOARD PREP PEDIATRICS MAIN HANDOUT BY DRS. DE VERA AND PUNONGBAYAN Page 20 of 106
For inquiries visit www.topnotchboardprep.com.ph or email us at topnotchmedicalboardprep@gmail.com
This handout is only valid for October 2022 PLE batch. This will be rendered obsolete for the next batch since we update our handouts regularly.
TOPNOTCH MEDICAL BOARD PREP PEDIATRICS MAIN HANDOUT BY DRS. DE VERA AND PUNONGBAYAN
For inquiries visit www.topnotchboardprep.com.ph or https://www.facebook.com/topnotchmedicalboardprep/
This handout is only valid for October 2022 PLE batch. This will be rendered obsolete for the next batch since we update our handouts regularly.
✔ GUIDE QUESTIONS • Category III – single or multiple transdermal bites or scratches,
Persistent inconsolable crying, seizures and, rarely, acute neurologic contamination of mucous membrane with saliva from licks;
illness may follow administration of whole cell vaccine of: exposure to bat bites or scratches (severe exposure)
A. Pertussis • Member of the family Rhabdoviridae
B. Diphtheria
C. Tetanus
• From saliva of rabid dogs, cats, bats, and cattle: bite of rabid
D. BCG animal or by licking the mucosa or open wound
A grade schoolboy sustained a superficial scratch on his right leg while • Variable period of communicability
playing with his pet Pomeranian. There was apparently neither break • Incubation period: 20-180 days (9 days/7 years)
in the skin nor bleeding. His mother brings him to you for advice. You
Madali lang tandaan. Category I – nag-inarte lang.
will wash the wound with soap and water and:
Category III – SPONTANEOUS BLEEDING or Cat II in head and neck
A. Not vaccinate Dr. De Vera
B. Give rabies immunization MANAGEMENT OF BITES:
C. Give rabies immunoglobulin • Active anti-rabies vaccine – 0.5 ml IM on days 0, 3, 7, between
D. Give rabies immunization and immunoglobulin
14-28
RABIES PEP • Passive rabies vaccine –
• Category I – touching or feeding animals, licks on the skin (no 1. HRIG – 20 U/kg (1/2 of the dose IM & ½ of the dose
exposure) infiltrated around the wound), or;
• Category II – nibbling of uncovered skin, minor scratches or 2. ERIg – 40 U/kg (needs skin testing)
abrasions without bleeding, licks on broken skin (exposure)
WHO 2018 UPDATE ON RABIES IMMUNIZATION
Category I Category II Category III
• Washing of wound
PLUS
• Cat II management PLUS
• 2-sites ID on day 0,3,7
• RIG
OR
• Washing of wound • Infiltrate as much as possible
Immunologically Naïve • 1-site IM day 0,3,7 and
• No PEP to wound and area around it
between 14-28
• No need to give the remaining
OR
via IM
• 2-sites IM day 0 and 1-site IM
day 7, 21
• Washing of wound
PLUS
Previously Immunized
• 1-site ID on days 0 and 3
Those who received PreP or PEP • Washing of wound • Same as Cat II
OR
within 3 months requires NO • No PEP • No RIG
• 4-sites ID on day 0
Vaccination
OR
• 1-site IM days 0 and 3
Take note that this is the latest guidelines. In the 20th edition of nelson’s the much older one is used (5 doses of vaccine, now only 4. IG remaining dose is infiltrated
on the deltoid which now is no longer recommended)
Dr. De Vera

Before sending the patient home…. SUPPLEMENT: QUICK SHEET


• Check tetanus immunization status IMPORTANT POINTS:
• Who needs antibiotic prophylaxis? Category III bites • Simultaneous administration of Vaccines
• Drug of choice? o No contraindication except for
o Co-Amoxiclav at 40 mg/kg/day for 7 days 1. MCV and DPT, and
2. Yellow fever and Cholera
TETANUS PROPHYLAXIS IN ROUTINE WOUND MANAGEMENT • Lapsed immunization schedules
Adapted from Nelson Textbook of Pediatrics, 20th ed.
o Does not require reinstitution of series
Clean, Minor All Other
(DTP, IPV, Hep B, & Hib)
wounds Wounds*
• Unknown Immunization status
HISTORY OF
o Assume not immunized and proceed with regular schedule
ABSORBED TDAP TDAP
TIG‡ TIG‡
TETANUS OR TD† OR TD†
TOXOID ✔ GUIDE QUESTION
A 10-month-old infant had measles, what will you give in an exposed
Uncertain or <3
Yes No Yes Yes unvaccinated child to prevent measles?
doses A. Vitamin A
3 or more doses No§ No Noǁ No B. Vaccinate immediately
*Such as, but not limited to, wounds contaminated with dirt, feces, and saliva; C. Ribavirin within the first 72 hours
puncture wounds; avulsions; wounds resulting from missiles, crushing, burns, D. Amoxicillin clavulanate
and frostbite.
†For children younger than 7 years of age, DTaP if preferred to tetanus toxoid
COVID-19 VACCINE IN CHILDREN
alone if <3 doses of DTaP have been previously given. If pertussis vaccine is
contraindicated, DT id given. For persons 7 years of age or older, Td (or Tdap • Emergency Use Authorization and Health Technology
for adolescents 11-18 years of age) is preferred to tetanus toxoid alone. Tdap Assessment Committee (HTAC) Recommendation
is preferred to Td for adolescents 11-18 years of age who have never received • 12-17 years old (as of Feb 2022)
Tdap. Td is preferred to tetanus toxoid for adolescents who received Tdap o Pfizer-BioNTech
previously or when Tdap is not available. § Primary series: 0.3mL 2 doses 21 days apart
‡TIG should be administered for tetanus-prone wounds in HIV-infected
§ Booster: 0.3mL 28 days after primary series
patients regardless of the history of tetanus immunizations.
§Yes, if 10 years or longer since the last tetanus toxoid-containing vaccine o Moderna
dose. § Primary series: 0.5mL 2 doses 28 days apart
ǁ Yes, if 5 years or longer since the last tetanus toxoid-containing vaccine § Booster: 0.25mL 28 days after primary series
dose. o Primary series: 2 doses
(More frequent boosters are not needed and can accentuate adverse events.) • 5-11 years old (as of Feb 2022)
DT, diphtheria and tetanus toxoid vaccine; DTaP, combined diphtheria o Pfizer-BioNTech
toxoid-tetanus toxoid-acellular pertussis vaccine; Td, tetanus toxoid and § Primary series: 0.2mL 2 doses 21 days apart
reduced diphtheria toxoid vaccine; Tdap, tetanus toxoid, reduced diphtheria
§ Booster: 0.2mL 28 days after primary series
toxoid, and acellular pertussis vaccine; TIG, tetanus immune globulin.

TOPNOTCH MEDICAL BOARD PREP PEDIATRICS MAIN HANDOUT BY DRS. DE VERA AND PUNONGBAYAN Page 21 of 106
For inquiries visit www.topnotchboardprep.com.ph or email us at topnotchmedicalboardprep@gmail.com
This handout is only valid for October 2022 PLE batch. This will be rendered obsolete for the next batch since we update our handouts regularly.
TOPNOTCH MEDICAL BOARD PREP PEDIATRICS MAIN HANDOUT BY DRS. DE VERA AND PUNONGBAYAN
For inquiries visit www.topnotchboardprep.com.ph or https://www.facebook.com/topnotchmedicalboardprep/
This handout is only valid for October 2022 PLE batch. This will be rendered obsolete for the next batch since we update our handouts regularly.
POST EXPOSURE PROPHYLAXIS
• Who receives: Household, Playmate, Hospital workers
DEHYDRATION
• Varicella
https://qrs.ly/dkdufgw
o VZIg within 96 hours 1 vial (125 U/1.25 ml) per 10 kg IM
(never IV!!) max of 5 vials
o Vaccine: healthy children within 3-5 days
✔ GUIDE QUESTION
o Newborn: give if onset of varicella <5 d pre delivery or within
A toddler was brought to you for multiple bouts of watery diarrhea
48 hours without vomiting. He is tachycardic, with weak pulses, reduced skin
o Acyclovir? turgor, and sunken eyes. He is lethargic but irritable to touch. He gulps
• Meningococcemia vigorously the oral rehydration solution offered to him. What is your
o Rifampicin, Ceftriaxone or Ciprofloxacin (adults) assessment?
• Diphtheria A. No dehydration
o All close contact regardless of immune status B. Some dehydration
C. Severe dehydration
o Give booster if none within last 5 years
D. Dehydration with danger signs
o Erythromycin 40-50 mg/kg/d or Benzathine Pen G
SIGNS CLASSIFICATION TREATMENT
IMPORTANT POINTS TO REMEMBER: Two of the
• For IM injections, the choice of site is based on the volume of the following signs: • Plan C
injected material and the size of the muscle. • abnormally • Refer urgently to
• In children younger than 1 year of age, the anterolateral aspect of sleep or difficult the hospital
the thigh provides the largest muscle and is the preferred site to awaken Severe • If child is 2 years
• In older children, the deltoid muscle is usually large enough for • Sunken eyes dehydration old and there is
IM injection. • Not able to drink cholera in the
• The upper, outer aspect of the buttocks should not be used for or drinks poorly area, give
active immunization because the gluteal region is covered by a • Skin pinch goes Tetracycline
significant layer of subcutaneous fat and because of the back slowly
possibility of damaging the sciatic nerve. Two of the
following signs: • Plan B
IRON SUPPLEMENTATION • Restless irritable Some • Continue feeding
AGE PREPARATION DOSE • Sunken eyes dehydration • Follow up in 5
0.3 ml once a day • Drinks eagerly, days
Low birth Drops: 15 mg to start at 2 months thirsty
weight elemental iron/ 0.6 ml of age until 6 • Give fluid and
months Not enough signs food
Infants 6-11 0.6 ml OD for 3 to classify as some • Plan A
Drops No dehydration
months months or severe • Follow up in 5
Children 1-5 5 ml OD for 3 dehydration days if not
Syrup: 30 mg/5 ml
years old months improving
Tablet with 60 mg iron • Treat for 5 days
Adolescents OD
and 400 mcg folic acid with Co-
• Notes Blood in stool Dysentery trimoxazole and
o Preferably taken on empty stomach return after 2
o Preferably not mixed with milk days
Amount given Amount of ORS to
DEWORMING PLAN A after each loose provide for use at
• Recommended for all children 1-12 years old stool home
• Albendazole < 24 mo old 50-100mL 500/day
o 12 months to 23 months – 200 mg single dose every 6 months 2-10 y/o 100-200mL 1000/day
o 24 months and up – 400 mg single dose every 6 mo As much as
• Mebendazole > 10 y/o 2000/day
wanted
o 12 months and above – 500 mg single dose every 6 mo
Amount needed =
• Contraindications to Deworming
PLAN B • Weight (g) x 0.075
o Severe malnutrition
• Weight (kg) x 75
o High grade fever
o Profuse diarrhea FIRST, give THEN 70mL/kg
PLAN C
o Abdominal Pain 30mL/kg for: for:
o Serious illness < 12 mo old 1 hr 5 hrs
o Hypersensitivity to anti-helminthic drug > 12 mo old 30 mins 2 ½ hrs

INTEGRATED MANAGEMENT OF CHILDHOOD ✔ GUIDE QUESTION


A patient is admitted for diarrhea and was diagnosed to have severe
ILLNESSES dehydration, what is the appropriate fluid to use for this patient
LETHARGY EXCEPT?
UNCONSCIOUSNESS A. ORS
B. Plain Lactated Ringer’s Solution
C. Plain NaCl
D. D5 0.9 NaCl

VOMITING
DANGER CONVULSIONS
COMPOSITION OF THE OLD AND REFORMULATED ORS:
OLD WHO-ORS Reformulated ORS
SIGNS (mEq or mmol/L) (mEq or mmol/L)
Glucose 111 75
Sodium 90 75
Chloride 80 65
INABILITY TO DRINK Potassium 20 20
OR BREASTFEED Citrate 10 10
Osmolarity 311 245

TOPNOTCH MEDICAL BOARD PREP PEDIATRICS MAIN HANDOUT BY DRS. DE VERA AND PUNONGBAYAN Page 22 of 106
For inquiries visit www.topnotchboardprep.com.ph or email us at topnotchmedicalboardprep@gmail.com
This handout is only valid for October 2022 PLE batch. This will be rendered obsolete for the next batch since we update our handouts regularly.
TOPNOTCH MEDICAL BOARD PREP PEDIATRICS MAIN HANDOUT BY DRS. DE VERA AND PUNONGBAYAN
For inquiries visit www.topnotchboardprep.com.ph or https://www.facebook.com/topnotchmedicalboardprep/
This handout is only valid for October 2022 PLE batch. This will be rendered obsolete for the next batch since we update our handouts regularly.
This is very important. ORS is one of the most important managements ANTHROPOMETRICS
for AGE. Most diarrhea in children is viral. It will resolve spontaneously. Weight
Reformulated ORS has lower osmolarity thus less chance of actually 1-6 months
pulling water out into the lumen. The glucose:sodium ratio is also
modified 1:1. This is to utilize the Na-Gluc co-transporter. J 𝑊𝑒𝑖𝑔ℎ𝑡 𝑖𝑛 𝑔𝑟𝑎𝑚𝑠 = 𝐴𝑔𝑒 𝑖𝑛 𝑚𝑜𝑛𝑡ℎ𝑠 × 𝟔𝟎𝟎 + 𝐵𝑊
Dr. De Vera
7-11 months
𝑊𝑒𝑖𝑔ℎ𝑡 𝑖𝑛 𝑔𝑟𝑎𝑚𝑠 = 𝐴𝑔𝑒 𝑖𝑛 𝑚𝑜𝑛𝑡ℎ𝑠 × 𝟓𝟎𝟎 + 𝐵𝑊
TREATMENT OF > 6 MONTHS OLD
WITH ACUTE DIARRHEA: 1-6 years old
𝑊𝑒𝑖𝑔ℎ𝑡 𝑖𝑛 𝒌𝒈 = 𝐴𝑔𝑒 𝑖𝑛 𝒚𝒆𝒂𝒓𝒔 × 2 + 8
• Duration: MD-10.44 hrs (-21.13 to 0.25)
o modestly decreased the duration by half a day Take note that BW is doubled at 4 months and tripled at 1 year
Dr. De Vera
• Acute diarrhea persisting beyond 7 days: RR 0.73 (0.61 to 0.88)
o likelihood of prolonged diarrhea is decreased by 30%
Length and Height
• Duration if with Moderate Malnutrition: MD -26.98 hrs (-14.92
0-3 mos 9 cm
to -39.34)
4-6 mos 8 cm
o decreased the duration by more than a day
6-9 mos 5 cm
ROLE OF SUPPLEMENTAL ZINC THERAPY: 10-12 mos 3 cm
Ave gain in length during 1st year 25 cm
• Reduction in the severity and duration of diarrhea:
o 25% reduction of diarrhea
GROWTH PARAMETERS
o 30% reduction in stool volume
• Prevention of subsequent episodes of diarrhea: • Mnemonics for length / height:
o 10-20 mg of Zn per day for 10-14 days reduces the number of o 𝐻𝑡 𝑖𝑛 𝑐𝑚𝑠 = 𝐴𝑔𝑒 𝑖𝑛 𝒚𝒆𝒂𝒓𝒔 × 5 + 80
diarrheal episodes in the 2-3 months after supplementation o 𝐻𝑡 𝑖𝑛 𝑖𝑛𝑐ℎ𝑒𝑠 = 𝐴𝑔𝑒 𝑖𝑛 𝒚𝒆𝒂𝒓𝒔 × 2 + 32

✔ GUIDE QUESTION SUPPLEMENT: QUICK SHEET


A 4-year-old child is currently being treated with Cefuroxime for • At 2 yrs old: 1/2 mature height for boys
Pneumonia but is not improving. If you suspect that he has Mycoplasma • At 3 yrs old: 3 feet tall
infection, you expect him to improve with: • At 4 yrs old: 2x the birth length
A. Cefuroxime for 1 more week
• At 13 yrs old: 3x the birth length
B. Azithromycin
C. Ceftriaxone
D. Vancomycin HEAD CIRCUMFERENCE
• at birth 33-35 cm
PLEASE READ MORE ON: • taken up to 3 years old
• Updates on IMCI • Attained 90% of adult head size by age 2
• approximates adult head at 6 years old
Remember! You get a big head first!
GROWTH AND DEVELOPMENT Dr. De Vera

✔ GUIDE QUESTION
A mother brought in her 8-month-old daughter and was concerned that
her child has no erupted teeth yet. Her daughter had unremarkable
birth and maternal history and is otherwise well. What will you advise
the mother?
A. work up for congenital hypothyroidism as this may manifest
with delayed tooth eruption
B. This is definitely abnormal because at 8 months the child should
have at least 2 teeth
C. Request for a panoramic x-ray
D. Observe and re-assure the mother

ALL ABOUT THE TEETH


Much like immunizations, G&D entails memorization. Understanding the • # 𝑜𝑓 𝑡𝑒𝑒𝑡ℎ = 𝐴𝑔𝑒 𝑖𝑛 𝑚𝑜𝑛𝑡ℎ𝑠 − 6
principles below is VERY IMPORTANT! • Suspect thyroid disorder if no teeth by 13 mos
Dr. De Vera
PRINCIPLES IN GROWTH AND DEVELOPMENT • Usually begins at 6 months
• a continuous process • Central mandibular incisors
• Progresses in a cephalocaudal manner & proximodistal pattern • All 20 primary teeth have erupted by 3 years old
• Responses proceed from generalized reflexes to discrete
voluntary actions FIRST YEAR OF LIFE
• Developmental sequence is orderly and predictable GROWTH
• Rate varies from child to child • Objective assessment: weight, length, head circumference
• Average newborn: 3 kilos or 6 lbs.
DEVELOPMENTAL MILESTONES • Babies double their birth weight during the 4th-5th months and
• Though an age level is assigned to each milestone, the actual age triple at 1 year old
at which a milestone is achieved by a normal child may range • Average birth length: 50 cms
within 2 months of this age level in the first year of life to 4 • Average length at 1 year old: 75 cms.
months from the second year on. • Average head circumference at birth: 33-35 cms.
• Average HC at 1 year old: 45 cms.
DELAY
• Child’s development is not as advanced as it should be
• the rate of development has been slower than what is usually DEVELOPMENTAL
acceptable MILESTONES
✔ GUIDE QUESTION https://qrs.ly/elduf9x
A 5-month-old baby was brought in for consult. The child was born full
term and his birth and maternal history was unremarkable. His birth
weight is 2900g. Based on your knowledge in growth and development,
you expect the child’s weight at present to be?
A. 5,800 C. 4,000g
B. 7,000g D. 9,000g

TOPNOTCH MEDICAL BOARD PREP PEDIATRICS MAIN HANDOUT BY DRS. DE VERA AND PUNONGBAYAN Page 23 of 106
For inquiries visit www.topnotchboardprep.com.ph or email us at topnotchmedicalboardprep@gmail.com
This handout is only valid for October 2022 PLE batch. This will be rendered obsolete for the next batch since we update our handouts regularly.
TOPNOTCH MEDICAL BOARD PREP PEDIATRICS MAIN HANDOUT BY DRS. DE VERA AND PUNONGBAYAN
For inquiries visit www.topnotchboardprep.com.ph or https://www.facebook.com/topnotchmedicalboardprep/
This handout is only valid for October 2022 PLE batch. This will be rendered obsolete for the next batch since we update our handouts regularly.
DEVELOPMENTAL MILESTONES • 6 mos: looks to see where • 6 mos: imitates sounds;
dropped toy is babbles: “da-da, ba-ba, pa-
• 7 mos: responds to name- pa”
calling; inhibits to “no”; • 10 mos: says “mama”,
follows 1-step command “dada”
with gesture • 12 mos: speaks first real
• 10 mos: follows 1-step word; 2-word vocabulary
command w/o gesture • 15 mos: jargon
• 18 mos: points to body • 18 mos: speaks 10-15
parts words; names pictures

2-5 YEARS OLD


2 3 4 5
Dry by day Dry by night Hops Can skip
GROSS & FINE MOTOR SKILLS: 1 YEAR OLD Runs Rides a Draws a Draws a
• 10 mos: sits up alone and • 8 mos: pincer grasp Follows 2/3 tricycle cross square
indefinitely without • 12 mos: turns pages of commands Knows name Knows colors Knows ABC
support with back straight; book; releases object to Parallel play age and sex VA is 20/20 Can write
pulls to stand another person on request Draws a name
• 12 mos: rises • 13 mos: scribbles circle
independently; takes • 15 mos: builds tower of 2-3
several steps cubes; makes a line with
• 15 mos: walks alone; crawls crayon
up stairs • 18 mos: imitates vertical
• 18 mos: runs stiffly; walks line; tower of 4 cubes Mnemonics above shows what a
upstairs with one hand held child can draw. Circle at 3 y/o, cross
at 4, square at 5, triangle at 6yo.
RECEPTIVE & EXPRESSIVE LANGUAGE: 1 YEAR OLD Here is a drawing that helped me
• 1.5 mos: smiles in response • 1 mo: throaty gurgling remember these.
to face & voice sounds
• 2 mos: smiles on social • 2 mos: vocalizes, coos
contact • 3 mos: says “aah”
• 3 mos: turns head to • 4 mos: laughs aloud Dr. De Vera
sound/voice • 5 mos: “ah-goo”
DEVELOPMENTAL MILESTONES IN THE 1ST 2 YEARS OF LIFE
Adapted from Nelson Textbook of Pediatrics, 20th ed.
AVERAGE AGE OF
MILESTONE DEVELOPMENTAL IMPLICATIONS
ATTAINMENT (MO)
GROSS MOTOR
Holds head steady while sitting 2 Allows more visual interaction
Pulls to sit, with no head lag 3 Muscle tone
Brings hands together in midline 3 Self-discovery of hands
Asymmetric tonic neck reflex gone 4 Can inspect hands in midline
Sits without support 6 Increasing exploration
Rolls back to stomach 6.5 Truncal flexion, risk of falls
Walk alone 12 Exploration, control of proximity to parents
Runs 16 Supervision more difficult
FINE MOTOR
Grasps rattle 3.5 Object use
Reaches for objects 4 Visuomotor coordination
Palmar grasp gone 4 Voluntary release
Transfers object hand to hand 5.5 Comparison of objects
Thumb-finger grasp 8 Able to explore small objects
Turns pages of book 12 Increasing autonomy during book time
Scribbles 13 Visual-motor coordination
Builds tower of 2 cubes 15 Uses objects in combination
Builds tower of 6 cubes 22 Requires visual, gross, and fine motor coordination
COMMUNICATION AND LANGUAGE
Smiles in response to face, voice 1.5 More active social participant
Monosyllabic babble 6 Experimentation with sound, tactile sense
Inhibits to “no” 7 Response to tone (nonverbal)
Follows one-step command with gesture 7 Nonverbal communication
Follows one-step command without gesture 10 Verbal receptive language (e.g., “Give it to me”)
Says “mama” or “dada” 10 Expressive language
Points to object 10 Interactive communication
Speaks first real word 12 Beginning of labeling
Speaks 4-6 words 15 Acquisition of object and personal names
Speaks 10-15 words 18 Acquisition of object and personal names
Speaks 2-word sentences (e.g., “Mommy shoe” 19 Beginning grammaticalization, corresponds with 50-word vocabulary
COGNITIVE
Stares momentarily at spot where object Lack of object permanence
2
disappeared (out of sight, out of mind [e.g., yarn ball dropped])
Stares at own hand 4 Self-discovery, cause and effect
Bangs 2 cubes 8 Active comparison of objects
Uncovers toy (after seeing it hidden) 8 Object permanence
Egocentric symbolic play (e.g., pretends to
12 Beginning symbolic though
drink from cup)
Uses stick to reach toy 17 Able to link actions to solve problems
Pretend play with doll (e.g., gives doll bottle) 17 Symbolic thought
TOPNOTCH MEDICAL BOARD PREP PEDIATRICS MAIN HANDOUT BY DRS. DE VERA AND PUNONGBAYAN Page 24 of 106
For inquiries visit www.topnotchboardprep.com.ph or email us at topnotchmedicalboardprep@gmail.com
This handout is only valid for October 2022 PLE batch. This will be rendered obsolete for the next batch since we update our handouts regularly.
TOPNOTCH MEDICAL BOARD PREP PEDIATRICS MAIN HANDOUT BY DRS. DE VERA AND PUNONGBAYAN
For inquiries visit www.topnotchboardprep.com.ph or https://www.facebook.com/topnotchmedicalboardprep/
This handout is only valid for October 2022 PLE batch. This will be rendered obsolete for the next batch since we update our handouts regularly.
QUICK SHEET: Please memorize this table. This shows at what age primitive reflexes are
OTHER IMPORTANT STUFF TO REMEMBER expected to disappear. Remember, primitive reflexes should disappear in
• Handedness usually at 3 years old order for voluntary actions to happen. For patients with cerebral palsy,
• Bed wetting it is common to find persistence of these reflexes.
Dr. De Vera
o Up to 4 y/o in Girls
o Up to 5 y/o in Boys
• Ties shoes at 6 y/o
• Language explosion at 2 y/o (20-50 words by end of 2nd year)
• Interest in Basic sexuality at 4-5 years old

✔ GUIDE QUESTION
Among the following reflexes, which normally persist throughout life?
A. Moro reflex
B. Rooting
TONIC NECK REFLEX
https://www.healthline.com/health/baby/tonic-neck-reflex
C. Tonic Neck reflex
D. Parachute
E. Sucking reflex
This reflex is elicited by supporting the infant in a semi-erect position
and then allowing the infant’s head to fall backwards onto the
examiner’s hand. To which the expected response is symmetric
extension and abduction of the fingers and upper extremities, followed
by flexion of the upper extremities and an audible cry
A. Landau Reflex
B. Parachute Reflex LANDAU REFLEX
C. Moro Reflex https://www.playstreet.in/2020/09/23/primitive-reflex-integration/

D. Tonic Neck Reflex


✔ GUIDE QUESTIONS
FULLY The average hour of sleep of a 3-month-old baby is?
REFLEX ONSET DURATION A. 15
DEVELOPED
B. 15.5
Palmar 28 wk 32 wk 2-3 mo C. 16
grasp gestation gestation postnatal D. 16.5
Less prominent
32 wk 36 wk
Rooting after 1 mo
gestation gestation
postnatal
28-32 wk 37 wk 5-6 mo
Moro
gestation gestation postnatal
Tonic 35 wk 1 mo 6-7 mo
Neck gestation postnatal postnatal
7-8 mo 10-11 mo Remains
Parachute
postnatal postnatal throughout life

REFLEXES
• Moro reflex is elicited by supporting the infant in a semi-erect
position and then allowing the infant’s head to fall backwards
onto the examiner’s hand
o normal = symmetric extension and abduction of the fingers
and upper extremities, followed by flexion of the upper
extremities and an audible cry
o asymmetric response = fractured clavicle, brachial plexus
injury, or hemiparesis
o No response = significant CNS dysfunction
• Grasp response is elicited by placing a finger in the open palm
of each hand
o 37 wk of gestation, strong enough that the examiner can lift the
infant from the bed with gentle traction
• Tonic neck reflex is produced by manually rotating the infant’s
head to 1 side and observing for the characteristic fencing
posture
• Parachute reflex
o slightly older infants Nelson Textbook of Pediatrics, 20th ed.
o evoked by holding the infant’s trunk and then suddenly A 5-year-old male was brought in for consult due to poor school
lowering the infant as if he or she were falling performance. The child was noted to have poor participation in group
o The arms will spontaneously extend to break the infant’s fall activities. Most of the time the child would not look at you when he/she
o this reflex a prerequisite to walking is being talked to. He was noted to be mostly busy arranging his favorite
toy cars. Upon further history there was note of developmental delay
AGE OF AGE OF in verbal and cognitive domains. This is probably a case of?
REFLEX
APPEARANCE DISAPPEARANCE A. ADHD
Rooting 32 weeks AOG 1 month B. Global developmental delay
Palmar 28 weeks AOG 2-3 months C. Autism Spectrum Disorder
Placing/Stepping 37 weeks AOG 4-5 months D. Generalized Epilepsy
Which among the following is NOT key feature of Autism Spectrum
28-32 weeks
Moro 5-6 months Disorder?
AOG A. Deficits in social-emotional reciprocity
Tonic neck B. Deficit in non-verbal communicative behavior
35 weeks AOG 6-7 months
reflex C. Restricted pattern of behavior
Plantar 11 weeks AOG
7-9 months D. Impulsivity
Covered up by
Landau 3 months
voluntary action
Covered up by
Parachute reflex 4-9 months
voluntary action
TOPNOTCH MEDICAL BOARD PREP PEDIATRICS MAIN HANDOUT BY DRS. DE VERA AND PUNONGBAYAN Page 25 of 106
For inquiries visit www.topnotchboardprep.com.ph or email us at topnotchmedicalboardprep@gmail.com
This handout is only valid for October 2022 PLE batch. This will be rendered obsolete for the next batch since we update our handouts regularly.
TOPNOTCH MEDICAL BOARD PREP PEDIATRICS MAIN HANDOUT BY DRS. DE VERA AND PUNONGBAYAN
For inquiries visit www.topnotchboardprep.com.ph or https://www.facebook.com/topnotchmedicalboardprep/
This handout is only valid for October 2022 PLE batch. This will be rendered obsolete for the next batch since we update our handouts regularly.

AUTISM SPECTRUM DISORDER (DSM V) A. Inattention (6 or more of the ff:)


1. Often fails to give close attention to details or makes
• Encompasses the following:
careless mistakes in school
1. Autistic disorder (autism) 2. Often has difficulty sustaining attention in tasks or play
2. Asperger disorder
activities
3. Childhood disintegrative disorder
3. Often does not seem to listen when spoken to directly
4. Rett disorder
4. Often does not follow through on instructions and fails to
5. Pervasive developmental disorder NOS
finish schoolwork
• Characterized by deficits in 2 core domains: 5. Often has difficulty in organizing tasks
1. Deficits in social communication and social interaction 6. Often avoids, dislikes or is reluctant to engage in tasks
2. Restricted repetitive patterns of behavior, interests, and that require sustained mental effort
activities 7. Often loses things necessary for tasks
These are the 2 most important things to remember in ASD. Please 8. Often easily distracted by external stimuli
remember this 2 core domain deficits. 9. Often forgetful in daily activities
Dr. De Vera
B. Hyperactivity and impulsivity (6 or more of the ff:)
AUTISM SPECTRUM DISORDER 1. Often fidgets with hands or feet or squirms in seat
A. Persistent deficits in social communication & social interaction 2. Often leaves seat in classroom
1. Deficits in social-emotional reciprocity (failure of normal 3. Often runs about or climbs excessively
back-and-forth conversation; reduced sharing of emotions, 4. Often has difficulty playing or engaging in leisure
interests, or affect; failure to initiate or respond to social activities quietly
interactions) 5. Often “on the go”
2. Deficits in nonverbal communicative behaviors used for 6. Often talks excessively
social interaction (abnormal eye contact and body 7. Often blurts out answers before questions have been
language, deficit in understanding and use of gestures; total completed
lack of facial expressions) 8. Often has difficulty awaiting turn
3. Deficits in developing, maintaining, and understanding 9. Often interrupts or intrudes on others
relationships (absence of interest in peers, difficulties in *** SEVERAL symptoms were present before 12 years of age.
making friends) C. Several inattentive or hyperactive-impulsive symptoms
B. Restricted, repetitive patterns of behavior, interests, or are present in 2 or more settings (ex: at home, school, or
activities as manifested by at least 2 of the following work; with friends or relatives; in other activities)
currently or by history: D. There is clear evidence that symptoms interfere with, or reduce
1. Stereotyped or repetitive motor movements, use of objects, or the quality of, social, academic, or occupational functioning.
speech (lining up of toys, echolalia, flipping objects) E. The symptoms do not occur exclusively during the course
2. Insistence on sameness, inflexible adherence to routines, of schizophrenia or another psychotic disorder and are
or ritualized patterns of verbal or nonverbal behavior not better explained by another mental disorder.
(extreme distress at small changes, rigid thinking patterns,
difficulties with transition, need to take same route or eat MANAGEMENT
same food every day) • Therapy provided by:
3. Highly restricted, fixated interests that are abnormal in 1. Pediatrician
intensity or focus (preoccupation with unusual objects, 2. Clinical psychologist
perseverative interests) 3. Occupational therapist
4. Hyper- or hypo- reactivity to sensory input or unusual 4. Parents
interest in sensory aspects of the environment (apparent 5. Other members of the family (siblings, grandparents, caregivers)
indifference to pain/temperature, excessive smelling or PLEASE READ MORE ON:
touching of objects, visual fascination with lights or Specific developmental milestones especially during the first
movement, adverse reaction to sounds/textures) and second years of life – Memory work
C. Symptoms must be present in the early developmental period
D. Symptoms cause clinically significant impairment in social, GASTROENTEROLOGY
occupational, or other important areas of current functioning.
✔ GUIDE QUESTIONS
E. These disturbances are not better explained by
Cholelithiasis in children is usually due to:
intellectual disability or global developmental delay. A. Abnormal lipid metabolism
The best-established prognostic factors for individual B. Cholesterolosis
outcome are: C. Hemolytic dyscrasia
D. Abnormal calcium deposition
• presence or absence of associated intellectual disability
A 3-month old baby boy was brought for jaundice. Patient was noted to
• language impairment develop jaundice on the 10th day of life. He was exclusively breast fed
o Currently, as many as 15% of ASD are associated with a known and daily sun exposure was done. Mother reported the dark staining
genetic mutation; males > females (4:1) urine in the diaper and whitish stool which she thought were normal.
The child is well, no fever, has good appetite and activity. Family history
ATTENTION-DEFICIT / HYPERACTIVITY DISORDER (ADHD) is unremarkable, patient is full term. On physical examination there is
generalized jaundice and hepatomegaly. Bilirubin was done Total
• DSM V changes: 15mg/dl, direct 10mg/dl, indirect is 5mg/dl. The following are possible
o Age at onset: several inattentive or hyperactive-impulsive differential diagnosis EXCEPT?
symptoms were present prior to age 12 A. Neonatal Hepatitis
o a comorbid diagnosis with ASD is now allowed B. Biliary Atresia
• A neurodevelopmental disorder defined by impairing levels of C. Alagille Syndrome
inattention, disorganization, and/or hyperactivity- D. Breastfeeding Jaundice
If you are suspecting biliary atresia, what is the gold standard for
impulsivity; males > females (2:1)
diagnosis of this disease?
• Inattention and disorganization: unable to stay on task, seeming A. HIDA Scan
not to listen, losing materials at levels that are inconsistent with B. Ultrasound
age or developmental level C. Biopsy
• Hyperactivity-impulsivity: overactivity, fidgeting, inability to stay D. Direct Cholangiography
seated, intruding into other people’s activities, inability to wait
• Symptoms are excessive for age and developmental level NEONATAL CHOLESTASIS
• Frequently overlaps with oppositional defiant disorder and • prolonged elevation of the serum levels of conjugated bilirubin
conduct disorder beyond the 1st 14 days of life
If ASD is characterized by deficit in social communication / interaction + • Mechanical Obstruction of bile flow
restricted repetitive behavior, ADHD also has 2 dominant features to o Biliary Atresia
remember, Inattention / disorganization + hyperactivity-impulsivity. • Functional impairment of hepatic excretory function and bile
Dr. De Vera
secretion
TOPNOTCH MEDICAL BOARD PREP PEDIATRICS MAIN HANDOUT BY DRS. DE VERA AND PUNONGBAYAN Page 26 of 106
For inquiries visit www.topnotchboardprep.com.ph or email us at topnotchmedicalboardprep@gmail.com
This handout is only valid for October 2022 PLE batch. This will be rendered obsolete for the next batch since we update our handouts regularly.
TOPNOTCH MEDICAL BOARD PREP PEDIATRICS MAIN HANDOUT BY DRS. DE VERA AND PUNONGBAYAN
For inquiries visit www.topnotchboardprep.com.ph or https://www.facebook.com/topnotchmedicalboardprep/
This handout is only valid for October 2022 PLE batch. This will be rendered obsolete for the next batch since we update our handouts regularly.

ACUTE DIARRHEA
• excessive loss of fluid and electrolyte in the Stool
• sudden onset of excessively loose stools of >10 mL/kg/day in
infants and >200 g/24 hr in older children, which lasts < 14 days
• > 14 days = chronic or persistent diarrhea

DIARRHEA
https://qrs.ly/uxdugfi

Patients presenting with non-inflammatory diarrhea


Day care center
Rotavirus, ETEC
Infants and Toddlers
Profuse diarrhea and
vomiting
Cholera
Flecks of mucous on
Watery voluminous diarrhea
diarrhea, Profuse diarrhea after
Vibrio
vomiting eating raw oysters or
Nelson Textbook of Pediatrics, 20th ed. parahaemolyticus
undercooked shellfish
Greasy stool after camping
BILIARY ATRESIA Giardiasis
Gas
• a.k.a. Non-cystic Obliterative Cholangiopathy Water diarrhea and vomiting
• 2 types: Fetal and Perinatal ETEC
after history of travel
• Most common form (85%) – obliteration of the entire
extrahepatic biliary tree at or above the porta hepatis CHOLERA
• Most are normal at birth with postnatal progressive obliteration PATHOGENESIS
of bile ducts • Colonization of upper small intestine → produces enterotoxin →
• Post-natal onset is usually immune or infection mediated promotes secretion of fluids & electrolytes into the lumen of the
SI → enterotoxin’s B subunit binds to GM1 ganglioside receptor
BILIARY ATRESIA VS. NEONATAL HEPATITIS → allows A subunit to enter cells → activates adenylate cyclase
• Difficult to differentiate → ↑ cAMP → blocks NaCl absorption → promotes secretion of
• No single biochemical test Cl & H2O by crypt cells → massive outpouring of isotonic fluid
into the SI → overwhelms the absorptive capacity of the LI →
• Ultrasound
severe diarrhea
o Not diagnostic
o Important to rule out other surgical causes of cholestasis (e.g. TREATMENT
Choledochal cyst) • Fluid & electrolyte replacement
o Triangular Cord sign – cone-shaped fibrotic mass cranial to the • When do you use antibiotics?
bifurcation of the portal vein, may be seen in patients with 1. To shorten the duration of illness
biliary atresia 2. To reduce period of excretion of the organism
• Hepatobiliary scintigraphy – sensitive but not specific 3. To decrease the requirements for fluid therapy
• Liver biopsy – important to discriminate between the various DRUGS USED IN CHOLERA
causes of cholestasis
• Tetracycline 12.5 mg/kg/dose 4x day PO qid for 3 days (max 2
• Cholangiography – gold standard. To determine the presence g/day); not for <9 yrs old; or
and site of obstruction
• Doxycycline 5 mg/kg PO single dose (max 200 mg/day)
BILIARY ATRESIA NEONATAL HEPATITIS • Alternatives: TMP-SMX 8-10 mg/kg/day bid PO; Erythromycin
Location Extrahepatic Intrahepatic 40 mg/kg/day every 6 hours max. 2g/day; or Furazolidone 5-8
Familiar mg/kg/day max 400 mg/day
Unlikely 20%
Incidence
Premature ✔ GUIDE QUESTIONS
Uncommon Common
and SGA What is the mechanism behind the bleeding diarrhea in shigella?
Persistent A. exotoxin
Yes +/-
acholic stool B. invasion of the intestinal epithelium
Severe, diffuse C. increased cGMP
hepatocellular disease, D. ADP ribosylation
Bile duct proliferation, distortion of lobular A 10-year-old boy presents with 9 days intermittent fever with Tmax =
bile plugs, portal or architecture, marked 39.2°C, malaise, and soft stools. 3 days PTC, he had on & off abdominal pain
perilobular edema and infiltration with which increased in intensity. Coherent, tachycardic, febrile, no TPC, clear
Biopsy
fibrosis, INTACT basic inflammatory cells and breath sounds, flat abdomen, hyperactive bowel sounds, tenderness in the
hepatic lobular focal hepatocellular periumbilical area, full pulses. If you want to isolate the causative organism,
architecture necrosis. the best specimen for sample collection would be?
BILE DUCT SHOWS LESS A. Blood C. Stool
ALTERATION B. Urine D. All of the above
BILIARY ATRESIA MANAGEMENT ENTERIC FEVER
• Nutritional support • Ingestion of food or water contaminated with S. typhi from
• Kasai hepatoportoenterostomy human feces – most common mode of transmission
o Golder period of 60 days. Success rate is much higher(90%) if • Period of communicability: throughout the duration of fecal excretion
performed before 8 weeks of life.
• Incubation period: 7-14 days
• Definitive management = liver transplant o More dramatic presentation in <5 years old
✔ GUIDE QUESTIONS WHAT ARE THE SIGNS/SYMPTOMS OF TYPHOID FEVER?
The most common cause of viral gastroenteritis in infants is: • High-grade fever, malaise, myalgia, cough, abdominal pain,
A. Rotavirus C. Respiratory syncytial virus hepatosplenomegaly, anorexia, diarrhea / constipation
B. Norwalk virus D. Parvovirus B19 • Maculopapular rashes (rose spots) in 25% - visible on day 7-10
By definition acute diarrhea DOES NOT last longer than ___days? of illness on the lower chest or abdomen and lasts 2-3 days
A. 11 C. 13
• If no complications occur, the symptoms & physical findings
B. 12 D. 14
gradually resolve within 2-4 weeks
TOPNOTCH MEDICAL BOARD PREP PEDIATRICS MAIN HANDOUT BY DRS. DE VERA AND PUNONGBAYAN Page 27 of 106
For inquiries visit www.topnotchboardprep.com.ph or email us at topnotchmedicalboardprep@gmail.com
This handout is only valid for October 2022 PLE batch. This will be rendered obsolete for the next batch since we update our handouts regularly.
TOPNOTCH MEDICAL BOARD PREP PEDIATRICS MAIN HANDOUT BY DRS. DE VERA AND PUNONGBAYAN
For inquiries visit www.topnotchboardprep.com.ph or https://www.facebook.com/topnotchmedicalboardprep/
This handout is only valid for October 2022 PLE batch. This will be rendered obsolete for the next batch since we update our handouts regularly.
Typhoid is one of your great mimickers. A clue would be prolonged fever PROGNOSIS
+ weird signs and symptoms (may present with UTI, pneumonia, GI • Despite appropriate Tx, 2-4% may relapse after initial clinical
symptoms, myocarditis etc.) response to treatment.
Dr. De Vera
WHAT ARE THE COMPLICATIONS? • Those who excrete S. typhi for more than 3 months are chronic carriers.
• Intestinal hemorrhage (<1%) and perforation (0.5-1%) is • Risk for becoming a carrier is <2% for all infected children
infrequent in children.
• If with perforation: RLQ pain, tenderness, vomiting, features of
peritonitis
• Rare: toxic myocarditis, delirium, increased ICP, pyelonephritis,
meningitis, endocarditis
TREATMENT OF TYPHOID FEVER IN CHILDREN
Adapted from Nelson Textbook of Pediatrics, 20th ed.

Optimal Therapy Alternative Effective Drugs


DAILY DOSE DAILY DOSE
SUSCEPTIBILITY ANTIBIOTIC DAYS ANTIBIOTIC DAYS
(mg/kg/day) (mg/kg/day)
UNCOMPLICATED TYPHOID FEVER
Fluoroquinolone, e.g.,
Fully sensitive Chloramphenicol 50-75 14-21 15 5-7*
ofloxacin or ciprofloxacin
Multidrug – Amoxicillin 75-100 14
Azithromycin 8-10 7
resistant Fluoroquinolone 15 5-7
or
Cefixime 15-20 7-14
Quinolone – Cefixime 15-20 7-14
Azithromycin 8-10 7
resistant† Cefixime 20 7-14
or
Ceftriaxone 75 10-14
SEVERE TYPHOID FEVER
Fluoroquinolone, e.g., Chloramphenicol 100 14-21
Fully sensitive 15 10-14
ofloxacin Amoxicillin 100
Multidrug – Ceftriaxone 60 10-14
Fluoroquinolone 15 10-14
resistant or
Cefotaxime 80 10-14
Quinolone – Ceftriaxone 60 10-14
Azithromycin 10-20 7
resistant Cefotaxime 80 10-14
Fluoroquinolone 20 7-14
*A 3-day course is also effective, particularly for epidemic containment.
†The optimum treatment for quinolone-resistant typhoid fever has not been determined. Azithromycin, third-generation cephalosporins, or high-dose

fluoroquinolones for 10-14 days is effective.


INFLAMMATORY DIARRHEA/DYSENTERY SHIGELLOSIS
Patients presenting with PATHOGENESIS
inflammatory diarrhea/dysentery • Basic virulence trait: ability to invade intestinal epithelial cells
Trophozoites with ingested RBC’s Amoebiasis • Pathologic changes occur in the colon
Abdominal cramps, systemic Clostridium • Shigella cross the colonic epithelium through M cells overlying
toxicity after antibiotic use difficile the Peyer patches
Bloody Abdominal cramps, tenesmus, • S. dysenteriae serotype 1 produce Shiga toxin, a potent protein
Shigella synthesis-inhibiting exotoxin causes HUS.
diarrhea abundant pus and WBC in stool
High fever, headache, drowsiness,
Pus and confusion, meningismus, seizures, FACTS ABOUT SHIGELLA
WBC in abdominal distention Salmonella • Incubation period: 12 hrs to several days before symptoms ensue
stool History of eating eggs, poultry, • Period of communicability: as long as the person excretes the
unpasteurized milk bacteria (1-4 wks)
Diarrhea with blood after eating • Symptoms: severe abdominal pain, high fever, emesis, anorexia,
EHEC
hamburger generalized toxicity, painful defecation, urgency
• PE findings: abdominal distention & tenderness, hyperactive
Patients presenting with
bowel sounds, tender rectum on digital exam
inflammatory diarrhea/dysentery
Ham, potato salad, cream pastries Staphylococcus
Abdominal WHAT TO EXPECT IN SHIGELLOSIS:
Reheated fried rice Bacillus cereus
cramps • Watery voluminous diarrhea initially evolving into frequent
Home-canned food
Diarrhea small-volume, bloody mucoid stools → significant dehydration
Muscle weakness
Sweating Botulism • Neurologic manifestations (40%) but the cause is not
Diplopia
No fever understood
Blurring of vision
• Mortality rate is high (20%) when sepsis occurs
WHAT ARE THE 5 MAJOR GROUPS OF E. COLI?
• ETEC - infantile explosive diarrhea with dehydration; few or no DIFFERENTIAL DIAGNOSIS FOR BLOODY DIARRHEA:
structural changes in the gut mucosa • Campylobacter jejuni
• EIEC – cause colonic lesions like dysentery • Salmonella
• EPEC – non-bloody diarrhea with mucus; prolonged • Enteroinvasive E.coli (EIEC)
• EAEC- significant dehydration; prolonged diarrhea • Entamoeba histolytica
• EHEC - shiga-toxin producing E.coli; bloody diarrhea • Clostridium difficile
✔ GUIDE QUESTION • Yersinia enterocolitica
An 8-year-old male child was brought to the clinic for diarrhea with
bloody watery stools. He had 3 episodes of diarrhea today which DIAGNOSTIC TESTS:
started suddenly accompanied by fever which was checked at 38.5°C, • Presumptive dx: fecal leukocytes (>50-100 PMNS/hpf) & fecal
he had 2 episodes of vomiting and intermittent abdominal pain, the boy blood; ↑ WBC in CBC
also complains of painful defecation, the stools are characterized as
initially watery then becoming bloody with mucus, no family members
• Definitive dx: culture of stool & rectal swab specimen
or close friends with the same illness was noted, which of the ff. is the (MacConkey agar, xylose-lysine deoxycholate, SS agar)
most likely diagnosis? o use clinical judgment in the management of clinical
A. Rotavirus C. Vibrio cholera syndromes consistent with shigellosis
B. Shigella D. lactose intolerance

TOPNOTCH MEDICAL BOARD PREP PEDIATRICS MAIN HANDOUT BY DRS. DE VERA AND PUNONGBAYAN Page 28 of 106
For inquiries visit www.topnotchboardprep.com.ph or email us at topnotchmedicalboardprep@gmail.com
This handout is only valid for October 2022 PLE batch. This will be rendered obsolete for the next batch since we update our handouts regularly.
TOPNOTCH MEDICAL BOARD PREP PEDIATRICS MAIN HANDOUT BY DRS. DE VERA AND PUNONGBAYAN
For inquiries visit www.topnotchboardprep.com.ph or https://www.facebook.com/topnotchmedicalboardprep/
This handout is only valid for October 2022 PLE batch. This will be rendered obsolete for the next batch since we update our handouts regularly.
TREATMENT DIAGNOSIS
• Empirical therapy: • Plain abdominal X-ray:
o Cefixime 8 mg/kg/day po every 12 hrs for 5 days; (+) density
o Ceftriaxone 50 mg/kg/day IV or IM once a day for 5 days; or • Barium enema: coiled-
o Azithromycin 12 mg/kg/day po 1st day followed by 6 spring sign (thin rim of
mg/kg/day for the next 4 days barium trapped around
• WHO recommendation for all patients with bloody diarrhea the invaginating part
irrespective of age: Ciprofloxacin 30 mg/kg/day in 2 divided doses within the
• Zinc 20 mg/day for 14 days improves immune response to Shigella intussuscipiens;
Again, let’s make things simple. edematous mucosal folds
Acute diarrhea in children = virus unless proven otherwise of the returning limb of
Acute bloody diarrhea = Shigella NOT amoebiasis (<30%, look for E. intussusceptum outlined
histolytica TROPHOZOITES, NOT cyst) by the contrast)
DOC for Shigella = Ciprofloxacin • Ultrasound: tubular mass
Definitive diagnosis Shigella = culture & a doughnut or target
Dr. De Vera
sign (98-100%
✔ GUIDE QUESTIONS sensitivity)
A 1-and-a-half-year-old male baby was brought into the clinic due to Ultrasound finding: target sign
crying spells accompanied by knee flexion. There are intermittent
MODALITY ADVANTAGES DISADVANTAGES
episodes that he feels well. The patient reportedly has vomited once
during an episode of abdominal pain, vomitus consists of previously Success rate under Recurrent cases
Hydrostatic
ingested food and sometimes clear watery fluid. There is poor appetite. fluoroscopic or noted in 5-8%
(saline
Which among the ff. should be asked? ultrasonic guidance (more common in
A. any recent change in diet reduction)
in 80-85% hydrostatic reduction)
B. any illness in the family Fewer complications
C. any blood in the stool Air Perforation rate (0-
and lower radiation
D. any bulging of the umbilicus reduction 1-0.2%)
In relation to the case above, what is the initial diagnostic test of choice
exposure
for this patient? Barium and
Bowel perforation at
A. Barium enema hydrostatic
0.5-2.5%
B. Ultrasound of the abdomen reduction
C. Upper GI endoscopy
✔ GUIDE QUESTION
D. Abdominal CT scan
A 2-month-old boy was noted to have vomiting after every feeding for
In relation to the case above, what is the most likely finding on workup?
the past week. There were no other symptoms noted. PE: irritable,
A. Double bubble sign
slightly tachycardic, no retractions, clear breath sounds, (+) firm,
B. Increased pyloric thickness
movable 2x2 cm mass on the abdomen. What is your impression?
C. Donut ring sign
A. Duodenal Atresia C. Malrotation
D. Absence of parasympathetic postganglionic cell bodies
B. Pyloric Stenosis D. Intussusception

PRESENTING AS INTESTINAL OBSTRUCTION PYLORIC STENOSIS

INTESTINAL
OBSTRUCTION
https://qrs.ly/4odugg0

SUPPLEMENT: QUICK SHEET


THINGS TO ASK TO RULE OUT OBSTRUCTION:
1. Onset and duration – early onset vomiting in the first few days of
life is almost always obstructive (i.e. duodenal atresia)
2. Bilious or non-bilious? – bilious is commonly associated with
DIAGNOSTIC TESTS
obstruction
3. Abdominal distention? • Ultrasound is confirmatory: pyloric thickness >4mm or length
4. BM or flatus present? – vomiting without BM may point to >14mm
obstruction. AGE can initially present with vomiting followed by • Barium: elongated pyloric channel, a bulge of the pyloric muscle
LBM. However, if patient has been vomiting for several days into the antrum (shoulder sign) & streaks of barium in the
without BM, think obstruction narrowed channel (double tract sign)
✔ GUIDE QUESTION
INTUSSUSCEPTION A 48-hr old boy born 34 wks AOG was noted to vomit greenish material
• Occurs when a portion of the alimentary tract is telescoped into after every feeding. PE: awake, alert, no respiratory distress, clear
an adjacent segment breath sounds, distinct heart sounds, non-distended abdomen with
hyperactive bowel sounds. What is your impression?
• Most common cause of intestinal obstruction between 3 months A. Volvulus C. Pyloric Stenosis
– 6 years old; M > F B. Malrotation D. Duodenal Atresia
• Unknown cause in most cases
• Correlation with Adenovirus DUODENAL ATRESIA
CLINICAL MANIFESTATIONS FACTS ABOUT DUODENAL ATRESIA:
• Severe paroxysmal colicky pain that recurs at frequent intervals • Due to failure to recanalize the lumen during the 4th-5th week of
with straining efforts gestation
• legs & knees are flexed with loud crying • 25-40% of all intestinal atresias
• 60% of infants pass currant jelly stool • 50% are premature
• obstruction usually distal to the ampulla of Vater
PE FINDINGS • may have other congenital anomalies
• Palpation of abdomen: slightly tender sausage-shaped mass in • Hallmark: bilious vomiting without abdominal distention
the RUQ which may increase in size & firmness during a usually noted on the 1st day of life
paroxysm of pain • Polyhydramnios in 50% due to failure of absorption of amniotic
• Child looks well in between the paroxysms of pain fluid in the distal intestine
• If not reduced → shock-like state • Jaundice in 1/3 of patients
• Dx: double-bubble sign on plain abdominal X-ray due to
distended & gas-filled stomach & proximal duodenum
• Tx: NGT/OGT decompression with IV fluid replacement; surgery
– Duodenoduodenostomy with gastrostomy tube
TOPNOTCH MEDICAL BOARD PREP PEDIATRICS MAIN HANDOUT BY DRS. DE VERA AND PUNONGBAYAN Page 29 of 106
For inquiries visit www.topnotchboardprep.com.ph or email us at topnotchmedicalboardprep@gmail.com
This handout is only valid for October 2022 PLE batch. This will be rendered obsolete for the next batch since we update our handouts regularly.
TOPNOTCH MEDICAL BOARD PREP PEDIATRICS MAIN HANDOUT BY DRS. DE VERA AND PUNONGBAYAN
For inquiries visit www.topnotchboardprep.com.ph or https://www.facebook.com/topnotchmedicalboardprep/
This handout is only valid for October 2022 PLE batch. This will be rendered obsolete for the next batch since we update our handouts regularly.
• Absence of Meissner & Auerbach plexus & hypertrophied
bundles with high concentrations of acetylcholinesterase
between the muscular & submucosa layers
o Decreased motility in the affected bowel segment
o Lack of propagation of peristaltic waves into the aganglionic colon
o Abnormal or absent relaxation of this segment and of the
internal anal sphincter

Duodenal atresia Duodenostomy


Double bubble sign
✔ GUIDE QUESTIONS
A 3-year-old boy presents with chronic constipation since he was an
infant. You find a boy who looks active and playful. His abdomen was
globular, non-tender and rectal exam revealed a normal sphincteric
tone and no feces on the examining finger. What is your most likely
impression?
A. Hirschsprung Disease C. Meckel Diverticulum
B. Functional Constipation D. Duodenal Atresia
From the case above, what part of the digestive tract is most probably
PATHOGENESIS
affected?
A. Duodenum • Symptoms usually begin at birth with the delayed passage of
B. Large Intestine meconium
C. Small Intestine • Some present with history of chronic constipation
D. Recto-sigmoid • Failure to pass stool → dilatation of the proximal bowel &
What is the definitive diagnostic tool for this disease? abdominal distention → ↑ intraluminal pressure → ↓ blood flow
A. Upper GI Series & deterioration of mucosal barrier → bacteria proliferates →
B. Barium Enema
enterocolitis
C. Biopsy
D. Clinical DIAGNOSIS
• Dx: rectal suction biopsy – gold standard (easy and reliable)
HIRSCHSPRUNG DISEASE • Abdominal X-ray:
FACTS ABOUT HIRSCHSPRUNG DISEASE o transition zone between normal dilated proximal colon
• Congenital aganglionic megacolon o a smaller caliber obstructed distal colon due to nonrelaxation
• Most common cause of lower intestinal obstruction in neonates; M > F of the aganglionic bowel
• Arrest of neuroblast migration from the proximal to distal bowel MANAGEMENT
– absence of ganglion cells in the bowel wall beginning in the • Treatment:
internal anal sphincter o a temporary colostomy & wait until the infant is 6-12 mos old
• Dominant & recessive patterns seen to perform definitive repair
• RET genes on chr 10q11 & the EDNRB gene on chr 13q22 o definitive procedure (Swenson, Duhamel, Boley endorectal
• Aganglionic segment limited to the rectosigmoid in 80% pull-through via laparoscopy)
Patients presenting as obstruction
• Can’t pass NG tube
• Omega sign or Coffee bean sign on AXR Volvulus
• Severe pain and emesis
• Sausage-shaped RUQ mass / absence of
• Bloody Currant jelly stools bowel sounds on RLQ Intussusception
Abdominal
• Coiled-spring sign
pain
• Post-prandial vomiting, non-bilious • Olive-shaped mass
Vomiting
• Abdominal distention • Barium studies: Shoulder sign, Double Pyloric stenosis
± Distention
• Down’s syndrome Tract Sign
• Normal history or recurrent obstructive symptoms
• Painless rectal bleeding • Scintigraphy scan to detect gastric tissue Meckel Diverticulum
• Intermittent pain
✔ GUIDE QUESTION
A three-year old male was brought to clinic for a chief complaint of
constipation. Mother noted child to be defecating only once a week in
the last 4 months. Patient was noted to be irritable at times with
decrease in appetite. There was no weight loss, failure to thrive, nor
abdominal distention. The child cries when he passes stool which was
observed to be seldomly blood streaked. Mother was also concerned
that his child was already toilet trained and yet there would be times
that the boy would stool in his underwear while attending daycare.
According to the Rome criteria for functional constipation in children
less than 4 years of age, how many criteria must be met in order to
diagnose functional constipation?
A. 1
Radiographic findings in patients presenting with obstruction B. 2
Leftmost – coffee bean/omega sign; Mid upper – bird's beak sign; C. 3
Mid lower – coiled spring sign; right most – double bubble sign D. 4
SUPPLEMENT: QUICK SHEET
MECKEL DIVERTICULUM RULE OF 2 FUNCTIONAL CONSTIPATION
• 2% of the population is affected • Constipation – delay or difficulty in defecation present for 2 wk
• 2 yrs old on presentation (usually) or longer and significant enough to cause distress to the patient.
• 2 times more females are affected • Differentiate from organic causes with a GOOD history and
• 2 inches long physical examination
• 2 feet from the ileocecal valve • Usually starts AFTER the neonatal period
• 2 types of common ectopic tissue (gastric and pancreatic) • Vicious cycle
• Does not apply 2 everyone! • Hard stool → anal irritation → stool retention → Hard Stool

TOPNOTCH MEDICAL BOARD PREP PEDIATRICS MAIN HANDOUT BY DRS. DE VERA AND PUNONGBAYAN Page 30 of 106
For inquiries visit www.topnotchboardprep.com.ph or email us at topnotchmedicalboardprep@gmail.com
This handout is only valid for October 2022 PLE batch. This will be rendered obsolete for the next batch since we update our handouts regularly.
TOPNOTCH MEDICAL BOARD PREP PEDIATRICS MAIN HANDOUT BY DRS. DE VERA AND PUNONGBAYAN
For inquiries visit www.topnotchboardprep.com.ph or https://www.facebook.com/topnotchmedicalboardprep/
This handout is only valid for October 2022 PLE batch. This will be rendered obsolete for the next batch since we update our handouts regularly.
• Encopresis is common TREATMENT
o voluntary or involuntary passage of feces into inappropriate • Patient education
places at least once a month for 3 consecutive months once a o Toilet training
chronologic or developmental age of 4 years has been reached o Dietary modification
To simplify functional constipation. It’s a behavioral disorder. Where the • Relief of impaction
child thinks that it is painful to poo and therefore avoid defecating. This • Stool softener
causes problem of impaction, hard stools thus a vicious cycle. To address o Polyethylene glycol
this the patient should be toilet trained, this will only be possible if pain is o Lactulose
removed. Thus, decompaction and stool softeners are mainstay. o Mineral oil
Lactulose is given for around 4-6 months. Common pitfall of some o Prolonged stimulants (Senna or Bisacodyl) should be
practitioners is to give stool softeners as PRN basis, this will not break the
AVOIDED
cycle.
Dr. De Vera
CHRONIC CONSTIPATION: ROME III CRITERIA ✔ GUIDE QUESTION
Adapted from Nelson Textbook of Pediatrics, 20th ed.
A 14-year-old female started having foul smelling watery soft stools in
INFANTS AND TODDLERS the morning. She noticed this after she started taking breakfast cereal
• Must include 1 mo of at least 2 of the following in infants up to with milk. She says that she usually drinks coffee in the mornings and
4 years of age: is not accustomed to drinking milk. What is the most likely mechanism
o ≤2 Defecations per week of diarrhea in this case?
o ≥1 Episode of incontinence after the acquisition of toilet A. Secretory
training skills B. Osmotic
C. Exudative
o History of excessive stool retention
D. Inflammatory
o History of painful or hard bowel movements
o Presence of a large fecal mass in the rectum
DIFFERENCE BETWEEN OSMOTIC & SECRETORY DIARRHEA:
o History of a large -diameter stool that might obstruct the toilet
Accompanying symptoms may include irritability, decreased appetite, OSMOTIC SECRETORY
and/or early satiety. The accompanying symptoms disappear immediately Volume of stool <200 ml/day >200 ml/day
following passage of a large stool. Response to Diarrhea
Diarrhea stops
fasting continues
CHILDREN WITH A DEVELOPMENTAL AGE OF 4-18 YEARS
Stool Na <70 mEq/L >70 mEq/L
• Must include 2 or more of the following in a child with a
developmental age of at least 4 years with insufficient criteria Reducing
positive negative
substances
for diagnosis of irritable bowel syndrome*:
o ≤2 Defecations per week Stool pH <5 >6
o ≥1 Episode of fecal incontinence per week
o History of retentive posturing or excessive volitional stool
retention MECHANISMS
o History of painful or hard bowel movements OF DIARRHEA
o Presence of a large fecal mass in the rectum https://qrs.ly/8tduggc
o History of a large-diameter stool that might obstruct the toilet

MECHANISMS OF DIARRHEA
Adapted from Nelson Textbook of Pediatrics, 20th ed.

PRIMARY
DEFECT STOOL EXAMINATION EXAMPLES COMMENT
MECHANISM
Cholera, toxigenic Escherichia coli; Persists during fasting; bile
Decreased
Watery, normal carcinoid, VIP, neuroblastoma, salt malabsorption can
absorption, increased
Secretory osmolality with ion gap < congenital chloride diarrhea, also increase intestinal
secretion, electrolyte
100 mOsm/kg Clostridium difficile, water secretion; no stool
transport
cryptosporidiosis (AIDS) leukocytes
Maldigestion, Watery, acidic, and Stop with fasting;
transport defects reducing substances; Lactase deficiency, glucose- increased breath hydrogen
Osmotic ingestion of increased osmolality galactose malabsorption, with carbohydrate
unabsorbable with ion gap > 100 lactulose, laxative abuse malabsorption; no stool
substances mOsm/kg leukocytes
Loose to normal-
Irritable bowel syndrome, Infection can also
Increased Decreased transit appearing stool,
thyrotoxicosis, postvagotomy contribute to increased
motility time stimulated by gastrocolic
dumping syndrome motility
reflex
Defect in neuromuscular
Decreased Loose to normal- Possible bacterial
unit(s) stasis Pseudoobstruction, blind loop
motility appearing stool overgrowth
(bacterial overgrowth)
Decreased
Might require elemental
surface area Decreased functional Short bowel syndrome, celiac
Watery diet plus parenteral
(osmotic, capacity disease, rotavirus enteritis
alimentation
motility)
Inflammation,
Salmonella, Shigella infection;
Mucosal decreased colonic Blood and increased Dysentery evident in
amebiasis; Yersinia,
invasion reabsorption, WBCs in stool blood, mucus, and WBCs
Campylobacter infection
increased motility

✔ GUIDE QUESTION FOREIGN BODY INGESTION


A 3-year-old girl presents with sudden onset of dysphagia. No other
symptoms noted. Her older brother witnessed her to have accidentally OTHER IMPORTANT POINTS:
swallowed a round object 1 hr PTC. PE: well-nourished, coherent, • 30% may be asymptomatic
fearful-looking, slightly tachycardic, clear & equal breath sounds, soft & • Initial Sx: choking, gagging, coughing followed by excessive
non-tender abdomen? What is your impression? salivation, dysphagia, food refusal, emesis, pain in the neck or
A. Asthma throat
B. Foreign body Aspiration • If oropharynx or proximal esophagus is perforated: cervical
C. Foreign body Ingestion
swelling, erythema, subcutaneous crepitations
D. None of the above

TOPNOTCH MEDICAL BOARD PREP PEDIATRICS MAIN HANDOUT BY DRS. DE VERA AND PUNONGBAYAN Page 31 of 106
For inquiries visit www.topnotchboardprep.com.ph or email us at topnotchmedicalboardprep@gmail.com
This handout is only valid for October 2022 PLE batch. This will be rendered obsolete for the next batch since we update our handouts regularly.
TOPNOTCH MEDICAL BOARD PREP PEDIATRICS MAIN HANDOUT BY DRS. DE VERA AND PUNONGBAYAN
For inquiries visit www.topnotchboardprep.com.ph or https://www.facebook.com/topnotchmedicalboardprep/
This handout is only valid for October 2022 PLE batch. This will be rendered obsolete for the next batch since we update our handouts regularly.
WHEN IS ENDOSCOPY INDICATED? CLINICAL MANIFESTATIONS
• Sharp objects, disc button batteries, or FBs with respiratory • Abdominal pain (epigastric and may radiate to the back, steady,
symptoms mandate urgent removal appears acutely ill), persistent vomiting, fever
• Failure to visualize object + symptomatic = URGENT endoscopy • Pain increases in intensity for 24-48 hrs with vomiting
Additional reminder… • Severe acute type: rare in children; severe nausea, vomiting,
• Asymptomatic: blunt objects may be observed up to 24 hrs in shock, jaundice, high fever, Cullen sign, Grey Turner sign,
anticipation of passage into the stomach necrotic pancreas; mortality rate of 50%

Criteria for the diagnosis of pancreatitis in children


CAUSTIC INGESTIONS
• 2 of 3 of the following:
• Ingestion results in esophagitis, necrosis, perforation & stricture o abdominal pain
formation o serum amylase and/or lipase activity at least 3 times greater
• Liquid alkali produces liquefaction necrosis than the upper limit of normal
• Acidic agents are bitter so less may be ingested – coagulation o and imaging findings characteristic of, or compatible with,
necrosis acute pancreatitis.
• Vomiting, drooling, refusal to drink, oral burns, dysphagia,
abdominal pain AMYLASE VS LIPASE?
• Circumferential ulcers, white plaques, sloughing of mucosa • Serum amylase level typically elevated for up to 4 days
• Serum lipase is more specific than amylase for acute
• Absence of oropharyngeal inflammatory pancreatic disease
lesions does not exclude the o Rises by 4-8 hrs, peaks at 24-48 hrs, remains elevated 8-14
possibility of significant days LONGER than serum amylase
esophagogastric injury!
• Circumferential ulcers, white OTHER LABORATORY FINDINGS:
plaques, sloughing of mucosa • Hemoconcentration
→ increased risk of strictures • Coagulopathy
Oral lesions (plaques) after caustic ingestion • Leukocytosis
• Hyperglycemia
Severity of esophageal injury post-caustic ingestion as graded • Hyperbilirubinemia
by endoscopic appearance of esophageal mucosa • High glutamyl transpeptidase
• Grade 0: normal mucosa IMAGING STUDIES:
• Grade 1: erythema • Abdominal x ray:
• Grade 2: erythema, sloughing, ulceration, and exudates (not o Sentinel loop
circumferential) o Dilated transverse colon (cutoff sign)
• Grade 3: deep mucosal sloughing and ulceration o Ileus
(circumferential) o Blurring of the left psoas margin
o Peripancreatic extraluminal gas bubbles
• Grade 4: eschar, full thickness injury, and perforation

MANAGEMENT
• Upper endoscopy – for all symptomatic patients to rapidly
identify tissue damage
• Dilution by water/milk is recommended.
• Neutralization, induced emesis & gastric lavage are
contraindicated.
• Surgical resection: dilation of strictures, SI or colon interposition

✔ GUIDE QUESTION
A 10-year-old girl was diagnosed to have systemic viral infection 1-
week PTC. She was managed conservatively. 2 days PTC, she had
epigastric pain that increased in intensity associated with 6 vomiting
episodes for the past few hours. PE: febrile, toxic-looking, flat, tensed
Distention of the colon to the Sentinel loop – a focal dilated
abdomen with tenderness radiating to the back. What is your treatment
transverse colon with a paucity of proximal jejunal loop in the left
of choice for this patient?
gas distal to the splenic flexures upper quadrant
A. Exploratory Laparotomy with appendectomy
B. NPO, TPN, Antibiotics, Analgesics • Abdominal ultrasound and CT scan:
C. Analgesics Alone o Pancreatic enlargement
D. Barium decompression o Hypoechoic, sonolucent, edematous
o Pancreatic masses
o Fluid collections
ACUTE PANCREATITIS o Abscess
• Most common pancreatic disorder in children *20% of children may have normal findings
• Common causes:
o blunt abdominal injury TREATMENT
o mumps & other viral illnesses • Medical mgt: to relieve pain and to restore metabolic
o multisystem disease homeostasis
o congenital anomalies • Analgesia
o biliary microlithiasis • Fluid electrolyte & mineral balance restored & maintained
o drugs & toxins • Prophylactic antibiotics useful in severe cases to prevent
infected pancreatic necrosis
PATHOGENESIS • Refeed when vomiting has resolved, serum amylase is falling,
• Theory: following an insult, lysosomal hydrolase co-localizes and clinical symptoms are resolving
with pancreatic proenzymes within the acinar cell → • Endoscopic therapy – if due to anatomic abnormalities
pancreastasis with continued synthesis of enzymes occur → (strictures or stones)
proenzymes are activated by cathepsin leading to auto-digestion • Severe acute type: enteral or TPN, antibiotics, gastric acid
→ further activation & release of active proteases → lecithin is suppression, peritoneal lavage
activated by Phospholipase A2 into the toxic lysolecithin

TOPNOTCH MEDICAL BOARD PREP PEDIATRICS MAIN HANDOUT BY DRS. DE VERA AND PUNONGBAYAN Page 32 of 106
For inquiries visit www.topnotchboardprep.com.ph or email us at topnotchmedicalboardprep@gmail.com
This handout is only valid for October 2022 PLE batch. This will be rendered obsolete for the next batch since we update our handouts regularly.
TOPNOTCH MEDICAL BOARD PREP PEDIATRICS MAIN HANDOUT BY DRS. DE VERA AND PUNONGBAYAN
For inquiries visit www.topnotchboardprep.com.ph or https://www.facebook.com/topnotchmedicalboardprep/
This handout is only valid for October 2022 PLE batch. This will be rendered obsolete for the next batch since we update our handouts regularly.
PROGNOSIS TIME
HBsAg Anti-HBS Anti-HBc HBeAg
• Uncomplicated cases: recover over 2-5 days PERIOD
Incubation
• If associated with trauma or systemic disease: prognosis is
related to the associated medical conditions Period + - - +
• Ranson criteria and APACHE score – prognostic systems used in Acute
infection + - + IgM +
adults are inappropriate for use in children
Window
Period - - +IgM -
✔ GUIDE QUESTION
Complete
A 15-year-old sexually active male came in due to history of fever, chills
Recovery - + +IgG -
and right upper quadrant abdominal pain. Upon physical examination
you noted icteric sclera, hepatomegaly and jaundice. You suspect an Chronic
acute hepatitis B infection. What test would you expect to find? Carrier + - +IgG -
A. HBsAg (+), Anti-HBc (+), IgM Anti-HBc (+), Anti-HBs (–) Chronic
B. HBsAg (+), Anti-HBc (+), IgM Anti-HBc (-), Anti-HBs (–) Active + - +IgG +
C. HBsAg (-), Anti-HBc (+), Anti-HBs (+)
D. HBsAg (-), anti-HBc (-), Anti-HBs (+)
Vaccinated - + - -

VIRAL HEPATITIS NEPHROLOGY


• Identified Antibodies: When assessing patient with edema that is related to the kidneys, one
o Anti-HAV, IgM anti-HAV approach is to classify the patient as having nephritic VS nephrotic
o Anti-HBsAg, IgM anti-HBsAg, anti-HBcAg, anti-HBeAg syndrome.
Dr. De Vera
o Anti-HCV
o Anti-HDV SUPPLEMENT: QUICK SHEET
o Anti-HEV, IgM anti-HEV Nephritic vs. Nephrotic Mnemonics
o Anti-HGV Nephritic – HOHA
Hypertension
HEPATITIS B ANTIGENS & ANTIBODIES Oliguria
Hematuria
• First clinical evidence of HBV infection is elevation of ALT levels
Azotemia
which occurs about 6-7 wks after exposure
• Anti-HBcAg – most valuable single serologic marker of acute
Nephrotic – PALE
HBV infection because it is present as early as HbsAg and
Proteinuria (nephrotic range)
continues to be present later in the course of the disease when
Albumin low
HBsAg has disappeared
Lipid High
• HBcAg – inner portion of the virion that encodes the viral DNA
Edema
• HBeAg – serves as a marker of active viral replication;
identification of infected people at increased risk of transmitting ✔ GUIDE QUESTION
HBV The most common cause of gross hematuria in pediatric population is?
• HBsAg – first serologic marker to appear & its rise coincides A. PSGN C. Malignancy
B. Trauma D. Urinary Tract Infection
with the onset of symptoms; detection of acutely or chronically
infected people; antigen used in hepatitis B vaccine HEMATURIA
• Anti-HBs – identification of people who have resolved infections
• More than 5 RBCs/hpf (centrifuged)
with HBV; determination of immunity after immunization
o RBC morphology
• Anti-HBe – identification of infected people with lower risk of
• Dipstick (based on the peroxidase reaction with hemoglobin)
transmitting HBV
• Pathophysiology: RBC casts formed when they enmesh in protein
• Anti-HBc – identification of people with acute, resolved, or
matrix
chronic HBV infection
• False negative hematuria
• IgM anti-HBc – identification of people with acute or recent HBV
o Formalin
infections (including HBsAg-negative people during the
o High urinary ascorbic acid
“window” phase of infection)
• False positive
HBsAg Negative o Alkaline urine (pH>8)
anti-HBc Negative Susceptible o Hydrogen peroxide
anti-HBs Negative • Presence of RBC is suggested by >10 RBC/uL or +1 dipstick in a
HBsAg Negative freshly voided and centrifuged urine
anti-HBc Positive Immune due to natural infection
anti-HBs Positive IMPORTANT POINTS
HBsAg Negative
Immune due to hepatitis B • Grossly red urine with or without blood clots – from the lower
anti-HBc Negative
vaccination urinary tract
anti-HBs Positive
HBsAg Positive
• Brown-colored / tea-colored urine – glomerular in origin
anti-HBc Positive • Cloudy urine – can be normal; due to crystal formation at room
Acutely infected temp.
IgM anti-HBc Positive
anti-HBs Negative • Uric acid crystals form in acidic urine
HBsAg Positive • Phosphate crystals form in alkaline urine
anti-HBc Positive • Normal specific gravity – 1.015-1.025
Chronically infected
IgM anti-HBc Negative • Falsely high SG if with glucose, abundant protein and with
anti-HBs Negative iodine-containing contrast materials
Interpretation unclear; four
possibilities: SOURCES OF HEMATURIA?
HBsAg Negative 1. Resolved infection (most common) • Glomerular – brown, cola, tea, burgundy colored urine.
anti-HBc Positive 2. False-positive anti-HBc, thus Proteinuria >100mg/dl on dipstick, RBC casts, dysmorphic
anti-HBs Negative susceptible
RBCs (acanthocytes)
3. “Low level” chronic infection
4. Resolving acute infection
• Tubular system – leukocytes or renal tubular cast
• Lower urinary tract – gross hematuria, terminal hematuria,
blood clots, normal morphology, minimal proteinuria
<100mg/dl on dipstick
Three things are very useful when comparing lower tract vs glomerular
hematuria. Protein content, color, and morphology.
Dr. De Vera

TOPNOTCH MEDICAL BOARD PREP PEDIATRICS MAIN HANDOUT BY DRS. DE VERA AND PUNONGBAYAN Page 33 of 106
For inquiries visit www.topnotchboardprep.com.ph or email us at topnotchmedicalboardprep@gmail.com
This handout is only valid for October 2022 PLE batch. This will be rendered obsolete for the next batch since we update our handouts regularly.
TOPNOTCH MEDICAL BOARD PREP PEDIATRICS MAIN HANDOUT BY DRS. DE VERA AND PUNONGBAYAN
For inquiries visit www.topnotchboardprep.com.ph or https://www.facebook.com/topnotchmedicalboardprep/
This handout is only valid for October 2022 PLE batch. This will be rendered obsolete for the next batch since we update our handouts regularly.
✔ GUIDE QUESTIONS SUPPLEMENT: QUICK SHEET
A 10-year-old male patient comes to the clinic for swelling. His mother IMPORTANT POINT
noticed that he had been having swelling especially of the lower Best single Ab titer to document skin infection is DNAse B
extremities. She also noticed him to be weak and sometimes lethargic.
antigen
He had an infected skin infection a month ago which resolved with
unrecalled antibiotics. Physical exam revealed BP 130/90, HR 102, RR
24, T 36.5°C, (+) bipedal edema, clear breath sounds bilaterally, non- What are the only causes of Renal Insufficiency that cause
hyperemic posterior pharynx, tonsils not enlarged, Urinalysis: revealed decreased C3?
RBC 25-30, protein +1, bacteria +1, epithelial cells +1, with RBC casts, 1. PSGN
serum creatinine is within normal limits, what is the most likely 2. MPGN
diagnosis in this patient? 3. Lupus Nephritis
A. Minimal change disease
B. Post-streptococcal glomerulonephritis
C. Membranoproliferative glomerulonephritis Indications for Renal Biopsy in APSGN
D. Membranous glomerulonephritis • Acute renal failure
Ask yourself, is this nephrotic or nephritic? This is definitely nephritic. • Nephrotic syndrome
Hypertension and a history of pyoderma. Classic PSGN. Some might • Absence of evidence of strep infection
be misled by the protein +1. RBCs in urine causes false positive • Normal complement level
proteinuria. • Hematuria and proteinuria
Dr. De Vera
A 10-year-old male patient started having headaches and blurring of
• Low C3 that persists more than 2 months
vision. He is brought to this clinic by his mother. Initial examination In both nephrotic and nephritic, we do biopsy when the picture does not
revealed bipedal edema but no symptoms of dysuria, no abdominal perfectly fit the most common causes (i.e. PSGN, MCN). So, for PSGN, this
pain. The mother recalls that the patient had cough and sore throat 1 should be predominantly nephritic, if we have nephrotic features,
week ago. BP is 150/90, HR 106, RR 20 T 36.9°C, which among the ff. persistently low C3, and other features listed above, we do biopsy.
is an expected laboratory finding? Dr. De Vera

A. Increased C3 C. Increased C5
B. Decreased C3 D. Decreased C5 ALPORT SYNDROME (HEREDITARY NEPHRITIS)
You have to understand how to use C3. C3 is a dead giveaway. If C3 is • Bilateral sensorineural hearing loss (never congenital)
decreased and you are considering glomerulonephritis in children, • Ocular abnormalities (30-40%) such as anterior lenticonus (its
almost always that is PSGN (C3 should normalize after 8 weeks).
presence is pathognomonic), macular flecks, & corneal erosions
Other two choices are MPGN and SLE which presents with mixed
nephrotic-nephritic picture. • Critical in the diagnosis: family history, screening urinalysis of
Dr. De Vera 1st-degree relatives, audiogram, ophthalmologic exam
In relation to the above case, all of the ff. statements are true regarding • Highly likely: (+) hematuria and at least 2 of the ff: macular
the patient’s condition above except? flecks, recurrent corneal erosions, GBM thickening or thinning,
A. complete recovery occurs in 95% of children
sensorineural deafness
B. early systemic antibiotic therapy does not eliminate the risk of
this disease
C. administering antibiotics can alter the natural history of this disease PROGNOSIS AND TREATMENT
D. serum complement level is usually low in this immune-complex • NO specific therapy is available
mediated disease • ACE inhibitors can slow the rate of progression
• Manage renal failure complications such as hypertension,
APSGN anemia, electrolyte imbalance
• Age group: 5-12 years old. Rare before 3 years old • ESRD occurs before 30 years old in 75% of hemizygotes with X-
• Clinical spectrum from asymptomatic to renal failure linked AS
• Documentation of prior strep infection • Patients with ESRD are treated with dialysis and kidney
o Throat – ASO transplantation
o Skin – antideoxyribonuclease B
• Most commonly sporadic THIN BASEMENT MEMBRANE DISEASE
o Serotype following throat infection: 1, 2, 4 , 12, 18 25 • Presence of persistent microscopic hematuria and isolated
o Serotype following skin infection: 49, 55, 57, 60 thinning of the GBM on E/M
• Clinical manifestation: patient develops nephritic syndrome • Often initially observed in childhood and is intermittent
o How many weeks after throat infection? 1-2 • Isolated hematuria in multiple family members without renal
o How many weeks after skin infection? 3-6 dysfunction is termed as benign familial hematuria
• When will the abnormalities resolve and normalize? • Important to monitor for hypertension, progressive proteinuria,
o Acute phase : 6-8 weeks or renal insufficiency
o Urinary protein excretion and hypertension: 4-6 weeks ✔ GUIDE QUESTIONS
o Microscopic hematuria : 1-2 years A 4-year-old male patient was brought to the clinic because of swelling.
o C3 level : 6-8 weeks The mother started to notice it 3 days ago initially on the legs. She also
• What is the pathogenesis of PSGN? started to notice it that her child had puffy eyelids whenever he wakes
o Ab is produced & combines with a circulating Ag unrelated to up in the morning and decreases throughout the day. There was no
the kidney → IC accumulate in glomeruli & activate the fever or abdominal pain, (+) anorexia, PE revealed: HR 101, RR 26,
complement system → immune injury T>36.7°C, (+) bipedal edema, (-) murmurs, clear bilateral breath
sounds, urinalysis was done which revealed +4 proteinuria, RBC 0-1,
• What hypothesis is given to explain the predilection to
bacteria +1, serum creatinine within normal limits, 24-hour urinary
damage the glomeruli? protein is 4g/24hr, what is the next best step for this patient?
o Localized in the glomeruli due to negatively charged capillary A. Treat with steroids
wall, mesangial trapping, hydrodynamic forces B. Renal biopsy
• Why does serum C3 decrease? C. Renal ultrasound
o Inflammatory reaction follows immunologic injury due to D. Urine culture and sensitivity
activation of classic and alternative pathways which converge In relation to the case above, after 8 weeks of steroid therapy, patient’s
at C3 → lysis of cell membranes → hypocomplementemia urinary protein is +3, what is the next best step in the management of
this patient?
• What major noxious products of complement activation are A. Restart treatment with steroids
produced after C3 activation? B. Renal biopsy
o anaphylatoxin (increases vascular permeability) & C5a C. Renal ultrasound
(release substances that damage cells & basement D. Abdominal CT scan
membranes) E. Treat with cyclophosphamide
In relation to the case above, what is the most likely diagnosis in this
MANAGEMENT: patient?
• Fluid restriction A. Membranous glomerulonephritis
B. Minimal change disease
• Diuretics (Furosemide) C. Mesangial proliferative
• BP control D. Focal segmental glomerulosclerosis
TOPNOTCH MEDICAL BOARD PREP PEDIATRICS MAIN HANDOUT BY DRS. DE VERA AND PUNONGBAYAN Page 34 of 106
For inquiries visit www.topnotchboardprep.com.ph or email us at topnotchmedicalboardprep@gmail.com
This handout is only valid for October 2022 PLE batch. This will be rendered obsolete for the next batch since we update our handouts regularly.
TOPNOTCH MEDICAL BOARD PREP PEDIATRICS MAIN HANDOUT BY DRS. DE VERA AND PUNONGBAYAN
For inquiries visit www.topnotchboardprep.com.ph or https://www.facebook.com/topnotchmedicalboardprep/
This handout is only valid for October 2022 PLE batch. This will be rendered obsolete for the next batch since we update our handouts regularly.
PHYSICAL EXAM /
HISTORY DIAGNOSIS
DIAGNOSTICS
History of hemoptysis and severe
Goodpasture syndrome
respiratory distress
Patient presenting with Purpuric rash most
hematuria, hypertension, With SVI 3 weeks ago prominent at the buttocks HSP
and signs of renal Pain in the joints
insufficiency History of vomiting, bloody diarrhea,
Dehydrated hepato-
and fever HUS
splenomegaly
History of eating hamburger
Sudden onset of gross Patient is febrile with LRTI or GIT infection Normal C3 level IgA Nephropathy
hematuria History of Strep throat or pyoderma Severely depressed C3 level APSGN
Child discovered to have Patient wears very thick glasses and
asymptomatic microscopic wear hearing aid
Normal labs Alport Syndrome
hematuria during yearly Child has uncles with the same
check-up at school condition

SUMMARY OF PRIMARY RENAL DISEASES THAT MANIFEST AS ACUTE GLOMERULONEPHRITIS


Adapted from Nelson Textbook of Pediatrics, 20th ed.
IDIOPATHIC RAPIDLY
POSTSTREPTOCOCCAL GOODPASTURE
DISEASES IgA NEPHROPATHY PROGRESSIVE
GLOMERULONEPHRITIS SYNDROME
GLOMERULONEPHRITIS
CLINICAL MANIFESTATIONS
All ages, mean 7 years, 2:1
Age and sex 10-35 yr. 2:1 male 15-30 years, 6:1 male Adults, 2:1 male
male
Acute nephritic
90% 50% 90% 90%
syndrome
Asymptomatic
Occasionally 50% Rare Rare
hematuria
Nephrotic syndrome 10-20% Rare Rare 10-20%
Hypertension 70% 30-50% Rare 25%
Acute renal failure 50% (transient) Very rare 50% 60%
Pulmonary
Follows viral
Other Latent period of 1-3 wk hemorrhage; iron None
syndromes
deficiency anemia
↑ ASO titers (70%)
Positive anti-GBM
Laboratory findings Positive streptozyme (95%) ↑ Serum IgA (50%) Positive ANCA in some
antibody
↓C3-C9; normal C1, C4
Immunogenetics HLA-B12, D “EN” (9)* HLA-Bw 35, DR4 (4)* HLA-DR2 (16)* None established
RENAL PATHOLOGY
Focal → diffuse
Light microscopy Diffuse proliferation Focal proliferation proliferation with Crescentic GN
crescents
Immunofluorescence Granular IgG, C3 Diffuse mesangial IgA Linear IgG, C3 No immune deposits
Electron microscopy Subepithelial humps Mesangial deposits No deposits No deposits
95% resolve spontaneously 75% stabilize or
Slow progression in 75% stabilize or improve if
Prognosis 5% RPGN or slowly improve if treated
25-50% treated early
progressive early
Uncertain (options Plasma exchange,
Treatment Supportive include steroids, fish steroids, Steroid pulse therapy
oil, and ACE inhibitors) cyclophosphamide

PROTEINURIA • Dipstick is POSITIVE if


o > trace in a sample with SG <1.01
• charge and size selective properties of the glomerular capillary
o >/= +1 if >1.015
wall prevent significant amounts of albumin, globulin, and other
• 24hr urine protein determination is more accurate
large plasma proteins from entering the urinary space
o Normal is </= 4mg/m2/hr
• 3 mechanisms
o Abnormal 4-40mg/m2/hr
o Glomerular proteinuria – ineffective filtration
o Nephrotic range > 40mg/m2/hr
o Tubular Proteinuria – ineffective reabsorption
• Urine protein: creatinine ratio
o Increase plasma protein – ex. rhabdomyolysis
o <0.5 in <2 years old is normal
• Upper limit in healthy children is 150mg/day
o <0.2 in >2 years old is normal
• Urine dipstick
o >2 is nephrotic range proteinuria
o Negative
o Trace = 10-29mg/dl The cardinal feature that we look for in nephrotic syndrome is
o 1+ = 30mg/dl proteinuria. Memorize the cut-off for nephrotic range proteinuria either
24hr value OR UPCR (urine protein:crea ratio). If PSGN is for nephritic,
o 2+ = 100mg/dl Minimal change disease is for nephrotic. It is the most common cause,
o 3+ = 300mg/dl treatment is steroids.
o 4+ = >2g/dL Dr. De Vera

• Non-pathologic (<1000mg/day) ✔ GUIDE QUESTION


o Postural A 13-year-old / male came in for annual physical examination. The rest
o Febrile (>38.4°C) of the findings are normal except for a urinalysis with trace proteinuria
o Exercise – resolves after 48h on dipstick test. Mother is worried. You will advise the following
• False positive EXCEPT?
o Very high urine PH (>7) A. This is a benign condition. No evaluation or therapy is
necessary
o Highly concentrated urine B. Majority of children will have negative evaluations on repeated
o Contamination with blood dipstick test and protein quantification
• False negative C. This is more common in adolescents and younger children
o Dilute urine D. You will request for a UPCR
o Urine wherein the predominant protein is not albumin
TOPNOTCH MEDICAL BOARD PREP PEDIATRICS MAIN HANDOUT BY DRS. DE VERA AND PUNONGBAYAN Page 35 of 106
For inquiries visit www.topnotchboardprep.com.ph or email us at topnotchmedicalboardprep@gmail.com
This handout is only valid for October 2022 PLE batch. This will be rendered obsolete for the next batch since we update our handouts regularly.
TOPNOTCH MEDICAL BOARD PREP PEDIATRICS MAIN HANDOUT BY DRS. DE VERA AND PUNONGBAYAN
For inquiries visit www.topnotchboardprep.com.ph or https://www.facebook.com/topnotchmedicalboardprep/
This handout is only valid for October 2022 PLE batch. This will be rendered obsolete for the next batch since we update our handouts regularly.
CAUSES OF TRANSIENT PROTEINURIA NON-RESPONDERS
• Fever A. Steroid resistant
• Exercise B. Frequent relapser
• Dehydration C. Steroid dependent
• Cold exposure __________1. Relapse within 28 days of stopping prednisone
• Congestive heart failure therapy
• Seizure __________2. Patients who respond well to prednisone therapy
• Stress but relapse 4 or more times in a 12-month period
__________3. Children who fail to respond to prednisone therapy
✔ GUIDE QUESTION
within 8 weeks
Still worried, mother decided to have a second opinion a month later. 4 __________4. Relapse on alternate-day steroid therapy
Serial determination of urinalysis revealed SG 1.2 and ++ proteinuria. __________5. Children who continue to have proteinuria (+2 or
The following is true regarding this condition? greater) after 8 weeks of steroid therapy
A. This is still transient proteinuria
Answers: C B A C A
B. Further investigation should be done
C. No further investigation should be done • The major complication of nephrotic syndrome is infection.
D. Refer mother to psych • The most frequent type of infection is spontaneous bacterial
FIXED PROTEINURIA peritonitis.
• First morning void, 3 separate occasion • The most common organism causing peritonitis is
• Proteinuria ≥ 1+ with SG >1.015 pneumococcus.
• UPCR ≥ 2 Management
• Further evaluation needed • Edema – sodium restriction. Cautious Albumin and diuretics
• Indicates potential glomerular or tubular disorder • Dyslipidemia – dietary modification
• Infection – usually 3rd gen cephalosporin
✔ GUIDE QUESTION • Thromboembolism – anticoagulation if necessary
An 8-year-old female was brought to a physician because of swelling of
✔ GUIDE QUESTION
the face, arms and legs. A 24-hour collection of urine yielded 5 grams
of protein, rare red cells, and no red cell casts. The child was given A 13-year-old female patient presents to the clinic due to bipedal edema
corticosteroids. All symptoms and findings vanished, and the physician of 3 days duration; it is gradually progressive, she wakes up in the
concluded, even without renal biopsy, that the child most likely had: morning with puffy eyelids, urinalysis reveals RBC 8-10 with RBC casts,
A. Acute post-streptococcal glomerulonephritis protein +3, WBC 0-2 no fever or rash noted, patient is HbsAg (+), which
B. IgA nephropathy of Berger she says she got from her mother at birth. Biopsy reveals thickened
C. Nephritic syndrome glomerular capillary walls with splitting from the interposition of the
D. Minimal change disease mesangium, with C3 lobular deposits in immunofluorescence, what is
the most likely diagnosis?
A. FSGS
NEPHROTIC SYNDROME B. Minimal change disease
• Idiopathic Nephrotic Syndrome C. Membranous glomerulonephritis
o Minimal change disease D. Membranoproliferative glomerulonephritis
Where is the site of the pathology as seen in the electron microscope in
o Focal segmental glomerulosclerosis
the condition described above?
o Mesangial proliferation A. Subepithelial
• Glomerular diseases B. Subendothelial
o Membranous nephropathy C. Both A and B
o Membranoproliferative glomerulonephritis D. None of the above

NEPHROTIC SYNDROME (MINIMAL CHANGE DISEASE) PATHOLOGY CROSS-OVER


• The urinalysis reveals 3+ or 4+ proteinuria FINDINGS DISEASE
• Spot urine protein/creatinine ratio exceeds 2.0 and urinary LM: Diffuse thickening of
protein excretion exceeds 3.5 g/24 hr in adults and 40 glomerular capillary walls
mg/m2/hr in children. IF: Granular IgG and C3 Membranous
• The serum albumin level is generally <2.5 g/dL, and the serum EM: Sub-epithelial deposits of Glomerulonephritis
cholesterol and triglyceride levels are elevated. electron dense material
• C3 and C4 levels are normal. “Spike and Dome” appearance
• Renal biopsy is not required for diagnosis in most children. LM: normal
• >95% with MCNS respond to corticosteroid therapy EM: diffuse effacement of
Minimal Change Disease
• Pathogenesis epithelial foot process, no
o Increased permeability of glomerular capillary wall → deposits
proteinuria and hypoalbuminemia LM: Large, hypercellular
o Podocytes play a crucial role in filtering proteins glomeruli
Membranoproliferative
§ Effacement of foot process, decrease number of podocytes Increased mesangial matrix
Glomerulonephritis
and altered slit diaphragm integrity = LEAKY Membranes Double Contour or Tram-track
• Management of MCNS appearance
o Corticosteroid LM: Focal segmental sclerosis
o Tuberculosis must be first ruled out and hyalinosis Focal Segmental
o Uncomplicated nephrotic syndrome between 1-8 years old are EM: Loss of foot process, Glomerulosclerosis
likely to have steroid responsive MCNS epithelial denudation
§ Initiate steroids even without biopsy
o Do biopsy if with features less likely of MCNS ✔ GUIDE QUESTION
§ Gross hematuria, hypertension, renal insufficiency, The following are true regarding UTI in children EXCEPT?
hypocomplementemia, < 1 year old or > 12 years old A. It is more common in males during the first year of life
B. E. coli is one of the most common pathogens
o Prednisone or Prednisolone (60mg/BSA/day or C. Pyelonephritis may manifest only with fever
2mg/kg/day, max 60mg daily) for 4-6 weeks D. Involvement of the renal parenchyma is termed pyelitis
o Alternate day (40mg/BSA or 1.5mg/kg) for 8 weeks to 5
months
o Response is remission attained within initial 4 weeks
GLOMERULONEPHRITIS
o Remission is UPCR < 0.2 or < +1 protein on dipstick for 3
consecutive days vs. NEPHROTIC SYNDROME
o Relapse is UPCR >2 or ≥ +3 dipstick for 3 consecutive days https://qrs.ly/duduggt

TOPNOTCH MEDICAL BOARD PREP PEDIATRICS MAIN HANDOUT BY DRS. DE VERA AND PUNONGBAYAN Page 36 of 106
For inquiries visit www.topnotchboardprep.com.ph or email us at topnotchmedicalboardprep@gmail.com
This handout is only valid for October 2022 PLE batch. This will be rendered obsolete for the next batch since we update our handouts regularly.
TOPNOTCH MEDICAL BOARD PREP PEDIATRICS MAIN HANDOUT BY DRS. DE VERA AND PUNONGBAYAN
For inquiries visit www.topnotchboardprep.com.ph or https://www.facebook.com/topnotchmedicalboardprep/
This handout is only valid for October 2022 PLE batch. This will be rendered obsolete for the next batch since we update our handouts regularly.
GLOMERULONEPHRITIS VS NEPHROTIC IN CHILDREN DEFINITION OF TERMS (PPS GUIDELINES)
GLOMERULO- NEPHROTIC • 2 or more acute
NEPHRITIS SYNDROME Recurrent pyelonephritis/presumptive UTI OR
“HOHA” “PALE” UTI • 3 or more cystitis OR
• Hypertension • Proteinuria • 1 pyelonephritis + 1-2 cystitis
Clinical features • Oliguria • Albumin low • clinical symptoms + urinalysis findings
Presumptive
• Hematuria • Lipid high suggestive of UTI, NOT supported by urine
UTI
• Azotemia • Edema culture.
Disease commonly
Minimal change LABORATORY TESTS IN UTI (PPS GUIDELINES)
encountered in PSGN
disease • < 2 years old = clean catch urine collection
children
• >2 years old = mid-stream sample
Important Decrease C3 that 24hr urine
• Wee bag is only useful if NEGATIVE
diagnostic test normalizes in 8 weeks protein or UPCR Collection of
RESULTS
• Supportive Urine
• Suprapubic aspiration or urethral
• 95% will resolve
95% will catheterization if and only if above
• Monitor C3
Management respond to methods cannot be done properly
• Fluid limit
steroids URINE DIPSTICK
• Diuretics
• Nitrate reductase present in most gram-
• Anti-hypertensive negative uropathogenic rods
Nitrite
UTI IN CHILDREN • Requires sufficient bladder incubation
time (around 4 hours)
• Prevalence varies with age
o 1st year of life = male predominance • May be associated with UTI and non-
o Beyond 1-2 years old = female predominance infectious renal diseases
Leukocytes
• Primarily caused by colonic bacteria • False negative: glycosuria, high specific
o 75-90% caused by E. coli in girls, followed by Klebsiella and gravity, contamination with debris
Proteus Leukocyte + Leukocyte –
• 3 basic forms Send Urine
Nitrite Urine CS
o Pyelonephritis CS
+ Treat as UTI
o Cystitis Treat as UTI
Interpretation
o Asymptomatic Bacteuria Urine CS
• Nearly all are ascending infections Nitrite Treat IF
Not UTI
– WITH
DIFFERENT FORMS OF UTI IN CHILDREN SYMPTOMS
PYELONEPHRITIS URINE MICROSCOPY
• is characterized by any or all of the following: abdominal, back,
• WBC >5/HPF in centrifuged urine
or flank pain; fever; malaise; nausea; vomiting; and, occasionally, Pyuria
• WBC >10/uL in uncentrifuged
diarrhea.
• May be positive from contamination
• Fever may be the only manifestation.
Bacteria • Not significant unless accompanied by
• Newborns - nonspecific symptoms
presence of WBCs
• Most common serious bacterial infection in younger than 24
Pyuria + Pyuria –
months
Send Urine
CYSTITIS Urine CS
Bacteriuria CS
• Involvement of bladder Treat as
+ Treat as
• dysuria, urgency, frequency, suprapubic pain, incontinence and UTI
Interpretation UTI
malodorous urine Urine CS
• Cystitis does not cause fever and does not result in renal Injury Bacteriuria Treat IF
• Acute hemorrhagic cystitis – E. coli Not UTI
– WITH
• Eosinophilic cystitis SYMPTOMS
• Interstitial Cystitis – irritative voiding symptoms relieved by • Gold standard in a properly collected
voiding. Negative urine culture. Adolescent girls. Idiopathic sample
ASYMPTOMATIC BACTERIURIA • Indications:
• Positive urine culture without any manifestation of infection Urine Culture o Significant urinalysis findings
• Most common in girls o Complicated UTI
• Does not cause renal injury except in pregnant women (can o Seriously ill children
result in symptomatic UTI) o Recurrent UTI
KUB • Indications
RISK FACTORS FOR UTI Ultrasound o Culture proven pyelonephritis
• Female gender • Tight clothing (underwear) with post void o Febrile presumptive UTI
• Uncircumcised male • Pinworm infestation study o Recurrent UTI
• Vesicoureteral reflux* • Constipation • VCUG to check for reflux
• Toilet training • Bacteria with P fimbriae Other Test • DMSA renal scan
• Voiding dysfunction • Anatomic abnormality • Acute phase reactants: Procalcitonin
• Obstructive uropathy (labial adhesion)
• Urethral instrumentation • Neuropathic bladder MANAGEMENT OF UTI
• Wiping from back to front • Sexual activity NELSON’S
in girls • Pregnancy Co-trimoxazole •
• Bubble bath? Nitrofurantoin Acute Cystitis •
✔ GUIDE QUESTION Amoxicillin – high rate of resistance •
The following are true regarding the diagnosis of UTI in children EXCEPT Ceftriaxone •
A. Urine culture is necessary Cefotaxime •
B. In toilet trained children, midstream urine sample is usually Acute
Ampicillin + Aminoglycoside •
satisfactory
C. In children who are not toilet trained, a catheterized or
Pyelonephritis
Oral 3rd generation cephalosporin like •
suprapubic aspirate should be obtained Cefixime is as effective but does not
D. Use of adhesive, sealed, sterile collection bag after cover pseudomonas
disinfection is useful only if with a negative culture • Aminoglycoside
E. NOTA Pseudomonas
• Fluoroquinolone (ciprofloxacin)
TOPNOTCH MEDICAL BOARD PREP PEDIATRICS MAIN HANDOUT BY DRS. DE VERA AND PUNONGBAYAN Page 37 of 106
For inquiries visit www.topnotchboardprep.com.ph or email us at topnotchmedicalboardprep@gmail.com
This handout is only valid for October 2022 PLE batch. This will be rendered obsolete for the next batch since we update our handouts regularly.
TOPNOTCH MEDICAL BOARD PREP PEDIATRICS MAIN HANDOUT BY DRS. DE VERA AND PUNONGBAYAN
For inquiries visit www.topnotchboardprep.com.ph or https://www.facebook.com/topnotchmedicalboardprep/
This handout is only valid for October 2022 PLE batch. This will be rendered obsolete for the next batch since we update our handouts regularly.
DOH RECOMMENDATION PATHOGENESIS
• <2 months old: Cefotaxime PLUS o Thickening of capillary walls → narrowing of capillary lumen
Amikacin for 10-14 days → widening of mesangium → fibrin thrombi in glomerular
E. coli capillaries & arterioles → cortical necrosis → localized clotting
• >2 moths to 18 years old. Duration (7-
Klebsiella o Microangiopathic anemia due to mechanical damage to
14 days)
Enterobacter RBCs as they pass through the altered vasculature
o Oral co-amoxiclav
Enterococcus • Thrombocytopenia is due to intrarenal platelet adhesion or
o Cefuroxime
Group B strep damage
o Nitrofurantoin
o Ampicillin Sulbactam • Most common in <4 yrs old
Recurrent UTI, • Onset is preceded by gastroenteritis (fever, vomiting, abdominal
Catheter-related, pain, bloody diarrhea)
comorbidities • Ceftriaxone and/or Amikacin for 7-14 • Sudden onset of pallor, irritability, weakness, lethargy & oliguria
Enterobacteriaceae days usually occurs 5-10 days after the initial gastroenteritis or
Pseudomonas respiratory illness
Enterococcus • PE: dehydration, petechiae, hepatosplenomegaly, marked irritability
• If culture shows different sensitivity • Peripheral smear: helmet cells, burr cells, fragmented RBCs
Antibiotic switch pattern and/or poor response after • Increased reticulocyte count, negative Coombs test,
48-72 hours leukocytosis, thrombocytopenia, anemia, hematuria &
proteinuria, normal PT & PTT, stool culture is often negative
✔ GUIDE QUESTION • Supportive care: fluids & electrolytes, early institution of
A 4-year-old male was diagnosed with UTI based on positive cultures, peritoneal dialysis, BP control, red cell transfusion
urinalysis, and symptoms. After 10 days of oral Cefixime patient
improved and was asymptomatic. What will you do next? • Platelets NOT generally administered regardless of platelet
A. Request for repeat urine culture C. Request for VCUG count (almost immediately consumed by active coagulation and
B. Request for Ultrasound D. NOTA can worsen the clinical course)
• NO antibiotic therapy (can lead to increased toxin release)
IMAGING STUDIES IN UTI • Those who survived the acute phase of HUS need long-term
• Goal: identify anatomic abnormalities that predispose to UTI, follow-up because complications may not be apparent for up to
determine if there is active renal involvement, asses renal 20 years (hypertension, chronic renal insufficiency, proteinuria)
function How is this different from thrombotic thrombocytopenic
purpura (TTP)?
Guideline Recommendations for Diagnostic Evaluation • TTP has the same triad but can include CNS involvement and
Following a Febrile Urinary Tract Infection in Infants fever and has a more gradual onset
LATE • Fewer cases of TTP can occur after diarrhea compared to HUS
GUIDELINE UTZ VCUG DMSA
SCAN Laboratory criteria for diagnosis:
National Institute The following are both present at some time during the illness:
for Health & Care 1. Anemia (acute onset) with microangiopathic changes
Excellence (NICE) (schistocytes, burr cells, helmet cells) on peripheral blood smear
American Academy 2. Renal injury (acute onset) evidenced by hematuria,
Yes If abnormal UTZ No proteinuria, or elevated creatinine level: equal or > 1 mg/dL
of Pediatrics
Italian Society for If abnormal UTZ in < 13 years old or >1.5 mg/dL in > 13 years old or equal or
If abnormal >50% increase over baseline
Pediatric Yes or if risk factors
UTZ or VUR
Nephrology (ISPN) are present*
*Abnormal antenatal UTZ of fetal urinary tract, family history of reflux,
septicemia, renal failure, age younger than 6 months in a male infant, likely
family noncompliance, incomplete bladder emptying, no clinical response to
appropriate antibiotic therapy within 72 hours or infection with organism
other than E. coli.

✔ GUIDE QUESTION
A 9-year-old boy presents with a 5-day history of abdominal pain &
diarrhea. One day PTC, he had bloody stools. He is admitted for IVF
replacement & further workup. After 4 days, his abdominal pain Helmet & Burr cells
subsided but the urine is grossly bloody & he looked pale. His blood
smear showed fragmented RBCs. What is the most common etiologic CASE CLASSIFICATION
agent for this disease?
PROBABLE
A. Shigella C. C. Difficile
B. E. coli D. P. aeruginosa • An acute illness diagnosed as HUS or TTP that meets the lab
criteria in a patient who does not have a clear history of acute or
HEMOLYTIC-UREMIC SYNDROME bloody diarrhea in the preceding 3 weeks, or
• One of the most common cause of acute renal failure in young • An acute illness diagnosed as HUS or TTP that:
children o Has onset within 3 weeks after onset of an acute or bloody
• Microangiopathic hemolytic anemia, thrombocytopenia, and diarrhea, and
renal insufficiency o Meets the lab criteria except that microangiopathic changes
• Caused by infection, genetics, drugs (calcineurin inhibitors, are not confirmed
cytotoxic agents, Clopidogrel and Ticlopidine, quinine), and other CONFIRMED
diseases associated with microvascular injury (like SLE and • An acute illness diagnosed as HUS or TTP that both meet the
HELLP syndrome – Hemolytic anemia, Elevated Liver enzymes, laboratory criteria and began within 3 weeks after onset of an
Low Platelet count syndrome) episode of acute bloody diarrhea
• Most common form of HUS is due to toxin-producing E. coli
• Asia and southern Africa: the shiga toxin of Shigella dysenteriae ACUTE KIDNEY INJURY
type 1 is causative • Abrupt loss of kidney function leading to a rapid decline in the
• Western countries: verotoxin-producing E. coli (VTEC) 0157:H7 glomerular filtration rate (GFR), accumulation of waste
• Microvascular injury with endothelial cell damage is products such as BUN and CREA, and dysregulation of
characteristic of all forms of HUS. extracellular volume and electrolyte homeostasis
• KDIGO (kidney disease improving global outcomes) definition:
o increase in crea by >/= 0.3mg/dl from baseline within 48 hrs
o increase in crea to >/= 1.5 times within the prior 7 days
o urine volume </= 0.5ml/kg/hr for 6 hours
TOPNOTCH MEDICAL BOARD PREP PEDIATRICS MAIN HANDOUT BY DRS. DE VERA AND PUNONGBAYAN Page 38 of 106
For inquiries visit www.topnotchboardprep.com.ph or email us at topnotchmedicalboardprep@gmail.com
This handout is only valid for October 2022 PLE batch. This will be rendered obsolete for the next batch since we update our handouts regularly.
TOPNOTCH MEDICAL BOARD PREP PEDIATRICS MAIN HANDOUT BY DRS. DE VERA AND PUNONGBAYAN
For inquiries visit www.topnotchboardprep.com.ph or https://www.facebook.com/topnotchmedicalboardprep/
This handout is only valid for October 2022 PLE batch. This will be rendered obsolete for the next batch since we update our handouts regularly.
KDIGO STAGING OF ACUTE INJURY Classification Description Common causes
STAGE SERUM CREATININE URINE OUTPUT Congenital conditions
1.5-1.9 times baseline, OR <0.5mL/kg/hr for 6- (posterior urethral
1 Post-renal Obstruction of
≥0.3mg/dL increase 12hrs valve, ureteropelvic
AKI urinary tract
<0.5mL/kg/hr for junction obstruction).
2 2.0-2.9 times baseline
≥12hrs Urolithiasis, tumors
3.0 times baseline, OR Specific form of AKI related to spontaneous
SCr ≥4.0mg/dL, OR Tumor lysis or chemotherapy-induced cell lysis in
<0.3mL/kg/hr for
Initiation of renal syndrome patients with lymphoproliferative
3 ≥24hrs, OR
replacement therapy, OR malignancies
Anuria for ≥12hrs
eGFR <35mL/min per
1.73m2 (<18yrs) ACUTE KIDNEY INJURY (RIFLE)
Estimated CCl Urine Output
CATEGORIES OF AKI Risk eCCl decreased by 25% <0.5 ml/kg/hr for 8 hrs
Classification Description Common causes <0.5 ml/kg/hr for 16
Injury eCCl decreased by 50%
Diminished hrs
effective eCCl decreased by 75%
Dehydration, sepsis, <0.3 ml/kg/hr for 24
circulating Failure or eCCl <35
hemorrhage, hrs or anuria for 12 hrs
Pre-renal arterial volume ml/min/1.73m 2
hypoalbuminemia, Loss Persistent failure >4 wks
which leads to
cardiac failure
inadequate renal End
Persistent failure >3 mos
perfusion stage
Glomerulonephritis, acute
Renal
tubular necrosis.
parenchymal
Endogenous toxins
damage, ACUTE KIDNEY INJURY
Intrinsic (myoglobin, hemoglobin)
including https://qrs.ly/6edugk0
renal and exogenous toxins
sustained
(medications, poisons,
hypoperfusion
radioactive materials) that
and ischemia
causes AKI

© Topnotch Medical Board Prep

CHRONIC KIDNEY DISEASE HEMATOLOGY/ONCOLOGY


CRITERIA FOR DEFINITION OF CHRONIC KIDNEY DISEASE ✔ GUIDE QUESTIONS
(NKF KDOQI GUIDELINES) A one-month old male was brought in for consultation. The patient was
• Patient has chronic kidney disease (CKD) if either of the born full term and the antenatal and perinatal course was
following criteria are present unremarkable. The patient developed jaundice to which the attending
physician requested for CBC and Bilirubin levels on the 10th DOL. A
o Kidney damage for ≥3mo, as defined by structural or
final diagnosis of breastfeeding jaundice was made, and the patient
functional abnormalities of the kidney, with or without currently is well. The mother however was concerned that her CBC
decreased GFR, manifested by one or more of the following showed a Hgb level of 10g/dl (NV: 12-14). Physical examination is
features: unremarkable. The following is/are true regarding this condition
§ Abnormalities in the composition of the blood or urine EXCEPT?
§ Abnormalities in imaging tests A. This condition is expected in the first 8-12 weeks of life
§ Abnormalities on kidney biopsy B. This is explained by the down regulation of EPO production
o GFR <60 mL/min/1.73m2 for ≥3mo, with or without the other C. No therapy is indicated for both term and pre-term infants
D. Nadir Hgb levels rarely falls between 100mg/dl
signs of kidney damage described above
The predominant hemoglobin present at birth in an infant is:
A. Hemoglobin A C. Hemoglobin F
STANDARD TERMINOLOGY FOR STAGES OF CHRONIC B. Hemoglobin Gower D. Hemoglobin A2
KIDNEY DISEASE (NKF KDOQI GUIDELINES)
GFR
STAGE DESCRIPTION PHYSIOLOGIC ANEMIA OF INFANCY
(mL/min/1.73m2)
Kidney damage with normal or • Normal full-term infants have higher hemoglobin and larger red
1 ≥90
increased GFR blood cells than older children and adults
Kidney damage with mild • 1st week of life – progressive decline in Hgb that persists 6-8
2 60-89
decrease in GFR weeks
3 Moderate decrease in GFR 30-59 • Usually lasts 8-12 weeks
4 Severe decrease in GFR 15-29 • Hb levels around 11g/dL, rarely falls below 10g/dL
<15 or on • No hematologic problem, no therapy required unless
5 Kidney failure
dialysis exacerbated by other ongoing processes
TOPNOTCH MEDICAL BOARD PREP PEDIATRICS MAIN HANDOUT BY DRS. DE VERA AND PUNONGBAYAN Page 39 of 106
For inquiries visit www.topnotchboardprep.com.ph or email us at topnotchmedicalboardprep@gmail.com
This handout is only valid for October 2022 PLE batch. This will be rendered obsolete for the next batch since we update our handouts regularly.
TOPNOTCH MEDICAL BOARD PREP PEDIATRICS MAIN HANDOUT BY DRS. DE VERA AND PUNONGBAYAN
For inquiries visit www.topnotchboardprep.com.ph or https://www.facebook.com/topnotchmedicalboardprep/
This handout is only valid for October 2022 PLE batch. This will be rendered obsolete for the next batch since we update our handouts regularly.
• Pathophysiology
o Respiration at birth → more O2 to bind Hb (50% to 95% or
more)
o Switch from fetal to adult Hb (lower affinity to O2)
o Net result is down regulation of EPO production
→ suppression of erythropoiesis
o Hb decreases until tissue O2 needs become greater than O2
delivery (8-12th week of life) → EPO production increases

PHYSIOLOGIC ANEMIA OF PREMATURITY


• More extreme and rapid decline in Hb
• Minimal levels up to 7-9g/dL
• Same physiologic factors exacerbated by other factors such as
o Iatrogenic blood loss
o Shortened RBC life span (40-60 days)
o Accelerated expansion of RBC mass due to rapid rate of growth
in preterm babies
• Not considered a benign condition, requires transfusion or EPO ANEMIA CLASSIFICATION
administration MICROCYTIC NORMOCYTIC MACROCYTIC
✔ GUIDE QUESTION
MCV <80 (TAILS) MCV 80-100 MCV >100
A 3-year-old child is brought to you for pica and pallor. He is described • Thalassemia • Anemia of • Folate deficiency
as irritable with poor appetite. He prefers purely milk diet. What is the • Iron deficiency chronic disease • Vit B12
most likely diagnosis? • Lead poisoning • Uremia deficiency
• Sideroblastic • Hypothyroidism • Drug and
Answers: Iron Deficiency Anemia
anemia • Bone marrow alcohol-induced
failure (aplastic anemia
ANEMIA anemia)
• Anemia is defined as a reduction of the hemoglobin IDA vs. THALASSEMIA
concentration or red blood cell (RBC) volume below the range of FEATURES IDA THALASSEMIA
values occurring in healthy persons. Nutritional
Cause Others: GI Bleed, Hereditary
CHILD PRESENTING Parasitism
WITH ANEMIA RBC Size Microcytic Microcytic
https://qrs.ly/6pdurlg RBC Color Hypochromic Hypochromic
RDW Elevated Normal
Low Iron
Hemoglobin
Other Test Low Ferritin
Electrophoresis
High TIBC
Reticulocyte
Low Low
Count
NORMAL MEAN AND LOWER LIMITS OF NORMAL FOR HEMOGLOBIN, HEMATOCRIT, AND MEAN CORPUSCULAR VOLUME
MEAN CORPUSCULAR VOLUME
HEMOGLOBIN (g/dL) HEMATOCRIT (%)
(µM3)
Age (years) Mean Lower Limit Mean Lower Limit Mean Lower Limit
0.5-1.9 12.5 11.0 37 33 77 70
2-4 12.5 11.0 38 34 79 73
5-7 13.0 11.5 39 35 81 75
8-11 13.5 12.0 40 36 83 76
12-14 female 13.5 12.0 41 36 85 78
12-14 male 14.0 12.5 43 37 84 77
15-17 female 14.0 12.0 41 36 87 79
15-17 male 15.0 13.0 46 38 86 78
18-49 female 14.0 12.0 42 37 90 80
18-49 male 16.0 14.0 47 40 90 80

Nelson Textbook of Pediatrics, 20th ed.

IRON DEFICIENCY ANEMIA • At 6-10 mg/dL


• The most common hematologic disease of infancy and childhood o Compensatory mechanisms
• Most common in 9-24 months of age. o Almost no symptoms if these mechanisms are efficient
• Most common cause is poor dietary intake. • At 5 mg/dL
o irritable and anorexic
• Pallor is the most important sign (7-8g/dl)
TOPNOTCH MEDICAL BOARD PREP PEDIATRICS MAIN HANDOUT BY DRS. DE VERA AND PUNONGBAYAN Page 40 of 106
For inquiries visit www.topnotchboardprep.com.ph or email us at topnotchmedicalboardprep@gmail.com
This handout is only valid for October 2022 PLE batch. This will be rendered obsolete for the next batch since we update our handouts regularly.
TOPNOTCH MEDICAL BOARD PREP PEDIATRICS MAIN HANDOUT BY DRS. DE VERA AND PUNONGBAYAN
For inquiries visit www.topnotchboardprep.com.ph or https://www.facebook.com/topnotchmedicalboardprep/
This handout is only valid for October 2022 PLE batch. This will be rendered obsolete for the next batch since we update our handouts regularly.
• Tachycardia, cardiac dilation, and systolic murmurs • 3/4 foci (Hemoglobin H disease): splenomegaly, increased ß4
• Non-hematologic effects • 4/4 foci (hydrops fetalis): incompatible with life
o Pica • Dx: decreased reticulocyte count (due to ineffective
o Pagophagia – ICE hematopoiesis), microcytic, hypochromic RBCs, normal RDW,
o Neurocognitive effects target cells and Heinz bodies on PBS
DIAGNOSTIC FINDINGS o *definitive diagnosis with Hb electrophoresis
• Dx: low reticulocyte count, microcytic, hypochromic RBCs, high • Tx: frequent transfusions, chelation therapy with Deferoxamine
RDW, low serum iron & ferritin, high TIBC as needed, splenectomy

MOLECULAR STRUCTURE OF HEMOGLOBIN


https://phescreening.blog.gov.uk/2016/03/18/reasons-for-differing-haemoglobin-variant-percentages-in-carriers/
• Why are the measurements of the total iron binding capacity BETA THALASSEMIA
important? • due to a decrease in beta-globin chain synthesis or absence of beta
o TIBC is high in iron deficiency anemia and low in anemia of chain of hemoglobin
chronic disease. Both illnesses have low serum iron levels. • common in people of Mediterranean origin, and SEA descent
One of the basic tests for anemia is CBC and RBC indices. IDA and • types:
Thalassemia both present as micro, hypo anemia with low reticulocyte o Beta-thalassemia major (Cooley anemia)
count. IDA has high RDW while thalassemia has normal RDW! § no beta-globin production
Dr. De Vera
MANAGEMENT § anemia at 6 mos (decline in normal Hgb F & rise in Hgb A),
• Prevention splenomegaly, extramedullary hematopoiesis)
o Premature infants: 7.5mg OD for 3 months o Beta-thalassemia minor
o 4mo-11mo: 15mg OD for 3 months § asymptomatic carriers, mild or no anemia
o 1-5 y/o: 30mg OD for 3 months • What is the pathophysiology of this condition?
o Adolescents: Iron 60mg + Folic Acid 400mcg o ↓ or absent beta chain of Hgb → normal alpha Hgb chains build
• Treatment up → form insoluble aggregates that precipitate within the
o 3-6mg/kg/day elemental iron in 2-3 div doses RBCs → hemolysis or damage to RBCs → susceptible to
o Max 150-200mg per day macrophage destruction & splenic sequestration → small &
o 2-3 months until blood values normalize pale RBCs
• Mild anemia only, repeat CBC after 4 weeks • Dx: same as alpha-thalassemia, definitive diagnosis with Hb
o Increase in Hb by 1-2g/dL electrophoresis (increased HbF, HbA2)
• Expected rise in Hb = 0.1-0.4g/dL/day depending on the severity • Tx: aggressive transfusions, chelation therapy as needed,
of anemia (more severe anemia = more rapid rise in Hb) hydroxyurea, splenectomy (to treat the resultant hypersplenism)
• Blood transfusion rarely necessary o *Beware of Yersinia enterocolitica infection which eats up the
free iron!
RESPONSE TO IRON THERAPY
12-24 hours Subjective improvements in patients EVALUATION OF MICROCYTIC ANEMIA
Iron Lead Chronic
26-28 hours Initial bone marrow response Deficiency poisoning
Thalassemia
disease
28-72 hours Reticulocytosis Hemoglobin ↓ N or ↓ ↓ ↓
4-30 days Increasing hemoglobin level MCV ↓ N or ↓ N or ↓ N or ↓
1-3 months Repletion of iron stores RDW ↑ N or ↑ N N or ↑
✔ GUIDE QUESTION Number of
RBCs
↓ ↓ N or ↑ N or ↓
An 18-month-old boy was brought to the clinic by his mother when she
noted him to be sallow-looking and dyspneic for about 2 weeks. The Serum Iron ↓ N N ↓
mother claims her son has been healthy up until now. PE reveals TIBC ↑ N N ↓
tachycardia, pale mucous membranes. Lab results: Hgb 5.5 g/dL, Ferritin ↓ N N ↑
decreased reticulocyte count, RBC indices showed microcytic
hypochromic RBCs with normal RDW, normal WBC and platelet. What
is your impression? EVALUATION OF
A. IDA C. Acute Leukemia
B. Aplastic Anemia D. Thalassemia MICROCYTIC ANEMIA
https://qrs.ly/xsdugla
THALASSEMIA
• Inherited microcytic hemolytic anemia due to abnormal ✔ GUIDE QUESTION
hemoglobin synthesis An 8-month-old male is noted to have jaundice & dyspnea. PE reveals
tachycardia and splenomegaly. The mother recalls a family history of
• Normal predominant HbA is tetramer made up of 2 alpha globin
“blood disease”. Lab results: Hgb 8.5 g/dL, Hct 29%, MCV 85 fl (NV=
chains and 2 beta globin chains 80-100 fl), MCH is normal. Reticulocyte and MCHC increased. WBC and
• Result when 1 or more of globin genes mutates Platelet counts are normal. What is the most likely diagnosis?
• Clinical phenotype determined by which and how many genes A. G6PD deficiency
are mutated B. DIC
C. Spherocytosis
ALPHA THALASSEMIA D. ITP
• due to a decrease in alpha-globin chain synthesis In the exams, when MCHC is increased, chances are the examiner is
• common in Asians and African-Americans looking for spherocytosis. Among the indices we usually look at MCH
and MCV. MCHC is important for the diagnosis of spherocytosis.
• symptoms depend on how many of the four foci are deleted or Normochromic, normocytic RBC with increased MCHC points to
mutated spherocytosis.
• 1/4 foci (silent): asymptomatic Dr. De Vera

• 2/4 foci (thalassemia trait): mild anemia


TOPNOTCH MEDICAL BOARD PREP PEDIATRICS MAIN HANDOUT BY DRS. DE VERA AND PUNONGBAYAN Page 41 of 106
For inquiries visit www.topnotchboardprep.com.ph or email us at topnotchmedicalboardprep@gmail.com
This handout is only valid for October 2022 PLE batch. This will be rendered obsolete for the next batch since we update our handouts regularly.
TOPNOTCH MEDICAL BOARD PREP PEDIATRICS MAIN HANDOUT BY DRS. DE VERA AND PUNONGBAYAN
For inquiries visit www.topnotchboardprep.com.ph or https://www.facebook.com/topnotchmedicalboardprep/
This handout is only valid for October 2022 PLE batch. This will be rendered obsolete for the next batch since we update our handouts regularly.

HEMOLYTIC ANEMIA
• Results from a decrease in RBC survival
• Common findings are pallor, jaundice due to increased indirect
bilirubin, increased LDH,
• Normocytic, normochromic RBCs with increased retic ct.,
characteristic cells on PBS
• Etiologies:
o intrinsic RBC defects (structural or metabolic abnormalities)
o autoimmune destruction
o intravascular destruction

HEMOLYTIC ANEMIA DUE TO INTRINSIC RBC DEFECTS


SPHEROCYTOSIS
• Defect in membrane spectrin or ankyrin (erythrocyte skeletal
proteins) → ↓membrane-volume ratio → causing loss of Radiographic appearance of sickle cell disease
biconcave shape → cells become more fragile → cells trapped in
the spleen & destroyed • Dx: sickle cells and Howell-Jolly bodies on PBS; definitive
diagnosis with Hb electrophoresis (HbS)
• Autosomal dominant
• What blood tests would be useful to establish the diagnosis?
o Osmotic fragility test confirms the presence of fragile sphere-
shaped RBCs.
o MCHC is increased due to a reduction in membrane surface
area
o MCV remains normal because the overall volume remains
stable.
o High reticulocyte count
o ↑ indirect bilirubin level • Tx: analgesia and hydration for acute crises, hydroxyurea,
• Treatment of spherocytosis: vaccinations against encapsulated organisms, folate
o Splenectomy is curative and should be considered in patients supplementation
with more severe disease.
o Folate supplementation may also be useful. ✔ GUIDE QUESTIONS
A 3-year-old child presents with a petechial rash but is otherwise well
✔ GUIDE QUESTION and without physical findings. Platelet count is 20,000/cu mm.
An 18-year-old African-American female visiting in Manila complains of Hemoglobin and WBC count are normal. The most likely diagnosis is:
episodes of extreme pain and discomfort in her legs and lower back. A. Systemic lupus erythematosus
She has been experiencing these recurrent episodes, with extreme B. Idiopathic Thrombocytopenic Purpura
fatigue, since she was a child. On PE, she is jaundiced with a Hct 23% C. Acute Lymphoblastic Leukemia
and Hgb of 7 g/dL. She reports she has family members who D. Disseminated Intravascular Coagulopathy
experienced the same symptoms. What is your most likely diagnosis?
IDIOPATHIC
A. Hereditary Spherocytosis
B. G6PD Deficiency THROMBOCYTOPENIC
C. DIC PURPURA
D. Sickle Cell Disease https://qrs.ly/oyduglh
An 18-month-old male child was brought to the ER because of swelling
SICKLE CELL DISEASE of bilateral ankles and hematoma of the thighs. No previous bleeding
• What is the pathophysiology? episodes were noted. PE revealed bilateral ankle swelling, hematoma,
o Develops at around 6 mos old when HbS (result of a single and ecchymosis over the thighs, what is the most likely diagnosis in this
missense mutation in the B-globin gene of Hgb) replaces HbF case?
o Episodes of painful crisis due to hypoxic tissue injury from A. Von Willebrand Disease
microvascular occlusions B. Idiopathic Thrombocytopenic Purpura
o Hgb becomes susceptible to polymerization in conditions of C. Hemophilia
D. Henoch Schönlein Purpura
low oxygen or dehydration → reduces flexibility of the RBC
In relation to the above case, what is the expected laboratory finding?
membrane A. Normal platelet count, increased bleeding time, normal PT,
o Any organ can be affected by vascular congestion, thrombosis, prolonged PTT
and infarction caused by sickling cells B. Normal platelet count, normal bleeding time, normal PT,
• Complications of Sickle Cell Disease: prolonged PTT
o Painful (vaso-occlusive) crisis C. Decreased platelet count, increased bleeding time, normal PT,
o Aplastic crisis normal PTT
o Splenic sequestration crisis leading to auto-splenectomy → D. Decreased platelet count, increased bleeding time, prolonged PT,
prolonged PTT
↑susceptibility to encapsulated organisms
o Priapism We consider Hemophilia if there is an ISOLATED PTT derangement.
o Stroke Looking at the pathway below, the most common form of Hemophilia A
o Leg ulcers (VIII) or B (IX) affects the intrinsic pathway. Thus, PTT derangement.
o Acute chest syndrome (fat emboli & infection) An interesting fact that most students miss out is that Vit K deficiency /
• Important Facts re: SCD warfarin toxicity will present with BOTH PT and PTT derangement.
o How prevalent is this? Again, look at the pathway below. In this disease, Vit K dependent factors
IX, X, VII, II are affected. Both intrinsic and extrinsic. Just so happens that
§ Autosomal recessive disorder with an 8% carrier rate in VII has the shortest half-life so PT is a better test because it will be
African Americans deranged before PTT does.
§ About 0.2% of African Americans Dr. De Vera

o What are the typical radiologic findings?


§ Erythropoiesis must increase to compensate for hemolysis →
marrow expansion → resorption of bone → new bone
formation on the external aspect of the skull (“crew cut” or
“hair on end” appearance on skull radiographs)

TOPNOTCH MEDICAL BOARD PREP PEDIATRICS MAIN HANDOUT BY DRS. DE VERA AND PUNONGBAYAN Page 42 of 106
For inquiries visit www.topnotchboardprep.com.ph or email us at topnotchmedicalboardprep@gmail.com
This handout is only valid for October 2022 PLE batch. This will be rendered obsolete for the next batch since we update our handouts regularly.
TOPNOTCH MEDICAL BOARD PREP PEDIATRICS MAIN HANDOUT BY DRS. DE VERA AND PUNONGBAYAN
For inquiries visit www.topnotchboardprep.com.ph or https://www.facebook.com/topnotchmedicalboardprep/
This handout is only valid for October 2022 PLE batch. This will be rendered obsolete for the next batch since we update our handouts regularly.
TREATMENT OF VON WILLEBRAND DISEASE
• Mild bleeding in type 1: desmopressin (causes release of vWF
from endothelial stores)
• Severe disease: Factor VIII concentrates which contain high vWF Ag
LABORATORY FINDINGS IN BLEEDING DISORDERS
BLEEDING PLATELET
PT PTT
TIME COUNT
Hemophilia
N ↑ N N
A&B
vWd N ↑ ↑ N or ↓
ITP N N ↑ ↓
Vitamin K
↑ N or ↑ N N
deficiency
DIC ↑ ↑ ↑ ↓

BLEEDING DISORDERS
https://qrs.ly/jldugm3

HEMOPHILIA
• X-linked recessive deficiencies of – ✔ GUIDE QUESTIONS
o factor VIII (Hemophilia A) – more common Most children with leukemia present with:
A. Unexplained fever, pallor, and hemorrhage
o factor IX (Hemophilia B or Christmas disease)
B. Hepatomegaly
• severity of bleeding is proportional to factor levels C. Massive GI bleeding
o mild (5-25% normal activity): bleeding only after major trauma D. Abdominal pain
or surgery Which type of leukemia in childhood will have the best response to
o moderate (1-5%): bleeding with mild trauma chemotherapy?
o severe (<1%): spontaneous bleeding A. AML
B. ALL
MISCELLANEOUS HEMATOLOGY C. CML
What is the most common hereditary von Willebrand D. CLL
bleeding disorder? Disease
What is the most common and most serious
congenital coagulation factor deficiencies?
Hemophilia A ACUTE LYMPHOCYTIC LEUKEMIA
SIGNS/SYMPTOMS OF ALL
What is the most common hereditary
Factor V Leiden • acute onset < 4 wks duration of Sx
hypercoagulable disorder?
Prolonged • non-specific (anorexia, irritability, lethargy)
What is the hallmark of hemophilia? • signs of marrow failure (anemia, bleeding, purpuric/petechial
bleeding
What is the earliest joint hemorrhages in children? Ankles lesions, low-grade fever)
• signs of infiltration (bone pain, lymphadenopathy, splenomegaly
✔ GUIDE QUESTIONS
> hepatomegaly)
A 2-year-old boy is brought to his doctor by his parents who are
For the board exam, please study ALL. This is the most common childhood
concerned about the multiple bruises on the boy’s shins and hands.
malignancy.
They report the child seems to get large bruises with minimal injury
A similar disease is Aplastic Anemia (AA). Both presents with
and bleeds profusely when his teeth are brushed. On questioning, they
PANCYTOPENIA (in CBC affectation of 2 or more cell lines: RBC, WBC
reveal the child has a grandmother with a bleeding disorder. Lab tests:
platelet). The difference is that leukemia is pancytopenia +
bleeding time 14 mins, PT 12 secs, PTT 41 secs. What is the
hypercellularity (blasts), while AA is pancytopenia + hypocellularity. On
management of choice for this disease?
PE leukemia presents with SIGNS OF INFILTRATION while AA does not.
A. Desmopressin Dr. De Vera
B. Platelet Transfusion WHAT IS THE ETIOLOGY OF THESE PE FINDINGS?
C. Factor IX Transfusion
• Result from leukemic expansion and crowding out of the
D. Steroids or IVIG
normal marrow
• Anemia and thrombocytopenia, bone or joint pain from invasion
VON WILLEBRAND into the periosteum
DISEASE • Fever results from pyrogenic cytokines released from leukemic cells
https://qrs.ly/nzduglw • Painless enlargement of the scrotum & CNS symptoms -
extensive extramedullary invasion
What is the most likely diagnosis?
• Von Willebrand disease - the most common inherited bleeding PROGNOSTIC / PREDICTIVE FACTORS
disorder • The single most important prognostic factor in ALL is the
• Result of a quantitative (type 1 or 3) or qualitative (type 2) defect in vWF treatment.
• vWF is a large protein made by endothelial cells and megakaryocytes; a • 3 of the most important predictive factors:
carrier for factor VIII and is a cofactor for platelet adhesion 1. Age of the patient at the time of diagnosis
2. Initial leukocyte count
VON WILLEBRAND DISEASE 3. Speed of response to treatment
§ Average risk: age between 1-10 yrs old and leukocyte count
• What clinical findings are commonly associated with this
of <50,000/uL
condition?
o Disturbs both primary & secondary hemostasis SUPPLEMENT: QUICK SHEET
o Role in platelet adhesion to exposed subendothelium leads to IMPORTANT FACTS RE: ALL
increased bleeding time → mucous membrane bleeding, • sites of relapse: bone marrow, CNS (increased ICP and isolated
petechiae, purpura cranial nerve palsies), testes (painless swelling of one or both
o Often have a (+) family history testes in 1-2% of males)
• Dx: • Where does it spread? Liver, spleen, lymph nodes
o ↓ vWF levels • poor prognostic factors:
o ↓ Factor VIII o < 2 yrs or > 10 yrs
o prolonged bleeding time o male
o abnormal platelet adhesion o WBC > 100,000 u/L on presentation
o ↑ PTT o presence of CNS leukemia
o Ristocetin cofactor assay (measures vWF antigen levels and activity) o presence of a mediastinal mass
TOPNOTCH MEDICAL BOARD PREP PEDIATRICS MAIN HANDOUT BY DRS. DE VERA AND PUNONGBAYAN Page 43 of 106
For inquiries visit www.topnotchboardprep.com.ph or email us at topnotchmedicalboardprep@gmail.com
This handout is only valid for October 2022 PLE batch. This will be rendered obsolete for the next batch since we update our handouts regularly.
TOPNOTCH MEDICAL BOARD PREP PEDIATRICS MAIN HANDOUT BY DRS. DE VERA AND PUNONGBAYAN
For inquiries visit www.topnotchboardprep.com.ph or https://www.facebook.com/topnotchmedicalboardprep/
This handout is only valid for October 2022 PLE batch. This will be rendered obsolete for the next batch since we update our handouts regularly.

✔ GUIDE QUESTION • Dx: abdominal CT scan (calcification & hemorrhage), 24-hr urine
An asymptomatic 2-year-old girl is noted to have an abdominal mass on VMA & HVA (elevated in 95% of cases), BUN/Crea, CXR, bone
the left flank by her mother. IVP reveals an enlarged kidney with scan, LFTs, CBC
distorted pelvis and calyces. The px most likely has: • International Neuroblastoma Staging System
A. Wilms tumor C. Adrenal tumor • Prognostic factors: age at diagnosis, stage of disease, Shimada
B. Malignant lymphoma D. Granulosa cell tumor histology

WILMS TUMOR INSS


• renal tumor of embryonal origin; 2nd most common malignant • St 1: tumors confined to the organ or structure of origin
abdominal tumor in childhood • St 2: tumors extend beyond the structure of origin but not across
• 2-5 years old the midline with (2B) or without (2A) ipsilateral lymph node
• Incidence of bilateral Wilms tumor is 7% involvement
• associated with: • St 3: extend beyond the midline w/ or w/o bilateral lymph node
o Neurofibromatosis involvement
o Beckwith-Wiedemann syndrome (hemihypertrophy, • St 4: disseminated to distant sites (bone, BM, liver, distant lymph nodes)
visceromegaly, macroglossia) • St 4S: <1 year old w/ dissemination to liver, skin, or BM w/o bone
o WAGR syndrome (Wilms tumor, Aniridia, Genitourinary involvement & with a primary tumor
abnormalities, mental retardation)
PROGNOSIS OF NEUROBLASTOMA
SUPPLEMENT: QUICK SHEET • With localized disease: regardless of age, 80-90% 5-year survival rate
IMPORTANT FACTS RE: WILMS TUMOR • Overall, at younger age at diagnosis carries a more favorable
• S/Sx: painless abdominal enlargement with flank mass that does not prognosis
cross the midline, hematuria (12-25%), hypertension • 5-year survival rate:
• Dx: abdominal UTZ, CT scan or MRI, abdominal X ray, liver & kidney o 83% for infants
function tests, chest x ray (pulmonary metastasis) o 55-60% for 1-5 yrs old
• Tx: transabdominal nephrectomy & post-surgical chemotherapy
o 40% for children >5 yrs old
• Worse prognosis: large tumor (>500 grams), st III & IV, unfavorable
histologic type (anaplastic)
• More than 60% of patients generally survive TREATMENT
• St I-III have a cure rate of >90% • Low risk St 1 & 2: surgery
• Observation for St 4S (nearly 100% survival)
✔ GUIDE QUESTIONS • St 2: cure rate is >90%
A 9-year-old girl is receiving chemotherapy for ALL. The drug of first • Local recurrence: chemotherapy and radiation
choice for PCP prophylaxis is: • If with spinal cord compression at Dx: chemotherapy, surgery, or
Answers: Co-Trimoxazole radiation
• Intermediate risk: chemotherapy (cisplatin, cyclophosphamide,
A 2-year-old male child is brought to the clinic due to an abdominal
swelling which the mother found out while she was bathing her child. etoposide & doxorubicin), surgery, or radiation
The child has no complaints, but the mother has seen occasional jerky • St 3 & 4: surgery + chemotherapy; 90% survival
movements of his leg. On PE, the mass palpated was found to cross the • High-risk: chemotherapy, stem cell transplantation, surgery,
midline, there were also some bluish subcutaneous nodules palpated. radiation
What is the most likely diagnosis in this case?
In WILMS vs Neuroblastoma. Please note the ff differences: origin (renal
A. Wilms tumor C. Lymphoma
vs elsewhere), crosses midline, associated symptoms, CT scan findings.
B. Neuroblastoma D. Rhabdomyosarcoma Dr. De Vera
In relation to the case above, which of the ff. approaches should you
request to completely diagnose the case? SUPPLEMENT: QUICK SHEET
A. Bone marrow aspirate + urine VMA and HVA Wilms Tumor vs. Neuroblastoma
B. Bone marrow aspirate and CT scan Wilms Tumor Neuroblastoma
C. Urine VMA and HVA + CT scan Age group 2-5 y/o 2-5 y/o
D. VMA and CT scan
Anywhere along the
In relation to the above case, which among the ff. is the mainstay in the Origin /
treatment of this condition? Kidneys neuroaxis, most
Location
A. Surgical Resection C. Chemotherapy commonly adrenals
B. Await Spontaneous Resolution D. Radiation Therapy Crosses
No Yes
In relation to the case above, which among the ff. is the most common midline?
site of metastasis? Many
A. Lungs C. Brain Horner syndrome,
B. Bones D. Liver Associated Hypertension, hypertension, cord
symptoms hematuria compression, periorbital
bruising, subcutaneous nodules,
NEUROBLASTOMA opsoclonus, myoclonus
• embryonal tumor of neural crest cell origin Metastasis Lungs Bones
• 3rd most common pediatric cancer Imaging, Biopsy (BMA),
• 8% of childhood malignancies Urine VMA and HVA
Diagnosis Imaging, biopsy
• Neuroblasts (pluripotent stem cells) invaginate and migrate along CT scan usually shows
the neuroaxis where neural crest cells are present – sympathetic calcifications
ganglia, adrenal glands Surgery ± Surgery ±
• median age at diagnosis is 2 yrs old & 90% of cases are diagnosed Treatment chemotherapy, chemotherapy,
by 5 yrs old radiotherapy radiotherapy
• associated with:
SUPPLEMENT: QUICK SHEET
o N-myc oncogene – tuberous sclerosis
o Neurofibromatosis – Pheochromocytoma DIFFERENTIAL DIAGNOSIS OF SMALL, ROUND, BLUE-CELL
o Hirschsprung disease TUMORS:
• S/Sx: reflect the tumor site & extent of disease; non-tender • Wilms tumor
abdominal mass which may cross the midline, Horner syndrome, • Acute leukemia
hypertension, cord compression (from a paraspinal tumor) • Rhabdomyosarcoma
• Metastases: periorbital bruising (“raccoon eyes”), • Mesothelioma / Medulloblastoma
subcutaneous tumor nodules, opsoclonus /myoclonus • Ewing sarcoma
(“dancing eyes, dancing feet”) • Retinoblastoma
• most common sites of metastasis: long bones & skull, BM, liver, • Primitive neuroectodermal tumor (PNET)
lymph nodes, skin • Neuroblastoma
*** W-A-R-M-E-R-P-N (warmer in the Philippines)
TOPNOTCH MEDICAL BOARD PREP PEDIATRICS MAIN HANDOUT BY DRS. DE VERA AND PUNONGBAYAN Page 44 of 106
For inquiries visit www.topnotchboardprep.com.ph or email us at topnotchmedicalboardprep@gmail.com
This handout is only valid for October 2022 PLE batch. This will be rendered obsolete for the next batch since we update our handouts regularly.
TOPNOTCH MEDICAL BOARD PREP PEDIATRICS MAIN HANDOUT BY DRS. DE VERA AND PUNONGBAYAN
For inquiries visit www.topnotchboardprep.com.ph or https://www.facebook.com/topnotchmedicalboardprep/
This handout is only valid for October 2022 PLE batch. This will be rendered obsolete for the next batch since we update our handouts regularly.

✔ GUIDE QUESTIONS ✔ GUIDE QUESTIONS


A 16-year-old male patient presents to the clinic due to weight loss over A 10-year-old male child presents with pallor, headaches, palpitations
the past 3 months, body malaise, (+) night sweats, and intermittent and easy fatigability. He also has noted gingival bleeding when
febrile episodes of 38-39°C. There was occasional dry cough, no brushing his teeth. He has a history repeated flu like symptoms over
dyspnea, occasional episodes of epistaxis. The patient had some cough the past 3 months. No history of rashes and solvent exposure, no
and colds 1 month ago, (+) episode of flu 2 weeks ago. No other relevant weight loss or anorexia, no muscle or joint pains. PE revealed: pale
history. Examination revealed: BP 90/60, HR 108, RR 20, T >37.2°C, palpebral conjunctiva, pale buccal mucosa, no lymphadenopathy, no
pale palpebral conjunctiva, pale buccal mucosa, (+) multiple nontender splenomegaly. A CBC was done which revealed pancytopenia, what is
cervical and supraclavicular lymphadenopathy, no splenomegaly, clear the most likely diagnosis in this case?
bilateral breath sounds, what is the next best step in the diagnosis in A. AML C. Non-Hodgkin Lymphoma
this px? B. Myelofibrosis D. Aplastic Anemia
A. CXR C. urinalysis • Aplastic anemia – results from failure or autoimmune
B. ECG D. lymph node biopsy destruction of myeloid stem cells → pancytopenia
In relation to the above case, what is the most likely diagnosis? • What is the most likely cause?
A. Pulmonary Tuberculosis o Most cases are idiopathic. Other possible causes: viruses
B. Infectious Mononucleosis (parvovirus B19, hepatitis viruses, HIV, Epstein-Barr virus),
C. Hodgkin Lymphoma drugs & chemicals (alkylating & antimetabolite agents,
D. HIV acute retroviral syndrome
chloramphenicol, insecticides, benzene), and radiation

LYMPHOMAS APLASTIC ANEMIA


• neoplasms of lymphoid cells (lymphocytes, histiocytes & their • What are the typical peripheral blood smear findings in this
precursors) - 6% of childhood cancers condition?
o Hypocellularity and pancytopenia
• Ann Arbor Staging Classification:
o Stage I one lymph node group involved • What other test would be useful in confirming the
o Stage II two lymph node groups involved on the same side of diagnosis?
the diaphragm o Bone marrow biopsy (hypocellular bone marrow with a fatty
o Stage III two lymph node groups involved on both sides of the infiltrate)
diaphragm or extra-lymphatic involvement
o Stage IV diffuse extra-lymphatic site involvement (usually PANCYTOPENIA
bone marrow, CNS, liver) 1. FANCONI ANEMIA
HODGKIN NON-HODGKIN • autosomal recessive
• Painless, non-tender, firm, rubbery cervical • aplastic anemia with microcephaly, microphthalmia, hearing
or supraclavicular adenopathy loss, limb anomalies (absent radii & thumbs)
Hx • B symptoms important in staging • Dx: clinical, cytogenetic analysis
(weight loss >10% of TBW over 3 months, unexplained • Tx: steroids, BMT, supportive care
high-grade fever, drenching night sweats)
• systemic 2. APLASTIC ANEMIA
regional adenopathy adenopathy • associated with ionizing radiation exposure, drugs, viral
PE hepatosplenomegaly • more extranodal infections
is rare involvement • Dx: clinical, BMA (hypocellular marrow)
• hepatosplenomegaly
node biopsy, whole TREATMENT OF APLASTIC ANEMIA
node biopsy, CXR,
Dx body CT Scan, BM • RBC and platelet transfusion
BM biopsy, LP
biopsy, LP • Allogenic BM transplant (sibling)
Chemotherapy • Granulocyte colony-stimulating factor or granulocyte
COPAD: macrophage colony-stimulating factor
cyclophosphamide, • radiation for CNS • Withdrawal of any toxic causative agent
vincristine, prednisone, involvement
doxorubicin or COMP: ✔ GUIDE QUESTION
Tx • chemotherapy for B
cyclophosphamide, A 2-year-old male patient is brought to the clinic by his mother for an
vincristine, Sx, St. III & IV abdominal mass which she incidentally found while bathing the child.
methotrexate, • possible BMT There was no other associated symptoms. PE: revealed an abdominal
prednisone, 6- mass from the right quadrant extending past the midline. A CT scan
mercaptopurine was done which revealed that the mass had calcifications and
• bimodal age hemorrhage. What is the most likely diagnosis in this case?
• 60% of all A. Wilms tumor
distribution
lymphomas in B. Neuroblastoma
• 20-30 yrs & > 50
children & C. Non-Hodgkin Lymphoma
Peak age & yrs; developing D. Rhabdomyosarcoma
adolescents; ↑
risk factors countries: early
prevalence with age MISCELLANEOUS ONCOLOGY
peak before
• EBV (Burkitt Most common malignancy? Leukemia (ALL)
adolescence; M>F
lymphoma) Most common solid tumor? Brain Tumors
• EBV, CMV, HHV-6
Reed-Sternberg cell Most common solid tumor
Neuroblastoma
(arises from the outside CNS?
Histology --
germinal center of B Most common soft tissue tumor? Rhabdomyosarcoma
cells) Malignancy with highest
contiguous lymph non-contiguous lymph Brain (PNET)
Distribution mortality?
node spread node spread
OSTEOSARCOMA EWING SARCOMA
• nodular sclerosis
2nd decade. M>F 2nd decade. M>F
• mixed cellularity
All races Primarily whites
lymphocyte-
low, intermediate & Spindle cell producing Undifferentiated small
Variants predominant
high-grade osteoid round cell of neural origin
(best prognosis)
• lymphocyte- Metaphysis of long bones Diaphysis of long bones
depleted Local pain and swelling, often Local pain and swelling with
history of injury fever
Onion-skinning/ moth
Sunburst pattern
eaten appearance
Spreads to lungs and bones Spreads to lungs and bones
Surgery + Chemotherapy Surgery + chemotherapy
TOPNOTCH MEDICAL BOARD PREP PEDIATRICS MAIN HANDOUT BY DRS. DE VERA AND PUNONGBAYAN Page 45 of 106
For inquiries visit www.topnotchboardprep.com.ph or email us at topnotchmedicalboardprep@gmail.com
This handout is only valid for October 2022 PLE batch. This will be rendered obsolete for the next batch since we update our handouts regularly.
TOPNOTCH MEDICAL BOARD PREP PEDIATRICS MAIN HANDOUT BY DRS. DE VERA AND PUNONGBAYAN
For inquiries visit www.topnotchboardprep.com.ph or https://www.facebook.com/topnotchmedicalboardprep/
This handout is only valid for October 2022 PLE batch. This will be rendered obsolete for the next batch since we update our handouts regularly.

PLEASE READ MORE ON: • Seizure disorder – a general term to include any one of the ff:
epilepsy, febrile seizures, and seizures secondary to metabolic,
• Staging of Tumors
infectious, or other etiologies
• Soft Tissue Malignancies
• Epilepsy – disorder of the brain characterized by an enduring
o Rhabdomyosarcoma
predisposition to generate seizures and by the neurobiological,
• Osteosarcoma
cognitive, psychological, and social consequences of this
• Ewing Sarcoma condition; two or more unprovoked seizures occurring in a
Chemotherapeutic Regimens time frame of >24 hrs
o Epileptic syndrome – disorder that manifests one or more
NEUROLOGY specific seizure types and has a specific age of onset and
SEIZURES & EPILEPSY prognosis
• Seizure type is the primary determinant of the type of
• Seizures/convulsions are paroxysmal, time-limited changes medications the patient is likely to respond to
in motor activity and/or behavior that result from abnormal
• Epilepsy syndrome determines the type of prognosis one could
electrical activity in the brain
expect.
o Transient occurrence of signs/symptoms resulting from abnormal
excessive or synchronous neuronal activity in the brain
CLASSIFICATION FOR EPILEPSY SYNDROMES WITH AN INDICATION OF
AGE OF ONSET, DURATION OF ACTIVE EPILEPSY, PROGNOSIS, AND THERAPEUTIC OPTIONS
Adapted from Nelson textbook of Pediatrics 20th ed.
AGE OF AGE AT MONOTHERAPY OR
SPECIFIC SYNDROMES PROGNOSIS POSSIBLE ADD-ON SURGERY
ONSET REMISSION ADD-ON
EPILEPSIES OF UNKNOWN CAUSE OF INFANCY AND CHILDHOOD
Benign infantile seizures
Infant Infant Good PB -- No
(nonfamilial)
Benign childhood
epilepsy with 3-13 yrs 16 yrs Good CBZ, LEV, OXC, VPA -- No
centrotemporal spikes
Early and late-onset 12 yrs or
2-8 yrs; 6-17
idiopathic occipital younger; 18 Good CBZ, LEV, OXC, VPA -- No
yrs
epilepsy yrs
FAMILIAL (AUTOSOMAL DOMINANT) EPILEPSIES
Benign familial neonatal Newborn to Newborn to
Good PB -- No
convulsions young infant young infant
Benign familial infantile
Infant Infant Good CBZ, PB -- No
convulsions
Autosomal dominant
CBZ, GBP, OXC, PHT,
nocturnal frontal lobe Childhood -- Variable CLB, LEV, PB, PHT No
TPM
epilepsy
Familial lateral temporal Childhood to CBZ, GBP, OXC, PHT,
-- Variable CLB, LEV, PB, PHT No
lobe epilepsy adolescence TPM, VPA
Generalized epilepsies Childhood to
-- Variable ESM, LTG, TPM, VPA CLB, LEV No
with febrile seizures plus adolescence
STRUCTURAL-METABOLIC FOCAL EPILEPSIES
Limbic Epilepsy
Mesial temporal lobe
School-age or CBZ, LEV, OXC, TPM, CLB, GBP, LAC, PB,
epilepsy with Long lasting Variable Temporal resection
earlier VPA PHT, ZON
hippocampal sclerosis
Mesial temporal lobe
CBZ, LEV, OXC, TPM, CLB, GBP, LAC, PB,
epilepsy defined by Variable Long lasting variable Temporal resection
VPA PHT, ZON
specific causes
Other types defined by CBZ, LEV, OXC, TPM, CLB, FBM, GBP, LAC, Lesionectomy ±
Variable Long lasting variable
location and causes VPA PB, PHT, ZON temporal resection
Neocortical epilepsies
Plasmapheresis, CBZ, LAC, PB, PHT, Functional
Rasmussen syndrome 6-12 yrs Progressive Ominous
immunoglobulins TPM hemispherectomy
Hemiconvulsion- CBZ. LEV, OXC, TPM, CLB, GBP, LAC, PB, Functional
1-5 yrs Chronic Severe
hemiplegia syndrome VPA PHT, ZON hemispherectomy
Other types defined by CBZ, LE, OXC, TPM, PHT, PB, CLB, GBP, Lesionectomy ±
Variable Long lasting Variable
location and cause VPA LAC, ZON cortical resection
Migrating partial Bromides, CBZ, LEV,
Infant No remission Ominous BDZ, LAC, ZON No
seizures of early infancy PB, PHT, TPM, VPA
GENERALIZED EPILEPSIES OF UNKNOWN CAUSE
Benign myoclonic
3 mo-3 yrs 3-5 yrs Variable LEV, TPM, VPA BDZ, ZON No
epilepsy in infancy
BDZ, ketogenic diet,
Epilepsy with myoclonic
3-5 yrs Variable Variable ESM, TPM, VPA LEV, LTG, steroids, No
astatic seizures
ZON
Childhood absence Acetazolamide,
5-6 yrs 10-12 yrs Good ESM, LTG, VPA No
epilepsy ketogenic diet, ZON
Epilepsy with myoclonic
1-13 yrs Variable Guarded ESM, VPA BDZ, ZON No
absences
ACTH, adrenocorticotropic hormone; BDZ, benzodiazepine; CBZ, carbamazepine; CLB, clobazam; DZP: diazepam; ESM, ethosuximide; FBM: felbamate; GBP,
gabapentin; IVIG, intravenous immunoglobulin; LAC, lacosamide; LEV, levetiracetam; LTG, lamotrigine; n/a, not applicable; OXC: oxcarbazepine; PB, phenobarbital;
PHT, phenytoin; PRM, primidone; RFD, rufinamide; TPM, topiramate; VGB: vigabatrin; VPA, valproic acid; ZON, zonisamide.
CATEGORY OF EPILEPTIC SEIZURES o Subdivided into:
International Classification of Epileptic Seizures § Focal Seizures without impairment of consciousness
• Focal Seizures (previously known as simple partial seizures)
o Formerly known as partial seizure § Focal Seizures with impairment of consciousness a.k.a. focal
o initial activation of a system of neurons limited to part of 1 dyscognitive seizures (previously complex partial seizures)
cerebral hemisphere • Generalized Seizures
o Clinical and EEG changes indicate synchronous involvement of
all or both hemispheres
• Febrile Seizures

TOPNOTCH MEDICAL BOARD PREP PEDIATRICS MAIN HANDOUT BY DRS. DE VERA AND PUNONGBAYAN Page 46 of 106
For inquiries visit www.topnotchboardprep.com.ph or email us at topnotchmedicalboardprep@gmail.com
This handout is only valid for October 2022 PLE batch. This will be rendered obsolete for the next batch since we update our handouts regularly.
TOPNOTCH MEDICAL BOARD PREP PEDIATRICS MAIN HANDOUT BY DRS. DE VERA AND PUNONGBAYAN
For inquiries visit www.topnotchboardprep.com.ph or https://www.facebook.com/topnotchmedicalboardprep/
This handout is only valid for October 2022 PLE batch. This will be rendered obsolete for the next batch since we update our handouts regularly.
International League Against Epilepsy (ILAE) classification of ✔ GUIDE QUESTION
etiology of epilepsy A 4-year-old male was brought to your clinic for evaluation. Patient
• Genetic epilepsy (previously known as idiopathic) was noted to wake up at night with twitching of the right side of face
o direct result of a known or presumed genetic defect(s) in and drooling. Patient is unable to speak during the episodes but is
awake and follows command. PE is unremarkable. MRI is normal. ECG
which the genetic defect is not causative of a brain structural
showed broad-based centrotemporal spikes. This type of benign
or metabolic disorder other than the epilepsy epilepsy syndrome with focal seizure is expected to be outgrown at
• Structural/metabolic epilepsy (previously “symptomatic epilepsy”) what age
o distinct structural or metabolic entity that causes A. Pre-pubertal
seizure/epilepsy B. Adolescence
• Unknown Epilepsy (previously “cryptogenic epilepsy”) C. Adulthood
o Cause is unknown D. School age period

ILAE 2017 CLASSIFICATION OF SEIZURE TYPES: It’s called “Rolandic” because seizures originate in the Rolandic area
of the brain, the one that controls the face. Benign Rolandic epilepsy
FOCAL
is also called “benign childhood epilepsy with centrotemporal spikes.”
(either aware or GENERALIZED UNKNOWN ONSET Dr. Punongbayan
impaired awareness)
MOTOR MOTOR MOTOR
• Automatisms • Tonic • Tonic-clonic BENIGN EPILEPSY SYNDROMES WITH FOCAL SEIZURES
• Tonic • Clonic • Epileptic spasms • Most common type is benign childhood epilepsy with
• Clonic • Tonic-clonic (trunk flexion) centrotemporal spike
• Myoclonic • Atonic (limp) o 3-10 y/o
• Atonic • Myoclonic o Outgrown during adolescence
• Epileptic spasms o Focal seizure with buccal and throat tingling and tonic or
• Hyperkinetic clonic contractions of 1 side of the face, with drooling and
NON-MOTOR NON-MOTOR NON-MOTOR inability to speak but with preserved consciousness and
• Emotional (Absence) • Behavior arrest
comprehension
• Sensory • Typical (pausing, freezing,
• Autonomic • Atypical activity arrest) o ECG = broad-based centrotemporal spikes
• Behavior arrest • Myoclonic o MRI normal
• Cognitive o Responds well to AED such as carbamazepine
UNCLASSIFIED • Benign Epilepsy with occipital spikes
o Panayiotopoulos type
SUPPLEMENT: QUICK SHEET
§ Early childhood
ILAE (International League Against Epilepsy) § Complex partial seizures with ictal vomiting
2017 Classification of Seizure Types: o Gastaut Type
I. Focal (either aware or impaired awareness) § Later childhood
A. Motor onset (automatisms, myoclonic, etc.) § Complex partial seizure, visual auras, migraine headache
B. Non-motor onset (autonomic, emotional, etc.) o Both are outgrown in few years
II. Generalized
A. Motor (tonic-clonic, tonic, clonic, etc.) ✔ GUIDE QUESTION
B. Non-motor (absence) An 8-month-old baby boy was brought to your clinic for evaluation.
III. Unknown Patient was noted to have episodes of truncal flexion described by
A. Motor parents “as if doing sit ups” that occur multiple times throughout the
B. Non-motor day. Patient was also noted to have lost the ability to sit without
C. Unclassified support which he was able to do a month ago. EEG shows high voltage,
slow, chaotic background with multifocal spikes. What is your
SUPPLEMENT: QUICK SHEET diagnosis?
ILAE Classification: a. Lennox-Gastaut Syndrome
Modification for Seizures in the Neonate (Feb. 2021) b. West Syndrome
c. Ohtahara Syndrome
KEY POINTS:
d. Dravet Syndrome
• Seizures are considered focal at onset.
• Seizures can occur with or without clinical manifestations. West Syndrome
• Descriptors are determined by the predominant clinical • Usually between 2 to 12 months
• Triad:
feature and divided into motor, non-motor, and sequential.
o Infantile epileptic spasms
• It emphasizes the key role of EEG in the diagnosis of seizures o Developmental regression
in this age group. o EEG = hypsarrhythmia
Dr. Punongbayan

SEVERE GENERALIZED EPILEPSIES


• Severe generalized epilepsies are associated with intractable
seizures and developmental delay
• Early myoclonic infantile encephalopathy
o starts during the 1st 2 months of life with severe myoclonic
seizures and burst suppression pattern on EEG
o usually caused by inborn errors of metabolism
• Early infantile epileptic encephalopathy
(Ohtahara syndrome)
o similar age of onset and EEG but manifests tonic seizures and
is usually caused by brain malformations or syntaxin
binding protein 1 mutations
• Severe myoclonic epilepsy of infancy (Dravet syndrome)
Essential questions: o starts as focal febrile status epilepticus or focal febrile seizures
• Age group and later manifests myoclonic and other seizure types
• Acute or chronic • West Syndrome
• Associated symptoms and signs o Usually between 2 to 12 months
• Focal or generalized o Triad
• Motor or non-motor § Infantile epileptic spasms
• Medications § Developmental regression
• Breakthrough seizure § EEG = hypsarrhythmia
• Work up

TOPNOTCH MEDICAL BOARD PREP PEDIATRICS MAIN HANDOUT BY DRS. DE VERA AND PUNONGBAYAN Page 47 of 106
For inquiries visit www.topnotchboardprep.com.ph or email us at topnotchmedicalboardprep@gmail.com
This handout is only valid for October 2022 PLE batch. This will be rendered obsolete for the next batch since we update our handouts regularly.
TOPNOTCH MEDICAL BOARD PREP PEDIATRICS MAIN HANDOUT BY DRS. DE VERA AND PUNONGBAYAN
For inquiries visit www.topnotchboardprep.com.ph or https://www.facebook.com/topnotchmedicalboardprep/
This handout is only valid for October 2022 PLE batch. This will be rendered obsolete for the next batch since we update our handouts regularly.
• Lennox-Gastaut Syndrome Generalities in the management of Seizures and Epilepsy:
o 2 to 10 years of age • If the risk of recurrence is low (normal neurodevelopmental
o Triad: status, EEG and MRI), long term therapy is not indicated.
§ Developmental delay • AEDs reduce excitability by:
§ Multiple seizure type: absence, myoclonic, astatic and tonic o interfering with the sodium, potassium or calcium ion
§ EEG = 1-2Hz spike and slow waves, polyspike bursts in sleep channels
and slow background in wakefulness o reducing excitatory neurotransmitter release or function
✔ GUIDE QUESTION o enhancing GABAergic inhibition
Which of the following is incorrect regarding absence seizures? • medications acting on sodium channels are effective against
A. Typically starts at 5-8 years of age partial seizures
B. Usually preceded by aura
• medications acting on T-type calcium channels are effective
C. Lasts for only a few seconds
D. Does not have a post ictal period and is characterized by immediate
against absence seizures
resumption of what the patient was doing before the seizure • MOST MEDICATIONS HAVE MULTIPLE MOA
Absence seizures (formerly called ‘petit mal’) involve brief staring • TREATMENT IS INDIVIDUALIZED
spells that usually last for less than 15 seconds; usually resolves 2-5
years after its onset, usually at puberty
Dr. Punongbayan

Adapted from Nelson textbook of Pediatrics 20th ed.


ANTIEPILEPTIC DRUGS ✔ GUIDE QUESTIONS
Disease 1st line AED Alternative A. Benign Febrile Seizures C. Meningitis
Focal seizures Oxcarbazepine and B. Encephalitis D. Brain abscess
-- In relation to the case above, what is the next best step in the
and epilepsies Carbamazepine
Absence Lamotrigine and management of this patient?
Ethosuximide A. EEG D. cranial CT
seizures Valproate
Juvenile B. blood culture E. none of the above
Valproate and C. lumbar tap
Myoclonic --
Lamotrigine
Epilepsy
Clobazam, Valproate, SIMPLE FEBRILE SEIZURE
Lennox-Gestaut Topiramate, COMPLEX FEBRILE SEIZURE
-- (70-75%)
syndrome Lamotrigine, +/- • generalized • focal
Rufinamide
• less than 15 minutes • more than 15 minutes
Infantile Adrenocorticotropic
-- • no recurrence in the 1st 24 • may recur in the 1st 24
spasms hormone (ACTH)
Valproate + hours hours
Dravet benzodiazepine • absent focal signs in the • with focal signs in the post-
--
Syndrome (Clobazam or post-ictal period ictal period
Clonazepam)
Benzodiazepines, SUPPLEMENT: QUICK SHEET
Benign
Clonazepam,
Myoclonic Valproate Child Neurology Society of the Philippines Guidelines
Lamotrigine and
Epilepsy on a first febrile seizure (2017)
Topiramate
Severe • Lumbar puncture should be performed in all patients below
Topiramate, Clobazam,
Myoclonic
Valproate, Zonisamide
-- 18 months old for a first febrile seizure.
Epilepsy o For 18 months old and older, LP should be done in the
Partial and Phenobarbital, presence of clinical signs of meningitis.
Oxcarbazepine,
secondary Topiramate, o Neuroimaging studies should not be routinely done in
Levetiracetam,
generalized Lamotrigine,
Carbamazepine, children for a first simple febrile seizure.
tonic and clonic Clobazam or
seizures
Valproic acid
Clonazepam o Children who had febrile seizure have increased odds of
developing epilepsy (2-7%).
✔ GUIDE QUESTIONS • The use of continuous anticonvulsants is not recommended
A 3-year-old male presents to the ED due to seizures. His parents in children after a first febrile seizure.
describe the seizures as jerky movements of all of the extremities
lasting for about a minute. The patient has been having cough and colds
• The use of intermittent anticonvulsants (whether Diazepam
for the past 2 days, a sore throat, and a temperature of 39°C. This is the or Phenobarbital) is not recommended for the prevention of
first episode and the PE reveals a crying anxious child, no signs of recurrent febrile seizures.
nuchal rigidity, erythematous pharyngeal mucosa. What is the most • EEG should not be routinely requested for in children with a
likely diagnosis in this case? first simple febrile seizure.
TOPNOTCH MEDICAL BOARD PREP PEDIATRICS MAIN HANDOUT BY DRS. DE VERA AND PUNONGBAYAN Page 48 of 106
For inquiries visit www.topnotchboardprep.com.ph or email us at topnotchmedicalboardprep@gmail.com
This handout is only valid for October 2022 PLE batch. This will be rendered obsolete for the next batch since we update our handouts regularly.
TOPNOTCH MEDICAL BOARD PREP PEDIATRICS MAIN HANDOUT BY DRS. DE VERA AND PUNONGBAYAN
For inquiries visit www.topnotchboardprep.com.ph or https://www.facebook.com/topnotchmedicalboardprep/
This handout is only valid for October 2022 PLE batch. This will be rendered obsolete for the next batch since we update our handouts regularly.
WHEN TO DO LUMBAR TAP? ✔ GUIDE QUESTION
• all infants younger than 6 mo of age who present with fever and Which of the ff. factors if present will give the greatest risk for
seizure or if the child is ill-appearing or at any age if there are occurrence of subsequent epilepsy in this child?
clinical signs or symptoms of concern A. Focal complex febrile seizure
B. Neurodevelopmental abnormalities
• Is an option in a child 6-12 mo of age who is deficient in
C. Family history of epilepsy
Haemophilus influenzae type b and Streptococcus pneumoniae D. Recurrent febrile seizures
immunizations or for whom immunization status is unknown
RISK FACTORS FOR OCCURRENCE OF
✔ GUIDE QUESTIONS SUBSEQUENT EPILEPSY AFTER A FEBRILE SEIZURE
Adapted from Nelson Textbook of Pediatrics, 20th ed.
Which of the ff. if present in the above case is a major factor that can
RISK FOR
increase the risk of recurrence of febrile seizures?
A. Fever of 38-39°C RISK FACTOR SUBSEQUENT
B. Family history of febrile seizures EPILEPSY
C. Male gender Simple febrile seizure 1%
D. Hyponatremia Recurrent febrile seizures 4%
Complex febrile seizures (>15
RISK FACTORS FOR RECURRENCE OF FEBRILE SEIZURES 6%
min or recurrent within 24hrs
Adapted from Nelson Textbook of Pediatrics, 20th ed.
Fever <1 hr before febrile seizure 11%
• MAJOR
Family history of epilepsy 18%
o Age <1 year
Complex febrile seizures (focal) 29%
o Duration of fever <24 hours
Neurodevelopmental
o Fever 38-39°C (100.4-102.2°F) 33%
abnormalities
• MINOR
o Family history of febrile seizures ✔ GUIDE QUESTIONS
o Family history of epilepsy Which of the ff. patients with a febrile seizure are indicated to have a
o Complex febrile seizure lumbar puncture?
A. 8-month-old infant
o Daycare
B. A 4-year-old with otitis media and complaining of ear pain
o Male gender C. 2-year-old child with no signs of neck stiffness
o Lower serum sodium at time of presentation D. all of the above
No risk factors = 12% recurrence
1 risk factor = 25-50% Among the choices, letter A poses the highest risk for a possible CNS
>3 risk factors = 73-100% infection thus, a lumbar tap is needed to diagnose it. Ear infections,
unless stated to be chronic, do not correlate directly with a CNS
infection. Likewise, a young child with no signs of neck stiffness or
meningeal irritation is unlikely to have meningitis.
Dr. Punongbayan

COMPARISON OF RECOMMENDATIONS FOR THE TREATMENT OF PEDIATRIC EPILEPSY


Adapted from Nelson Textbook of Pediatrics, 20th ed

SEIZURE TYPE PEDIATRIC EXPERT


FDA SIGN AAN PEDIATRIC EXPERT
CONSENSUS
OR EPILEPSY NICE (2012) ILAE (2013) CONSESNSUS SURVEY
SYNDROME APPROVED (2005) (2004) SURVEY (NORTH
(EUROPE – 2007)
AMERICA – 2005)
A: OXC
CBZ, Ezogabine,
CBZ, CLB, CBZ, GBP, B: None
Lacosamide, LEV,
LTG, OXC, CBZ, LEV, LTG, LTG, OXC, C: CBZ, PB, PHT,
Partial-onset LTG, OXC, PB, CBZ, OXC CBZ, OXC
PHT, TPM, OXC, VPA PB, PHT, TPM, VGB, VPA
Perampanel,
VGB, VPA TPM D: CLB, CZP, LTG,
PHT, TPM, VGB
ZNS
A, B: None
Not
CBZ, LEV, LTG, Not C: CBZ, VPA
BCECT None specifically CBZ, OXC VPA
OXC, VPA surveyed D: GBP, LEV, OXC,
mentiones
STM
A: ESM, VPA
Childhood
ESM, LTG, B: None
absence ESM, VPA ESM, LTG, VPA LTG ESM VPA
VPA C: LTG
epilepsy
D: None
Juvenile
LEV, LTG, TPM, Not A, B, C: None
myoclonic LEV, LTG, TPM VPA LTG, VPA VPA
VPA surveyed D: TPM, VPA
epilepsy
CLB, FLB, LTG,
Lennox-Gastaut CLB, LTG, Not
Rufinamide VPA Not reviewed LTG, VPA VPA
syndrome VPA surveyed
(atonic), TPM
Nitrazepam, ACTH, VGB
Infantile Corticosteroids,
VGB TPM, VGB, (updated IS Not reviewed ACTH, VGB VGB
spasms VGB guidelines 2012)
VPA
A: None
Primary
B: None No
generalized
LEV, LTG, TPM TPM, VPA LTG, TPM, VPA C: CBZ, PB, PHT, evidence -- --
tonic-clonic
TPM, VPA given
seizures
D: OXC
ACTH – adrenocorticotropic hormone; BCECT – Benign Childhood Epilepsy with Centrotemporal Spikes; CBZ – Carbamazepine; CLB – Clozabam; CZP – Clonazepam;
ESM – Ethosuximide; FLB – Felbamate; GBP – Gabapentin; ILAE – International League Against Epilepsy; LEV – Leviteracetam; LTG – Lamotrigine; NICE – National
Institue of Clinical Excellence; OXC – Oxcarbazepine; PB – Phenobarbital; PHT – Phenytoin; SIGN – Scottish Intercollegiate Guidelines Network; STM – Sulthiame;
TPM – Topimirate; VGB – Vigabatrin; VPA – Valproic Acid; ZNS – Zonisamide

SIMPLE FEBRILE SEIZURE COMPLEX OR COMPLICATED FS


• Most common seizure disorder during childhood • when the duration is >15 min
• rare before 9 mo and after 5 yr. of age, peak age of onset is 14– • when repeated convulsions occur within 24 hr
18 mo of age • when focal seizure activity or focal findings are present during
• usually associated with a core temperature that increases the postictal period.
rapidly to ≥39°C.
• It is initially generalized and tonic-clonic in nature, lasts a few
seconds and rarely up to 15 min, is followed by a brief postictal
period of drowsiness, and occurs only once in 24 hr.

TOPNOTCH MEDICAL BOARD PREP PEDIATRICS MAIN HANDOUT BY DRS. DE VERA AND PUNONGBAYAN Page 49 of 106
For inquiries visit www.topnotchboardprep.com.ph or email us at topnotchmedicalboardprep@gmail.com
This handout is only valid for October 2022 PLE batch. This will be rendered obsolete for the next batch since we update our handouts regularly.
TOPNOTCH MEDICAL BOARD PREP PEDIATRICS MAIN HANDOUT BY DRS. DE VERA AND PUNONGBAYAN
For inquiries visit www.topnotchboardprep.com.ph or https://www.facebook.com/topnotchmedicalboardprep/
This handout is only valid for October 2022 PLE batch. This will be rendered obsolete for the next batch since we update our handouts regularly.

STATUS EPILEPTICUS MANAGEMENT OF FEBRILE SEIZURES:


• It is defined as continuous seizure activity or recurrent • general, antiepileptic therapy, continuous or intermittent, is
seizure activity without regaining of consciousness lasting not recommended for children with 1 or more simple febrile
for more than 5 min as part of an operational definition put seizures
forth within the past few years. • Seizure longer than 5 min, acute treatment with diazepam,
• In the past, the cutoff time was 30 min, but this has been reduced lorazepam, or midazolam is needed
to emphasize the risks involved with the longer duration. • Antipyretics can decrease the discomfort of the child but do not
• Convulsive and Non-convulsive types reduce the risk of having a recurrent febrile seizure
• Refractory status epilepticus is status epilepticus that has ✔ GUIDE QUESTION
failed to respond to therapy, usually with at least 2 medications A 5-year-old male patient presented to the ED with a single episode of
(such as a benzodiazepine and another medication). seizure characterized as generalized tonic clonic lasting less than a
minute. Two weeks prior, he had low grade fever of 38°C, dry cough,
• Management of Status Epilepticus:
headache, irritability, drowsiness, and malaise which was dismissed as
o securing airway, breathing, and circulation an episode of flu. At present, patient is lethargic, (+) nuchal rigidity, no
o Determine etiology other focal neurologic deficits observed. (-) vomiting, CXR was normal,
o glucose, sodium, calcium, magnesium, complete blood count, lumbar puncture revealed WBC 200 with lymphocyte predominance,
basic metabolic panel, CT scan, and continuous EEG, are glucose 30mg/dL, protein is 1000mg/dL. What is the most likely
needed for all patients diagnosis?
o Other tests (ex MRI, LP) depending on suspected clinical condition A. Bacterial meningitis C. Aseptic meningitis
o initial emergent therapy usually involves intravenous B. Viral meningitis D. TB meningitis
diazepam, lorazepam, or midazolam The given LP findings point to a bacterial cause. Letter A is a possible
o If intravenous access is not available, buccal or intranasal answer. However, the history of chronic cough increased the
midazolam, intranasal lorazepam, or rectal diazepam are possibility of a tuberculous process.
Dr. Punongbayan
effective options.
o “The strongest evidence for initial and emergent therapy is for Triad of Imaging Findings in TB Meningitis?
diazepam or lorazepam, followed by Phenytoin / • Hydrocephalus
Fosphenytoin and phenobarbital, then valproate and • Basal Enhancements
levetiracetam” –Nelson 20th • Infarcts
✔ GUIDE QUESTION
In patients developing febrile status epilepticus, what viral infection is HYDROCEPHALUS
most commonly associated in one third of the cases? • How does hydrocephalus develop?
A. Measles C. HHV-6 and HHV7 1. impaired circulation & absorption of CSF
B. Rhinovirus D. Enterovirus
2. from increased production of CSF
Due to the high fever and the ability of the virus to cross the blood-
brain barrier, 10-15% of children with Roseola will experience febrile TYPE DESCRIPTION CAUSE
seizure though it is mild and self-limited.
Dr. Punongbayan Obstructive Obstruction Abnormality of the
or non- within the aqueduct or a lesion
OTHER FACTS ABOUT FEBRILE SEIZURES
communicating ventricular in the 4th ventricle
• Simple febrile seizures do not have an increased risk of mortality
type system (aqueductal stenosis)
• Complex febrile seizures may have an approximately 2-fold
Obliteration of the
long-term increase in mortality (probably secondary to a co- Nonobstructive Follows a
subarachnoid
existing pathology) or subarachnoid
cisterns
• Patients with febrile seizures do not have any increase in the communicating hemorrhage
Malfunction of the
incidence of abnormalities of behavior, scholastic performance, type Leukemic infiltrates
arachnoid villi
neurocognitive function, or attention.
• Febrile seizures risk of recurrence ALL ABOUT CSF
o 30% of those experiencing a first episode • Where is CSF produced?
o 50% after 2 or more episodes o Choroid plexus epithelium within the cerebral ventricles
o 50% of infants younger than 1 year old at febrile seizure onset SUPPLEMENT: QUICK SHEET
• Only 2-7% of children who experience febrile seizures proceed CSF Flow Mnemonic
to develop epilepsy later in life Come Let Me Treat Sisa For Lunch Maybe Somewhere in Ayala
Choroid Plexus
APPROACH TO PATIENT WITH FEBRILE SEIZURES
Adapted from Nelson Textbook of Pediatrics, 20th ed. Lateral Ventricle
Foramen of Monro (Interventricular foramen)
Third Ventricle
Aqueduct of Sylvius (Cerebral aqueduct)
Fourth Ventricle
Foramina of Luschka
Foramen of Magendie
Superior Sagittal Sinus
Arachnoid Villi

• How is CSF reabsorbed?


o Arachnoid villus cells, which are located in the superior sagittal
sinus, return CSF to the bloodstream within vacuoles (via a
process called pinocytosis)

© Topnotch Medical Board Prep


TOPNOTCH MEDICAL BOARD PREP PEDIATRICS MAIN HANDOUT BY DRS. DE VERA AND PUNONGBAYAN Page 50 of 106
For inquiries visit www.topnotchboardprep.com.ph or email us at topnotchmedicalboardprep@gmail.com
This handout is only valid for October 2022 PLE batch. This will be rendered obsolete for the next batch since we update our handouts regularly.
TOPNOTCH MEDICAL BOARD PREP PEDIATRICS MAIN HANDOUT BY DRS. DE VERA AND PUNONGBAYAN
For inquiries visit www.topnotchboardprep.com.ph or https://www.facebook.com/topnotchmedicalboardprep/
This handout is only valid for October 2022 PLE batch. This will be rendered obsolete for the next batch since we update our handouts regularly.
✔ GUIDE QUESTION SUPPLEMENT: QUICK SHEET
A 5-year-old male patient presented to the ED with confusion and Absolute contraindications to a lumbar puncture are the
lethargy. 1 week prior, he was noted to have low grade fever of 38°C, following:
productive cough, headache, irritability, drowsiness, anorexia, and 1. Signs of elevated intracranial pressure (decreased or fluctuating
malaise. No treatment was sought initially. At present, patient is level of consciousness, relative bradycardia and hypertension,
lethargic, HR 120, RR 22, T 39°C, (+) nuchal rigidity, no other focal focal neurological signs, abnormal posturing, unequal, dilated or
neurologic deficits observed. (-) vomiting, lumbar puncture revealed poorly responsive pupils, papilledema, and abnormal Doll’s eye
WBC 2000 with neutrophil predominance, glucose 20mg/dL, protein is movement);
500mg/dL. What is the most likely diagnosis? 2. Local infection at desired puncture site; and
A. Bacterial meningitis 3. Radiological signs (in cranial scan or MRI) of obstructive
B. Viral meningitis hydrocephalus, cerebral edema or herniation and the presence of
C. Aseptic meningitis an intracranial mass lesion or midline shift
D. TB meningitis Relative contraindications (lumbar puncture may be done but
The presence of a bulging fontanel in infants increase the probability only after appropriate diagnostic and therapeutic interventions
of bacterial meningitis by 3.5 times and neck stiffness increased the are done):
likelihood of acute bacterial meningitis by eight-fold. Complex 1. Signs of shock, sepsis or hypotension
seizures double the risk for bacterial meningitis. 2. Coagulation defects [disseminated intravascular coagulopathy
A systematic review of the meningeal signs such as neck stiffness, (DIC), platelet count <50,000/mm3, and therapeutic use of
Brudzinski’s and Kernig’s signs, as basis for the diagnosis of meningitis warfarin]
proved to be variable in sensitivity and specificity. Thus, these signs of 3. Focal neurological deficit (especially for suspected posterior fossa
meningeal irritation are not reliably predictive of meningitis if used lesions)
alone. 4. Glasgow coma score < 8
Dr. Punongbayan 5. Epileptic seizures
Philippine CPG on the Diagnosis and Management of Acute Bacterial Meningitis.
PIDSP Technical Working Group. PIDSP Journal vol. 16 no. 2 July-December 2015

CEREBROSPINAL FLUID FINDINGS IN CNS DISORDERS


Adapted from Nelson Textbook of Pediatrics, 20th ed.
PRESSURE PROTEIN GLUCOSE
CONDITION LEUKOCYTES (mm3) COMMENTS
(mm H2O) (mg/dL) (mg/dL)
>50 (or 75%
Normal 50-80 <5, ≥75% Lymphocytes 20-45 --
serum glucose)
COMMON FORMS OF MENINGITIS
Usually Decreased, usually Organisms usually seen on
Acute bacterial 100-10,000 or more; usually Usually
elevated <40 (or <50% Gram stain and recovered by
meningitis 300-2,00; PMNs predominate 100-500
(100-300) serum glucose) culture
5-10,000; PMNs usual but
Partially treated Normal or mononuclear cells may Usually Normal or Organisms may be seen on
bacterial meningitis elevated predominate if pretreated for 100-500 decreased Gram stain
extended period of time
Generally
Rarely >1,000 cells. Eastern HSV encephalitis is suggested by
normal; may
equine encephalitis and focal seizures or by focal
Normal or be decreased
lymphocytic choriomeningitis findings on CT or MRI scans or
Viral meningitis or slightly Usually 50- to <40 in some
may have cell counts of several EEG. Enteroviruses and HSV
meningoencephalitis elevated (80- 200 viral diseases,
thousand. PMNs early but infrequently recovered from
150) particularly
mononuclear cells predominate CSF. HSV and enteroviruses
mumps (15-
through most of the course may be detected by PCR of CSF
20% of cases)
UNCOMMON FORMS OF MENINGITIS
100-3,000; <50 in most Acid-fast organisms almost
may be cases; never seen on smear.
10-500; PMNs early, but
Tuberculous Usually higher in decreases with Organisms may be recovered in
lymphocytes predominate
meningitis elevated the time if culture of large volumes of CSF.
through most of the course
presence of treatment is Mycobacterium tuberculosis may
block not provided be detected by PCR of CSF
BRAIN ABSCESSES AND PARAMENINGEAL FOCUS
5-200; CSF rarely acellular; Normal unless
Usually lymphocytes predominate; if abscess No organisms on smear or
Brain abscess elevated abscess ruptures into ventricle, 75-500 ruptures into culture unless abscess ruptures
(100-300) PMNs predominate and cell ventricular into ventricular system
count may reach >100,000 system
No organisms on smear or
Usually
culture of CSF unless meningitis
Subdural empyema elevates 100-5,000; PMNs predominate 100-500 Normal
is also present; organisms found
(100-300)
on tap of subdural fluid

MENINGITIS CPG on Acute Bacterial Meningitis


PPS and PIDSP, 2015
• What if the child was started on oral antibiotics prior to LP? • Signs and symptoms of bacterial meningitis associated with
o The presumptive diagnosis of bacterial meningitis can be neutrophilia and increased serum CRP are highly suggestive
made despite negative cultures because pleocytosis, high CSF of bacterial meningitis.
protein level, and low CSF sugar persist for several days even • Procalcitonin may be used to differentiate viral from bacterial
after antibiotics. meningitis. In situations wherein a CSF analysis cannot be
• What do you do next if the LP is traumatic? performed immediately, it may be used as a basis to start
o It is prudent to rely on bacteriologic results because the antibiotics. However, it should not replace CSF analysis and
Gram stain, culture & glucose level may not be influenced culture to diagnose bacterial meningitis.
by a traumatic LP. MNEMONIC COMPLICATIONS OF BACTERIAL MENINGITIS
Note: a non-traumatic sample is CSF with less than 1,000 red blood cells
per hpf • H - hydrocephalus
• How reliable is a blood culture in cases of bacterial • A - abscess
meningitis? • C – cerebritis / cranial nerve involvement
o Blood cultures should be performed in all patients with • T - thrombosis
suspected meningitis. It reveals the responsible bacteria in • I - infarct
up to 80-90% of cases of meningitis. • V – ventriculitis / vasculopathy
• E – extra-axial fluid collection (empyema)
*** MRI > CT scan
TOPNOTCH MEDICAL BOARD PREP PEDIATRICS MAIN HANDOUT BY DRS. DE VERA AND PUNONGBAYAN Page 51 of 106
For inquiries visit www.topnotchboardprep.com.ph or email us at topnotchmedicalboardprep@gmail.com
This handout is only valid for October 2022 PLE batch. This will be rendered obsolete for the next batch since we update our handouts regularly.
TOPNOTCH MEDICAL BOARD PREP PEDIATRICS MAIN HANDOUT BY DRS. DE VERA AND PUNONGBAYAN
For inquiries visit www.topnotchboardprep.com.ph or https://www.facebook.com/topnotchmedicalboardprep/
This handout is only valid for October 2022 PLE batch. This will be rendered obsolete for the next batch since we update our handouts regularly.

Patients presenting with CNS infections POOR PROGNOSTICATING FACTORS


• Poor prognostic factors on presentation
• Prefrontal
o include hypothermia or extreme hyperpyrexia
headache Naegleria fowleri
• Swimming in o hypotension or shock
• High fever (brain-eating
warm water o purpura fulminans
• Disturbance in amoeba)
o seizures
smell
o leukopenia
• Systemic disease o thrombocytopenia (including DIC)
Borrelia
• Bell’s Palsy • Cutaneous lesions o Acidosis
burgdorferi
• Carditis o high circulating levels of endotoxin and tumor necrosis factor-α
• History of GIT Campylobacter • The presence of petechiae for <12 hr before admission, absence
infection jejuni of meningitis, and low or normal erythrocyte sedimentation rate
• Ascending
• History of indicate rapid, fulminant progression and poorer prognosis.
paralysis Haemophilus
Respiratory
influenza type b
Infection ✔ GUIDE QUESTIONS
• History of viral A 16-year-old female patient presents to the ED with fever, altered
• Lethargy and infection sensorium, nausea, and vomiting. She started having fever 12 hours ago
Reye Syndrome accompanied by sore throat and muscle pains. At present examination,
irritability • Chicken pox
she has multiple purpuric lesions over the lower extremities, her blood
• Aspirin use
pressure is 80/60, HR 130 RR 20, T 39.5°C. Her parents say that she
• Previous history of lives in a college dormitory and she doesn’t have a boyfriend as far as
• Ocular nerve Tuberculosis they know. What is the most likely diagnosis in this patient?
TB meningitis
palsies • Active A. Herpes simplex encephalitis
Tuberculosis B. Disseminated Gonococcal infection
C. Meningococcemia
✔ GUIDE QUESTION D. Henoch Schönlein Purpura
A 9-year-old patient with Tetralogy of Fallot is admitted due to In relation to the case above, which among the ff. diagnostic test
headache, fever and seizures. Cranial imaging showed multi-loculated establishes a definitive diagnosis?
abscess. The patient is being managed empirically with antibiotics and A. Culture of the causative agent in the blood
is on the second week of treatment. What will be the next appropriate B. Sputum gram stain and culture
management for this patient? C. Cranial CT scan
A. Continue antibiotics for 4-6 weeks D. Viral PCR
B. Serial monitoring of cranial CT scan In relation to the above case, carriage of the causative organism is
C. Surgical management for the abscess usually in the
D. Send patient home on oral antibiotics A. Nasopharynx C. Bronchi
B. Gallbladder D. Skin
BRAIN ABSCESS In relation to the case above, if the patient has a documented case of
severe anaphylaxis to penicillin according to her parents, all of the ff.
• Most common between 4-8 years old antibiotics are appropriate to give except?
• Etiology: embolization due to CHD with R->L shunts, meningitis, A. Chloramphenicol C. Meropenem
chronic OM & mastoiditis, face & scalp infections, orbital B. Ciprofloxacin D. Doxycycline
cellulitis, dental infections, penetrating head injuries, VP shunt
infections MENINGOCOCCEMIA
• Cerebrum 80%; occipital lobe, cerebellum & brain stem 20%
FACTS ABOUT MENINGOCOCCEMIA:
• Majority are single abscess; 30% multiple
• Mode of transmission: by aerosol droplets or through contact
with respiratory secretions
IMPORTANT FACTS ABOUT BRAIN ABSCESS:
• Period of communicability: until 24 hours after initiating
• Causes: S. aureus, Streptococci, anaerobes, Gram (-) aerobic effective treatment
bacilli (Proteus, Pseudomonas, Haemophilus, Citrobacter)
• Incubation period: 1-10 days or less than 4 days
• Early stage: nonspecific symptoms
• *seizures & focal neurologic signs occur less frequently than
• Vomiting, severe headache, seizures, focal neurologic signs, in patients with meningitis due to H. influenza b &
papilledema pneumococcus
• Dx: (+) blood culture in 10%; CSF not done to avoid herniation;
cranial CT scan & MRI (most reliable methods) CLINICAL MANIFESTATIONS
• Asymptomatic colonization to fulminant sepsis
MANAGEMENT OF BRAIN ABSCESS
• Infections of the GUT, conjunctiva, pharynx, CNS, heart, skin,
• Empiric antibiotics depend on the probable pathogenesis & adrenals
most likely organism
• Diffuse adrenal hemorrhage without vasculitis, DIC, coma, and
• Unknown cause: 3rd generation cephalosporin + Metronidazole death → Waterhouse-Friderichsen syndrome (fulminant
• Head trauma or neurosurgery: Oxacillin or Vancomycin with 3rd cases)
generation cephalosporin + Metronidazole
• Meropenem as monotherapy How is it different from other bacterial meningitis?
• Due to CHD: Penicillin + Metronidazole • Headache
• Infected VP shunt: Vancomycin + Ceftazidime • Photophobia
• Immunocompromised: broad spectrum + Amphotericin B • Lethargy
• Aspiration for encapsulated abscesses • Vomiting
• Indications for surgery: • Nuchal rigidity
1. (+) gas in the abscess
2. Multiloculated abscesses FREQUENCY OF SIGNS/SYMPTOMS
3. Posterior fossa location
• Fever (81%)
4. Fungal cause
• Petechiae and/or purpura (50%)
5. Assoc. infections like mastoiditis, periorbital abscess,
sinusitis • Hypotension or poor peripheral perfusion (41%)
• Vomiting (34%)
WHAT TO EXPECT: • Lethargy (30%)
• Duration of antibiotics: 4-6 wks • Irritability (21%)
• Rhinorrhea (10%)
• High mortality: multiple abscesses, coma
o * seizures (6%)
• Long-term sequela: behavior & learning problems,
hydrocephalus, seizures, hemiparesis

TOPNOTCH MEDICAL BOARD PREP PEDIATRICS MAIN HANDOUT BY DRS. DE VERA AND PUNONGBAYAN Page 52 of 106
For inquiries visit www.topnotchboardprep.com.ph or email us at topnotchmedicalboardprep@gmail.com
This handout is only valid for October 2022 PLE batch. This will be rendered obsolete for the next batch since we update our handouts regularly.
TOPNOTCH MEDICAL BOARD PREP PEDIATRICS MAIN HANDOUT BY DRS. DE VERA AND PUNONGBAYAN
For inquiries visit www.topnotchboardprep.com.ph or https://www.facebook.com/topnotchmedicalboardprep/
This handout is only valid for October 2022 PLE batch. This will be rendered obsolete for the next batch since we update our handouts regularly.
These pictures were taken on DRUG OF CHOICE
succeeding days of the same • Penicillin G 250,000-300,000 U/kg/day IV in 4-6 divided doses
patient who presented with acute for at least 5-7 days (DOC)
onset of fever, petechiae, and
• Ceftriaxone IM or IV (100 mg/kg/day once or twice a day) or
decreased activity and sensorium.
NOTE the quick progression of Cefotaxime IM or IV (200-300 mg/kg/day every 6 or 8 hours)
petechia to purpuric and * clinical improvement within 24-72 hrs
ecchymotic lesions in a span of less
than 24 hours
Dr. Punongbayan
TREATMENT OF NEISSERIA MENINGITIDIS INVASIVE INFECTIONS
Adapted from Nelson Textbook of Pediatrics, 20th ed
ROUTE OF DOSING MAXIMUM
DRUG DOSE NOTES
ADMINISTRATION INTERVAL (HR) DAILY DOSE
Does not clear carriage and
300,00 12-24 million
Penicillin G IM or IV 4-6 “prophylaxis” is required at
units/kg/day units
the end of treatment
Does not clear carriage and
200-400
Ampicillin IM or IV 6 6-12g “prophylaxis” is required at
mg/kg/day
the end of treatment
200-300 Recommended in the
Cefotaxime IM or IV 6-8 8-12g
mg/kg/day neonate
Preferred treatment as only
100
Ceftriaxone IM or IV 12-24 2-4g once or twice daily and may
mg/kg/day
reduce skin complications
ALTERNATIVE THERAPY IN THE FACE OF LIFE-THREATENING β-LACTAM ALLERGY
Chloramphenicol 50-100
IV 6 2-4g --
Monitor blood levels to avoid toxicity mg/kg/day
Ciprofloxacin 30-40
IV 12 1-1.5g --
For individuals ≥18 y/o mg/kg/day
Meropenem
60-120
Rate of cross-reactivity in penicillin- IV 8 1.5-6g --
mg/kg/day
allergic adults is 2-3%
Who are considered exposed? SUPPLEMENT: QUICK SHEET
• Household, school or day care contacts during the 7 days Empiric antibiotics for bacterial meningitis:
before onset of illness should receive antibiotic prophylaxis 1. For neonates: Ampicillin OR Cefotaxime PLUS
• Prophylaxis NOT routinely recommended for medical personnel aminoglycoside
except those with intimate exposure (intubation, suctioning,
mouth-to-mouth resuscitation) CEFTRIAXONE
What do you give to exposed persons? IN THE NEWBORN
• Children: Rifampicin 10 mg/kg po every 12 hrs for a total of 4 https://qrs.ly/ffdul3u
doses (max 600 mg); 5 mg/kg/dose for <1 month old
• Or Ceftriaxone 125 mg single dose IM for < 12 yrs old 2. 1 month–18 years old: Ceftriaxone OR Chloramphenicol
• > 18 yrs old: Ciprofloxacin 500 mg po as a single dose 3. For Hib – Ceftriaxone for 7-10 days
4. For S. pneumoniae – Penicillin for 10-14 days (alternative:
ANTIBIOTIC PROPHYLAXIS TO PREVENT NEISSERIA Ceftriaxone)
MENINGITIDIS INFECTION 5. For N. meningitidis – Penicillin for 7 days
Adapted from Nelson Textbook of Pediatrics, 20th ed
6. For E. coli – Cefotaxime for 21 days
DRUG DOSE DURATION
7. For GBS – Cefotaxime OR Ceftriaxone for 14 days
RIFAMPIN Philippine CPG on the Diagnosis and Management of Acute Bacterial Meningitis.
5 mg/kg PO every 12 PIDSP Technical Working Group. PIDSP Journal vol. 16 no. 2 July-December 2015
Infants < 1 mo 2 days (4 doses)
hours
10 mg/kg PO every ✔ GUIDE QUESTION
Children ≥1 mo 12 hours (Max: 2 days (4 doses) A term baby boy was born to a G1P1 mother via NSD with an APGAR
600mg) score of 8 & 9. He was noted to have a fleshy erythematous mass on the
600 mg PO every 12 midline of his lower back. There were no other associated signs.
Adults 2 days (4 doses)
hours Mother is asking if how she may be able to prevent this in the future?
CEFTRIAXONE A. Folic Acid Supplementation 0.4mg OD
Children <15 yrs 125 mg IM 1 dose B. Folic Acid Supplementation 4mg OD
Children ≥15 yrs 250 mg IM 1 dose C. Vitamin B12 supplementation
CIPROFLOXACIN D. This cannot be prevented
20 mg/kg (Max: 500 The recommended dose to prevent NTDs during the first pregnancy:
Children ≥1 mo 1 dose
mg) PO 400 micrograms or 0.4 mg of folic acid
In succeeding pregnancies or recurrent cases of NTDs: 4000
PATIENT PRESENTING WITH CNS INFECTIONS micrograms or 4 mg of folic acid (although an article in Fetal
Sepsis, seizure, Diagnosis and Therapy 2018; 44:161-165 by Dolin, et al concluded
irritability, Gram Negative that the 4 mg dose was too much and that it needed further studies to
Px is <2 months old
lethargy, bulging of Bacteria / GBS, reconsider minimizing the dose)
fontanelles, rigidity Dr. Punongbayan

No mention of proper
Hib
vaccination, <5 y/o NEURAL TUBE DEFECTS
Properly vaccinated
WHAT IS THE UNDERLYING CAUSE?
child, abrupt in onset,
Meningococcemia • Due to failure of the neural tube to close spontaneously
toxic looking with
Headache, fever, rashes all over between the 3rd & 4th week of in utero development
confusion, lethargy, Young adults Pneumococcus • Hyperthermia, drugs, malnutrition, chemicals, maternal obesity
nuchal rigidity, Renal transplant or diabetes, and genetic determinants adversely affect CNS
Listeria
vomiting patient development from the time of conception
Gradual onset of • Myelomeningocele – the most severe form of neural tube
signs and symptoms, Enterovirus defect (dysraphism); occurs in 1 in 1400 live births
not toxic looking
RBCs in the CSF
HSV
examination

TOPNOTCH MEDICAL BOARD PREP PEDIATRICS MAIN HANDOUT BY DRS. DE VERA AND PUNONGBAYAN Page 53 of 106
For inquiries visit www.topnotchboardprep.com.ph or email us at topnotchmedicalboardprep@gmail.com
This handout is only valid for October 2022 PLE batch. This will be rendered obsolete for the next batch since we update our handouts regularly.
TOPNOTCH MEDICAL BOARD PREP PEDIATRICS MAIN HANDOUT BY DRS. DE VERA AND PUNONGBAYAN
For inquiries visit www.topnotchboardprep.com.ph or https://www.facebook.com/topnotchmedicalboardprep/
This handout is only valid for October 2022 PLE batch. This will be rendered obsolete for the next batch since we update our handouts regularly.
MYELOMENINGOCELE EPIDEMIOLOGIC DATA:
• How can this be prevented? • F > M: adolescents
o maternal periconceptual use of folic acid reduces its • Younger than 10 yrs old: M > F
incidence by at least 50% (started before conception until at • More than 50% undergo spontaneous prolonged remission
least the 12th wk of gestation when neurulation is complete) after the 10th birthday
– 0.4 mg once a day
• What parts are affected? PATHOGENESIS:
o Dysfunction of the skeleton, skin, GUT, PNS, CNS How do you explain the aura in migraine?
o May be located anywhere along the neuroaxis (lumbosacral Cortical spreading depression (CSD) – a phenomenon associated
75%) with high CNS hydrogen & potassium ions with the release of
glutamate & nitrous oxide → leads to excitation of trigeminal-
vascular system → activates the release of VIP → vasodilation →
extravasation of plasma proteins from the dural vessels →
localized inflammation of dural vessels → excitation of pain
sensitive receptor → pain

When do you request for imaging studies?


Indications for cranial CT or MRI:
1. abnormal neurologic signs
2. behavioral changes, recent school failure, fall-off in linear
growth rate
3. headache awakens the child during sleep
4. migraine & seizure occur in the same episode
5. focal neurologic signs
http://body-disease.com/neural-tube-defects-spina-bifida-meningocele-and-myelomeningocele/
6. cluster headaches esp.in <5 yrs old
• What are the manifestations?
o flaccid paralysis of the LE TREATMENT OF MIGRAINE
o absence of DTRs • Avoid stimuli: stress, fatigue, anxiety
o lack of response to pain & touch • Analgesics (Acetaminophen 15 mg/kg or Ibuprofen 7.5-10
o hip subluxation mg/kg) or antiemetics (parenteral Metoclopramide)
o clubfeet
• What do you do with status migrainosus (persistent headache
o bowel & bladder incontinence
lasting > 3 days)? → Prochlorperazine IV 0.15 mg/kg max. 10 mg
o associated with hydrocephalus (type II Chiari)
When do you give prophylactic therapy?
ARNOLD CHIARI MALFORMATIONS
• More than 2-4 severe episodes monthly
• Chiari Type I: elongation of cerebellar tonsils extending in • Unable to attend school regularly
vertebral canal • PedMIDAS > 20
• Chiari Type II: elongation of inferior vermis and brainstem with • Drugs:
their displacement in cervical spinal canal with o Propranolol - 10-20 mg tid for > 7 yrs old
myelomeningocele and hydrocephalus; posterior fossa is o Flunarizine - 5 mg at bedtime
shallow, torcular is low, sometimes craniolacuna,
polymicrogyria
• Chiari Type III: extremely rare, bony defect at occipito-cervical
level with herniation of cerebellum into encephalocele

Piper R J, Pike M, Harrington R, Magdum S A. Chiari malformations: principles of diagnosis and management BMJ 2019;
365 :l1159 doi:10.1136/bmj.l1159
✔ GUIDE QUESTION
An 11-year-old boy presents with one-sided headache, on & off,
associated with nausea for the past 4 days. There was no fever, cough,
colds, or other symptoms. The parent claimed a family history of the
same nature of the headache. His neurologic examination did not show
any deficits. What is the appropriate management for this child?
A. Do Cranial Imaging and EEG
B. Do cranial imaging alone
C. Analgesics and reassurance
D. None of the above
NEUROCUTANEOUS SYNDROMES
• Tuberous sclerosis
• Neurofibromatosis
HEADACHE • Most disorders are familial
APPROACH TO HEADACHE • Arise from a defect in differentiation of the primitive
• What are the features of migraine? ectoderm
o Recurrent headache with symptom-free intervals & at least 3
of the ff: (F-R-U-A-N-T)
1. (+) Family history TUBEROUS SCLEROSIS
2. Relief following sleep FACTS ABOUT TUBEROUS SCLEROSIS
3. Unilateral location • Inherited as AD trait
4. Associated aura • Heterogeneous disease with a wide clinical spectrum varying
5. Abdominal pain from severe MR & incapacitating seizures to normal
6. Nausea & vomiting intelligence and a lack of seizures, often within the same family
7. Throbbing in character • Affects many organs like skin, brain, heart, kidney, eyes, lungs,
bone
TOPNOTCH MEDICAL BOARD PREP PEDIATRICS MAIN HANDOUT BY DRS. DE VERA AND PUNONGBAYAN Page 54 of 106
For inquiries visit www.topnotchboardprep.com.ph or email us at topnotchmedicalboardprep@gmail.com
This handout is only valid for October 2022 PLE batch. This will be rendered obsolete for the next batch since we update our handouts regularly.
TOPNOTCH MEDICAL BOARD PREP PEDIATRICS MAIN HANDOUT BY DRS. DE VERA AND PUNONGBAYAN
For inquiries visit www.topnotchboardprep.com.ph or https://www.facebook.com/topnotchmedicalboardprep/
This handout is only valid for October 2022 PLE batch. This will be rendered obsolete for the next batch since we update our handouts regularly.
What genes are mutated in this condition? NEUROFIBROMATOSIS 1
• Tuberous sclerosis complex gene 1 (TSC1) and gene 2 (TSC2) • NF-1: most prevalent type: diagnosed when any 2/7 of the ff are
are mutated. present:
• TSC1 encodes for the protein hamartin and TSC2 encodes for 1. 6 or more café au lait macules >5mm in diameter in
the protein tuberin. prepubertals & >15mm in postpubertal individuals: present at
• Both act as tumor suppressor genes → wide expressivity of the birth & increase in size, number & pigmentation with predilection
syndrome for the trunk & extremities with sparing of the face
• Characteristic brain lesions consist of tubers located in the 2. Axillary or inguinal freckling consists of multiple
convolutions of the cerebral hemispheres → typically present in hyperpigmented areas 2-3 mm in diameter
the subependymal region → undergo calcification & project 3. 2 or more iris Lisch nodules (hamartomas located within the iris)
into the ventricular cavity → produce a candle-dripping 4. 2 or more neurofibromas (along the skin, PNS, blood
appearance vessels & within viscera) or one plexiform neurofibroma
5. Distinctive osseous lesion
6. Optic glioma
7. 1st-degree relative with NF-1 whose diagnosis was based on
the aforementioned criteria
8. Majority of mutations in NF-1 occur in the paternal germline
9. High incidence of learning disabilities

CLINICAL MANIFESTATIONS
• May present during infancy with infantile spasms
• Typical hypopigmented skin lesions: ash leaf >90% of cases
• CT scan shows calcified tubers in the periventricular area
• Childhood: generalized seizure disorder & skin lesions;
shagreen patch – roughened, raised lesion with an orange-peel
consistency located primarily in the lumbosacral region
• Adolescence: subungual or periungual fibromas from the
fingers & toes
MORE ON TUBEROUS SCLEROSIS
• 2 types of retinal lesions: mulberry tumors or round, flat gray lesions
• Rhabdomyosarcoma of the heart
• Renal manifestations: bilateral angiomyolipomas and cysts
• Dx: high index of suspicion when assessing a child with infantile spasm
• Tx: seizure control & baseline studies like 2-D echo, chest X-ray,
renal UTZ

NEUROFIBROMATOSIS NEUROFIBROMATOSIS 2
• Von Recklinghausen disease – AD disorder; every system may • NF-2 is diagnosed when 1 of the ff is present:
be affected; complications may be delayed for decades 1. Bilateral 8th nerve masses (acoustic neuroma)
• Result of an abnormality of neural crest differentiation & 2. Parent, sibling, or child with NF-2 & either unilateral 8th
migration during the early stages of embryogenesis nerve masses or any 2 of the ff: neurofibroma, meningioma,
glioma, schwannoma
SUPPLEMENT: QUICK SHEET
o Management: genetic counseling & early detection of treatable
IMPORTANT!
conditions or complications
“Von Recklinghausen” has 17 letters and is located on chromosome 17!
SUMMARY OF NEUROCUTANEOUS SYNDROMES
SYNDROME MANIFESTATIONS PE FINDING DIAGNOSTICS TREATMENT
Axillary or inguinal Genetic counseling & early
Neurofibromatosis Café au lait macules that
freckling, Lisch nodules, CT scan or MRI detection of treatable
(Von Recklinghausen) spare the face
optic glioma complications
Tubers in cerebrum CT scan or MRI of the
multisystemic; seizures, Seizure control;
Tuberous Sclerosis (candle-dripping); ash brain, heart, abdomen;
mental retardation multidisciplinary approach
leaf, shagreen patch 2D echo; renal UTZ
✔ GUIDE QUESTION GBS
A 10-year-old boy presented with progressive weakness in both of his • Postinfectious polyneuropathy involving mainly motor
legs that started 5 days PTC. He just recovered from a URTI about 2 ½
• Not hereditary; affects all ages
weeks PTC. On the day of consultation, he could not walk
independently. His neuro exam showed absence of patellar and • Paralysis usually follows a nonspecific viral infection (GIT or
Achilles tendon reflexes bilaterally. What is your expected CSF finding? RT) by 10 days (Campylobacter jejuni and herpesvirus)
A. Increased protein, normal glucose, normal cells • Weakness begins in the lower extremities & progressively
B. Increased protein, increased glucose, increased cells involves the trunk, upper limbs & bulbar muscles (Landry
C. Decreased protein, normal glucose normal cells, ascending paralysis)
D. Decreased protein, decreased glucose, decreased cells
What other physical findings are associated with this condition?
This is known as the Cytoalbuminocytologic dissociation finding in
CSF examination in GBS.
• Cranial nerve deficits leading to dysphagia, dysarthria, facial
Dr. Punongbayan weakness, papilledema, autonomic dysfunction, respiratory
muscle paralysis
GUILLAIN-BARRE SYNDROME • Miller-Fisher syndrome: acute ophthalmoplegia, ataxia, areflexia
What is the most likely diagnosis?
FACTS ABOUT GBS:
• Guillain-Barre syndrome (GBS) or acute demyelinating
• Benign clinical course with spontaneous recovery within 2-3 weeks
polyradiculoneuropathy (symmetric ascending muscle weakness
• Tendon reflexes usually the last function to recover & lower
or paralysis)
extremity weakness last to resolve
What is the etiology of this condition? • 3 clinical features are predictive of poor outcome with sequela:
• Autoimmune reaction that develops in response to a previous cranial nerve involvement, need for intubation & maximum
infection → leads to aberrant demyelination of peripheral disability at the time of presentation
nerves & ventral motor nerve roots
TOPNOTCH MEDICAL BOARD PREP PEDIATRICS MAIN HANDOUT BY DRS. DE VERA AND PUNONGBAYAN Page 55 of 106
For inquiries visit www.topnotchboardprep.com.ph or email us at topnotchmedicalboardprep@gmail.com
This handout is only valid for October 2022 PLE batch. This will be rendered obsolete for the next batch since we update our handouts regularly.
TOPNOTCH MEDICAL BOARD PREP PEDIATRICS MAIN HANDOUT BY DRS. DE VERA AND PUNONGBAYAN
For inquiries visit www.topnotchboardprep.com.ph or https://www.facebook.com/topnotchmedicalboardprep/
This handout is only valid for October 2022 PLE batch. This will be rendered obsolete for the next batch since we update our handouts regularly.
DIAGNOSTIC TESTS FOR GBS ✔ GUIDE QUESTIONS
What laboratory finding is likely in this condition? A 3-year-old boy, who lives in a farm, was brought to the ER for sudden
• CSF analysis: increased protein, normal glucose, no pleocytosis episode of decrease in sensorium. Lola reported that the patient was
• Dissociation between high CSF protein and a lack of cellular active few hours prior and crying and that she was trying to console him
by rocking him back and forth. At the ER, patient arrived in active
response is diagnostic (albuminocytologic dissociation)
seizure. Pertinent findings included wasting, bruises on the
• Reduced motor NCVs extremities, retinal hemorrhages on fundoscopy. What is your
impression?
MANAGEMENT OF GBS A. Abusive head trauma
• Acute stage: admit for observation because ascending paralysis B. Meningococcemia
may occur within 24 hrs C. Organophosphate poisoning
• Rapidly progressive ascending paralysis: IVIG (0.4 grams / kg / D. Cavernous sinus thrombosis
day for 5 consecutive days) Biomechanics of children less than 2 years old that lead to shaken
• Supportive care; prevention of ulcers baby syndrome or abusive head trauma:
• High-dose pulse methylprednisolone IV for relapses 1. Large size of the head relative to the body
2. Weakness of cervical muscle
3. High water content of the brain
✔ GUIDE QUESTION
4. Large subarachnoid space
A 5-year-old boy is brought to his doctor by his mother for a follow-up Dr. Punongbayan
appointment. Two months ago, the patient was seen for a chief
complaint of morning headache, vomiting, and decreased activity. At
this visit, the mother reports that her son continues to have worsened PEDIATRIC STROKE
symptoms as well as new onset falling and a stumbling gait. • Important cause of acquired brain injury in children & newborns
What is your most likely diagnosis? • ischemic varieties of arterial ischemic stroke (AIS) and cerebral
Answers: Medulloblastoma sinovenous thrombosis (CSVT) are more common than brain
malignancy
Consider a space-occupying lesion in young children who presents
with headache, signs of increased intracranial pressure, and change
in gait and/or behavior. ARTERIAL ISCHEMIC STROKE (AIS)
In this question, the answer is based on the most common malignant • Strokes most often involve the middle cerebral artery
brain tumor in children which is medulloblastoma. It accounts for
territory but can occur in any cerebral artery of any size
about 20% of primary CNS neoplasms and approximately 40% of all
posterior fossa tumors. • Usually delayed diagnosis
Dr. Punongbayan o Subtle and non-specific signs
o >50% of initial CT Scans are normal
MEDULLOBLASTOMA • The acute onset of a focal neurologic deficit in a child is
stroke until proven otherwise.
• Most often found in the cerebellum
• Diagnostic of choice = MRI
• Most common malignant brain tumor in children
• CT scan demonstrates mature AIS and rules out hemorrhage
• 4-8 years old; M > F
• 3 main etiologies
• MRI is used to visualize the extent of the tumor.
o Arteriopathy – disorders of cerebral arteries; leading cause
• Medulloblastomas are heterogenous enhancements in the
(>50%)
cerebellum, often invading the 4th ventricle, and can cause
o Cardiac – cardioembolic strokes; 25%
obstructive hydrocephalus.
§ Complex congenital heart disease (ex. TOF)
• Homer-Wright rosettes - circular patterns of tumor cells
o Hematologic
surrounding a center of neutrophils
§ Sickle cell anemia increases risk for stroke by 400-fold
§ Iron deficiency anemia
INFRATENTORIAL TUMORS § Coagulation disorders
• Cerebellar astrocytoma – most common & with the best
prognosis; cystic; causes hydrocephalus; resection with 90% 5- For cardioembolic etiology: maximal embolic risk is concurrent with
catheterization, surgical repair, or ventricular assistive device use.
year survival rate Dr. Punongbayan
• Medulloblastoma – < 7-year-old; can spread to extracranial sites; MANAGEMENT OF AIS
surgery + irradiation; 80-90% 5-year survival rate • Antithrombotic Strategies
• Brain stem glioma o Heparin
• Ependymoma o Aspirin
• Neuroprotective strategies
SUPRATENTORIAL TUMORS o Glucose control
• Craniopharyngioma – solid & cystic areas that tend to calcify; o Temperature control
short stature; pressure to optic chiasm produces bitemporal o Seizure prevention
visual field defects o Maintenance of cerebral perfusion
• Optic nerve glioma – decreased visual acuity & pallor of the • Secondary Stroke Prevention (antiplatelet therapy in
discs; 25% have neurofibromatosis; hyperalert & euphoric arteriopathy and anticoagulation in cardiogenic causes)
despite being emaciated; invasion of the hypothalamus leads to • Rehabilitation
obesity or DI
• Astrocytoma CEREBRAL SINOVENOUS THROMBOSIS (CSVT)
• Choroid plexus papilloma
• Greatest risk in the neonatal period
• Thrombotic occlusion of venous structures create increased ICP,
BRAIN TUMORS
cerebral edema, and venous infarction or hemorrhage
• Cranial MRI – can delineate tumors
• Clinical presentations are typically gradual, variable, and
• Cranial irradiation + chemotherapy
nonspecific compared to AIS
• Brachytherapy – implantation of radiation seeds
• Diagnostic of choice = contrast CT venography or MR
venography
✔ GUIDE QUESTIONS
• Non-contrast CT scan = low sensitivity
The most common cause of arterial ischemic stroke in pediatric
population is:
A. Arteriopathy C. Hematologic MANAGEMENT OF CSVT
B. Cardiac D. Idiopathic • Anticoagulation therapy – unfractionated or low molecular
This type of intracranial hemorrhage is almost always associated with weight heparin
trauma:
A. Subdural C. Subgaleal
B. Epidural D. Intra-ventricular

TOPNOTCH MEDICAL BOARD PREP PEDIATRICS MAIN HANDOUT BY DRS. DE VERA AND PUNONGBAYAN Page 56 of 106
For inquiries visit www.topnotchboardprep.com.ph or email us at topnotchmedicalboardprep@gmail.com
This handout is only valid for October 2022 PLE batch. This will be rendered obsolete for the next batch since we update our handouts regularly.
TOPNOTCH MEDICAL BOARD PREP PEDIATRICS MAIN HANDOUT BY DRS. DE VERA AND PUNONGBAYAN
For inquiries visit www.topnotchboardprep.com.ph or https://www.facebook.com/topnotchmedicalboardprep/
This handout is only valid for October 2022 PLE batch. This will be rendered obsolete for the next batch since we update our handouts regularly.
COMMON RISK FACTORS FOR CSVT HEMORRHAGIC STROKE (HS)
Adapted from Nelson Textbook of Pediatrics, 20th ed
• Clinical presentations vary according to location, cause, and rate
• Blood Coagulation
o Prothrombotic conditions of bleeding
§ Factor V Leiden, prothrombin gene mutation 20210A, Protein C • Acute hemorrhagic stroke
deficiency, Protein S deficiency, Antithrombin III deficiency, o instantaneous or thunderclap headache
Lipoprotein a, Antiphospholipid antibodies (lupus anticoagulant, o loss of consciousness
anticardiolipin antibodies), pregnancy / puerperium o nuchal rigidity
o Dehydration (e.g., gastroenteritis, neonatal failure to thrive) o focal neurologic deficits
o Iron-deficiency anemia
o Seizures
o Drugs and toxins (e.g., L-asparaginase, OCPs)
o Acute systemic illness (e.g., sepsis, DIC) o may be rapidly fatal
o Chronic systemic illness (e.g., IBD, SLE, Leukemia) • CT is highly sensitive
o Nephrotic syndrome • Abusive head trauma
o Inborn errors of metabolism (e.g., Homocystinuria) o Primarily subdural or intraparenchymal
• Blood Vessel • Epidural hemorrhage
o Infection / thrombophlebitis o Nearly all are caused by trauma
§ Otitis media, mastoiditis, bacterial meningitis, sinusitis, dental
• Subdural hemorrhage
abscess, pharyngitis
§ Lemierre syndrome o May occur spontaneously in children with brain atrophy
§ Sepsis (stretching of the bridging veins)
o Trauma: skull fractures, closed head trauma • AV malformation is the most common cause of childhood
o Compression: birth, occipital bone compression in neonates in supine subarachnoid and intraparenchymal hemorrhagic stroke
lying • Emergent neurosurgical intervention for large or rapidly
o Iatrogenic: neurosurgery, jugular lines, ECMO expanding hemorrhage
o Venous malformations (e.g., dural AV fistulas)
• Neuroprotective measures
• Reversal of anti-coagulant therapy if indicated
DIFFERENTIAL DIAGNOSIS OF STROKE-LIKE EPISODES IN CHILDREN
Adapted from Nelson Textbook of Pediatrics, 20th ed
CLINICAL DISTINCTION FROM
DISORDER IMAGING DISTINCTION FROM STROKE
STROKE
Evolving or “marching” symptoms, short
Typically normal
Migraine duration, complete resolution, headache,
Migrainous infarction is rare
personal or family history of migraine
Positive symptoms, Todd paralysis is Normal or may identify source of seizures (e.g.,
Seizure
postseizure and limited malformation, old injury, etc.)
Normal or signs of encephalitis / cerebritis, which are typically
Fever, encephalopathy, gradual onset,
Infection diffuse and bilateral. Arterial ischemic stroke and cerebral
meningismus
sinovenous thrombosis can occur in bacterial meningitis
Gradual onset, multifocal symptoms, Multifocal lesions, characteristic appearance (e.g., patchy in
encephalopathy acute disseminated encephalomyelitis, ovoid in multiple
Demyelination
Accompanying optic neuritis or sclerosis), typical locations (e.g., pericallosal in multiple
transverse myelitis sclerosis), less likely to show restricted diffusion
Risk factor e.g., insulin therapy), Bilateral, symmetrical
Hypoglycemia related to meals, additional systemic May see restricted diffusion
symptoms Posterior dominant pattern
Watershed infarction caused
Risk factor (e.g., hypotension, sepsis, Bilateral, symmetric restricted diffusion in border zones
by global hypoxic-ischemic-
heart disease), bilateral deficits between major arteries (watershed zones)
encephalopathy
Hypertensive encephalopathy Documented hypertension, bilateral Posterior dominant, bilateral, patchy lesions involving gray
(posterior reversible
leukoencephalopathy)
visual symptoms, encephalopathy and white matter, usually no restricted diffusion
May have restricted diffusion lesions but bilateral,
Preexisting delays / regression, symmetrical, not within vascular territories. MR
Inborn errors in metabolism multisystem disease, abnormal spectroscopy changes
biochemical profiles (e.g., high lactate in mitochondrial myopathy, encephalopathy, lactic
acidosis, and stroke-like episodes)
Symptoms limited to vertigo, imbalance
Vestibulopathy Normal
(i.e., no weakness), gradual onset
Sudden-onset bilaterally symmetric
Acute cerebellar ataxia Normal
ataxia, postviral
Syndromic cluster of symptoms not
Channelopathy localizing to single lesion. Gradual Normal
onset, progressive evolution
History contralateral events
Alternating hemiplegia --
Choreoathetosis / dystonia
PLEASE READ MORE ON: PHYSICAL EXAMINATION:
• Cerebral Palsy • RR – be familiar with normal range of RR per age group
o Different types • O2 saturation level – 95% and above at room air
• Different types of Migraine
• Signs of distress – alar flaring, retractions, adventitious breath
• Myasthenia Gravis
• Bell palsy
sounds

PULMONOLOGY RESPIRATORY PROBLEM BY SEVERITY:


RESPIRATORY DISTRESS RESPIRATORY FAILURE
APPROACH TO COUGH
Clinical state
HISTORY TAKING Clinical state of inadequate
characterized by
• Onset of cough – acute? chronic? oxygenation, ventilation, or both
abnormal respiratory rate
• Wet cough? Dry cough? End stage of respiratory distress
and effort
• Timing of cough – daytime? nighttime? no predilection?
• Associated symptoms – fever, colds, watery and/or itchy eyes, rash

TOPNOTCH MEDICAL BOARD PREP PEDIATRICS MAIN HANDOUT BY DRS. DE VERA AND PUNONGBAYAN Page 57 of 106
For inquiries visit www.topnotchboardprep.com.ph or email us at topnotchmedicalboardprep@gmail.com
This handout is only valid for October 2022 PLE batch. This will be rendered obsolete for the next batch since we update our handouts regularly.
TOPNOTCH MEDICAL BOARD PREP PEDIATRICS MAIN HANDOUT BY DRS. DE VERA AND PUNONGBAYAN
For inquiries visit www.topnotchboardprep.com.ph or https://www.facebook.com/topnotchmedicalboardprep/
This handout is only valid for October 2022 PLE batch. This will be rendered obsolete for the next batch since we update our handouts regularly.
RESPIRATORY DISTRESS RESPIRATORY FAILURE ✔ GUIDE QUESTIONS
Respiratory effort: • Tachypnea (early), bradypnea (late) The thumb sign is a manifestation of an edematous and enlarged
• Increased: nasal flaring, • Increased, decreased, or no epiglottis seen on lateral soft tissue X ray of the neck. It projects as a
retractions, use of rounded soft tissue structure into the hypopharynx.
respiratory effort Dr. Punongbayan
accessory muscles • poor to absent distal air A 3 y/o male comes in with a 2-day history of runny nose and mild dry
• Inadequate: e.g., movement cough with a fever of 38°C. His elder brother was noted to have cough
hypoventilation, • Tachycardia (early), bradycardia and colds 5 days prior. He was brought in for consult today because the
bradypnea • Cyanosis mother noted worsening of cough with a barking character. The child’s
voice was also hoarse. PE showed HR 102, RR 43, T 38°C, seen sitting
• Stupor, coma (late) upright, looks anxious whenever examined, erythematous posterior
Change in airway sounds pharynx, and stridor. What is the most likely diagnosis?
Associated changes in skin A. Atypical pneumonia D. Epiglottitis
color and mental status B. Bronchiolitis E. Laryngotracheobronchitis
C. Bacterial tracheitis
✔ GUIDE QUESTIONS Buzz words/phrases for LTB: history of URTI, fever, ill family member,
A newborn term baby girl had respiratory distress shortly after birth barking cough, stridor.
Dr. Punongbayan
for which she was intubated. PE reveals a coloboma of the right eye and A 5 y/o male patient comes in with a 2-day history of runny nose and
abnormally formed and low-set ears. An NGT cannot be passed mild dry cough with a fever of 39°C. He was brought in for consult today
although there was no cleft or other mass lesions. The most
because the mother noted worsening of brassy cough, with hoarseness,
appropriate next diagnostic step is:
and with production of copious purulent sputum. PE revealed HR 102,
A. CT scan of the head C. MRI of the head
RR 58, T 39°C. The child is seen on semi-recumbent position with his
B. Karyotype D. Flexible bronchoscopy mother on the examination bed, looks anxious, with erythematous
The case is about a newborn who had an acute onset of respiratory posterior pharynx, (+) rhonchi on auscultation. What is the most likely
distress. There was mention of abnormal physical features as well diagnosis?
(coloboma and low-set ears) implying a syndrome. Part of this A. Atypical pneumonia D. Epiglottitis
syndrome is the need to insert an NGT implying an obstruction. B. Bronchiolitis E. Laryngotracheobronchitis
CHARGE syndrome would come to mind for this patient and one of its C. Bacterial tracheitis
features is a choanal atresia, the most common anomaly of the nose Buzz words/phrases for Bacterial Tracheitis: high fever, patient is in
occurring in 1/7000 live births and mostly affecting females. The distress, acutely ill, brassy cough, copious purulent sputum
atretic plate may be directly seen by a flexible rhinoscopy Dr. Punongbayan
Nelson Textbook of Pediatrics 21st ed. 2020 If a chest x-ray was done for the patient, what is the most likely finding
Dr. Punongbayan
seen?
SUPPLEMENT: QUICK SHEET A. Thumb sign
CHARGE syndrome: B. Ragged air column sign
• C – coloboma of the eye C. Normal chest x-ray
D. Steeple sign
• H – heart defect
E. Diffuse interstitial infiltrates
• A – atresia choanae
Finding of irregular lining of the trachea due to pseudomembranes on
• R – retarded growth and development or CNS anomalies lateral neck X ray of patients with bacterial tracheitis
• G – genital anomalies or hypogonadism Dr. Punongbayan

• E – ear anomalies LARYNGOTRACHEOBRONCHITIS


TYPE OF RESPIRATORY PROBLEM • also called viral croup
• most common etiology is parainfluenza virus
A. SIGNS OF UPPER AIRWAY OBSTRUCTION: • SSx: rhinorrhea, pharyngitis, and low-grade fever 1-3 days before
(Croup, Foreign Body Aspiration, Anaphylaxis) signs of stridor, hoarse voice, barking cough
• Increased rate and effort (retractions, nasal flaring, use of • neck x-ray: subglottic narrowing - “steeple sign” (X ray findings
accessory muscles) do not correlate well with disease severity)
• Changes in voice (e.g., hoarseness), cry, or presence of barking
CROUP is diagnosed CLINICALLY….
cough
Clinical signs of respiratory distress:
• Stridor (usually inspiratory but may be expiratory)
• Stridor
• Poor chest rise
• Retractions
• Poor air entry on auscultation
• Cyanosis
• Drooling, snoring, or gurgling sounds
• Level of consciousness
✔ GUIDE QUESTIONS • Air entry
Respiratory problems rank high among Filipino children. The nature of ***Indications for admission: depends on severity of respiratory distress
this problem is frequently:
A. Congenital C. Allergic
B. Metabolic D. Infectious ROLE OF X-RAY IN CROUP
A 4-year-old male child is brought to the emergency room due to https://qrs.ly/vzdung9
difficulty breathing. He has high fever of 39°C. 12 hours ago, he started
having sore throat and a mild fever which progressed and prompted
consult. Patient is seen drooling and the neck is slightly held MANAGEMENT OF CROUP:
hyperextended. What is the most likely diagnosis in this case?
SEVERITY OF
A. Acute Laryngotracheobronchitis INTERVENTION
B. Acute Bronchiolitis CROUP
C. Acute Epiglottitis Dexamethasone (0.6 mg/kg/dose po/IV/IM;
MILD
D. Foreign Body Aspiration max dose 16 mg)
The main clinical features of acute epiglottitis are highlighted in this Administer humidified O2 (O2 sat <92%)
case. It is one of the common upper airway obstructions in a young child. Put on NPO temporarily
Dr. Punongbayan Administer nebulized epinephrine (0.5
Which among the ff. is the best next step in the management of this MODERATE mL/kg/dose max 5 mL using 1:1000 dilution
patient? TO SEVERE over 15 minutes)
A. Start IV and antibiotics
B. Chest x-ray
Administer dexamethasone
C. Secure airway with intubation Re-assess at least after 2 hours after
D. Nebulize with racemic epinephrine intervention
Lookout for clues that denote respiratory distress in the case such as
Administer a high concentration of O2: use
drooling and a hyperextended neck which would need immediate non-rebreathing mask if available
stabilization of the airway. IMPENDING Assisted ventilation (bag mask ventilation)
Dr. Punongbayan RESPIRATORY for <90% O2sat
In relation to the above case, if an x-ray was done in this patient, what is FAILURE Administer dexamethasone IV/IM
the expected finding? Perform endotracheal intubation if indicated
A. Consolidation C. Steeple sign
B. Thumb sign D. Ragged air column sign
(use smaller ET tube size)

TOPNOTCH MEDICAL BOARD PREP PEDIATRICS MAIN HANDOUT BY DRS. DE VERA AND PUNONGBAYAN Page 58 of 106
For inquiries visit www.topnotchboardprep.com.ph or email us at topnotchmedicalboardprep@gmail.com
This handout is only valid for October 2022 PLE batch. This will be rendered obsolete for the next batch since we update our handouts regularly.
TOPNOTCH MEDICAL BOARD PREP PEDIATRICS MAIN HANDOUT BY DRS. DE VERA AND PUNONGBAYAN
For inquiries visit www.topnotchboardprep.com.ph or https://www.facebook.com/topnotchmedicalboardprep/
This handout is only valid for October 2022 PLE batch. This will be rendered obsolete for the next batch since we update our handouts regularly.
VIRAL CROUP EPIGLOTTITIS
Age group 3 mos to 3 yrs 3-7 yrs
Stridor 88% 8%
parainfluenza H. influenzae
Pathogen
virus type B
prodrome
Onset rapid (4-12 hrs)
(1-7 days)
Fever severity low grade high grade
barking cough, muffled voice,
Associated Sx
hoarseness drooling
✔ GUIDE QUESTIONS
Response to racemic stridor A 3-year-old male is brought to the ER due to acute onset of noisy
none
epinephrine improves breathing. Patient is coughing from time to time, no cyanosis noted, and
“thumbprint/ he points to the neck when asked about pain. Mother says that the
CXR “steeple sign”
leaf sign” patient was apparently well and playing with his toys before the onset
of the symptoms. What is the most likely diagnosis?
A. Bronchiolitis
B. Asthma
C. Foreign body aspiration
D. Bronchitis
Sudden onset of respiratory distress and/or noisy breathing in a
STRIDOR BARKING COUGH previously well young child (esp. in the 1st 3 years of life) highly
OF CROUP signifies foreign body airway obstruction.
https://qrs.ly/h2dungb Dr. Punongbayan
https://qrs.ly/n1dunge
Which foreign body is most commonly obtained from respiratory tracts
The succeeding X rays are the typical findings in croup (steeple sign, the of children?
one on the left) and in epiglottitis (thumbprint sign, the one on the right). A. Marbles C. Nuts
Dr. Punongbayan
B. Beads D. Berries
In the following table, the main presenting feature is colds with sore
throat and nasal congestion. Additional clinical features will narrow
down your initial impression to the most likely diagnosis.
Dr. Punongbayan

PATIENTS PRESENTING WITH UPPER AIRWAY OBSTRUCTION


CLINICAL VIGNETTE CXR FINDINGS DX TX
Patchy infiltrates & ragged
Bacterial tracheitis antibiotics
tracheal column
If vaccinated: Staph. aureus; if unvaccinated:
“Thumb’s sign” Acute epiglottitis
Hib
All levels of respiratory distress:
Croup or Dexamethasone; moderate to severe
Subglottic narrowing
Patient presents with laryngotracheobronchitis distress: steroid and add racemic
s/sx of upper epinephrine
respiratory obstruction Air trapping on the right
Foreign body airway
lung w/ mediastinal shift bronchoscopy
obstruction
towards the right lung
All levels of respiratory distress:
Dexamethasone; moderate to severe
“Steeple sign” Croup of LTB
distress: steroid and add racemic
epinephrine
PATIENT PRESENTING WITH COMMON COLDS ✔ GUIDE QUESTIONS
CLINICAL Do you remember?
KEY CLUES DIAGNOSIS
VIGNETTE • What are the sinuses present at birth?
Prominent itching & sneezing, • What sinuses are pneumatized at 4 years old?
Allergic rhinitis
nasal eosinophilia • What sinuses begin to develop at 7-8 years old?
Unilateral foul-smelling Foreign body in Answers: Maxillary and Ethmoid, Sphenoid, Frontal
discharge, bloody nasal secretion the nose
Headache, facial pain,
Patient Clue: ME present at birth (ME is maxillary and ethmoid)
periorbital edema, rhinorrhea Sinusitis
presents with Dr. Punongbayan
for >2 weeks
sore, scratchy ✔ GUIDE QUESTION
Persistent rhinorrhea w/
throat, nasal Congenital A 4-year-old male was brought to the clinic due to coughing and
onset in the first 3 mos of life
obstruction syphilis wheezing usually worse at night. The patient has been having these
(snuffles)
and rhinorrhea symptoms intermittently for the past year. However, his coughing and
Hx of prolonged use of topical Rhinitis
or oral decongestant medicamentosa wheezing has progressively become worse with the mother noting
Paroxysms of cough leaving difficulty of breathing that prompted consult. PE revealed HR 105, RR
Pertussis or 46, T 37°C, chest PE presence of wheezing on bilateral lung fields,
the baby breathless &
whooping cough occasional rhonchi, and a prolonged expiratory phase. What is the most
subconjunctival hemorrhages
likely diagnosis?
(snuffles is a catarrhal discharge from the nasal mucous membranes in A. Atypical pneumonia D. Bronchitis
congenital syphilis) B. Bronchial asthma E. Pertussis
Dr. Punongbayan
C. Bronchiolitis
SINUSITIS
Typical presentation of asthma is coughing and wheezing usually
• Organisms: S. pneumoniae (30%), non-typable H. influenzae more often and severe at night that interrupts sleep; the PE findings
(20%), M. catarrhalis (20%) of wheezing and a prolonged expiratory phase reinforces the
• Anaerobes are uncommon causes of acute sinusitis in children diagnosis of asthma
• Anything that impairs mucociliary transport or causes nasal Dr. Punongbayan

obstruction predisposes to sinusitis


• S/Sx: colds and cough >10-14 days, purulent nasal discharge for
3-4 consecutive days, headache, tenderness over the sinuses
• X-ray: air-fluid levels, opacification of the sinuses
• Tx: antibiotics x 14 days (Co-Amoxiclav)
• Complications are abscess, meningitis
TOPNOTCH MEDICAL BOARD PREP PEDIATRICS MAIN HANDOUT BY DRS. DE VERA AND PUNONGBAYAN Page 59 of 106
For inquiries visit www.topnotchboardprep.com.ph or email us at topnotchmedicalboardprep@gmail.com
This handout is only valid for October 2022 PLE batch. This will be rendered obsolete for the next batch since we update our handouts regularly.
TOPNOTCH MEDICAL BOARD PREP PEDIATRICS MAIN HANDOUT BY DRS. DE VERA AND PUNONGBAYAN
For inquiries visit www.topnotchboardprep.com.ph or https://www.facebook.com/topnotchmedicalboardprep/
This handout is only valid for October 2022 PLE batch. This will be rendered obsolete for the next batch since we update our handouts regularly.

B. SIGNS OF LOWER AIRWAY OBSTRUCTION BRONCHIOLITIS ASTHMA


(Bronchiolitis, Asthma) • Clinical diagnosis heard) may indicate
• Tachypnea • Typical course of illness impending respiratory
• Wheezing (most commonly expiratory but may be inspiratory peaks at about 3-4 days failure
or biphasic) • Chest X ray is not routine • Triggers: viral URTIs (most
• Increased respiratory effort (retractions, nasal flaring, and (AAP 2014 guidelines) common in children),
prolonged expiration) • CXR findings: environmental triggers
• Prolonged expiratory phase associated with increased hyperinflation, possibly
with atelectasis and GINA guidelines 2020
expiratory effort (i.e., expiration is an active rather than passive
process) infiltrates that do not • Classify based on severity
correlate with severity o Intermittent
• Cough
• Supportive management: o Persistent (mild,
• Possible decreased air movement on auscultation
assess hydration status and moderate, severe)
LOWER AIRWAY OBSTRUCTION CONDITIONS oxygenation • Classify levels of asthma
BRONCHIOLITIS ASTHMA • Conflicting results: control
• Common in the 1st 2 years • Chronically inflamed nebulized hypertonic o Uncontrolled
of life airways and increased saline, heated humidified o Partly controlled
• Due to RSV production of mucus HFNC O2 o Controlled
• Prodrome of cough, colds, • Nocturnal cough • Systematic reviews: no role
fever • Bronchospasm for bronchodilators
• Signs of distress notably • Wheezing is prominent;
wheezing on auscultation silent chest (no wheezes
ASTHMA
APPROACH TO THE DIAGNOSIS OF ASTHMA IN CHILDREN <6 YEARS OLD BASED ON PATTERNS OF RESPIRATORY SYMPTOMS
(PAPP Asthma 2021, GINA 2021)
coughing, wheezing,
DURATION OF coughing, wheezing, heavy breathing for coughing, wheezing, heavy breathing for
heavy breathing for
SYMPTOMS >10 days during URTI >10 days during URTI
<10 days during URTI
FREQUENCY OF 2 to 3 episodes per more than 3 episodes per year OR severe more than 3 episodes per year OR severe
SYMPTOMS year episodes and/or night worsening episodes and/or night worsening
PROPORTION OF
YOUNG CHILDREN FEW have asthma SOME have asthma MOST have asthma
WITH ASTHMA
• Between the episodes, the child has
SYMPTOMS Between episodes, the child may have cough, wheeze, or heavy breathing during
BETWEEN NO symptoms occasional cough, wheeze, or heavy play or when laughing
EPISODES breathing • Known to have allergic sensitization, AD,
food allergy, of family history of asthma
ASTHMA Less likely to be
Asthma suspect More likely to be asthma
CATEGORY asthma
APPROACH TO THE DIAGNOSIS OF ASTHMA IN 6-18 YEARS OLD BASED ON PATTERNS OF RESPIRATORY SYMPTOMS
(PAPP Asthma 2021, GINA 2021)
DIAGNOSTIC FEATURE CRITERIA FOR DIAGNOSIS
First, a history of variable respiratory symptoms
• usually more than 1 type of respiratory symptom
• occurs variably over time and vary in intensity
Wheeze, shortness of breath, chest
• often worse at night or on waking
tightness, and cough
• often triggered by exercise, laughter, allergens, colds air
• often appears or worsens with viral infections
Second, confirmed variable expiratory airflow limitation, any of the following
Documented expiratory airflow limitation At a time when FEV1 is reduced, confirm that FEV1/FVC is reduced (usually >0.90)
Documented excessive variability in lung
The greater the variations, the more confident the diagnosis.
function
Positive BD reversibility test (more likely
to be + if BD medication is withheld before Increase in FEV1 >12% predicted
test: SABA ≥ 4 hours; LABA ± 15 hours
Excessive variability in twice-daily PEF
average daily diurnal PEF variability >13%
over 2 weeks
Positive exercise challenge test Fall in FEV1 >12% predicted, or PEF >15%
Excessive variation in lung function
Variation in FEV1 of >12% or 15% in PEF between visits
between visits
CLASSIFICATION OF ASTHMA EXACERBATION
(PAPP Asthma Guidelines 2021)
Age < 6 years old Ages 6-18 years old
PARAMETERS /
Severe /
CLASSIFICATION Mild Moderate Mild Moderate Severe Life-threatening
Life-threatening
Breathless, unable Able to talk Breathless
Activity / Able to
Able to talk to talk, confused, or only in or unable to Drowsy, confused
Sensorium talk
drowsy phrases talk
Respiratory rate, No No
< 40/min > 40/min < 30/min > 30/min
cpm increase increase Proceed or triage
100- 100- 100- immediately to an
Cardiac rate, bpm <140/min >140/min >125/min
120/min 120/min 120/min acute care facility or
Pulse oximetry >95% >92% <92% 95% 90-95% <90% Emergency
30% to 50% of Department
Lung Function N/A > 50% of personal best
personal best
TOPNOTCH MEDICAL BOARD PREP PEDIATRICS MAIN HANDOUT BY DRS. DE VERA AND PUNONGBAYAN Page 60 of 106
For inquiries visit www.topnotchboardprep.com.ph or email us at topnotchmedicalboardprep@gmail.com
This handout is only valid for October 2022 PLE batch. This will be rendered obsolete for the next batch since we update our handouts regularly.
TOPNOTCH MEDICAL BOARD PREP PEDIATRICS MAIN HANDOUT BY DRS. DE VERA AND PUNONGBAYAN
For inquiries visit www.topnotchboardprep.com.ph or https://www.facebook.com/topnotchmedicalboardprep/
This handout is only valid for October 2022 PLE batch. This will be rendered obsolete for the next batch since we update our handouts regularly.
CONSIDER HOSPITAL ADMISSION WHEN THE PATIENT HAS o also for those with past history of exacerbations requiring OCS
ANY OF THE FOLLOWING CLINICAL CRITERIA: o Prednisone / Prednisolone 1-2 mg/kg/day, max 20 mg/day (<2
• use of more than 6-8 SABA puffs in the past 24 hours yrs old), 30 mg/day (2-5 yrs old), 40 mg/day (6 yrs and above)
• PEF 50-75% of personal best o OR Methylprednisolone 1 mg/kg 6-hourly on day 1
• history of severe exacerbations warranting ICU admission o OR Dexamethasone IV 0.6 mg/kg/day divided into 3-4 hours
• hospital admission or previous exacerbation for the past 12 months for 2 days (below 6 yrs old)
• other considerations: psychosocial problems, physical disability • Inhaled corticosteroids
or learning difficulties, exacerbation despite adequate dose of o upon discharge: should be given to prevent future severe
oral steroids, in a remote location without transportation / exacerbations and hospitalizations and to reduce deaths
communication • ICS-LABA combination
o like formoterol and budesonide or beclomethasone improves
GENERAL PRINCIPLES ON THE MANAGEMENT OF ACUTE symptom control and addresses inflammation at its onset and
ASTHMA EXACERBATIONS, GINA 2021 reduces risk of exacerbations and hospitalization VS SABA only tx
• Target O2 sat for 6-11 y/o is 94-98% and in adolescents is 93-95% o insufficient evidence on safety and efficacy in <4 yrs old
• less than 92% is a predictor for hospitalization and less than • Ipratropium bromide
90% warrants aggressive therapy o in the ER: moderate to severe exacerbations across all ages –
• patients not on controller meds should be started on regular greater improvement in lung function and fewer
ICS-containing Tx hospitalizations
• Delivery via pMDI is the most cost effective • Magnesium sulfate
• In moderate to severe exacerbations: ipratropium combination can o NOT routinely given for acute exacerbation
be added for 3 consecutive doses if initial salbutamol tx is insufficient o add-on Tx in the 1st hour in severe exacerbations
• pMDI plus spacer with face mask (0-3 years); pMDI plus spacer o single dose 40-50 mg/kg (max. 2 grams) by slow IV infusion
with mouthpiece (4-5 years) (20-60 minutes)
• Systemic corticosteroids o reduces hospitalization
o equal effectiveness of oral and IV form o role of MgSO4 in <5 years old is still not established
o moderate and severe exacerbation who fail to respond to initial • Leukotriene receptor antagonists
bronchodilator tx after the 1st hour or worsening of symptoms o limited evidence to support its use in the ER setting

MANAGEMENT OF ASTHMA IN ACUTE EXACERBATION IN <6 YEARS OLD IN AN OUTPATIENT SETTING, PAPP 2021

TOPNOTCH MEDICAL BOARD PREP PEDIATRICS MAIN HANDOUT BY DRS. DE VERA AND PUNONGBAYAN Page 61 of 106
For inquiries visit www.topnotchboardprep.com.ph or email us at topnotchmedicalboardprep@gmail.com
This handout is only valid for October 2022 PLE batch. This will be rendered obsolete for the next batch since we update our handouts regularly.
TOPNOTCH MEDICAL BOARD PREP PEDIATRICS MAIN HANDOUT BY DRS. DE VERA AND PUNONGBAYAN
For inquiries visit www.topnotchboardprep.com.ph or https://www.facebook.com/topnotchmedicalboardprep/
This handout is only valid for October 2022 PLE batch. This will be rendered obsolete for the next batch since we update our handouts regularly.
MANAGEMENT OF ASTHMA IN ACUTE EXACERBATION IN 6-18 YRS OLD IN AN OUTPATIENT SETTING, PAPP 2021

Maintenance and reliever therapy (MART) with ICS-LABA (Budesonide or Beclometasone with Formoterol) has been recommended by GINA 2021 for this age group
MANAGEMENT OF ASTHMA IN ACUTE EXACERBATION IN <6 YEARS OLD IN A HOSPITAL SETTING, PAPP 2021

TOPNOTCH MEDICAL BOARD PREP PEDIATRICS MAIN HANDOUT BY DRS. DE VERA AND PUNONGBAYAN Page 62 of 106
For inquiries visit www.topnotchboardprep.com.ph or email us at topnotchmedicalboardprep@gmail.com
This handout is only valid for October 2022 PLE batch. This will be rendered obsolete for the next batch since we update our handouts regularly.
TOPNOTCH MEDICAL BOARD PREP PEDIATRICS MAIN HANDOUT BY DRS. DE VERA AND PUNONGBAYAN
For inquiries visit www.topnotchboardprep.com.ph or https://www.facebook.com/topnotchmedicalboardprep/
This handout is only valid for October 2022 PLE batch. This will be rendered obsolete for the next batch since we update our handouts regularly.
MANAGEMENT OF ASTHMA IN ACUTE EXACERBATION IN 6-18 YEARS OLD IN AN ED SETTING, PAPP 2021

GINA symptom screening tool: Assessment of symptom PREFERRED


IF: RELIEVER
control CONTROLLER TX
In the past 4 wks, has the low-dose ICS-LABA
Symptoms most
patient had: or medium-dose ICS
days or waking
Daytime symptoms more than or very low dose
at night once or
2x/week? MART (ICS- As needed SABA
more in a week
❏ Yes formoterol) or low-dose ICS-
❏ No Symptoms most formoterol
SABA reliever needed more days or waking medium-dose ICS- reliever for
LEVEL OF ASTHMA
than 2x/week? at night ≥ once a LABA or low-dose MART
SYMPTOM CONTROL
❏ Yes (how many “yes” ticked): week and/or ICS formoterol MART
❏ No 0 well controlled low lung or refer
1-2 partially controlled function
Any night waking due to
asthma? 3-4 uncontrolled
Potentially modifiable risk factors for exacerbations:
❏ Yes
• Medications: ICS not prescribed, poor adherence, incorrect
❏ No
inhaler technique, high SABA use
Any activity limitation due to
• Comorbidities: obesity, chronic rhinosinusitis, GERD,
asthma?
confirmed food allergy, anxiety, depression, pregnancy
❏ Yes • Exposures: smoking, air pollution, allergen exposure
❏ No • Setting: major socioeconomic problems
• Lung function: low FEV1, esp. if <60% predicted
Pharmacological tx for 6-11 years old with asthma, wheezing, • Prior intubation or in intensive care for asthma; having 1 or
or suspected asthma (PAPP Asthma 2021) more severe exacerbations in the last 12 months
PREFERRED • Others: sputum/blood eosinophilia
IF: RELIEVER
CONTROLLER TX
low-dose ICS GINA guidelines 2021 / PAPP Asthma 2021:
Symptoms less
whenever SABA is As needed SABA • Every adult and adolescent with asthma should receive ICS-
than 2x a month
taken or low-dose ICS- containing controller medication to reduce their risk of serious
Symptoms 2x a formoterol exacerbation, even patients with infrequent symptoms.
month or more reliever for • Every patient with asthma should have a reliever inhaler,
daily low-dose ICS
but less than MART either low-dose ICS-formoterol or SABA.
daily

TOPNOTCH MEDICAL BOARD PREP PEDIATRICS MAIN HANDOUT BY DRS. DE VERA AND PUNONGBAYAN Page 63 of 106
For inquiries visit www.topnotchboardprep.com.ph or email us at topnotchmedicalboardprep@gmail.com
This handout is only valid for October 2022 PLE batch. This will be rendered obsolete for the next batch since we update our handouts regularly.
TOPNOTCH MEDICAL BOARD PREP PEDIATRICS MAIN HANDOUT BY DRS. DE VERA AND PUNONGBAYAN
For inquiries visit www.topnotchboardprep.com.ph or https://www.facebook.com/topnotchmedicalboardprep/
This handout is only valid for October 2022 PLE batch. This will be rendered obsolete for the next batch since we update our handouts regularly.

ASSESS FOR DISCHARGE ARRANGE AT DISCHARGE PATIENTS PRESENTING WITH WHEEZING


• Symptoms improved not • Reliever: continue as Hx of viral infection among the
Bronchiolitis
needing SABA needed family members. Px is 2 y/o
• PEF improving and >60- • Controller: start or step up Hx of atopy in the family. Recurrent
80% of personal best or • Prednisolone: continue for wheezing especially after mild viral Bronchial asthma
predicted 5-7 days infection or after exercise
• O2 sat >94% room air • Follow-up within 2-7 days Wheezes are heard loudest over the
• Resources at home trachea. Persistent wheezing never Chondromalacia
adequate seems to go away
Absent breath sounds on right lung. Foreign body airway
• Management of acute attacks:
Patient is 3 y/o obstruction
o short-acting inhaled beta 2-agonist
o oral or IV steroids (Prednisolone/Methylprednisolone) Bronchiolitis – again, history of wheezing with exposure to ill family
o anticholinergics (ipratropium bromide) – never used alone members, affecting young children points to this diagnosis
Chondromalacia – think structural abnormality especially when
o methylxanthines (theophylline, aminophylline): NOT first line
wheezing was detected on early infancy
• Management in between attacks: Foreign body airway obstruction – age was mentioned to highlight
o inhaled corticosteroids the increased incidence of FBAO in this age group
o long-acting inhaled beta 2-agonist Dr. Punongbayan

o leukotriene modifiers (Montelukast)


✔ GUIDE QUESTIONS
✔ GUIDE QUESTIONS Management of infectious croup in a hospitalized 15-month-old child:
An 8-month-old boy was brought to the ER by his mother due to fast A. Immediate intubation
breathing and wheezing. Mother is known to be a smoker and has a B. Ampicillin and chloramphenicol
history of bronchial asthma. The mother says that her baby has been C. Erythromycin
well until 2 days ago when he started having runny nose and a mild D. Aerosolized racemic epinephrine
undocumented fever. PE of the infant reveals wheezing on both lung A hospitalized toddler for infectious croup denotes moderate to severe
fields. What is the most likely diagnosis in this case? respiratory distress, hence the need for additional racemic
A. Asthma C. Pneumonia epinephrine in the management.
B. Bronchiolitis D. Laryngotracheobronchitis Racemic epinephrine works by relaxing the muscles in the airways and
The main features of bronchiolitis are: fever, cough, and wheezing in tightening of the blood vessels. Thus, bronchial and tracheal
the 1st 2 years of life; there is inflammation of the lining of epithelial secretions and airway wall edema decreases.
cells of bronchioles causing mucus production, inflammation and Racemic epinephrine stimulates both α- and β-adrenergic receptors.
cellular necrosis. It acts on vascular smooth muscle to produce vasoconstriction which
Bronchiolitis is caused by obstruction and collapse of the small markedly decreases blood flow at the capillary level. This shrinks
airways during expiration. It affects the younger children because upper respiratory mucosa and reduces edema.
bronchiolar wall thickening significantly affects airflow. Resistance is Dr. Punongbayan
inversely proportional to the 4th power of the radius of the Characteristics finding in the CXR of a patient with staphylococcal
bronchiolar passage. pneumonia include the following, EXCEPT:
Bronchiolitis is more common in boys, non-breastfed, those who live in A. Hemothorax C. Pleural effusion
crowded conditions, with mothers who smoked during pregnancy, B. Pneumatocele D. Pneumothorax
Dr. Punongbayan
Hemothorax is most commonly brought about by trauma.
If a chest x-ray is requested for this infant, what will be the expected Dr. Punongbayan
finding?
A. Hyperinflated lungs with patchy atelectasis
B. Steeple sign C. SIGNS OF LUNG TISSUE DISEASE
C. Lobar consolidation (Pneumonia, Pulmonary Edema, Trauma, Infiltrative Disease, Toxins)
D. Diffuse interstitial infiltrates • Tachypnea (often marked)
A general radiographic presentation of obstruction is hyperaeration • Increased respiratory effort
or hyperinflation of lungs especially with flattening of the diaphragm • Grunting
on a lateral CXR view.
Dr. Punongbayan • Crackles (rales) and decreased air movement
Management of BRONCHIOLITIS: American Academy of Pediatrics
• Diminished breath sounds
2014 guidelines: SUPPORTIVE MANAGEMENT • Tachycardia
The AAP no longer recommends a trial dose of a bronchodilator, such as • Hypoxemia (despite giving supplemental O2)
Albuterol or Salbutamol, because evidence to date shows that
bronchodilators are ineffective in changing the course of bronchiolitis
(evidence quality: B, strong recommendation). Bronchodilators do not
shorten the days of symptoms, admission rates, or length of stay. Systemic
steroids and racemic epinephrine are also not recommended.
However, if the choices would mention a bronchodilator, that would still
be the most plausible answer as actual local practice still include it in the
management of bronchiolitis for its dilating properties.
Dr. Punongbayan

TOPNOTCH MEDICAL BOARD PREP PEDIATRICS MAIN HANDOUT BY DRS. DE VERA AND PUNONGBAYAN Page 64 of 106
For inquiries visit www.topnotchboardprep.com.ph or email us at topnotchmedicalboardprep@gmail.com
This handout is only valid for October 2022 PLE batch. This will be rendered obsolete for the next batch since we update our handouts regularly.
TOPNOTCH MEDICAL BOARD PREP PEDIATRICS MAIN HANDOUT BY DRS. DE VERA AND PUNONGBAYAN
For inquiries visit www.topnotchboardprep.com.ph or https://www.facebook.com/topnotchmedicalboardprep/
This handout is only valid for October 2022 PLE batch. This will be rendered obsolete for the next batch since we update our handouts regularly.
✔ GUIDE QUESTIONS PNEUMONIA
A 9-month-old male was brought to the clinic due to cough and • etiologies in young infants:
difficulty breathing. The mother noted that he initially had runny nose o 0-28 days: GBS, E. coli, Listeria S. pneumoniae
with clear nasal discharge and dry cough 3 days prior. At present, the
o 3 wks – 3 mos: RSV*, parainfluenza, Chlamydia, Mycoplasma,
infant has a fever of 38.5°C, fast breathing, and cough which prompted
consult. PE revealed HR of 158, RR 68, T 38.8°C; he is irritable, S. pneumoniae*
coughing, (+) nasal flaring, (+) subcostal retractions, (+) crackles over • etiologies in older infants and children:
bilateral lung fields, and decreased breath sounds over the right lung o 4 mos – 5 yrs: viruses*, S. pneumoniae*, H. influenzae type b,
field. What is the most likely diagnosis? Mycoplasma
A. Pneumonia D. Bronchitis o 5-15 yrs: Mycoplasma*, S. pneumoniae
B. Bronchial asthma E. Pertussis
C. Bronchiolitis Clinical signs and symptoms that accurately diagnose
Triad of pneumonia: fever, cough, tachypnea (the latter is the most pneumonia in 3 months to 18 years old
sensitive indicator); significant finding of crackles in PE as it implies (2021 PCAP guidelines by PAPP)
alveolar-filled inflammatory fluid interacting with air as one inhales • A patient who has cough, fever PLUS any of the following:
Dr. Punongbayan
In relation to the above case, where should the patient be sent? o Grunting
A. Home C. Regular ward o Retractions or chest indrawing
B. OPD D. ICU o O2 saturation <95% at room air
If a chest X ray was done in the above case which revealed bilateral o Nasal flaring
interstitial infiltrates, and the assessment made was pneumonia, what o Tachypnea
would be the classification of the above patient? § 3-12 months old: ≥ 50 breaths/min
A. Minimal risk D. High risk § more than 1 yr old to 5 yrs old: ≥ 40 breaths/min
B. PCAP B E. Low risk
§ more than 5 years to 12 yrs old: ≥ 30 breaths/min
C. PCAP C
Which of the ff. is considered the most consistent clinical manifestation § more than 12 years old: ≥ 20 breaths/min
of pneumonia in children?
A. Fever C. Cough
B. Tachypnea D. Crackles on auscultation
PAPP GUIDELINE ON PCAP 2021
RISK CLASSIFICATION FOR PNEUMONIA-RELATED MORTALITY
PARAMETERS AT SITE-OF-CARE
LOW RISK (Non-severe) HIGH RISK (Severe)
Formerly classified as pCAP A pCAP B pCAP C pCAP D
CLINICAL PARAMETERS
1. Respiratory signs
Cyanosis / Hypoxemia None Present
Head bobbing None Present
Chest indrawing / retractions None Present
Apnea None Present
Grunting None Present
2. Central nervous system signs
Altered sensorium None or irritable but consolable Lethargic / Stuporous / Comatose / GCS <13
Convulsion None Present
3. Circulatory signs
Poor perfusion None CRT >3 secs or in shock
Pallor None Present
4. General Considerations
Malnutrition None or mild Moderate to severe
Refusal or inability to drink / feed / take oral meds No Yes
Dehydration None With some or severe signs
Age < 6 months No Yes
ANCILLARY PARAMETERS
5. Chest radiograph or ultrasound findings of
consolidation, multifocal disease, moderate to large None Present
effusion, abscess, air leak
6. Sustained O2 saturation at RA using pulse oximetry
≥ 94% ≤ 93%
for 20-30 minutes
DIAGNOSTIC AIDS TO CONFIRM THE PRESENCE OF SEVERE For severe PCAP, regardless of S. pneumoniae immunization
PNEUMONIA IN A HOSPITAL SETTING, PCAP 2021 status, any of the following is considered:
• Chest X ray is strongly recommended as an initial diagnostic aid • Pen G at 200,000 units/kg/day Q6 if with complete Hib
for patients classified as severe (high-grade evidence) vaccination OR Ampicillin at 200 mg/kg/day Q6 if with no or
• Point-of-care chest ultrasonography for severe cases (high- incomplete or unknown Hib vaccination
grade evidence)
• Procalcitonin (moderate evidence) For severe PCAP, regardless of S. pneumoniae immunization
• sputum Gram stain and culture are NOT considered to be done status, any of the following is considered:
routinely in severe cases (low-grade evidence) • start Cefuroxime at 100-150 mg/kg/day Q8 OR Ceftriaxone
• CBC, ABG, serum electrolytes are considered as necessary based 75-100 mg/kg/day Q12-24 OR Ampicillin-Sulbactam 200
on clinician’s evaluation (expert opinion) mg/kg/day Q6 in settings with documented high-level penicillin
resistant pneumococcal or beta-lactamase producing HIb based
EMPIRIC TREATMENT FOR PCAP BACTERIAL ETIOLOGY: on local resistance date
For non-severe PCAP, regardless of immunization status • add Clindamycin at 20-40 mg/kg/day Q6-Q8 when
against S. pneumoniae and Hib, any of the following is Staphylococcal pneumonia is highly suspected based on clinical
considered: and chest X ray features
• Amoxicillin at 40-50 mg/kg/day Q8 for 7 days OR at 80-90 • in cases of severe and life-threatening conditions, Vancomycin
mg/kg/day Q12 for 5-7 days at 40-60 mg/kg/day Q6-Q8 is preferred
• Amoxicillin-clavulanate at 80-90 mg/kg/day Q12 for 5-7 days
OR Cefuroxime at 20-30 mg/kg/day Q12 for 7 days in settings
with documented high-level penicillin resistant pneumococci or
beta-lactamase producing Hib based on local resistance data or
hospital antibiogram
TOPNOTCH MEDICAL BOARD PREP PEDIATRICS MAIN HANDOUT BY DRS. DE VERA AND PUNONGBAYAN Page 65 of 106
For inquiries visit www.topnotchboardprep.com.ph or email us at topnotchmedicalboardprep@gmail.com
This handout is only valid for October 2022 PLE batch. This will be rendered obsolete for the next batch since we update our handouts regularly.
TOPNOTCH MEDICAL BOARD PREP PEDIATRICS MAIN HANDOUT BY DRS. DE VERA AND PUNONGBAYAN
For inquiries visit www.topnotchboardprep.com.ph or https://www.facebook.com/topnotchmedicalboardprep/
This handout is only valid for October 2022 PLE batch. This will be rendered obsolete for the next batch since we update our handouts regularly.
For patients with known hypersensitivity to penicillin • Mucokinetic and mucolytic agents are NOT considered as
classified as: adjunctive tx for PCAP.
• Non-type 1 hypersensitivity, Cefuroxime 20-30 mkday PO or • Insufficient evidence to recommend the use of the ff as
100-150 mkday IV Q8 OR Ceftriaxone 75-100 mkday Q12-Q24 adjunctive tx for PCAP:
• Type 1 hypersensitivity, any of the ff: Azithromycin 10 mkday PO 1. oral folate 4. virgin coconut oil
or IV Q24 for 3 days OR 10 mkday day 1 followed by 5 mkday Q24 2. probiotics 5. nebulization with saline solution
for days 2-5; Clarithromycin 15 mkday Q12 for 7 days; Clindamycin 3. vitamin C 6. steam inhalation
10-40 mkday PO or 20-40 mkday IV Q6-Q8 for 7 days
PATIENTS PRESENTING WITH PNEUMONIA
When an atypical pathogen is suspected: • Poorly nourished
• Azithromycin 10 mkday PO or IV for 5 days esp. in less than 6 • Unvaccinated Measles
months old in whom pertussis is suspected OR 10 mkday Q24 • w/ onset of rashes all over the pneumonia
for 3-5 days OR 10 mkday day 1 followed by 5 mkday Q24 for body
days 2-5 • Px has CF
• Clarithromycin 15 mkday Q12 for 7-14 days • Px has CGD
Px with Pseudomonas
• Px is a burn px
fever, cough,
• Px is neutropenic
For proven viral etiology of pneumonia: and
• Px is a teen/ young adult
• Oseltamivir (twice a day for 5 days and dose depends on tachycardia
• Lives in a dormitory Mycoplasma
weight) is strongly recommended to be started immediately • Initial cough is non-productive
within 36 hours of laboratory-confirmed infection. • Px has his own aviarium in his
o <1 year old: 3 mg/kg/dose Psittacosis
home
o 1 year and older and 15 kilos or less: 30 mg • Hx of eye discharge during the
o more than 15-23 kgs: 45 mg Chlamydia
1st 5-14 days of neonatal period
o more than 23-40 kgs: 60 mg Mycoplasma – features mentioned above are common presentation of an
o more than 40 kgs: 75 mg atypical infection
Psittacosis – also known as parrot fever, and ornithosis—is a zoonotic
Other treatment for PCAP (PAPP 2021) infectious disease in humans caused by a bacterium called Chlamydia
• Vitamin A is strongly recommended (high-grade evidence) psittaci and contracted from infected parrots; fever, chills, dry cough,
• Bronchodilators in the presence of wheezing (expert opinion) weakness, fatigue
Chlamydia – history of conjunctivitis, subacute onset, sharp cough, fever
• Zinc is NOT considered as adjunctive tx for severe PCAP as it
usually absent, no wheeze; usually affects 3 week-3-month-old infants
does not have any effect in shortening recovery time. Dr. Punongbayan
• Vitamin D is NOT considered as adjunctive tx for severe PCAP as
it does not reduce the length of hospital stay.
Pneumonia S/SX CXR, CBC TX
diffuse streaky infiltrates;
Viral cough, wheezing, stridor supportive
lymphocytosis
(0-2 months) Ampicillin + Aminoglycoside
cough, high fever, dyspnea, dullness
Bacterial lobar consolidation, neutrophilia (2 months-5 years) Ceftriaxone or
to percussion
Cefuroxime + Ampicillin or Amoclav
Mycoplasma less ill-looking, non-productive interstitial pattern, usually lower (>5 yrs) Erythromycin, Clarithromycin, or
“walking pneumonia” cough lobes Azithromycin
6 wks to 6 mos, “staccato” cough, hyperinflation, “ground-glass”
Chlamydia Erythromycin PO x 14 days
maternal Hx of infection appearance; eosinophilia
✔ GUIDE QUESTIONS D. SIGNS OF DISORDERED CONTROL OF BREATHING
A 3-year-old male child was brought to the clinic due to intermittent (Neuromuscular Disease, Metabolic, Drug Overdose, Increased ICP)
episodes of continuous coughing until the child turns purple followed
by a deep loud inspiration, 1 week ago, the mother recalled that he had
• Variable or irregular respiratory rate and pattern (tachypnea
episodes of sneezing, rhinorrhea, what is the most likely diagnosis? alternating with bradypnea)
A. Diphtheria C. Bronchiolitis • Variable respiratory effort
B. Pertussis D. Pneumonia • Shallow breathing with inadequate effort (frequently resulting
Buzz phrases for a case of pertussis are: intermittent cough, in hypoxemia and hypercarbia)
paroxysms of cough, usually well and active in between the paroxysms • Central apnea (apnea without respiratory effort)
of cough. • Normal or decreased air movement
Dr. Punongbayan
In relation to the question above, which among the ff. is the best
treatment for the above condition?
A. Azithromycin C. Ceftriaxone
B. Penicillin G D. Doxycycline

INCREASED ICP POISONING / OVERDOSE NEUROMUSCULAR DISEASE


• If trauma is suspected, open the airway, • Antidote if appropriate • Consider noninvasive or invasive
head in midline, stabilize the cervical • Give Naloxone for opiate overdose ventilatory support
spine • Support airway and adequate ventilation
• If with poor perfusion, give 20 mL/kg IV • Suction the airway in case of vomiting
crystalloid • Diagnostic tests as indicated (ABG, ECG,
• Give pharmacologic therapy like osmotic chest X ray, electrolytes, glucose, drug
agents, hypertonic saline screen
• Avoid hyperthermia
• Period of communicability: from 7 days after exposure to 4 wks
after onset of typical paroxysms; most infectious during the
WHOOPING COUGH catarrhal stage
https://qrs.ly/2bdungh • Incubation period: 3-12 days
• 3 stages lasting 2 weeks each
o Catarrhal, Paroxysmal, Convalescent
PERTUSSIS
• Purely or predominantly cough. Absent
• Bordetella pertussis (gm- coccobacilli) o Fever, malaise, exanthem, sore throat, hoarseness, tachypnea,
• Whooping cough wheeze, crackles
• Source: secretions from RT of infected persons
• Transmitted by close contact via large aerosol drops

TOPNOTCH MEDICAL BOARD PREP PEDIATRICS MAIN HANDOUT BY DRS. DE VERA AND PUNONGBAYAN Page 66 of 106
For inquiries visit www.topnotchboardprep.com.ph or email us at topnotchmedicalboardprep@gmail.com
This handout is only valid for October 2022 PLE batch. This will be rendered obsolete for the next batch since we update our handouts regularly.
TOPNOTCH MEDICAL BOARD PREP PEDIATRICS MAIN HANDOUT BY DRS. DE VERA AND PUNONGBAYAN
For inquiries visit www.topnotchboardprep.com.ph or https://www.facebook.com/topnotchmedicalboardprep/
This handout is only valid for October 2022 PLE batch. This will be rendered obsolete for the next batch since we update our handouts regularly.
• Complications CLINICAL
CXR FINDINGS DIAGNOSIS
o Hemorrhage (subconjunctival or intracranial) VIGNETTE
o Seizure Lobar consolidation Pneumococcus
o Otitis media Hyperinflation w/ bilateral
Child with
interstitial infiltrates & RSV
o Atelectasis cough,
peribronchial cuffing
o Pneumonia colds, fever,
Prominent areas of cavitation
• Drug of choice: Macrolide (Erythromycin or Clarithromycin) wheezing, Staphylococcus
& multiple pneumatoceles
stridor
CARE OF HOUSEHOLD AND OTHER CLOSE CONTACTS: Mycobacterium
Right sided hilar adenopathy
tuberculosis
• A macrolide agent should be given promptly to all household
contacts and other close contacts regardless of age, history of Pneumococcus is the most common cause of lobar consolidation
Dr. Punongbayan
immunization, and symptoms.
• The same age-related drugs and doses are used for TYPE OF COUGH
treatment. Staccato Chlamydia
CAUSATIVE
Brassy Staphylococcus
PATHOLOGIC PROCESS Barking “Seal” Parainfluenza
AGENT
• Attaches to respiratory epithelium Whooping
Bordetella
• Inhibits cellular destruction Post tussive vomiting
Mycoplasma
• Sloughed cellular debris and inflammatory Most severe in the morning Asthma
cells and mucus cause airway obstruction With vigorous exercise Exercise-induced asthma
• Extensive areas of hemorrhagic necrosis Disappears with sleep Habit cough
• Irregular areas of cavitation Tight sounding w/ wheezing Asthma
Staphylococcus
• Pneumatoceles, empyema and
bronchopulmonary fistulas Staccato cough has a sharp quality with sudden bursts
• Diffuse infection with interstitial pneumonia Staphylococcus is the most common cause of bacterial tracheitis) – brassy
• Necrosis of tracheobronchial mucosa, cough is metallic in quality
formation of large amounts of exudate, edema, GABHS Parainfluenza – Croup
and local hemorrhage Bordetella – Pertussis
• Involvement of lymphatic vessels and pleural Dr. Punongbayan

• Local edema that aids in the proliferation of


organisms and spread into adjacent areas Pneumococcus
resulting in focal lobar involvement
ACUTE INFLAMMATORY UPPER AIRWAY OBSTRUCTION
CROUP (LTB) ACUTE EPIGLOTTITIS ACUTE INFECTIOUS LARYNGITIS
• URTI and low-grade fever 1-3 days
• Acute high fever, sore throat,
prior to UAO presentation • Sore throat, cough
dyspnea, and rapidly progressing
• “barking cough”, hoarseness • generally mild
Signs and obstruction (difficult swallowing,
• fever may become high-grade • respiratory distress is unusual
symptoms labored breathing in a few hours)
• symptoms worse at night and • hoarseness out of proportion to
• drooling and hyperextended neck;
agitation may aggravate symptoms symptoms
• no family member is ill
• other family members are ill
• In a tripod position
• Stridor, coryza, normal to
• stridor occurs late • Unremarkable except for pharyngeal
PE findings moderately inflamed pharynx, slight
• large, cherry red swollen epiglottis inflammation
tachypnea
by laryngoscopy
• Mainly clinical;
• X ray of the neck: subglottic • Clinical and by laryngoscopy
• Lateral X ray of the upper airway:
narrowing or “steeple sign” (may be (inflammatory edema of the vocal
• thumb sign
Diagnosis absent in patients with croup or cords and subglottic tissue)
• direct or flexible fiberoptic
present as a normal variant) • primary site of obstruction is the
laryngoscopy for direct visualization
• X rays do not correlate well with subglottic area
disease severity
• Airway management and treatment
of hypoxia
• Medical emergency: airway;
• nebulized racemic epinephrine for
Treatment Cefotaxime, Ceftriaxone, or • Supportive
moderate to severe croup
Meropenem IV
• single dose of oral dexamethasone
0.6 mg/kg
SPASMODIC CROUP BACTERIAL TRACHEITIS
• 1-3 years old; similar to LTB EXCEPT the history of a viral
prodrome and fever in the patient and family are often • Mean age 5-7 years old; M > F
absent; causes are viral, allergic, psychologic • often follows a viral respiratory infection
Signs and
• mostly nighttime symptoms with mild to moderate coryza • brassy cough
symptoms
and hoarseness • high fever and “toxicity” can occur immediately or after a
• awakens with a metallic, barking cough, noisy inspiration, few days of apparent improvement
appears frightened
• Usually afebrile
• Can lie flat, does not drool, no dysphagia
• severity usually diminishes within several hours and the
PE findings • mucosal swelling at the level of cricoid cartilage with
following day may appear well
copious, thick, purulent secretions
• episodes recur often several times
• Based on bacterial upper airway disease (high fever, purulent airway
secretions, absence of classic findings of epiglottitis)
Diagnosis • clinical
• lateral X ray of the neck: pseudomembranes detachment in the
trachea
Treatment • Same as croup (LTB) • Vancomycin or Clindamycin AND a 3rd generation cephalosporin
• Most common is S. aureus
• More of an allergic reaction to viral antigens than direct • MRSA, Strep. pneumoniae, Strep. pyogenes, Moraxella
Etiology
infection catarrhalis, non-typeable H. influenzae, anaerobic
organisms
TOPNOTCH MEDICAL BOARD PREP PEDIATRICS MAIN HANDOUT BY DRS. DE VERA AND PUNONGBAYAN Page 67 of 106
For inquiries visit www.topnotchboardprep.com.ph or email us at topnotchmedicalboardprep@gmail.com
This handout is only valid for October 2022 PLE batch. This will be rendered obsolete for the next batch since we update our handouts regularly.
TOPNOTCH MEDICAL BOARD PREP PEDIATRICS MAIN HANDOUT BY DRS. DE VERA AND PUNONGBAYAN
For inquiries visit www.topnotchboardprep.com.ph or https://www.facebook.com/topnotchmedicalboardprep/
This handout is only valid for October 2022 PLE batch. This will be rendered obsolete for the next batch since we update our handouts regularly.

LTB Epiglottitis Bacterial Tracheitis Spasmodic Croup


Age 3 mos – 6 yrs 2-5 yrs 5-7 yrs 1-3 yrs
Parainfluenza, RSV, H. influenza, S. pyogenes, S. S. aureus, M. catarrhalis, Viral, allergic,
Etiology
Adenovirus pneumoniae, S. aureus Anaerobes psychologic
Location Subglottic Supraglottic Trachea Subglottic
Onset Variable Rapid Biphasic Sudden
Fever Low/high High High None
Voice Hoarse Muffled No change/hoarse Hoarse
Cough Barking Muffled No change/hoarse Hoarse
Dysphagia - + (w/ drooling) - -
Toxicity +/- + + -
Position Comfortable Tripod Comfortable Comfortable
Stridor Ins > Exp Ins Ins & Exp Ins
Radiographic Steeple sign Thumb sign Ragged air column Normal
Pale mucosa
Deep red mucosa Cherry red epiglottis Deep red mucosa
Endoscopic Subglottic
Subglottic narrowing Ary-epiglottic swelling Copious secretions
narrowing
Racemic epinephrine, Intubation, antibiotics,
Treatment Intubation, antibiotics Cool mist
steroids, cool mist tracheal suctioning
Prognosis Excellent Excellent w/ treatment Excellent Excellent
• Presumptive extrapulmonary TB – refers to anyone having
UPPER AIRWAY s/sx specific to the suspected extrapulmonary site with or
OBSTRUCTION without constitutional s/sx such as unexplained fever or weight
https://qrs.ly/m4duqk9 loss, night sweats, fatigue, and loss of appetite
• Bacteriologically confirmed TB (BCTB) – a patient from
BACTERIAL TRACHEITIS whom a biological specimen is (+) for TB by smear microscopy,
culture, or rapid diagnostic tests (such as Xpert MTB/RIF, TB
• Diagnosis
LAMP, line probe assay for TB)
o Clinical picture
• Clinically diagnosed TB (CDTB) – a patient for which the
§ Toxic + absence of classic epiglottis
criterion for bacteriological confirmation is not met but
o X-rays
diagnosis is made on the basis of clinical findings, CXR
§ Not necessary
abnormalities, suggestive histology and/or biochemistry or
§ Findings of irregular lining of the trachea due to
imaging tests
pseudomembranes
§ Can have “steeple sign”
SYSTEMATIC SCREENING FOR TB IN 15 Y/O AND ABOVE
WITH UNKNOWN HIV INFECTION STATUS:

✔ GUIDE QUESTIONS
In most children, the only evidence of primary tuberculosis is:
A. Cough SYSTEMATIC SCREENING IN HEALTH FACILITIES:
B. Afternoon fever • For those who do not have the cardinal s/sx → offer chest X ray
C. Erythema nodosum if one has not been conducted in the past year
D. Recent conversion of PPD to positive
Tuberculin sensitivity develops ___ after its administration on the forearm.
• For PLHIV: screening by both CXR and symptoms should be
A. 5 days C. 3 months done at the time of diagnosis of HIV/AIDS and annually
B. 72 hours D. 7 months thereafter → one or more TB s/sy and/or a CXR suggestive of
The most common extrapulmonary form of tuberculosis in children is: TB, identify as presumptive TB in PLHIV
A. Meningitis C. Ileitis • Screening for extrapulmonary TB, all ages:
B. Scrofula D. TB verrucosa cutis o gibbus deformity
A 3-year-old boy has a positive tuberculin skin test. Which of the o non-painful enlarged cervical lymphadenopathy
following is suggestive of military TB? o nuchal rigidity and/or drowsiness
A. Infection of hilar lymph node B. Weight Loss
o pleural / pericardial effusion
C. Hepatosplenomegaly D. Chronic cough
o distended abdomen with ascites
PULMONARY TUBERCULOSIS o non-painful enlarged joint
• Primary complex (Ghon complex): o signs of tuberculin hypersensitivity (eg. erythema nodosum)
1. Primary pulmonary focus
2. Regional lymph nodes APPROACH TO DIAGNOSIS OF TB IN 15 Y/O AND BELOW:
3. Peritracheal lymph nodes • If CXR is strongly suggestive of TB based on the following,
4. Localized pleurisy between the middle & lower lobes classify as clinically diagnosed TB:
* Time between entry of the tubercle bacilli and tissue hypersensitivity ranges o markedly enlarged unequal hilar lymph gland >2 cm with
from 3-12 weeks or without opacification
o miliary mottling
NTP DOH 2020 Definition of terms: o large pleural effusion (equal or > ⅓ of pleural cavity, common
• Presumptive PTB - refers to any person having in >5 years old)
o (1) 2 weeks or longer of any of the following: o apical opacification with cavitation (rare in younger children,
§ Cough common in adolescents)
§ unexplained weight loss • If CXR finding is normal or uncertain and the child is stable,
§ unexplained fever follow up in 2 weeks and consider giving broad spectrum
§ night sweats, or; antibiotics for 1 week.
o (2) chest X ray finding suggestive of TB

TOPNOTCH MEDICAL BOARD PREP PEDIATRICS MAIN HANDOUT BY DRS. DE VERA AND PUNONGBAYAN Page 68 of 106
For inquiries visit www.topnotchboardprep.com.ph or email us at topnotchmedicalboardprep@gmail.com
This handout is only valid for October 2022 PLE batch. This will be rendered obsolete for the next batch since we update our handouts regularly.
TOPNOTCH MEDICAL BOARD PREP PEDIATRICS MAIN HANDOUT BY DRS. DE VERA AND PUNONGBAYAN
For inquiries visit www.topnotchboardprep.com.ph or https://www.facebook.com/topnotchmedicalboardprep/
This handout is only valid for October 2022 PLE batch. This will be rendered obsolete for the next batch since we update our handouts regularly.
• If the child still has persistent s/sx during follow up, may MOLECULAR DIAGNOSTIC TESTS
classify as clinically diagnosed TB if a contact of a known TB • GENE Xpert MTB/RIF ASSAY
case. o real time PCR-based molecular test that can simultaneously
• If not a contact of a TB case, perform TST. detect TB bacteria and rifampicin resistance in clinical
o If TST is positive, may classify as clinically diagnosed TB. specimens in less than 2 hours
o If TST is negative or not available, even if clinical s/sx remain o Meta-analysis on the diagnostic accuracy in children:
unresolved, it is less likely to be TB. § Expectorated sputum - 55-90%
• EPTB can be diagnosed clinically and bacteriologically using § Induced sputum – 40-100%
Xpert MTB/RIF. § Gastric lavage or aspirate – 40-100%
APPROACH TO DIAGNOSIS OF TB IN 15 Y/O AND BELOW o WHO recommendations on usage of Gene Xpert assay:
(NTP MOP 2020) § Used as replacement for conventional microscopy
§ Culture and drug susceptibility testing as the initial
diagnostic test
§ Suspected at risk for drug-resistant TB or have HIV-
associated TB
• INTERFERON GAMMA RELEASE ASSAY
o Blood sample mixed with antigens specific for MTB complex
strains and absent from the BCG vaccine strain
o WBCs from infected persons release IFN-g as a marker of
infection
o A (+) result suggests that MTB infection is likely
o Commercially available and FDA-approved: QuantiFERON TB
Gold in tube, ELISPOT-based T-SPOT TB
o Advantages: result available in 24 hours, booster
phenomenon does not occur, not affected by prior BCG
vaccination
o Limitations: expensive, needs to be processed immediately,
paucity of data in <5-year-old children and
immunocompromised patients

KEY FACTS TUBERCULIN SKIN TEST AND IGRA


• TST and IGRA generally should not be tested on persons with
low risk of TB infection and TB disease
• Neither TST nor IGRA can distinguish active TB from LTBI
• IGRA can distinguish LTBI from previous BCG administration
• Routine testing with TST and IGRA is not generally
recommended except when there is suspected active TB in
DEFINITION OF TERMS IN TREATMENT OF TB: immunocompromised patients or indeterminate results from
• RETREATMENT - has been treated before with anti-TB drugs either test
for at least 1 month and includes the ff: • TST is preferred over IGRA in children <5 years old.
o RELAPSE – declared cured and completed Tx but is presently *Tuberculosis in Infancy and Childhood. 4th ed. PPS Committee on Handbook on Childhood Tuberculosis. 2016

diagnosed with active TB


o TREATMENT AFTER FAILURE – previously treated but failed TREATMENT REGIMENS FOR DS-TB:
most recent course based on a (+) SM follow up at 5 months later REGIMEN ELIGIBLE TB PATIENTS
o TREATMENT AFTER LOST TO FOLLOW-UP – previously • PTB or EPTB (except CNS, bones, joints)
treated for TB but lost to follow up for at least 2 months whether new or retreatment with final
o PREVIOUS TREATMENT OUTCOME UNKNOWN Xpert result: MTB, RIF sensitive and MTB,
o PATIENTS WITH UNKNOWN PREVIOUS TB TREATMENT HISTORY Regimen 1 RIF indeterminate
PRIMARY DIAGNOSTIC TOOLS (WHO-ENDORSED): 2HRZE/4HR • new PTB or new EPTB (except CNS, bones,
automated molecular assay and is a rapid test joints) with (+) SM/TB LAMP or clinically
Xpert MTB / RIF diagnosed and Xpert not done and Xpert
that detects MTB and rifampicin resistance
manual molecular assay that can be read result is MTB not detected
TB LAMP
(Loop-mediated with the naked eye under UV light in • EPTB of CNS, bones, joints whether new or
isothermal under an hour to detect MTB but cannot retreatment, with final Xpert result: MTB,
amplification) detect rifampicin resistance RIF sensitive and MTB, RIF indeterminate
Regimen 2
Line probe based on PCR to detect MTB and drug • new EPTB of CNS, bones, joints with (+)
2HRZE/10HR
assay sensitivity to Rif and INH SM/TB LAMP or clinically diagnosed and
conventional test that serve as a basis for Xpert not done and Xpert result is MTB not
Smear diagnosis of TB, to monitor progress of detected
microscopy patients while on Tx, and confirm cure at • (H) INH 10 mg/kg/day PO (10-15 mkday; max 300 mg/day)
the end of Tx for DS-TB cases • (R) Rif 15 mg/kg/day PO (10-20 mgkday; max 600 mg/day)
TB culture and used in diagnosing and monitoring Tx • (Z) PZA 25 mg/kg/day PO (20-40 mkday; max 2 grams/day)
drug sensitivity response for DR-TB, for prevalence • (E) Ethambutol 15 mg/kg/day PO (15-25 mkday; max 1.2
test studies, drug resistance surveillance grams/day)
ADJUVANT DIAGNOSTIC TOOLS:
Chest X ray cannot differentiate DS-TB from DR-TB; has low specificity SUPPLEMENT: QUICK SHEET
• basic screening tool for TB infection among Additional notes on PTB drugs:
children • INH bactericidal and inhibits mycolic acid synthesis
• NOT a mandatory tool and its absence is not a • RIF inhibits DNA-dependent RNA polymerase
Tuberculin deterrent in diagnosing TB • PZA disrupts membrane energy metabolism
skin test • false (+) in recently BCG-vaccinated and false • ETM inhibits transferase enzymes involved in cell wall
(TST) or (-) in immunocompromised and severely synthesis
Mantoux malnourished children • For INH and RIF: discontinue if ALT/AST is more than 3-5x
test • TST is positive if there is induration of more the normal values; best absorbed on an empty stomach; no
than 5mm in immunocompromised children dose adjustment for renal dysfunction; common adverse
OR >10mm in children regardless of BCG reactions are hepatitis, allergic skin reactions, peripheral
vaccination neuropathy
TOPNOTCH MEDICAL BOARD PREP PEDIATRICS MAIN HANDOUT BY DRS. DE VERA AND PUNONGBAYAN Page 69 of 106
For inquiries visit www.topnotchboardprep.com.ph or email us at topnotchmedicalboardprep@gmail.com
This handout is only valid for October 2022 PLE batch. This will be rendered obsolete for the next batch since we update our handouts regularly.
TOPNOTCH MEDICAL BOARD PREP PEDIATRICS MAIN HANDOUT BY DRS. DE VERA AND PUNONGBAYAN
For inquiries visit www.topnotchboardprep.com.ph or https://www.facebook.com/topnotchmedicalboardprep/
This handout is only valid for October 2022 PLE batch. This will be rendered obsolete for the next batch since we update our handouts regularly.
SUPPLEMENT: QUICK SHEET
• For PZA: Discontinue if AST / ALT is more than 3-5x the
normal values; requires dose adjustment in renal failure;
adverse reactions are nausea, vomiting, polyarthralgia,
hypersensitivity reaction
• For Ethambutol: Previously omitted from regimens for
children <6 years old due to difficulty of monitoring optic
neuritis; new evidence shows safety in children at
recommended doses in the absence of renal impairment

RETROPHARYNGEAL ABSCESS
• 3-4 years old; M>F
• Retropharyngeal space located between the pharynx & the
cervical vertebrae & extending down into the superior
mediastinum
• PE: drooling, neck held in hyperextension, bulging of the
posterior pharyngeal wall, neck pain, muffled voice,
respiratory distress
• Group A streptococcus, anaerobes, Staphylococcus aureus;
others are Klebsiella and Haemophilus influenzae
• IV antibiotics with or without surgical drainage: 3rd gen
cephalosporin with Ampicillin-Sulbactam OR Clindamycin
✔ GUIDE QUESTIONS
TREATMENT OF TB IN CHILDREN A 4-year-old male child was brought to the clinic due to worsening
Dr. Arada cough now with purulent sputum. Symptoms started 3 days ago with
runny nose with clear nasal discharge, after which a frequent
PULMONARY intermittent dry hacking cough followed. Examination revealed HR 90,
RR 33, T 37.2°C, PE revealed: erythematous congested nasal mucosa,
TUBERCULOSIS slightly erythematous posterior pharynx, with some occasional coarse
https://qrs.ly/4vduqkz crackles and scattered high pitched wheezing, no nasal flaring, no
retractions. What is the most likely diagnosis?
A. Pneumonia
✔ GUIDE QUESTIONS
B. Bronchial asthma
A 3 y/o male patient presents with a 2-day history of gradual decreased
C. Bronchiolitis
oral intake, irritability, and fever of 38.5°C. The child points to the neck
D. Bronchitis
area that seems to bother him, and his voice was a little muffled. PE
E. Pertussis
revealed HR 108, RR 33, T 38.7 C with minimal movement and refusal
to move the neck, (+) neck stiffness. On oral examination. there is mild The main feature of acute bronchitis is the dry hacking cough; most
erythematous posterior pharynx and bulging of the posterior of the patients have low- to moderate-grade fever and do not appear
pharyngeal wall; chest PE revealed no crackles, occasional rhonchi. to be acutely ill; most commonly caused by viruses; supportive
What is the most likely diagnosis? management
Dr. Punongbayan
A. Retropharyngeal abscess
B. Peritonsillar abscess
C. Bacterial tracheitis RHEUMATOLOGY
D. Epiglottitis
E. Laryngotracheobronchitis SYMPTOMS SUGGESTIVE OF RHEUMATIC DISEASE:
• Joint pain, fever, fatigue, and rash
A common differential diagnosis for this is Acute Epiglottitis since they
both present acutely and in respiratory distress; consider • Arthralgia without physical findings of arthritis suggests:
retropharyngeal abscess based on PE finding of bulging of the o Infection
posterior pharyngeal wall; the red, swollen epiglottis can be directly o Malignancy
visualized by laryngoscopy (done in double set up in anticipation of o orthopedic conditions
possible respiratory arrest) in cases of epiglottitis. o benign syndromes
Dr. Punongbayan
o pain syndromes like fibromyalgia
A 16-year-old female patient presents with a 3-day history of cough and
sore throat. She sought consult because of worsening sore throat and • Pain syndrome (fibromyalgia) – poor sleep, debilitating
cough, fever, and trismus. There was also some dysphagia. PE revealed generalized joint pain that worsens with activity, school
an asymmetrical right tonsillar bulge with displacement of the uvula absences, and normal physical and lab findings in an adolescent
and erythematous posterior pharyngeal wall, BP 100/60, HR 98, RR 22, • Growing pains – children aged 3-10 years with a history of
T 38.8°C. What is the most likely diagnosis? episodic pain that occurs at night after increased daytime
A. Retropharyngeal abscess physical activity and is relieved by rubbing; no limp or
B. Peritonsillar abscess
complaints in the morning
C. Bacterial tracheitis
D. Epiglottitis • Patellofemoral syndrome – adolescent girl with knee pain
E. Laryngotracheobronchitis aggravated by walking upstairs and on patellar distraction
What is your management of choice for this patient? • Benign hypermobility syndrome – intermittent pain esp. in a
A. Antibiotics only girl aged 3-10 years that is increased with activity and is
B. Surgical management only associated with hyperextensible joints
C. Antibiotics and Surgery
D. Antibiotics ± surgical drainage ✔ GUIDE QUESTIONS
A 4-year-old female patient presents to the clinic with 5-day history of
fever with temperatures between 38-39°C. There was bilateral redness
PERITONSILLAR ABSCESS of the eyes with no discharge, decreased appetite and intake, physical
• Bacterial invasion through the capsule of the tonsils exam reveals dry cracked lips, erythematous oral and pharyngeal
mucosal areas, (+) cervical lymphadenopathy, maculopapular and
• Adolescents
scarlatiniform rashes most prominent at the groin areas, there was also
• Group A streptococcus and anaerobes noted mild swelling and erythema of the hands and feet, HR 108, RR 28
• Fever, sore throat, dysphagia, trismus T > 38.8°C what is the most likely diagnosis?
• PE: asymmetric tonsillar bulge with a displaced uvula A. Rubeola D. Kawasaki disease
• CT scan - ideally B. Steven Johnson syndrome E. Scarlet fever
• Surgical drainage (I & D or needle aspiration) and antibiotics C. Ritter disease
In a patient with known Kawasaki disease, which phase of the illness
gives the highest risk for sudden death?
A. Prodromal C. Subacute
B. Acute febrile D. Convalescence

TOPNOTCH MEDICAL BOARD PREP PEDIATRICS MAIN HANDOUT BY DRS. DE VERA AND PUNONGBAYAN Page 70 of 106
For inquiries visit www.topnotchboardprep.com.ph or email us at topnotchmedicalboardprep@gmail.com
This handout is only valid for October 2022 PLE batch. This will be rendered obsolete for the next batch since we update our handouts regularly.
TOPNOTCH MEDICAL BOARD PREP PEDIATRICS MAIN HANDOUT BY DRS. DE VERA AND PUNONGBAYAN
For inquiries visit www.topnotchboardprep.com.ph or https://www.facebook.com/topnotchmedicalboardprep/
This handout is only valid for October 2022 PLE batch. This will be rendered obsolete for the next batch since we update our handouts regularly.

KAWASAKI DISEASE TREATMENT OF KAWASAKI DISEASE


Adapted from Nelson Textbook of Pediatrics, 20th ed
(MUCOCUTANEOUS LYMPH NODE SYNDROME) Acute Stage
SUPPLEMENT: QUICK SHEET • Intravenous immunoglobulin 2g/kg over 10-12 hours and
KAWASAKI DISEASE • Aspirin 80-100 mg/kg/day divided every 6 hours orally until
• FEVER for at least 5 days +4 of the following features: patient is afebrile for at least 48 hours
o Conjunctivitis
§ Bilateral, bulbar injection without exudate Convalescent Stage
o Rash • Aspirin 3-5 mg/kg once daily orally until 6-8 weeks after illness
§ Any form of rash onset if normal coronary findings throughout course
o Adenopathy
§ Cervical adenopathy, non-suppurative Long-term Therapy for Patients with Coronary Abnormalities
o Strawberry tongue and other oropharyngeal changes • Aspirin 3-5 mg/kg once daily orally
§ Red cracked, swollen lips • Clopidogrel 1 mg/kg/day (Max: 75 mg/day)
o Hand changes • Most experts add Warfarin or LMWH for those patients at
§ Brawny edema, induration particularly high risk of thrombosis
§ Peeling around the nail beds
OTHER FEATURES OF KD: Acute Coronary Thrombosis
• Prompt fibrinolytic therapy with tPA or other thrombolytic
• medium-sized vasculitis that affects more males and is most
agent under supervision of a pediatric cardiologist
common in the 1st 5 years of life
• unknown cause
• aneurysm of the major coronary arteries is the most ROLE OF IVIG
characteristic finding IN KAWASAKI DISEASE
• Perineal desquamation is https://qrs.ly/awduqmd
common in the acute phase
especially in younger than IVIG RESISTANT KD
1-year-old patients • persistent or recrudescent fever 36 hr after completion of the
• Periungual desquamation initial IVIG infusion
begins 1-3 weeks after the • Increased risk for CAA
onset of illness • another dose of IVIG at 2 g/kg is administered to patients with
• Coronary artery IVIG resistance
aneurysms develop in the o Re-treatment of patients with refractory Kawasaki disease -
2nd-3rd week of illness in untreated cases. another IVIG 2g/kg
• Clinical phases of KD: o If there is a poor response to the 2nd IVIG, some patients have
1. Acute febrile phase: last 1-2 wks; fever and other acute signs of illness responded to IV methylprednisolone 30 mg/kg/day for 3 days.
2. Subacute phase: begins when fever & other acute signs have o Patients with a small solitary aneurysm should continue
abated; associated with desquamation, thrombocytosis, aspirin indefinitely.
Nelson Textbook of Pediatrics, 21st ed. 2020
development of coronary aneurysms, and highest risk of
sudden death in those who have developed aneurysms; lasts
until the 4th week MULTISYSTEM INFLAMMATORY SYNDROME
3. Convalescent phase: begins when all clinical signs have IN CHILDREN (MIS-C):
disappeared & continues until ESR & CRP return to normal • A serious, rare condition in children in which the body’s own
about 6-8 wks after the onset immune system overreacts to a stimulus
ATYPICAL KAWASAKI INCOMPLETE KAWASAKI • Results in inflammation of multiple organ systems → leads to
DISEASE DISEASE impaired organ function and organ failure
• Any infant or child with 5 or • Considered in all children • Diagnosed in <21 years of age in mostly Europe and the USA
more days of fever, with 2-3 with unexpected fever for 5 • Unknown cause
of the principal clinical or more days with 3 or less of • Mounting evidence that is linked to COVID-19
features, but have compatible the clinical criteria • Still needs more research and evidence
laboratory findings • More common in young
• May have manifestations infants
MIS-C CRITERIA BY CDC 2020:
such as renal failure, • Perineal erythema and
unilateral facial nerve palsy, desquamation • patient is 21 years of age or younger and has had a fever of
pulmonary infiltrates and/or • Higher risk to develop 38°C for at least 24 hours
effusion, testicular swelling, coronary artery lesions • blood work shows indications of inflammation
acute abdomen, • Laboratory evidence of • requires treatment at a hospital (often in the intensive care unit)
hemophagocytic syndrome systemic inflammation due to severe illness that includes dysfunction of two or more
ANCILLARY TESTS IN KD: organs, particularly the heart, blood vessels, GI organs, lungs,
• NO diagnostic test kidneys, skin, eyes, or nervous system
• WBC is normal to elevated • absence of other diagnoses that explain inflammatory
symptoms
• Elevated ESR, CRP for 4-6 weeks
• A positive test for current or past infection by SARS-CoV-2, or
• Normocytic, normochromic anemia
failing a positive test, evidence of exposure to someone with the
• Platelet count usually normal in the 1st week of illness but
virus within four weeks of the onset of symptoms of MIS-C
rapidly increases in the 2nd-3rd week
• Evidence of inflammation (including but not limited to 1 or more
• Sterile pyuria
of the ff):
• Mildly high liver transaminases
o Elevated CRP o Elevated LDH
• 2D echo should be performed at diagnosis and repeated o Elevated ESR o Elevated IL-6
after 2-3 weeks of illness o Elevated fibrinogen o Elevated neutrophils
o If both are normal, a repeat study should be done 6-8 wks after o Elevated procalcitonin o Reduced lymphocytes
onset of illness. o Elevated D-dimer o Low albumin
✔ GUIDE QUESTIONS o Elevated ferritin
Which of the ff. is the preferred treatment of Kawasaki in the acute stage Other evaluations:
of the disease? • ECG
A. IV Ig C. Warfarin
• 2D echocardiography
B. Low dose aspirin D. Both A and B
Refer to Table 166-3 seen below for the treatment of Kawasaki • Cardiac enzyme or troponin
disease lifted from Nelson textbook of Pediatrics 20th ed. • B-type natriuretic peptide
Dr. Punongbayan

TOPNOTCH MEDICAL BOARD PREP PEDIATRICS MAIN HANDOUT BY DRS. DE VERA AND PUNONGBAYAN Page 71 of 106
For inquiries visit www.topnotchboardprep.com.ph or email us at topnotchmedicalboardprep@gmail.com
This handout is only valid for October 2022 PLE batch. This will be rendered obsolete for the next batch since we update our handouts regularly.
TOPNOTCH MEDICAL BOARD PREP PEDIATRICS MAIN HANDOUT BY DRS. DE VERA AND PUNONGBAYAN
For inquiries visit www.topnotchboardprep.com.ph or https://www.facebook.com/topnotchmedicalboardprep/
This handout is only valid for October 2022 PLE batch. This will be rendered obsolete for the next batch since we update our handouts regularly.

MIS-C IN PEDIATRIC AGE GROUP: JUVENILE DERMATOMYOSITIS


• Kawasaki-like features but do not need to fulfill KD criteria • The most common of pediatric inflammatory myopathies
• History of exposure to Covid, recent/past infection with Covid • Systemic vasculopathy with characteristic cutaneous findings
• Need to do RT-PCR and serologic test and focal areas of myositis resulting in progressive proximal
• Difference between severe Covid and MIS-C: lungs in MIS-C not muscle weakness
significantly affected; heart is most commonly affected • Gottron Papules
• Check HEART RATE (significant tachycardia with MIS-C) o Alligator skin like appearance
o Skin over the metacarpal and
✔ GUIDE QUESTION proximal IPJ may be
An 18-year-old female presents with 3-month history of fever ranging hypertrophic and reddish pink
from 37.8 to 38.5°C, weight loss, irritability, and joint pains. She also
has some muscle weakness and has difficulty climbing the stairs and • Heliotrope eyelids / heliotrope
combing her hair sometimes. She also started having a violaceous rash
erythematous rash near the periorbital area that extends to the nose.
o Periorbital violaceous erythema
Current PE: HR 90, RR 18, T 37.5°C, characteristic rash over the face,
(+) muscle weakness 4/5 over biceps and hips, (+) reddish pink colored that may cross over the bridge of
skin over the dorsal surfaces of the metacarpal joints, what is the most the nose
likely diagnosis?
• SHAWL SIGN
A. Dermatomyositis
B. SLE
C. Photosensitive dermatitis
D. Vitamin c deficiency
E. Allergic contact dermatitis
When you encounter a case of a possible inflammatory myopathy or
CLINICAL MANIFESTATIONS:
rheumatic in origin, take note of the onset of disease (usually chronic)
• Neck flexor or abdominal muscle weakness
and the involved features such as presence or absence of fever,
• Unable to sit up or head lag during infancy, and Gower sign (use
myalgia, muscle weakness, arthralgia, arthritis, and rash. In this case,
most notable were the chronic history, fever, muscle weakness and the
of hands on thighs to stand from a sitting position)
pink-colored rash on the dorsum of metacarpophalangeal joints, all
• Derangement of upper airway function
pointing to dermatomyositis.
Dr. Punongbayan • Dysphagia is a severe prognostic sign
• Constipation, abdominal pain, or diarrhea
• Dilated cardiomyopathy
THE EULAR/ACR CLASSIFICATION CRITERIA FOR ADULT AND JUVENILE IDIOPATHIC INFLAMMATORY MYOPATHIES
When no better explanation for the symptoms and signs exists these classification criteria can be used
SCORE POINTS
NO WITH
VARIABLE DEFINITION
MUSCLE MUSCLE
BIOPSY BIOPSY
Age of onset
Age of onset of first symptom assumed to be related 18 ≤ Age (years) at onset of first symptom assumed to be related to the
1.3 1.5
to the disease ≥ 18 years and < 40 years disease < 40
Age of onset of first symptom assumed to be related Age (years) at onset of first symptom assumed to be related to the disease
2.1 2.2
to the disease ≥ 40 years ≥ 40
Muscle weakness
Weakness of proximal upper extremities as defined by manual muscle
Objective symmetric weakness, usually progressive,
0.7 0.7 testing or other objective strength testing, which is present on both sides
of the proximal upper extremities
and is usually progressive over time
Weakness of proximal lower extremities as defined by manual muscle
Objective symmetric weakness, usually progressive,
0.8 0.5 testing or other objective strength testing, which is present on both sides
of the proximal lower extremities
and is usually progressive over time
Neck flexors are relatively weaker than neck Muscle grades for neck flexors are relatively lower than neck extensors as
1.9 1.6
extensors defined by manual muscle testing or other objective strength testing
Muscle grades for proximal muscles in the legs are relatively lower than
In the legs proximal muscles are relatively weaker
0.9 1.2 distal muscles in the legs as defined by manual muscle testing or other
than distal muscles
objective strength testing
Skin manifestations
Purple, lilac-colored or erythematous patches over the eyelids or in a
Heliotrope rash 3.1 3.2
periorbital distribution, often associated with periorbital edema
Erythematous to violaceous papules over the extensor surfaces of joints,
Gottron papules 2.1 2.7 which are sometimes scaly. May occur over the finger joints, elbows, knees,
malleoli and toes
Erythematous to violaceous macules over the extensor surfaces of joints,
Gottron sign 3.3 3.7
which are not palpable
Other clinical manifestations
Difficulty in swallowing or objective evidence of abnormal motility of the
Dysphagia or esophageal dysmotility 0.7 0.6
esophagus
Laboratory measurements
Anti-Jo-1 (anti-histidyl-tRNA synthetase) Autoantibody test in serum performed with standardized and validated
3.9 3.8
autoantibody present test, showing positive result
Elevated serum levels of creatine kinase
(CK)* or lactate dehydrogenase (LDH)* or aspartate The most abnormal test values during the disease course (highest absolute
1.3 1.4
aminotransferase (ASAT/AST/SGOT)* or alanine level of enzyme) above the relevant upper limit of normal
aminotransferase (ALAT/ALT/SGPT)*
Muscle biopsy features- presence of:
Muscle biopsy reveals endomysial mononuclear cells abutting the
Endomysial infiltration of mononuclear cells
1.7 sarcolemma of otherwise healthy, non-necrotic muscle fibers, but there is
surrounding, but not invading, myofibers
no clear invasion of the muscle fibers
Perimysial and/or perivascular infiltration of Mononuclear cells are located in the perimysium and/or located around
1.2
mononuclear cells blood vessels (in either perimysial or endomysial vessels)
Muscle biopsy reveals several rows of muscle fibers which are smaller in
Perifascicular atrophy 1.9
the perifascicular region than fibers more centrally located
Rimmed vacuoles are bluish by Hematoxylin and Eosin staining and
Rimmed vacuoles 3.1
reddish by modified Gomori- Trichrome stains
*Serum levels above the upper limit of normal
TOPNOTCH MEDICAL BOARD PREP PEDIATRICS MAIN HANDOUT BY DRS. DE VERA AND PUNONGBAYAN Page 72 of 106
For inquiries visit www.topnotchboardprep.com.ph or email us at topnotchmedicalboardprep@gmail.com
This handout is only valid for October 2022 PLE batch. This will be rendered obsolete for the next batch since we update our handouts regularly.
TOPNOTCH MEDICAL BOARD PREP PEDIATRICS MAIN HANDOUT BY DRS. DE VERA AND PUNONGBAYAN
For inquiries visit www.topnotchboardprep.com.ph or https://www.facebook.com/topnotchmedicalboardprep/
This handout is only valid for October 2022 PLE batch. This will be rendered obsolete for the next batch since we update our handouts regularly.
LEVEL OF ✔ GUIDE QUESTIONS
SCORE In relation to the case above, what vessels are commonly affected?
PROBABILITY
A. Coronary and other muscular arteries
DEFINITE IIM Equal or >7.5 Equal or >90%
B. Large arteries
PROBABLE Equal or >5.5 to C. Medium arteries
Equal or >55 to <90%
IIM <7.5 D. Small arteries
POSSIBLE IIM Score of 5.3 or 5.4 Equal or >50 to <55% Skin biopsies have demonstrated vasculitis of the dermal capillaries and
postcapillary venules.
DIAGNOSTIC CRITERIA (BOHAN & PETER, 1975) Dr. Punongbayan
In relation to the above case, what is the best approach to treatment?
• Classic rash (heliotrope rash of the eyelids, Gottron papules) A. Supportive therapy
PLUS 3 of the ff: B. Mycophenolate or cyclophosphamide
1. Weakness (symmetric, proximal) C. Oral prednisone
2. Muscle enzyme elevation equal or >1 (CK, aspartate D. Plasmapheresis and IV Ig
aminotransferase, LDH, aldolase) Steroids are most often used to treat significant gastrointestinal
3. Electromyographic changes (myopathy, denervation) involvement or other life-threatening manifestations
Dr. Punongbayan
4. Muscle biopsy (necrosis, inflammation)

DIAGNOSIS AND LAB FINDINGS HENOCH-SCHÖNLEIN PURPURA


• PE of the nailfold capillaries show periungual avascularity with • IgA-mediated vasculitis of small vessels (dermal capillaries
capillary dropout and vessel dilatation with terminal bush and postcapillary venules)
formation • Most common cause of nonthrombocytopenic purpura in
• Alanine aminotransferase – most commonly elevated on children
initial presentation • Unknown cause; typically follows an URTI
• Antibodies to Pm/Scl identify a small, distinct group with a • 3-10 yrs old; male:female ratio 1.2-1.8:1
protracted disease course with pulmonary and cardiac • Linked with nephritis: HLA-B34 and HLA-DRB1*01
involvement • Pattern of crops of palpable purpura in the dependent parts
• MRI either full body or of the thigh and shoulder muscles • Lab tests are neither specific nor diagnostic
• Muscle biopsy confirms the diagnosis if the clinical and • ↑ ESR, anemia, 50% have ↑ IgA/IgM, (-) ANA & ANCAs
laboratory manifestations are inconclusive • Skin biopsy: leukocytoclastic angiitis
• Renal biopsy: IgA mesangial deposition

TREATMENT
• Symptomatic treatment
• Self-limiting
• Steroids for severe abdominal pain and joint pain
• Regardless of the severity of symptoms, patients require serial
urinalysis for 6 months after diagnosis especially those who
presented with hypertension or urinary abnormalities

✔ GUIDE QUESTIONS
A 10-year-old female patient presents with morning stiffness and joint pain
later in the day particularly in the knees and sometimes in the ankles it has
been bothering her for the past 2 months. PE revealed joint swelling with
limitation of range of motion of the knees and ankles. No other complaints
noted, HR 90, RR 20, T 37.2°C. What is the most likely the diagnosis?
A. Systemic onset JRA
B. Oligoarticular JIA
C. Polyarticular JIA
D. Rheumatic fever
Pauciarticular is the old term for oligoarthritis type of juvenile idiopathic
arthritis. Oligoarthritis involves less than 4 joints involved in the 1st 6
months of presentation. This is based on the 1997 ILAR (International
League of Associations for Rheumatology) classification.
TREATMENT Dr. Punongbayan
• Corticosteroids – Methylprednisolone for more severe cases Which of the ff. statements pertain to the above case and condition?
• Methotrexate decreases the length of treatment with steroids A. The disease is usually associated with RF positive
B. The disease is usually associated with ANA(+)
• Folic acid reduces toxicity and S/E of folate inhibition like oral C. It is the most aggressive or disabling form
ulcers, nausea, and anemia D. It is the least common type
• IVIG for severe cases About 40-85% of patients with pauciarticular or oligoarticular JIA will
have positive ANA
✔ GUIDE QUESTIONS Dr. Punongbayan

A 7-year-old male child presents to the clinic with a rash extending from the A 13 y/o female patient presents to the clinic with complaints of fever, rash,
buttocks to the lower extremities. The rash is characterized as raised and joint pains. It started 2 months ago with pain and swelling over her
pinkish to erythematous purpuric lesions. The mother recalled that he had knees and sometimes her ankles. She has also been having intermittent
just gotten well from the flu 1 week ago. He also has right knee pain with fever 1-2x/day ranging from 38-39°C returning to normal temperatures in
slight swelling but not warm and non-erythematous. He is presently between. She would also have faint, macular, salmon-colored evanescent
afebrile. Urinalysis reveals 6-8 RBC, protein +2, WBC 0-2, glucose (-), rash especially at the height of the fever. What is the most likely diagnosis?
bacteria +1. What is the most likely diagnosis in this case? A. Systemic onset JRA
A. Hemolytic Uremic Syndrome B. Pauciarticular JRA
B. Henoch Schönlein Purpura C. Polyarticular JRA
C. Dengue Fever D. Rheumatic fever
D. Meningococcemia Features of fever, rash, and joint pain and swelling for more than 6 weeks
In relation to the above case, what is the pathophysiologic mechanism point to a case of juvenile idiopathic arthritis. Letter D (rheumatic fever)
involved? is a close differential diagnosis but when Jones criteria is utilized, the
A. Deposition of IgA and immune complexes description of the skin lesion in this case is more of JIA rather than the
B. Microvascular thrombi leading to hemolysis erythema marginatum lesions seen in rheumatic fever.
C. Immune reaction against platelet with increased vascular Dr. Punongbayan
permeability
D. Endotoxin release resulting to DIC and shock

TOPNOTCH MEDICAL BOARD PREP PEDIATRICS MAIN HANDOUT BY DRS. DE VERA AND PUNONGBAYAN Page 73 of 106
For inquiries visit www.topnotchboardprep.com.ph or email us at topnotchmedicalboardprep@gmail.com
This handout is only valid for October 2022 PLE batch. This will be rendered obsolete for the next batch since we update our handouts regularly.
TOPNOTCH MEDICAL BOARD PREP PEDIATRICS MAIN HANDOUT BY DRS. DE VERA AND PUNONGBAYAN
For inquiries visit www.topnotchboardprep.com.ph or https://www.facebook.com/topnotchmedicalboardprep/
This handout is only valid for October 2022 PLE batch. This will be rendered obsolete for the next batch since we update our handouts regularly.

JUVENILE IDIOPATHIC ARTHRITIS


INTERNATIONAL LEAGUE OF ASSOCIATIONS FOR RHEUMATOLOGY (ILAR) CLASSIFICATION OF JUVENILE IDIOPATHIC ARTHRITIS:
CATEGORY DEFINITION
• Arthritis in equal or >1 joint with, or preceded by, fever for at least 2 weeks in duration
documented to be daily for at least 3 days and accompanied by 1 or more of the following:
1. Evanescent erythematous rash
SYSTEMIC
2. Generalized lymph node enlargement
3 .Hepatomegaly or splenomegaly or both
4. Serositis
• Arthritis affecting 1-4 joints during the 1st 6 months of disease:
OLIGOARTHRITIS 1. Persistent – equal or <4 joints affected throughout the course
2. Extended – affecting >4 joints after the 1st 6 months of disease
• RF NEGATIVE – affects 5 or more joints during the 1st 6 months of disease
POLYARTHRITIS • RF POSITIVE – affects 5 or more joints during the 1st 6 months of disease; 2 or more tests for RF at least
3 months apart during the 1st 6 months of disease are positive
• Arthritis and psoriasis, or arthritis and at least 2 of the ff:
1. Dactylitis (swelling of 1 or more digits in an asymmetric distribution that extends beyond the joint
PSORIATIC ARTHRITIS margin)
2. Nail pitting (minimum of 2 pits on any 1 or more nails at any time) and onycholysis
3. Psoriasis in first-degree relative
• Arthritis and enthesitis (tenderness at the insertion of a tendon, ligament joint capsule, or fascia
to the bone), or arthritis or enthesitis with at least 2 of the ff:
1. Presence of or history of sacroiliac joint tenderness or inflammatory lumbosacral pain, or both
ENTHESITIS-RELATED 2. Presence of HLA-B27 antigen
ARTHRITIS 3. Onset of arthritis in a male >6 years old
4. Acute (symptomatic) anterior uveitis
5. History of ankylosing spondylitis, enthesitis-related arthritis, sacroiliitis with IBD, Reiter syndrome, or
acute anterior uveitis in first-degree relative
UNDIFFERENTIATED
• Arthritis that fulfills criteria in no category or in 2 or more of the above categories
ARTHRITIS

AMERICAN COLLEGE OF RHEUMATOLOGY VS INTERNATIONAL LEAGUE OF ASSOCIATIONS FOR RHEUMATOLOGY


CLASSIFICATION OF CHILDHOOD CHRONIC ARTHRITIS
PARAMETER ACR (1977) ILAR (1997)
Term Juvenile rheumatoid arthritis • Juvenile Idiopathic arthritis
• Oligoarthritis
= or <4 joints in 1st 6 months
Pauciarticular o Persistent: <4 joints for course of disease
after presentation
o Extended: >4 joints after 6 months
>4 joints in 1st 6 months after • Polyarticular RF negative
Polyarticular
presentation • Polyarticular RF positive
Fever, rash, arthritis Systemic Systemic
• Psoriatic arthritis
Other categories included Exclusion of other forms • Enthesitis-related arthritis
• Undifferentiated
Inclusion of psoriatic arthritis,
No Yes
IBD, ankylosing spondylitis
• Goals of treatment: • Need individualized treatment plan and tailor management
1. To achieve disease remission based on:
2. To prevent or halt joint damage 1. Disease subtype and severity
3. To foster normal growth and development 2. Presence of poor prognostic indicators
3. Response to medications – Monitor potential medication toxicities
PHARMACOLOGIC TREATMENT
MEDICATION DOSE SUBTYPE SIDE EFFECTS
NSAIDs
• Naproxen • 15 mg/kg/day po BID Polyarthritis,
Gastritis, renal, and hepatic toxicity
• Ibuprofen • 40 mg/kg/day po TID oligoarthritis, systemic
• Meloxicam • 0.125 mg/kg po OD
Disease modifying anti-
Nausea, vomiting, oral ulcers, liver toxicity,
rheumatic drugs (DMARDs) Polyarthritis,
dyscrasias
• Methotrexate (oldest and least toxic) • 0.5-1 mg/kg po or SC weekly oligoarthritis, systemic
GI upset, allergic reaction, renal and liver
• Sulfasalazine • Initial 12.5 mkdose po daily; Polyarthritis
toxicity, SJS
maintain at 40-50 mg/kg BID
Anti-tumor necrosis factor • 0.8 mg/kg SC weekly (max dose • Polyarthritis, • Immunosuppressant, lupus-like reaction,
• Etanercept 50 mg/week) oligoarthritis, systemic demyelinating disease
Interleukin-1 inhibitor • 15-40 kg: 2 mg/kg/dose SC • Systemic • Immunosuppressant, headache, GI upset,
• Canakinumab every 8 weeks injection reaction
• For >40 kg: 150 mg SC every 8
weeks
Interleukin-6 receptor • Depends on the weight and if • IV: systemic and • Immunosuppressant, liver toxicity, GI
antagonist given IV or SC polyarthritis upset, dyslipidemia, cytopenia
• Tocilizumab • SC: polyarthritis
RECOMMENDED USE OF MEDICATIONS • Early aggressive therapy with methotrexate and etanercept may
• Indications of using DMARDs: result in earlier achievement of clinically inactive disease
o failure of methotrexate monotherapy • For sJIA: corticosteroids are started followed by IL-6 antagonist
o those with poor prognostic factors therapy to induce a rapid and dramatic response
o with severe disease onset
TOPNOTCH MEDICAL BOARD PREP PEDIATRICS MAIN HANDOUT BY DRS. DE VERA AND PUNONGBAYAN Page 74 of 106
For inquiries visit www.topnotchboardprep.com.ph or email us at topnotchmedicalboardprep@gmail.com
This handout is only valid for October 2022 PLE batch. This will be rendered obsolete for the next batch since we update our handouts regularly.
TOPNOTCH MEDICAL BOARD PREP PEDIATRICS MAIN HANDOUT BY DRS. DE VERA AND PUNONGBAYAN
For inquiries visit www.topnotchboardprep.com.ph or https://www.facebook.com/topnotchmedicalboardprep/
This handout is only valid for October 2022 PLE batch. This will be rendered obsolete for the next batch since we update our handouts regularly.
✔ GUIDE QUESTION ENTRY CRITERION
A 16-year-old female complains of fatigue, muscle and joint aches, and Antinuclear antibodies (ANA) at a titer of ≥1:80 on Hep-2 cells
fever that have lasted for 2 months. On PE, she had rashes over her or an equivalent positive test (ever)
cheeks and nose as well as friction rub on cardiac auscultation. Lab ⬇
findings: Hgb 10 g/dL, Hct 33%, platelet count of 145,00/mm3, WBC
4,300/mm3, Urinalysis 3+ proteinuria. The following is expected in her
If absent, do not classify as SLE
disease except? If present, apply additive criteria
A. Responds to steroids and other immunosuppressant ⬇
B. More common in females than males ADDITIVE CRITERIA
C. ANA has more than 90% specificity • Do not count a criterion if there is a more likely explanation
D. Anti-dsDNA reflects disease activity than SLE
ANA has 95-99% sensitivity but poor specificity to SLE; anti-Smith is • Occurrence of a criterion on a t least one occasion is sufficient
specific for the diagnosis of SLE • SLE classification requires at least one clinical criterion and
Dr. Punongbayan
≥10 points
• Criteria need not occur simultaneously
SYSTEMIC LUPUS ERYTHEMATOSUS • Within each domain, only the highest weighted criterion is
• Autoantibody production against self-antigens resulting in counter toward the total scores
inflammatory damage to target organs ⬇
• Unpredictable course of illness CLINICAL DOMAINS AND CRITERIA WEIGHT
• Decreased prevalence in whites; female : male 5:1 Constitutional
• Fibrinoid deposits found in blood vessel walls of affected organs Fever 2
• Skin, joints, kidneys, blood-forming cells, blood vessels, and Hematologic
CNS Leukopenia 3
Thrombocytopenia 4
Autoimmune hemolysis 4
Neuropsychiatric
Delirium 2
Psychosis 3
Seizure 5
Mucocutaneous
Non-scarring alopecia 2
Oral ulcers 2
Subacute cutaneous OR discoid lupus 4

IMMUNOLOGY DOMAINS AND CRITERIA WEIGHT
Antiphospholipid antibodies
Anti-cardiolipin antibodies OR Anti-β2GP1 2
antibodies OR Lupus anticoagulant
Complement proteins
Low C3 OR low C4 3
Low C3 AND C4 4
SLE-specific antibodies
Anti-dsDNA antibody OR Anti-Smith antibody 6
CLASSIFICATION CRITERIA
SUPPLEMENT: QUICK SHEET
CRITERIA SENSITIVITY SPECIFICITY CRITERIA FOR DIAGNOSIS OF SLE (1997):
American College of (SOAP BRAIN MD)
82.8% 93.4%
Rheumatology (1997) • Serositis (pleuritis, pericarditis)
Systemic Lupus • Oral ulcers (painless)
International Collaborating 96.7% 83.7%
• Arthritis (2 or more joints)
Clinics (2012)
• Photosensitivity
European League Against
96.1% 93.4% • Blood changes (anemia, leukopenia, low platelet)
Rheumatism / ACR (2019)
• Renal disorder (persistent proteinuria, cellular casts)
• ANA abnormal titer
2019 EULAR / ACR CLASSIFICATION CRITERIA FOR SLE:
• Immunological changes (anti-DNA Ab, anti-Sm)
• The entry criterion is necessary to classify SLE.
• Neurological signs (seizures, frank psychosis)
o Entry criterion: ANA at a titer of equal or >1:80 on HEp-2
• Malar rash
cells or an equivalent positive test.
• Discoid rash
• At least 1 clinical criterion required to classify SLE. Additional
additive (clinical or immunology) criteria are counted toward
the total score. AMERICAN COLLEGE OF RHEUMATOLOGY 1997
• Additive criteria: REVISED CLASSIFICATION CRITERIA FOR SYSTEMIC
o Do not count a criterion if there is a more likely explanation LUPUS ERYTHEMATOSUS
than SLE. • Malar rash
• Discoid rash
o Occurrence of a criterion in equal or >1 occasion is sufficient.
o Criteria need not occur simultaneously. • Photosensitivity
o Within each domain, only the highest-weighted criterion is • Oral or nasal ulcers
counted toward the total score if more than 1 is present. • Arthritis – Nonerosive, ≥2 joints
• SLE classification requires at least one clinical criterion and • Serositis – pleuritis, pericarditis, or peritonitis
≥10 points. • Renal manifestations
• Criteria need not occur simultaneously o Consistent renal biopsy
• Within each domain, only the highest weighted criterion is o Persistent proteinuria or renal casts
counted toward the total score • Seizure or psychosis
• Hematologic manifestations
o Hemolytic anemia
o Leukopenia (<4,000 leukocytes/mm3)
o Lymphopenia (<1,500 leukocytes/mm3)
o Thrombocytopenia (<100,000 thrombocytes/mm3)
TOPNOTCH MEDICAL BOARD PREP PEDIATRICS MAIN HANDOUT BY DRS. DE VERA AND PUNONGBAYAN Page 75 of 106
For inquiries visit www.topnotchboardprep.com.ph or email us at topnotchmedicalboardprep@gmail.com
This handout is only valid for October 2022 PLE batch. This will be rendered obsolete for the next batch since we update our handouts regularly.
TOPNOTCH MEDICAL BOARD PREP PEDIATRICS MAIN HANDOUT BY DRS. DE VERA AND PUNONGBAYAN
For inquiries visit www.topnotchboardprep.com.ph or https://www.facebook.com/topnotchmedicalboardprep/
This handout is only valid for October 2022 PLE batch. This will be rendered obsolete for the next batch since we update our handouts regularly.
• Immunologic abnormalities TREATMENT OF NON-RENAL SLE
o Positive anti-dsDNA or anti-Sm See Treatment of non-renal SLE algorithm below
o False-positive RPR, positive lupus anticoagulant test result,
or elevated anticardiolipin IgG or IgM CARDIOLOGY
• Positive ANA
*4 of the 11 establishes an SLE diagnosis CARDIAC MORPHOGENESIS
*Criteria were developed for clinical trial classification and not for • Early presomite embryo: the 1st identifiable cardiac precursors
clinical diagnosis are angiogenetic cell clusters arranged on both sides of the
Adapted from Nelson Textbook of Pediatrics, 20 ed
th
embryo’s central axis
DIAGNOSIS • Clusters form paired cardiac tubes by 18 days of gestation
• (+)ANA result is not required for diagnosis but its absence is rare • Paired tubes fuse in the midline on the ventral surface of the
• Renal biopsy to confirm lupus nephritis & to determine treatment embryo to form the primitive heart tube by 22 days
• ANA – excellent screening tool but can be (+) in other rheumatic • 20-22 days: embryonic heart begins to contract & exhibit phases
conditions; (+) in 95-99% of patients but has poor specificity of the cardiac cycle
for SLE • 22-24 days: heart tube begins to bend ventrally & toward the
• Anti-double-stranded DNA – more specific for lupus & reflects right (looping)
the degree of disease activity • 25 days: septation of the ventricles
• Low total hemolytic complement (CH50), C3, C4 in active disease • 30 days: septation of the atria
• anti-Smith Ab found only in SLE – specific for the diagnosis of SLE • 3 months: AV valve and semilunar valve formation is complete

FETAL CIRCULATION
• Oxygenated blood from placenta → 50% of umbilical
venous blood enters hepatic circulation → rest bypasses
liver & joins IVC via DV → RA → FO → LA →LV →
ascending aorta (fetal upper body and brain)
• Fetal SVC blood → RA → TV → RV → PA (only 10% of
RV outflow enters the lungs) → major portion
bypasses the lungs and flows through ductus
arteriosus → descending aorta → lower part of fetal
body → placenta via the 2 umbilical arteries
Cardiovascular structures unique to the fetus
important to maintain parallel circulation:
1. Ductus venosus: removal of the placenta from the
circulation result in its closure
2. Foramen ovale: most of the SVC blood goes to the RV;
about 1/3 of IVC blood is directed to the LA through
the FO whereas the 2/3 enters the RV and PA
3. Ductus arteriosus: less oxygenated blood in the PA
flows through the widely open DA to the descending
aorta and then to the placenta for oxygenation
• Fetal cardiac output
o The fetal heart is unable to increase stroke volume
when the HR falls because it has a low compliance.
o Thus, the fetal cardiac output depends on the HR;
when the HR drops, a serious fall in CO results.

FETAL CIRCULATION
https://qrs.ly/s9dur41

TRANSITIONAL CIRCULATION o Closure of the ductus venosus as a result of lack of blood return
from the placenta
CHANGES IN CIRCULATION AFTER BIRTH • Lung expansion results in the ff:
• The primary change after birth is a shift of blood flow for gas o Reduction of the PVR → an increase in pulmonary blood flow
exchange from the placenta to the lungs. → a fall in PA pressure
• Interruption of the umbilical cord results in the following: o Functional closure of the FO occurs due to increased pressure
o An increase in systemic vascular resistance due to removal of in the LA
the low-resistance placenta o Closure of PDA as a result of increased arterial O2 saturation
TOPNOTCH MEDICAL BOARD PREP PEDIATRICS MAIN HANDOUT BY DRS. DE VERA AND PUNONGBAYAN Page 76 of 106
For inquiries visit www.topnotchboardprep.com.ph or email us at topnotchmedicalboardprep@gmail.com
This handout is only valid for October 2022 PLE batch. This will be rendered obsolete for the next batch since we update our handouts regularly.
TOPNOTCH MEDICAL BOARD PREP PEDIATRICS MAIN HANDOUT BY DRS. DE VERA AND PUNONGBAYAN
For inquiries visit www.topnotchboardprep.com.ph or https://www.facebook.com/topnotchmedicalboardprep/
This handout is only valid for October 2022 PLE batch. This will be rendered obsolete for the next batch since we update our handouts regularly.
• With expansion of the lungs and the resulting increase in SYSTOLIC MURMURS
alveolar O2 tension: 1. EJECTION MURMUR
o there is an initial, rapid fall in the PVR (secondary to the
• interval between the S2 and onset of murmur
vasodilating effect of O2 on the pulmonary vasculature)
o Between 6-8 weeks of age, there is a slower fall in the PVR and • coincide with the ejection or flow of blood through the stenotic
PA pressure. or deformed semilunar valves or by increased flow through
o Functional closure of the DA occurs by constriction of the normal semilunar valves
medial, smooth muscle in the ductus within 10-15 hours after • audible at the 2nd LICS
birth. 2. REGURGITANT MURMUR
o Anatomic closure is completed by 2-3 weeks of age by • begins with S1
permanent changes in the endothelium and subintimal layers • caused by the flow of blood from a chamber that is at a higher
of the ductus. pressure throughout systole than the receiving chamber and
usually occur while the semilunar valves are still closed
• associated with only VSD, MR, & TR
• best heard at the left lower sternal border

DIASTOLIC MURMURS
• Occur between S2 and S1
• Early, mid-diastolic, late diastolic
• Early diastolic murmurs due to incompetence of aortic or
pulmonary valve
• ALWAYS pathologic!

TRANSITIONS OF THE
CARDIO-RESPIRATORY SYSTEM AT BIRTH
• Pulmonary vascular resistance decreases
• SVR (systemic vascular resistance) increases
o Removal of the placenta
• Closure of PDA
o Patency dependent on low O2 and high prostaglandins
§ High prostaglandins because of decreased pulmonary
circulation (metabolized in lungs) and increased production
in the placenta
• Functional Closure of PDA (15h)
o ↑ PO2, preferential blood flow from RV to lungs (↓pulmonary © Topnotch Medical board Prep

vascular resistance) Side Best


Valve Location Associated murmurs
Auscultated
• Anatomic Closure
2nd ICS,
o 2-3 weeks, fibrous proliferation of the intima Aortic Right lateral to
Ejection murmur
• Aortic stenosis
sternum
NEONATAL CIRCULATION 2nd ICS,
Ejection murmur
• A postnatal increase in O2 saturation of the systemic Pulmonic Left lateral to
• Pulmonic stenosis
sternum
circulation is the strongest stimulus for constriction of the
Pansystolic murmur
ductal smooth muscle which leads to closure of the ductus. • Tricuspid
• The responsiveness of the ductal smooth muscle to 02 is related Regurgitation
to the gestational age of the newborn; the ductal tissue of a 4th-5th ICS
• Ventricular Septal
over left
premature infant responds less intensively to oxygen than Tricuspid Left Defect
sternal
that of a full-term infant. border
Mid-to-late diastolic
murmur
Why is there decreased responsiveness of the immature ductus
• Tricuspid Stenosis
to 02? • Atrial Septal Defect
• due to its decreased sensitivity to O2-induced contraction Pansystolic murmur
• The DA is more likely to remain open in preterm infants after • Mitral Regurgitation
5th ICS MCL
birth because the premature infant’s ductal smooth muscle does Mitral Left Mid-to-late diastolic
(Apex)
not have a fully developed constrictor response to oxygen. murmur
• Mitral Stenosis
Early diastolic murmur
MURMURS Left
• Aortic Regurgitation
APPROACH TO MURMUR: -- -- sternal
• Pulmonic
border
• Timing of murmur Regurgitation
o Systolic INNOCENT PATHOLOGIC
o Diastolic
• Soft • Diastolic
o Continuous
• Systolic • Pansystolic
• Grade of murmur
• Short • Late systolic
• Associated symptoms
• Sounds "musical" • Continuous
"vibratory" "twangy • Thrill present on
GRADE QUALITY
• Symptomless examination
1 Soft, difficult to hear
• Special tests normal (x-ray, • Additional cardiac
2 Easily heard
ECG) abnormalities (e.g. clicks,
3 Louder but no thrill
• Standing / Sitting (vary abnormal splitting,
4 Associated with thrill asymmetric pulses
with position)
5 Thrill and audible with edge of stethoscope Second grade only (up to
6 Thrill and audible with stethoscope just off the chest Grade II only)
• AFTER 3, you have THRILL! • Sternal border (left or
midsternal)

TOPNOTCH MEDICAL BOARD PREP PEDIATRICS MAIN HANDOUT BY DRS. DE VERA AND PUNONGBAYAN Page 77 of 106
For inquiries visit www.topnotchboardprep.com.ph or email us at topnotchmedicalboardprep@gmail.com
This handout is only valid for October 2022 PLE batch. This will be rendered obsolete for the next batch since we update our handouts regularly.
TOPNOTCH MEDICAL BOARD PREP PEDIATRICS MAIN HANDOUT BY DRS. DE VERA AND PUNONGBAYAN
For inquiries visit www.topnotchboardprep.com.ph or https://www.facebook.com/topnotchmedicalboardprep/
This handout is only valid for October 2022 PLE batch. This will be rendered obsolete for the next batch since we update our handouts regularly.

Acyanotic Heart
MURMURS Disease Cyanotic Heart
https://qrs.ly/kodur9m Disease
(Left to Right Shunt)

ASSOCIATED VSD DECREASED PULMONARY BF:


SIDE LOCATION
MURMURS ASD Pulmonary Atresia
2nd ICS lateral to PDA Pulmonary Stenosis
Aortic R AS
sternum COA TOF
2nd ICS lateral to ECD Tricuspid Atresia
Pulmonic L PS
sternum Ebstein Anomaly
Pansystolic: TR, VSD
4th-5thICS over
Tricuspid L Mid-to-late INCREASED PULMONARY BF:
sternal border
diastolic: TS, ASD TOGA
Pansystolic: MR TAPVR
Mitral L 5th ICS MCL (apex) Mid-to-late Truncus Arteriosus
diastolic: MS
Early diastolic: AR, VENTRICULAR SEPTAL DEFECT
-- L Sternal border
PR • Enlargement of the LA, LV and main PA
• Increase in pulmonary vascular markings
QUICK SHEET APPROACH TO MURMURS • It is the LV that does volume overwork.
• All diastolic murmurs are pathologic. • Because the shunt occurs mainly during systole when the RV
• Systolic murmurs are either ejection type or regurgitant type. also contracts, the shunted blood goes directly to the PA
1. Systolic ejection or blowing murmur – heard best on the rather than remaining in the RV cavity.
base or at the 2nd ICS; blood flows through stenotic
structures thus producing a “blowing” sound
2. Systolic regurgitant murmur – heard best on the apex
or at the left lower sternal border; blood backflows from
one chamber / valve to another because of incompetent
structures
So, lahat ng systolic ejection / blowing murmur – nasa 2nd LICS
Ang systolic regurgitant murmur – nasa apex or LLSB
Functional or innocent murmurs are usually heard in young infants and
children, systolic in timing, varies with position, associated with normal
diagnostic results (CXR, ECG, 2D echo).
Dr. Punongbayan

CONGENITAL HEART DISEASE


APPROACH TO CONGENITAL HEART DISEASE
✔ GUIDE QUESTION
Baby Aaron is a full term 14-day-old infant who came in for routine
checkup. Upon auscultation you note a continuous murmur on the left
upper sternal border. Other significant PE findings are the presence of
bounding peripheral pulses and widened pulse pressure. The following
are true regarding this case EXCEPT?
A. the wall of the ductus is deficient in both mucoid endothelial
layer and the muscular media
B. There is a high chance of spontaneous closure
C. This will rarely close despite pharmacologic intervention
D. None of the above
The case posted here is that of PDA with the finding of continuous or
machinery-like murmur in the upper left sternal border or left 2nd ICS
plus bounding pulses and a wide pulse pressure.
When a term infant is found to have a PDA, the wall of the ductus is
deficient in both the mucoid endothelial layer and the muscular media,
whereas in the premature infant, the PDA usually has a normal
structure.
Thus, a PDA persisting beyond the 1st few wk of life in a term infant
rarely closes spontaneously or with pharmacologic intervention;
whereas, if early pharmacologic or surgical intervention is not SUPPLEMENT: QUICK SHEET
required in a premature infant, spontaneous closure occurs in most What happens if a large VSD is left untreated?
instances.
Dr. Punongbayan
Exposure of pulmonary artery system to high pressure and
A 3-year-old girl was brought to the clinic because of difficulty of increased flow → irreversible changes occur in the pulmonary
breathing for the past 2 days. History revealed 5 days cough and colds arterioles → progressive increase in PVR → when PVR exceeds
with undocumented fever. You learned that she has had recurrent SVR, ductal shunting reverses and becomes R->L → pulmonary
respiratory tract infections since she was an infant. She was apparently vascular obstructive disease (Eisenmenger's syndrome) → PA
diagnosed to have a “heart disease” but was lost to follow-up. On PE, is prominent with RVH and pulmonary hypertension →
she weighs 9 kgs, (+) gr 4/6 systolic regurgitant murmur heard loudest
bidirectional shunt causes cyanosis
at the left lower sternal border. Which chambers of the heart are likely
enlarged on chest radiograph? ✔ GUIDE QUESTIONS
A. Right side A 3-year-old girl was brought to your clinic for a well child visit. Upon
B. Left side auscultation, you noted a grade 3/6 systolic murmur described as
C. Both left and right “blowing” on the upper left 2nd ICS and a widely split S2. There is no
D. None of the above history of cyanosis. The other PE findings are unremarkable. 2D echo
This is a case of ventricular septal defect based on the characteristic revealed RVH and RAH with a shunt defect measuring 3mm at the site
murmur which is systolic regurgitant in character and timing and of the fossa ovalis. What is true about this disease EXCEPT?
heard best at the LLSB. The left side of the heart initially enlarges in A. Surgery should be attempted immediately
VSD because of the left to right direction of the shunt and more blood B. Spontaneous closure is 87% for lesions < 8mm
enters the pulmonary circulation leading to enlargement of the C. Large defects may lead to heart failure
pulmonary artery and the left chambers of the heart. D. LA is not enlarged
Dr. Punongbayan

TOPNOTCH MEDICAL BOARD PREP PEDIATRICS MAIN HANDOUT BY DRS. DE VERA AND PUNONGBAYAN Page 78 of 106
For inquiries visit www.topnotchboardprep.com.ph or email us at topnotchmedicalboardprep@gmail.com
This handout is only valid for October 2022 PLE batch. This will be rendered obsolete for the next batch since we update our handouts regularly.
TOPNOTCH MEDICAL BOARD PREP PEDIATRICS MAIN HANDOUT BY DRS. DE VERA AND PUNONGBAYAN
For inquiries visit www.topnotchboardprep.com.ph or https://www.facebook.com/topnotchmedicalboardprep/
This handout is only valid for October 2022 PLE batch. This will be rendered obsolete for the next batch since we update our handouts regularly.
This is a case of atrial septal defect. Main features are the ff: acyanotic, CYANOTIC CONGENITAL HEART DISEASES
systolic blowing murmur heard best on the left 2nd ICS with a widely
split S2 and right-sided enlargement. Surgery is not needed at this I. DECREASED PULMONARY BLOOD FLOW
point if the defect is 3mm or less. The most common type of ASD is • Obstruction (TV, RV or pulmonary valve level) & a pathway by
ostium secundum (50-70%) which is present at the site of fossa ovalis.
which systemic venous blood can shunt from R->L & enter the
The widely split S2 results partially from RBBB which delays both the
electrical depolarization of the RV and the ventricular contraction
systemic circulation
resulting in delayed closure of the pulmonary valve. • Tricuspid atresia
Dr. Punongbayan • Tetralogy of Fallot
What produces the widely split S2 in ASD? • Single ventricle with PS
• results partially from RBBB which delays both the electrical • Degree of cyanosis depends on the degree of obstruction to
depolarization of the RV and the ventricular contraction pulmonary blood flow
resulting in delayed closure of the pulmonary valve
TETRALOGY OF FALLOT
• Occurs in 10% of all CHDs
• Most common cyanotic heart defect beyond infancy
• 4 abnormalities: large VSD, RVOT obstruction, RVH,
overriding of the aorta
• RVOT obstruction is most frequently in the form of infundibular
stenosis (45%)

Manifestations of TOF:
• Ejection click; single S2; gr 3-5/6 systolic ejection murmur at the
mid and ULSB with radiation to the upper back (from PS)
• CXR: small heart size, decreased pulmonary vascular markings;
concave main PA with an upturned apex (couer en sabot or
CONGENITAL ACYANOTIC HEART DISEASES boot-shaped heart)
DISEASE HEART SOUNDS OTHER PE FINDINGS
• Systolic ejection
• Right sided
ASD murmur at 2nd LICS
enlargement
• widely split S2
• Left sided enlargement;
• Systolic regurgitant
• biventricular
VSD murmur at LLSB
hypertrophy if with
• loud and single S2
Eisenmenger syndrome
• Continuous • Bounding pulses;
“machinery-like” • wide pulse pressure;
PDA
murmur at the 2nd left • left-sided enlargement,
infraclavicular area • enlarged aorta
Boot-shaped heart of TOF
✔ GUIDE QUESTIONS ✔ GUIDE QUESTIONS
Which of the following is associated with the presence of an endocardial An 18-month-old male patient was brought to the clinic because of
cushion defect? intermittent cyanotic episodes more prominent in the lips, mouths,
A. Noonan syndrome fingernails, and toenails which lasts for a few minutes and goes away.
B. Marfan syndrome Sometimes the mother notices that the child assumes a squatting
C. Hunter-Hurler syndrome position. On examination there is a systolic murmur heard loudest over
D. Down syndrome the left sternal border. What is the most likely diagnosis?
Atrioventricular septal defect or ECD is associated with Down A. VSD C. TGA
syndrome. B. TOF D. TAPVR
Marfan syndrome is an inherited disorder of the connective tissue Given the age of the patient and features of cyanosis, systolic murmur
causing abnormalities in the eyes, bone, heart, and blood vessels on the left sternal border, and the relief noted upon squatting, these
(mitral valve prolapse and progressive enlargement of the aorta). are consistent with Tetralogy of Fallot, the most common cyanotic
Hunter syndrome or mucopolysaccharidosis (MPS II) – a rare genetic congenital heart disease in infants and young children.
disorder wherein glycosaminoglycans build up in body tissues; due to Dr. Punongbayan
a deficiency of iduronate-2-sulfatase causing heparan sulfate and What is the main pathophysiologic mechanism behind the
dermatan sulfate to accumulate in all body tissues → thickening of hypercyanotic spells or Tet spells in TOF?
cardiac valves resulting in improper valve closure A. due to increased systemic vascular resistance
Noonan syndrome – genetic disorder with facial anomalies, short B. due to overload and pulmonary congestion
stature, webbed neck, chest deformities, undescended testes; C. due to decreased pulmonary blood flow
pulmonary stenosis is the common cardiac defect. D. due to increased left to right shunting
Dr. Punongbayan
Short of doing a 2D echo, what is one method of distinguishing cyanotic
MECHANISM OF HYPOXIC SPELL
congenital heart disease from pulmonary disease?
A. Chest x-ray C. Hyperoxia test
B. ECG D. ABG
HYPEROXIA TEST
• 100% FiO2 (O2 hood/rebreather mask) for 10-15 mins
• Principle: in the absence of fixed cardiac shunt, 100% O2 will
increase alveolar pO2 → increase in pulmonary venous and
systemic arterial pO2

CHILD PRESENTING WITH CYANOSIS


BASIC CASE DIAGNOSIS
Cyanosis manifesting within
Transposition of great
few hours at birth or within
Arteries
few days of life
Cyanosis manifesting after the
first year of life, usually in an Tetralogy of Fallot
infant or a toddler

TOPNOTCH MEDICAL BOARD PREP PEDIATRICS MAIN HANDOUT BY DRS. DE VERA AND PUNONGBAYAN Page 79 of 106
For inquiries visit www.topnotchboardprep.com.ph or email us at topnotchmedicalboardprep@gmail.com
This handout is only valid for October 2022 PLE batch. This will be rendered obsolete for the next batch since we update our handouts regularly.
TOPNOTCH MEDICAL BOARD PREP PEDIATRICS MAIN HANDOUT BY DRS. DE VERA AND PUNONGBAYAN
For inquiries visit www.topnotchboardprep.com.ph or https://www.facebook.com/topnotchmedicalboardprep/
This handout is only valid for October 2022 PLE batch. This will be rendered obsolete for the next batch since we update our handouts regularly.
PATHOPHYSIOLOGY AND MANAGEMENT II. INCREASED PULMONARY BLOOD FLOW
• Not associated with obstruction to pulmonary blood flow
• Cyanosis due to either abnormal ventricular-arterial
connections or total mixing of systemic venous & pulmonary
venous blood within the heart
• Transposition of the great vessels
• Total anomalous pulmonary venous return
• Truncus arteriosus

TRANSPOSITION OF THE GREAT VESSELS


What is the pathophysiology of this condition?
• The aorta arises from the RV carrying desaturated blood to the
body; the PA arises posteriorly from the LV carrying oxygenated
blood to the lungs
• Result: complete separation of pulmonary & systemic
MANAGEMENT OF HYPOXIC SPELLS circulations → hypoxemic blood circulating throughout the
• Put the child in a knee-chest position (traps the systemic body & hyperoxemic blood circulating in the pulmonary
venous blood in the legs, decreasing the systemic venous return circuit
and helps calm the baby)
• Morphine sulfate suppresses the respiratory center &
abolishes hyperpnea (0.2 mg/kg/dose SC)
• NaHCO3 (1 mEq/kg) slow IV
• Oxygen may improve arterial oxygen saturation
How does squatting relieve the spell?

© Topnotch Medical Board Prep

• Defects that permit mixing of the 2 circulations (ASD, VSD,


PDA) are needed for survival
• About 50% have PFO or a small PDA
• VSD in 30-40%
• History of cyanosis from birth
• Vasoconstrictors like Phenylephrine raise SVR (0.02 mg/kg IV) • Signs of CHF with dyspnea & feeding difficulties develop during
• Propranolol may stabilize vascular reactivity of the systemic the newborn period
arteries preventing a sudden decrease in SVR (0.01-0.25 mg/kg • Most common cause of cyanotic congenital heart disease in
slow IV) newborns
• Ketamine (1-3 mg/kg IV over 60 secs) increases systemic • Moderate to severe cyanosis esp.in large, male newborns
vascular resistance • Single & loud S2; no heart murmur is heard in infants with an
intact ventricular septum
BLALOCK-TAUSSIG SHUNT • Severe arterial hypoxemia with or without acidosis (hypoxemia
• Palliative systemic-to-pulmonary artery shunt performed to unresponsive to oxygen inhalation)
augment pulmonary artery blood flow • X-ray: egg-shaped cardiac silhouette with a narrow, superior
• modified Blalock-Taussig shunt mediastinum
o Gore-Tex conduit anastomosed side to side from the subclavian
artery to the homolateral branch of the pulmonary artery
TRICUSPID ATRESIA
• Atretic (missing) tricuspid valve
• Hypoplastic right ventricle
• Ventricular septal defect
• Atrial septal defect
• Pulmonary stenosis

Egg on string sign


HEART OTHER
CONDITION PROCEDURE
SOUNDS FINDINGS
single &
Rashkind,
loud S2; no
Transposition egg-shaped Senning,
murmur if
of the great heart on X Mustard, or
with intact
arteries ray Jatene
ventricular
procedure
septum
minimal
© Topnotch Medical Board Prep single S2;
HEART OTHER cyanosis in
CONDITION PROCEDURE systolic
SOUNDS FINDINGS Truncus neonates; Rastelli
ejection
Systolic hypoplastic RV Glenn shunt; arteriosus older procedure
Tricuspid murmur at
regurgitant LAD and LVH Fontan children:
atresia LSB
murmur at LLSB in ECVG procedure heart failure
systolic ejection boot-shaped
Blalock-
systolic snowman
murmur at 2nd heart on X murmur at sign or Van Praagh
TOF Taussig TAPVR
LUSB; loud & ray (couer en LSB in mild figure-of-8 procedure
procedure
single S2 sabot) cases on X ray
TOPNOTCH MEDICAL BOARD PREP PEDIATRICS MAIN HANDOUT BY DRS. DE VERA AND PUNONGBAYAN Page 80 of 106
For inquiries visit www.topnotchboardprep.com.ph or email us at topnotchmedicalboardprep@gmail.com
This handout is only valid for October 2022 PLE batch. This will be rendered obsolete for the next batch since we update our handouts regularly.
TOPNOTCH MEDICAL BOARD PREP PEDIATRICS MAIN HANDOUT BY DRS. DE VERA AND PUNONGBAYAN
For inquiries visit www.topnotchboardprep.com.ph or https://www.facebook.com/topnotchmedicalboardprep/
This handout is only valid for October 2022 PLE batch. This will be rendered obsolete for the next batch since we update our handouts regularly.
TRUNCUS ARTERIOSUS
• Pulmonary arteries arise from aorta
• Truncal valve, occasionally quadracuspid, stenotic and/or
insufficient overrides the ventricular septal defect
• Ventricular septal defect, large

✔ GUIDE QUESTION
A 2-week-old baby is seen in the clinic for a well-baby check-up. On PE,
the baby’s femoral pulses are weak and delayed bilaterally. This
disease is highly associated with what syndrome?
A. Down Syndrome C. Klinefelter Syndrome
B. Noonan Syndrome D. Turner Syndrome
Congenital heart defects occur in up to 50% of patients with Turner
syndrome and the most common is left-sided heart defect such as
© Topnotch Medical Board Prep coarctation of the aorta which was described in the case as the baby
What is the characteristic anatomy in this condition? having weak and delayed femoral pulses.
Dr. Punongbayan
• The TA is the embryologic precursor to the aorta and pulmonary
artery → BOTH ventricles eject blood into a common vessel
• A VSD is ALWAYS present! COARCTATION OF THE AORTA
• Neural crest cells present in the TA grow in a spiral formation → • Males > females
separation of 2 outflow tracts → forms the aorta & PA • Which part of the aorta is typically affected?
• If this septum fails to form → single outflow tract persists → o The lesion is in the descending aorta, distal to the origin of
truncus arteriosus the left subclavian artery.
• Systolic ejection murmur with a thrill along the LSB o Disease course depends on the degree of obstruction, presence
• Normal S1 followed by a loud ejection click while S2 is loud and single of collateral circulation, and associated cardiac anomalies
• Apical diastolic low-pitched murmur due to increased flow
across the normal mitral valve is audible
• Tachypnea, tachycardia, excessive sweating, poor feeding begin
to appear due to increased volume load on the heart produced
by excessive blood flow through the pulmonary circulation
• Mortality by heart failure

TOTAL ANOMALOUS PULMONARY VENOUS RETURN


• All 4 pulmonary veins drain to the RA
• RV volume overload
• Total mixing of systemic venous and pulmonary venous blood
flow within the heart

© Topnotch Medical Board Prep


What is the pathophysiology?
• Pressure build-up in the proximal aorta and LV → hypertension in
the upper extremity
What are the manifestations?
• Most are asymptomatic but can have CHF if severe
• Weak, delayed, or absent femoral pulses
• Blood pressure higher in the arms > legs
• LVH may be seen in CXR or ECG

© Topnotch Medical Board Prep


How does it appear on X ray?
2 major clinical patterns depending on presence or absence
of obstruction: • Rib notching in children around
1. Neonates with severe obstruction to pulmonary venous 7 yrs old
return (most prevalent in infracardiac group) -- severe • Result of increased blood flow
cyanosis and respiratory distress through the interthoracic and
2. With mild or no obstruction to pulmonary venous return – intercostal vessels which serve as
heart failure develops as pulmonary vascular resistance collateral circulation
falls; systolic murmur at LSB
MANAGEMENT
SITE OF CONNECTION % WITH SIGNIFICANT • SBE prophylaxis
(% OF CASES) OBSTRUCTION • Anti-hypertensive Tx
Supracardiac (50%) – type I • Balloon angioplasty/stent placement in selected cases (usually
Left SVC (40) 40 recurrent CoA and adolescent/adult)
Right SVC (10) 75 • Neonates with severe CoA: PGE to reopen the ductus
Cardiac (25%) – type II • Surgical repair – treatment of choice: primary re-anastomosis
Coronary sinus (20) 10 or a patch aortoplasty (area of coarctation is enlarged with a
Right atrium (5) 5 roof of prosthesis)
Infracardiac (20%) – type III 95-100 CONDITION PE FINDINGS PROCEDURE
Mixed (5%) – type IV Weak or absent femoral
pulses;
• Without obstruction: mild cyanosis, tachypnea, feeding
BP arms > legs;
difficulties rib notching in Xray Primary re-
Coarctation
• With obstruction: rapid progression to dyspnea, weak pulses, systolic murmur at of the aorta
anastomosis or a
cardiorespiratory failure patch aortoplasty
3rd-4th LICS with
• Chest X ray: prominent PA and RV; increased pulmonary radiation to (L)
vascularity; “snowman sign” infrascapular area
TOPNOTCH MEDICAL BOARD PREP PEDIATRICS MAIN HANDOUT BY DRS. DE VERA AND PUNONGBAYAN Page 81 of 106
For inquiries visit www.topnotchboardprep.com.ph or email us at topnotchmedicalboardprep@gmail.com
This handout is only valid for October 2022 PLE batch. This will be rendered obsolete for the next batch since we update our handouts regularly.
TOPNOTCH MEDICAL BOARD PREP PEDIATRICS MAIN HANDOUT BY DRS. DE VERA AND PUNONGBAYAN
For inquiries visit www.topnotchboardprep.com.ph or https://www.facebook.com/topnotchmedicalboardprep/
This handout is only valid for October 2022 PLE batch. This will be rendered obsolete for the next batch since we update our handouts regularly.
CONDITION PE FINDINGS PROCEDURE • Newborns with HLHS will typically have lower-than-normal
Systolic ejection
Balloon valvuloplasty
oxygen saturations.
Pulmonic murmur at LUSB with o because all of the blood from the lungs mixes together in the single
valvotomy (Brock
stenosis radiation to the upper right ventricle before being pumped out of the lungs and body.
procedure)
back • Cyanosis may be the first clue to the presence of a serious
Balloon valvuloplasty underlying cardiac condition.
Aortic Systolic ejection
Ross procedure • Respiratory distress is often present because the lungs will tend
stenosis murmur at RUSB
(valve translocation)
to receive an excessively large amount of blood flow.
CXR FINDINGS DIAGNOSIS • There is often no or just a faint murmur present in newborns
Boot-shaped/Coeur en sabot Tetralogy of Fallot with HLHS.
Egg on string TGA, • Lethargy, poor feeding, and worsening respiratory distress may
Snowman TAPVR be seen as the ductus arteriosus closes. Ultimately, severe shock
Figure of 8 TAPVR resulting in seizures, renal failure, liver failure, and worsening
Rib notching Coarctation of the Aorta cardiac function may develop.
Inverted E Coarctation of the Aorta MATERNAL CONDITION FETAL HEART DEFECT
3 sign Coarctation of the Aorta Rubella PDA
DM TGA
Lupus Complete Heart Block
Persistent Pulmonary
Aspirin
Hypertension
Alcohol VSD and PS
Lithium Ebstein Anomaly
Ebstein anomaly – tricuspid valve is displaced toward the apex.
Dr. Punongbayan

✔ GUIDE QUESTIONS
A 16-year-old female develops acute onset of dyspnea characterized as
shortness of breath. She also has a high fever of 39°C together with joint
pains over the left knee and right ankle. She has been perfectly well ever
since she only recalls a skin infection with purulent discharge over her
lower back which resolved on its own. Which among the ff. is the best single
antibody titer to document cutaneous streptococcal skin infection?
A. Anti-Streptolysin O Titer
CXR: B. Anti-Deoxyribonuclease B
• small heart size, C. Streptozyme Test
D. Throat Culture And Sensitivity
• decreased pulmonary
vascular markings; • Antistreptolysin O (ASO) titer is well standardized; elevated in 80%
• concave main PA with an of patients; 333 Todd units in children and 250 Todd units in adults;
upturned apex (boot- a single low titer does not exclude RF
• Titers usually become elevated 2 weeks after strep infection, peaks
shaped heart)
at 4-6 weeks, and decreases after another 2 weeks
PHYSICAL EXAM DIAGNOSIS • A 4-fold rise in titer in 2 samples taken 10 days apart
Dr. Punongbayan
Late systolic murmur Which among these findings is considered a minor criterion?
MVP
with an opening click A. Polyarthralgia D. Subcutaneous Nodules
B. Carditis E. Polyarthritis
Disparity in pulsation & C. Chorea
BP in the arms & legs Tachycardia in rheumatic fever is significant if this is noted during:
A child is Weak popliteal, CoA A. Playing C. Crying
presented to posterior tibial, and B. Sleeping D. Feeding
you with dorsalis pedis pulses According to the Jones Criteria, which of the following is considered a
complaints of S2 widely split and fixed minor manifestation of acute rheumatic fever?
exercise in all phases of ASD A. Saint Vitus dance C. Migratory polyarthritis
intolerance, respiration B. Elevated ASO titers D. First-degree AV block
easy Loud, harsh, blowing
A 13 y/o female patient presents to the clinic with fever and joint pains.
fatigability VSD It started 3 days ago when she had fever of 38.8°C with right knee
holosystolic murmur swelling, which was warm, and very painful. At present, her right knee
Wide pulse pressure pain and swelling has resolved but now her right ankle and left knee is
Bounding peripheral swollen and painful. PE revealed BP 90/60, HR 125, RR 24, T 38.7°C,
PDA
arterial pulses (+) high pitched apical holosystolic murmur radiating to the axilla.
Continuous murmur What is the most likely dx in this case?
A. Systemic Onset JRA C. Polyarticular JRA
SURGICAL PROCEDURE INDICATION B. Pauciarticular JRA D. Rheumatic Fever
• Blalock-Taussig Shunt with Gore- This case mentioned the following main features of rheumatic fever:
Tex conduit fever, arthralgia, migratory polyarthritis, systolic regurgitant
• Aortopulmonary window shunt TOF murmur radiating to the axilla signifying mitral valve regurgitation.
• Waterson Cooley A common differential diagnosis is juvenile idiopathic arthritis as both
• Pott shunt conditions usually present with fever and joint swelling, but the
presence of a systolic murmur indicates RF instead.
• Rashkind Atrial Septostomy Dr. Punongbayan
• Jatene Arterial Switch TGA
• Senning and Mustard RHEUMATIC FEVER
• Fontan Procedure Tricuspid Atresia
QUICK SHEET JONES CRITERIA
• Norwood Procedure Hypoplastic Left JONES CRITERIA
• Glenn anastomosis Heart Syndrome • Required : evidence of recent strep infection
o ASO, Strep antibodies, Strep A culture, anti-DNase B, anti-
HYPOPLASTIC LEFT HEART SYNDROME hyaluronidase
• Major (JONES)
• all of the structures on the left side of the heart are severely o J – Joints / arthritis (poly)
underdeveloped. o O (©) – Carditis
• The right ventricle must then do a "double duty" of pumping o N – Nodules, subcutaneous
blood both to the lungs (via the pulmonary artery) and out to the o E – Erythema marginatum
body via a patent ductus arteriosus. o S – Sydenham chorea

TOPNOTCH MEDICAL BOARD PREP PEDIATRICS MAIN HANDOUT BY DRS. DE VERA AND PUNONGBAYAN Page 82 of 106
For inquiries visit www.topnotchboardprep.com.ph or email us at topnotchmedicalboardprep@gmail.com
This handout is only valid for October 2022 PLE batch. This will be rendered obsolete for the next batch since we update our handouts regularly.
TOPNOTCH MEDICAL BOARD PREP PEDIATRICS MAIN HANDOUT BY DRS. DE VERA AND PUNONGBAYAN
For inquiries visit www.topnotchboardprep.com.ph or https://www.facebook.com/topnotchmedicalboardprep/
This handout is only valid for October 2022 PLE batch. This will be rendered obsolete for the next batch since we update our handouts regularly.
QUICK SHEET JONES CRITERIA 3. Elevated acute phase reactants (CRP >3.0 mg/dL and ESR
• Minor (FRAPE) >60 mm/hr in low risk and >30 mm/hr in moderate- and high-
o F – Fever risk populations)
o R – Risk Factor (Previous RH or RHD) 4. Prolonged PR interval on the ECG
o A – Arthralgia
o P – Prolonged PR interval on ECG
o E – Elevated acute phase reactants: ESR / CRP / leukocytosis

JONES CRITERIA (1992)


MAJOR MANIFESTATIONS
1. Arthritis (70%)
• involves large joints simultaneously or in succession
• responds dramatically to salicylates

2. Carditis (50%)
• includes some or all of the following in increasing order of severity:
o Tachycardia (out of proportion to the fever)
o Heart murmur of valvulitis – MR or AR © Topnotch Medical Board Prep
o Pericarditis – friction rub, pericardial effusion, chest pain,
ECG changes
o Cardiomegaly – seen on chest X-ray
o Signs of CHF – gallop rhythm, distant heart sounds, cardiomegaly
SUPPLEMENT:
AHA Scientific Statement: Revision of Jones Criteria for the
Diagnosis of ARF in the Era of 2D Echo (March 2015): EVIDENCE OF ANTECEDENT GROUP A
• Valvulitis is the most consistent feature of ARF.
• 2D echocardiography is being used increasingly to diagnose carditis.
STREPTOCOCCAL INFECTION:
• Subclinical carditis refers exclusively to the circumstance in which 1. History of sore throat/scarlet fever unsubstantiated by
classic auscultatory findings of valvar dysfunction either are not laboratory data is not adequate evidence of recent infection.
present or are not recognized by the clinician but 2D echo reveal 2. Streptococcal antibody tests are the most reliable laboratory
mitral or aortic valvulitis. evidence. The onset of the clinical manifestations coincide with
AHA Statement Concludes the ff: (Mar.2015) the peak of the streptococcal antibody response.
• Echocardiography with Doppler should be performed in all cases of • Antistreptolysin O (ASO) titer is well standardized; elevated
confirmed and suspected ARF (Class I; Level of Evidence B).
in 80% of patients; 333 Todd units in children and 250 Todd
• Echocardiography/Doppler testing should be performed to assess
whether carditis is present in the absence of auscultatory findings, units in adults; a single low titer does not exclude RF
particularly in moderate- to high-risk populations and when ARF is *Titers usually become elevated 2 weeks after strep infection,
considered likely (Class I; Level of Evidence B). peaks at 4-6 weeks, and decreases after another 2 weeks
*a 4-fold rise in titer in 2 samples taken 10 days apart
3. Erythema marginatum (<6%)
• nonpruritic serpiginous or Any 1 of the ff can serve as evidence of preceding infection
annular erythematous (Jones criteria, March 2015):
evanescent rashes most 1. Increased or rising ASO titer or other streptococcal antibodies
prominent on the trunk and (anti-DNAse B) (Class I; Level of Evidence B). A rise in titer is better
inner proximal portions of evidence than a single titer result.
the extremities; 2. A positive throat culture for group A-beta hemolytic
• never on the face (disappear streptococci (Class I; Level of Evidence B).
on exposure to cold and 3. A positive rapid group A streptococcal carbohydrate
reappear after a hot shower or antigen test in a child whose clinical presentation suggests a
if covered with a blanket) high pretest probability of streptococcal pharyngitis (Class I; Level of
Evidence B).

4. Subcutaneous nodules (2-10%) DIAGNOSIS OF INITIAL ARF (MARCH 2015)


• particularly seen in recurrent cases; hard, painless, nonpruritic, • > 2 major manifestations, or;
freely movable swellings 0.2-2 cm in diameter • > 1 major plus 2 minor manifestations
• found symmetrically, singly or in clusters on the extensor • With evidence of previous streptococcal infection
surfaces of large & small joints, over the scalp or along the spine DIAGNOSIS OF RECURRENT RF (2015)
• last for weeks
• With a reliable past history of ARF or established RHD, and in the
face of documented GAS infection:
1. 2 major, or;
2. 1 major plus 2 minor, or;
3. 3 minor manifestations
• may be sufficient for a presumptive diagnosis (Class IIb; Level of Evidence C)
• When minor manifestations alone are present, the exclusion of
other more likely causes of the clinical presentation is
recommended before a diagnosis of an ARF recurrence can be
5. Sydenham chorea (10-15%) made (Class I; Level of Evidence C).
• occurs more often in prepubertal girls “POSSIBLE” RHEUMATIC FEVER (2015):
• choreic movements (spontaneous purposeless movements • Where there is genuine uncertainty, it is reasonable to consider
followed by motor weakness), hypotonia, emotional lability, offering 12 months of secondary prophylaxis followed by re-
hyperactivity, obsessions & compulsions evaluation to include a careful history and physical examination
in addition to a repeat echocardiogram (Class IIa; Level of
MINOR MANIFESTATIONS Evidence C).
1. Arthralgia – not considered a minor manifestation if arthritis • In a patient with recurrent symptoms (particularly involving the
is present joints) who has been adherent to prophylaxis recommendations
2. Fever – exceeds 38.5 C in low risk and >38 C in moderate- and but lacks serological evidence of group A streptococcal infection
high-risk populations and lacks echocardiographic evidence of valvulitis, it is
*In most settings, including all low-risk populations, fever associated with reasonable to conclude that the recurrent symptoms are not
ARF usually exceeds 38.5°C orally (Jones criteria March 2015). likely related to ARF, and discontinuation of antibiotic
prophylaxis may be appropriate (Class IIa; Level of Evidence C).
TOPNOTCH MEDICAL BOARD PREP PEDIATRICS MAIN HANDOUT BY DRS. DE VERA AND PUNONGBAYAN Page 83 of 106
For inquiries visit www.topnotchboardprep.com.ph or email us at topnotchmedicalboardprep@gmail.com
This handout is only valid for October 2022 PLE batch. This will be rendered obsolete for the next batch since we update our handouts regularly.
TOPNOTCH MEDICAL BOARD PREP PEDIATRICS MAIN HANDOUT BY DRS. DE VERA AND PUNONGBAYAN
For inquiries visit www.topnotchboardprep.com.ph or https://www.facebook.com/topnotchmedicalboardprep/
This handout is only valid for October 2022 PLE batch. This will be rendered obsolete for the next batch since we update our handouts regularly.
CLINICAL COURSE OF RF: ✔ GUIDE QUESTION
• Only carditis can cause permanent cardiac damage. Signs of After the acute episode of rheumatic fever, patient still with heart
mild carditis disappear rapidly in weeks but those of severe murmur but no other symptoms. The physician tells her that she needs
secondary antibiotic prophylaxis to prevent recurrence. She asks how
carditis may last for 2-6 months.
long she needs it.
• Arthritis subsides within a few days to several weeks even A. 5 years after the last attack
without treatment and does not cause permanent damage. B. Until she turns 30 y/o
• Chorea gradually subsides in 6 months or longer and usually C. 10 years or until 21 years of age
does not cause permanent neurologic sequelae. D. 10 years or until 40 years old
DIAGNOSTIC TESTS FOR RF DURATION OF PROPHYLAXIS FOR
• CBC • Chest X-ray PEOPLE WHO HAVE HAD ACUTE RHEUMATIC FEVER:
• ESR / CRP • ECG Recommendations of the American Heart Association
• throat culture • 2-D echo CATEGORY DURATION
• ASO titer Rheumatic fever without • 5 years or until 21 years of
carditis age, whichever is longer
AHA STATEMENT ON REVISION OF JONES CRITERIA
Rheumatic fever with
CONCLUDES THE FF (MARCH 2015):
carditis but without • 10 years or until 21 years of
• Echocardiography with Doppler should be performed in all residual heart disease (no age, whichever is longer
cases of confirmed and suspected ARF (Class I; Level of valvular disease*)
Evidence B). Rheumatic fever with • 10 years or until 40 years of
• Echocardiography/Doppler testing should be performed to carditis and with residual age, whichever is longer;
assess whether carditis is present in the absence of auscultatory heart disease (persistent • Sometimes lifelong
findings, particularly in moderate- to high-risk populations and valvular disease*) prophylaxis
when ARF is considered likely (Class I; Level of Evidence B). *Clinical or echocardiographic evidence
Adapted from Nelson Textbook of Pediatrics, 20th ed
ANTIBIOTIC THERAPY
SYSTOLIC EJECTION MURMURS
• Once the diagnosis of acute RF has been made and regardless of Atrial Septal
the throat culture results, the patient should receive 10 days of 2nd LICS with a widely split S2
Defect
oral Penicillin or Erythromycin or a single IM injection of
Pulmonic
benzathine Penicillin to eradicate GAS from the upper 2nd LICS with radiation to the upper back
Stenosis
respiratory tract.
Aortic Stenosis 2nd RICS with radiation to the upper back
• After this initial course of antibiotics, patient should be started
Coarctation of 3rd-4th LICS with radiation to the
on long-term antibiotic prophylaxis.
the aorta interscapular area
MANAGEMENT OF RF
• Antibiotic: to eradicate Streptococcus SYSTOLIC REGURGITANT MURMURS
o Penicillin VK - 200 – 500 mg QID x 10 days, or; Ventricular
Left lower sternal border
o Benzathine PCN - 0.6 – 1.2 MU IM, or; Septal Defect
o Erythromycin - 250 mg TID x 10 days Mitral Left lower sternal border with radiation to
• Anti-inflammatory agents: 6 – 8 weeks Regurgitation the left anterior axillary line
o ASA – 100 mg/kg/day 4-6 doses (arthritis and mild carditis) Here is my tip on how to approach a case that seem to describe a valvular
for 3-5 days then 75 mg/kg/day q 6hrs for 4 weeks heart disease: Determine if a certain valvular heart disease (MS, MR, and
o Prednisone – 2 mg/kg/day (carditis and cardiomegaly or AR), when suspected, is in failure or not in failure. Just think of the
CHF) q 6 hrs for 2-3 wks & taper location of the mitral and aortic valves (which chambers they are
situated). These conditions primarily would have DYSPNEA as the
PROPHYLAXIS FOR RF patient’s chief complaint when they are in failure. Of course, a 2D echo
• Primary prophylaxis prevents 1st episode of RF: would be the most appropriate diagnostic tool to identify the specific
o Treat streptococcal throat infection problem. But there are certain PE findings (especially the character and
• Secondary prophylaxis prevents recurrences of RF: location of the murmurs) that would help you come up with the most
o Pen VK 250 mg BID per orem, or: likely diagnosis.
Dr. Punongbayan
o Benzathine PCN 0.6-1.2 MU IM q21 days
Mitral valve involvement in ¾ of cases of valvular heart disease; ¼ of
✔ GUIDE QUESTIONS cases involve the aortic valve; tricuspid valve involvement is rare;
A 13 y/o female patient presents to the clinic with fever and joint pains. pulmonary valve involvement almost never occurs.
Pediatric Cardiology for Practitioners by Myung Park
It started 3 days ago when she had fever of 38.8°C with right knee
swelling, warm to touch, and very painful. At present, her right knee
pain and swelling has resolved but now her right ankle and left knee
are swollen and painful. PE revealed BP 90/60, HR125, RR24, T38.7°C, MITRAL VALVE STENOSIS
with apical holosystolic regurgitant murmur radiating to the axilla. • thickening of leaflets and fusion of commissures leading to
What is the most likely diagnosis in this case? calcification and immobility of the valves
A. Systemic onset JIA • LA and right-sided heart chambers are hypertrophied
B. Oligoarticular JIA
• valve is described as “fish mouth buttonhole deformity”
C. Rheumatic fever
Imagine this is the heart with its 4 chambers: (MV between the LA and
D. Valvular heart disease
LV; imagine it is stenotic or narrowed --- here are the consequences):
Right atrium Left atrium
VALVULAR • RA enlargement • LA enlargement
SYMPTOMS PE FINDINGS
HEART DISEASE • Right-sided failure • Increased LA pressure
• loud S1 at the apex and • Hepatic congestion • Pulmonary congestion
a narrowly split S2 with • Jugular vein distention • Pulmonary hypertension
MITRAL a loud P2 • Atrial fibrillation
STENOSIS • asymptomatic • narrow pulse pressure Right ventricle Left ventricle
if mild • opening snap with a • RVH • LV filling
• dyspnea with diastolic rumble • RV pressure overload
or without • systolic regurgitant • RV failure
MITRAL exertion murmur with transmission • dyspnea with or without exertion, orthopnea, PND, weak peripheral
pulses with narrowed pulse pressure
REGURGITATION • orthopnea, at the L axilla
nocturnal • S2 may widely split • loud S1 at the apex and a narrowly split S2 with a loud P2 indicating
pulmonary hypertension; mitral diastolic rumble at the apex
dyspnea, • diastolic murmur at the Dr. Punongbayan
palpitations upper and mid-sternal
AORTIC
left border with
REGURGITATION
radiation to the apex;
bounding pulses

TOPNOTCH MEDICAL BOARD PREP PEDIATRICS MAIN HANDOUT BY DRS. DE VERA AND PUNONGBAYAN Page 84 of 106
For inquiries visit www.topnotchboardprep.com.ph or email us at topnotchmedicalboardprep@gmail.com
This handout is only valid for October 2022 PLE batch. This will be rendered obsolete for the next batch since we update our handouts regularly.
TOPNOTCH MEDICAL BOARD PREP PEDIATRICS MAIN HANDOUT BY DRS. DE VERA AND PUNONGBAYAN
For inquiries visit www.topnotchboardprep.com.ph or https://www.facebook.com/topnotchmedicalboardprep/
This handout is only valid for October 2022 PLE batch. This will be rendered obsolete for the next batch since we update our handouts regularly.

MITRAL REGURGITATION
• shortened leaflets due to fibrosis; backflow of blood from LV to
LA; dilated LA and LV
• hallmark finding is a systolic regurgitant murmurs at the apex
with radiation to the left anterior axillary line
How can you differentiate the murmur from that of VSD since they both
have a systolic regurgitant type of murmur? VSD – no transmission to the
LAAL; MR – with transmission to the LAAL
Dr. Punongbayan

AORTIC REGURGITATION
• semilunar cusps are deformed and shortened; aortic valve is
dilated so that the cusps fail to appose tightly; backflow of blood
from aortic valve to LV
• hallmark PE finding is a high-pitched diastolic murmur loudest
at 3rd-4th LICS more audible when sitting and leaning forward
• other findings: diastolic thrill at 3rd LICS; hyperdynamic
precordium, bounding water hammer pulse or Corrigan pulse,
wide pulse pressure SUPPLEMENT:
2015 National Institute for Health and Care Excellence
INFECTIVE ENDOCARDITIS (NICE) Guidelines:
• Causative agents – polymicrobial 1. Antibiotic prophylaxis against IE is not recommended
o Most common: Staphylococcus aureus, viridans Streptococcus, routinely for people undergoing dental procedures
Enterococcus 2. Those at risk of developing IE: acquired valvular heart
o HACEK – Haemophilus, Actinobacillus, Cardiobacterium, disease; valve replacement; structural congenital heart
Eikenella, Kingella diseases except fully repaired ASD, PDA, VSD; hypertrophic
• Vegetations form at the site of endocardial or intimal erosion cardiomyopathy, previous IE
that result from the turbulent flow SUPPLEMENT:
• Develops in previously abnormal or damaged valves 2015 European Society of Cardiology (ESC) guidelines on
• May occur in children even without abnormal valves or cardiac high-risk procedures for which antibiotic prophylaxis is
malformations considered:
• Patients with CHD where there is turbulent blood flow due to a --- Consider for dental procedures requiring manipulation of
hole or a stenotic orifice are most susceptible to develop IE (VSD, the gingiva or periapical region of the teeth or perforation of the
AS, MVP) oral mucosa
1. Amoxicillin 2 grams orally 30-60 minutes prior to the
DUKE CRITERIA procedure
MODIFIED DUKE CRITERIA 2000: 2. If allergic to penicillin: Clindamycin 600 mg orally 30-60
DEFINITE INFECTIVE ENDOCARDITIS minutes before the procedure
• Pathologic Criteria 3. Empirical therapy: Vancomycin plus Gentamicin in
o Microorganisms demonstrated by results of cultures or patients without a prosthetic valve and when there is a high
histologic examination of a vegetation, a vegetation that has risk of S. aureus, enterococcus, and viridans Strep.
embolized, or an intracardiac abscess specimen; or 4. Native valve endocarditis due to viridans Strep and Strep.
o Pathologic lesions; vegetation, or intracardiac abscess bovis: aqueous Pen G Na (200,000 U/kg/day IV Q 4 or 6 hrs)
confirmed by results of histologic examination showing active OR Ceftriaxone (100 mg/kg/day IV od) OR Ceftriaxone plus
endocarditis Gentamicin (3 mg/kg/day IV od or Q 8 hrs)
• Clinical Criteria 5. For Oxacillin-resistant strains due to Staph: Vancomycin
o 2 major criteria, or (40 mg/kg/day IV Q 8 or 12 hrs)
o 1 major criterion and 3 minor criteria, or
o 5 minor criteria CHILD PRESENTING WITH CHF
POSSIBLE INFECTIVE ENDOCARDITIS BASIC CASE KEY CLUES DIAGNOSIS
• 1 major criterion and 1 minor criterion, or Neonate Neonate is 4 days
• 3 minor criteria presenting with old and is not
systemic cyanotic
MAJOR hypoperfusion and Grayish-blue color Hypoplastic left
1. Blood culture – viridans Strep or Strep bovis, HACEK shock with low indicative of heart disease
(Haemophilus, Actinobacillus, Cardiobacterium, Eikenella, cardiac output and cyanosis and poor
Kingella), Staphylococcus, Enterococcus weak peripheral perfusion will be
pulses. Severe apparent after 48
o 2 separate sites 12 hours apart
respiratory hours of life
o 3 or more 1 hour apart
2. Echocardiographic findings – oscillating mass vegetations, distress and Neonate is 4 days
TAPVR
regurgitant flow near a prosthesis, abscess, partial dehiscence of grunting old and is cyanotic
prosthetic valves, new wave regurgitant flow A 6-week-old baby presenting with
Coarctation of the
increasing respiratory distress,
Aorta
MINOR diaphoresis, lethargy
• Fever INFECTIVE ENDOCARDITIS
• Predisposing condition RISK FACTOR MOST LIKELY ORGANISM
• Vascular – emboli, pulmonary infarct, aneurysm, Janeway Normal person
Staphylococcus
lesions No underlying disease
• Immunologic – GN, Osler nodes, Roth Spots, RF Underlying heart disease
viridans Streptococci
• Microbiological evidence Dental procedure
• Echocardiographic findings GUT or lower bowel
Group D Streptococcus
Pertinent PE findings in Infective Endocarditis (IE) manipulation
• Osler nodes – tender, pea-sized intradermal nodules in the pads After open heart procedure Fungal
of fingers & toes Staphylococcus &
IV drug abusers
• Janeway lesions – painless small erythematous hemorrhagic Pseudomonas
lesions on the palms and soles + CVP
Coagulase-negative Staph
• Splinter hemorrhages – linear lesions beneath the nails + Prosthetic valves
TOPNOTCH MEDICAL BOARD PREP PEDIATRICS MAIN HANDOUT BY DRS. DE VERA AND PUNONGBAYAN Page 85 of 106
For inquiries visit www.topnotchboardprep.com.ph or email us at topnotchmedicalboardprep@gmail.com
This handout is only valid for October 2022 PLE batch. This will be rendered obsolete for the next batch since we update our handouts regularly.
TOPNOTCH MEDICAL BOARD PREP PEDIATRICS MAIN HANDOUT BY DRS. DE VERA AND PUNONGBAYAN
For inquiries visit www.topnotchboardprep.com.ph or https://www.facebook.com/topnotchmedicalboardprep/
This handout is only valid for October 2022 PLE batch. This will be rendered obsolete for the next batch since we update our handouts regularly.

CASE KEY CLUE ORGANISM EYE INFECTIONS


Chills, fever, • Red itchy eyes
Developing a new • Thin exudate
arthralgia, myalgia, Staph aureus Adenovirus
heart murmur • Pain & photophobia
acutely ill looking
• ± Cough & colds

CONJUNCTIVITIS
A child with Px has poor oral viridans
known CHD hygiene Streptococci • Red eyes
Staphylococcus
presents w/ fever, Px just underwent viridans • Presence of pus
fatigue, weight root canal Streptococci • Red eyes
loss, painful skin Recent repair of VSD Fungal • Pus Chlamydia
lesions on the Child is an IV drug • Inclusion bodies in scrapings
Pseudomonas
fingers user • In-turned eye lashes
• Corneal scarring Chlamydia
TREATMENT • Blindness
• Several weeks are required for a vegetation to organize and
• Stye Staphylococcus
complete therapy must be continued through this period to
avoid recrudescence • Bilateral eye lid swelling,

INFECTIONS
eosinophils, muscle pain, Hx of Trichinella

EYELID
• Total of 4-6 weeks is recommended
• Surgery for severe aortic or mitral valve involvement with infection
intractable heart failure, myocardial abscess, recurrent emboli, • Unilateral inflammation at bite site
Trypanosoma
new heart block, increasing size of vegetations while receiving around eye or mouth, Hx of travel
cruzi
therapy to Mexico or South America
Adenovirus – most common cause of viral conjunctivitis and
TREATMENT OF IE pharyngoconjunctival fever (red eye, red throat, fever)
ANTIBIOTIC Trichinella – nematode; pork worm
CLINICAL CONDITION Trypanosoma cruzi (Chagas disease or American trypanosomiasis) –
(DOSE AND FREQUENCY)
vector is kissing bug in Latin America
Patients without a
Trypanosoma brucei (African trypanosomiasis) – vector is tsetse fly;
prosthetic valve but with Vancomycin 40 mg/kg/day IV in sleeping sickness)
high risk for 2-3 equally divided doses plus Dr. Punongbayan

Staphylococcus aureus, Gentamicin 3 mg/kg/day IV OD


viridans Streptococcus, or every 8 hours SUPPLEMENT: QUICK SHEET
enterococcus • Cestodes (flatworms) – all are treated with Praziquantel
• aqueous Pen G 200,000 EXCEPT Echinococcus granulosus (Albendazole)
U/kg/day IV every 4-6 hours • All TREMATODES (flukes) are treated with Praziquantel
Native valve endocarditis for 4 weeks, or; • NEMATODES (roundworm, hookworm, whipworms)
due to viridans • Ceftriaxone 100 mg/kg/day IV • Albendazole for : Ascaris, Ancylostoma, Necator, Capillaria
Streptococcus and one dose for 4 weeks, or; • Mebendazole: Trichuris
Streptococcus bovis • Ceftriaxone plus Gentamicin 3 • Pyrantel pamoate: Enterobius
mg/kg/day IV in 1 dose or 3 • Ivermectin: Strongyloides
equally divided doses • DEC: Wuchereria and Brugia
Oxacillin 200 mg/kg/day IV • Thiabendazole: Trichinella
Endocarditis due to Staph every 4-6 hours for 6 weeks plus
without prosthetic optional Gentamicin 3
EYE INFECTIONS: ON FUNDOSCOPY
materials mg/kg/day IV every 8 hours for
3-5 days CHORIORETINITIS NEONATE
Cefazolin 100 mg/kg/day IV Retinopathy w/
For penicillin-allergic keratitis in a Mother has hx of
every 8 hours for 6 weeks plus Cytomegalovirus
patients with endocarditis newborn or a young drug abuse
optional Gentamicin 3
due to Staph without child
mg/kg/day IV every 8 hours for
prosthetic materials
3-5 days
For Oxacillin-resistant Vancomycin 40 mg/kg/day IV in ✔ GUIDE QUESTION
strains due to Staph 2-3 divided doses for 6 weeks Which drug will you give topically to eradicate the nasopharyngeal
carriage of Staphylococcus aureus in an adolescent with recurrent
carbunculosis?
INFECTIOUS DISEASES A. Mupirocin C. Bacitracin
APPROACH TO RED EYE IN NEONATES B. Neomycin D. Polymyxin

CAUSATIVE DRUG OF SKIN INFECTIONS


PND, KEY CLUES
AGENT CHOICE INFECTION KEY CLUE CAUSATIVE AGENT
red eye with purulent Neck, face, axillae,
discharge, tense edema Staphylococcus
Ceftriaxone 50 buttocks
of eyelids with marked Neisseria From neck down
mg/kg/day for Furuncle,
chemosis; incubation gonorrhoeae Has bath tub at
1 dose Carbuncle Pseudomonas
period 2-5 days; onset at home
birth or until >5 DOL (hot tub folliculitis)
Loves to play in the
mild to severe swelling of Chlamydia Erythromycin bath tub
eyelids with copious trachomatis 50 mg/kg/day Inflammation of
purulent discharge; (inclusion every 6 hours sebaceous glands Teenager Propionibacterium
incubation period 5-14 d blenorrhea) for 14 days & follicles
Days 5-18: edema and
Systemic PATIENT PRESENTING WITH CUTANEOUS LESIONS
erythema of eyelids,
Pseudomonas antibiotics; Mother brings her child
purulent discharge,
aeruginosa Gentamicin eye with multiple infected
pannus formation,
ointment wounds on the lower On PE, you see Ecthyma
endophthalmitis, sepsis
extremities. Mother multiply dry, (ulcerative
For Chlamydia – Giemsa stain epithelial cells scraped from the tarsal states that the lesions heaped up tightly form of
conjunctiva for intracytoplasmic inclusions
Dr. Punongbayan
started as mosquito adherent crusts impetigo)
bites and child kept on
scratching them.

TOPNOTCH MEDICAL BOARD PREP PEDIATRICS MAIN HANDOUT BY DRS. DE VERA AND PUNONGBAYAN Page 86 of 106
For inquiries visit www.topnotchboardprep.com.ph or email us at topnotchmedicalboardprep@gmail.com
This handout is only valid for October 2022 PLE batch. This will be rendered obsolete for the next batch since we update our handouts regularly.
TOPNOTCH MEDICAL BOARD PREP PEDIATRICS MAIN HANDOUT BY DRS. DE VERA AND PUNONGBAYAN
For inquiries visit www.topnotchboardprep.com.ph or https://www.facebook.com/topnotchmedicalboardprep/
This handout is only valid for October 2022 PLE batch. This will be rendered obsolete for the next batch since we update our handouts regularly.
PATIENT PRESENTING WITH CUTANEOUS LESIONS ✔ GUIDE QUESTIONS
Initially vesicular Ritter disease is an old name for staphylococcal scalded skin syndrome;
Honey crusted lesion exfoliatin A and B are distinct proteins that produce localized or
Catalase negative GABHS generalized skin manifestations; produce skin separation by splitting the
Mother brings a child desmosome changing the IC matrix in the stratum granulosum
No pain, no
with cutaneous lesion. Nikolsky sign – gentle stroking of the skin causes it to separate at the
systemic symptoms
Started from a bite of epidermal level
Initially vesicular
mosquito SSSS caused by group 2 phage S. aureus leading to the release of
Longer lasting and
Child kept on scratching exotoxins that causes separation of the epidermis beneath the
bigger bullae Staphylococcus
granular cell layer.
formation
Drug of choice: Oxacillin or if MRSA is suspected, start Vancomycin
Catalase positive Dr. Punongbayan

✔ GUIDE QUESTIONS
A 1-year-old male patient presents to the clinic with 2-day history of APPROACH TO RASH
fever and rash. Patient was noted to be irritable, weak, with poor • Identify the lesion….primary lesion? secondary lesion?
appetite, and fever of 38°C. One day PTC, an erythematous rash
appeared which started in the face and around the oral mucosa, axilla,
• Distribution
and groin areas. It proceeded to blistering and peeling off of the skin • Timing of appearance in relation to fever
easily with accompanying pain. Examination revealed conjunctival • Associated symptoms
erythema with fissuring and crusting around the nose and mouth with • Identify if there is absence or presence of desquamation
some superficial erosions on the lips, slightly erythematous posterior • Incubation period and period of communicability
pharynx, diffuse erythema of the skin with denuded areas over the
axillae, antecubital area, and groin. HR 110, RR 30, T 39°C. What is the
most likely diagnosis?
A. Rubeola
B. Steven Johnson Syndrome
VIRAL EXANTHEMS
C. Ritter Disease https://qrs.ly/zzdura2
D. Kawasaki
E. Scarlet Fever
VIRAL EXANTHEMS
CONDITION ETIOLOGY CHARACTERISTIC INCUBATION COMMUNICABILITY
• Coryza, Cough, Conjunctivitis
• High grade fever
• Maculopapular ashes appear at peak of fever
Measles RNA virus, 4 days before and 4
• Photophobia 8-12
(Rubeola) Paramyxoviridae days after onset of rash
• Cephalocaudal progression
• Branny desquamation when rash reaches soles
• Koplik Spot
Rubella • Low grade fever.
(German • No photophobia
Togaviridae, 7 days before 7 days
measles or • Cephalo-caudal rash. 14-21
RNA after rash
3-day • POSTERIOR AURICULAR LN
measles) • Forchheimer Spots
• Fever 3-5 days
Roseola
• High grade fever
(Exanthem
• Fussiness
Subitum, HHV 6 9-10 days unknown
• Seizures
Sixth
Disease) • RASHES APPEAR WHEN FEVER ABATES
• Nagayama spots
• Fever, malaise 1-2 days before rash 1-2 days before rash, 7
• Maculopapular, vesicular, pustular lesions present days after rash and ALL
Varicella VZV 14-16 days
simultaneously LESIONS HAVE
• Appear first on the trunk (similar to roseola) CRUSTED
Hand, foot Coxsackie Virus Respiratory tract
• Ulcers in tongue, buccal mucosa
and mouth A16 3-6 days shedding usually
• Tender ulcers on hands and feet
disease Enterovirus 71 limited to a week or less
• Slapped cheek appearance
Erythema • Lacy pattern Before onset of the rash
Infectiosum Parvovirus B19 • Spread to trunk 4-14 until after the onset of
5th disease • Spares palms and soles rash
• Complication: Aplastic Crisis

✔ GUIDE QUESTIONS ✔ GUIDE QUESTIONS


An 8-year-old male patient comes to the clinic for fever and rash. Two In relation to the case above, if the patient has a 4-year-old young
days prior, the patient had fever of 38°C, malaise, anorexia, headache, brother what can be given to prevent or modify the disease in the
and mild abdominal pain. After a day, pruritic erythematous macules younger sibling
started appearing over the face and trunk. These rashes eventually A. Immunoglobulin within 96 hours
became vesicles some of which started to erupt, then new lesions B. Live vaccine + immunoglobulin within 96 hours
started to appear over the extremities as well. Persistence of symptoms C. Live vaccine within 3-5 days
prompted consult. PE reveals some vesicles on a red base over the D. None of the above
extremities, some crusted lesions over the face and trunk. What is the Issue of POST-EXPOSURE PROPHYLAXIS for Varicella in a younger
most likely diagnosis? child: the live varicella vaccine itself can be given within 3-5 days of
A.. Varicella exposure to modify the course.
B. Steven Johnson Syndrome Passive immunization with Varicella Zoster IG (VZIG) is for
C. Ritter Disease (1) immunocompromised,
D. Kawasaki (2) pregnant, and
(3) newborns whose mother had chickenpox 5 days before up to 2 days
after delivery
Key phrase for varicella: simultaneous appearance of all stages of the lesions VZIG 125 U/10 kg IM (max. 625 units) to be given within 96 hours
Dr. Punongbayan
after exposure
Dr. Punongbayan

TOPNOTCH MEDICAL BOARD PREP PEDIATRICS MAIN HANDOUT BY DRS. DE VERA AND PUNONGBAYAN Page 87 of 106
For inquiries visit www.topnotchboardprep.com.ph or email us at topnotchmedicalboardprep@gmail.com
This handout is only valid for October 2022 PLE batch. This will be rendered obsolete for the next batch since we update our handouts regularly.
TOPNOTCH MEDICAL BOARD PREP PEDIATRICS MAIN HANDOUT BY DRS. DE VERA AND PUNONGBAYAN
For inquiries visit www.topnotchboardprep.com.ph or https://www.facebook.com/topnotchmedicalboardprep/
This handout is only valid for October 2022 PLE batch. This will be rendered obsolete for the next batch since we update our handouts regularly.

VARICELLA • Management for Sporotrichosis: Itraconazole; Amphotericin B for the severely ill
• Empiric treatment for LGV – Doxycycline 100 mg orally 2x a day for 21 days
ISSUES IN THE MANAGEMENT OF VARICELLA: • Drug of choice for Syphilis: single dose of benzathine Penicillin G 2.4
Issue #1: Post-exposure Prophylaxis million units IM
• active vaccine can be given within 5 days of exposure to • Drug of choice for W. bancrofti: single dose Diethylcarbamazine
modify course (contraindicated in patients coinfected with onchocerciasis) regardless
• Anti-VZV Ig for immunocompromised, pregnant & newborns whether symptoms or microfilaremia are present; with the addition of
exposed to maternal varicella: 125 units/10 kg IM given within Doxycycline 200 mg/day for 4-6 weeks for its macrofilaricidal activity
Dr. Punongbayan
96 hrs after exposure (max. 625 units)
***newborns whose mother had varicella 5 days before to 2 days PATIENTS PRESENTING WITH GRANULOMATOUS LESIONS
after delivery & DRAINING SINUS TRACT
Jaw area swelling
Issue #2: Is Acyclovir routinely given?
Sinus tract formation
• It is not recommended routinely for treatment of Yellow exudate
uncomplicated cases in the otherwise healthy child because of: Actinomyces
Carious teeth
1. the marginal benefit Dental procedure
2. the cost of the drug Px w/
Yellow granules in exudates
3. the low risk for complications of varicella granulomatous
Mycobacterium
• Oral Acyclovir 20 mg/kg/dose - max. 800 mg/dose – given as 4 lesions, Tropical fish enthusiasts
marinum
doses/day for 5 days draining sinus
Subcutaneous swelling of
tracts
Issue #3: Can we give Acyclovir at any point in the course of shoulder
the illness? Nocardia
Sinus Tract formations
• It is MOST effective if given within 24 hours of the onset of Granules
the rash. Usually in the cervical area Mycobacterium
When is intravenous Acyclovir indicated? Child has chronic cough tuberculosis
• For severe disease and for immunocompromised patients • M. marinum infection commonly develops as a complication of skin
• For patients with disseminated VZV: Acyclovir 500 mg/m2 every 8 and soft tissue injuries exposed to aquatic equipment such as fish lines
hrs IV initiated within 72 hrs of development of symptoms → and fishhooks. After an incubation period of 2 to 3 weeks, solitary
continued for 7 days or until no new lesions have appeared or 48 hrs erythematous to violaceous, hyperkeratotic papules form on the
elbows, hands, feet, or knees. The gold standard of treatment is
APPROACH TO PATIENTS PRESENTING Clarithromycin with Ethambutol. Rifampicin is typically added if
WITH VESICULAR LESIONS deeper organ infection is present such as osteomyelitis.
Herpes zoster • Actinomyces is a facultatively anaerobic gram-positive bacteria. It is
Preceded with neurological pain
(Shingles) commonly found in the nose and throat. Conventional therapy for
Very large bullous lesions actinomycosis is high-dose intravenous Penicillin at a dosage of 18-24
Progressing rapidly million U daily for 2-6 weeks, followed by oral Penicillin or Amoxicillin
Staphylococcal Scalded for 6-12 months.
Preceded by fever, fatigue, malaise
Skin Syndrome • Nocardiosis is an acute, subacute, or chronic infectious disease that
Toxic looking patient occurs in cutaneous, pulmonary, or disseminated forms. Nocardia are
Denuded areas obligate aerobic, partially acid fast, beaded, branching, gram-positive
Multiple vesicular lesions, some bacilli. Trimethoprim-Sulfamethoxazole is the first-line treatment but
papules, some crusted all at Varicella in patients with a sulfa allergy, Imipenem, Ceftriaxone, or Linezolid are
different ages options for first-line therapy.
Dr. Punongbayan
PATIENTS PRESENTING WITH CELLULITIS OR
OTHER CUTANEOUS LESIONS
SUBCUTANEOUS LESIONS
Pustule to dark red
Red, raised, butterfly rash Erysipelas Malignant Bacillus anthracis
fluid filled to necrosis
in appearance with derma (GAS – Strep pustule (Anthrax)
to black eschar
Inflamed pain & rapid spread pyogenes)
Target lesion Fever, headache; Borrelia burgdorferi
erythematous Following contact with Vibrio Bull’s eye Rash border; Bite site (Lyme disease)
skin, tender, saltwater or oysters vulnificus
and warm Burn px Bacillus anthracis
• gram positive rod-shaped bacteria found naturally in soil
Blue green pus Pseudomonas
1. Cutaneous – most common form and occurs in exposed areas
Grape-like odor 2. Inhalation – occurs when spores are aerosolized or as
Erysipelas appears to be like a red plaque with a well-defined margin weaponized spore preparations; cause hemorrhagic of thoracic
and has involved the upper dermis extending into superficial lymphatics; lymph nodes leading to hemorrhagic mediastinitis; rapidly
also called St. Anthony’s fire; most common cause is Streptococcus fulminant bacteremic phase
3. Gastrointestinal syndrome – occur following the consumption
Vibrio vulnificus is a gram- negative bacterium that causes serious
of undercooked infected meat from animals infected with
wound infections, septicemia, and diarrhea and the leading cause of
anthrax; necrotic ulcers and GI hemorrhage
shellfish-associated deaths in the US. From cellulitis to myositis and
DOC: bactericidal agent plus protein synthesis inhibitor: Ciprofloxacin
necrotizing fasciitis, the infection may spread rapidly especially in high-
30 mg/kg/day every 8 hrs (not to exceed 400 mg per dose PLUS
risk individuals (those with liver disease, hemochromatosis, and chronic
Clindamycin 40 mg/kg every/day every 8 hrs (not to exceed 900 mg/dose)
illness).
Borrelia burgdorferi
Presumptive diagnosis: fever, hypotension, bullous skin lesions, rapid
• Lyme disease due to a tick bite; Northeast and upper Midwest;
deterioration to shock, exposure to saltwater / shellfish / oysters
erythema migrans, arthritis, and facial palsy
Blood culture and stool culture (TCBS media)
o Erythema migrans – lesion that appears at the site of tick bite 7-14
Treatment: Doxycycline (100 mg orally 2x daily) PLUS either Cefotaxime
days after; occurs most often on the head, neck, arms, legs, back
or Ceftriaxone
Dr. Punongbayan • Clinical stages: early localized, early disseminated, late Lyme disease
• DOC: one of these 3 drugs which have equivalent efficacy: Doxycycline,
PATIENTS PRESENTING WITH LYMPHATIC OBSTRUCTION Amoxicillin, or Cefuroxime (Doxycycline has the best penetration into
OR LYMPHOCUTANEOUS LESIONS the CNS and can treat potential coinfecting agents; approved in the US
Solitary or lymphocutaneous lesions to be given to <8-year-old children if given for <21 days)
Sporothrix schenckii
Rose gardeners • Doxycycline 2.2 mg/kg twice daily (max. 100 mg/dose) for 10-14 days
(Sporotrichosis)
Likes lying in the garden Dr. Punongbayan

With hx of painful genital ulcer Chlamydia trachomatis PATIENTS PRESENTING WITH INFECTED WOUND
and inguinal and femoral (Lymphogranuloma Hx of animal bite
lymphadenopathy venereum) Pasteurella
Cellulitis with lymphadenitis
Fever, headache, elephantiasis Wuchereria bancrofti Hx of cat bite or cat scratches
of limbs or genitalia (Lymphatic filariasis) Or no hx of bite but with pet kitty
Bartonella henslae
Painless, discrete solitary Treponema pallidum Lymphadenopathy with stellate
genital ulcer (Syphilis) granulomas

TOPNOTCH MEDICAL BOARD PREP PEDIATRICS MAIN HANDOUT BY DRS. DE VERA AND PUNONGBAYAN Page 88 of 106
For inquiries visit www.topnotchboardprep.com.ph or email us at topnotchmedicalboardprep@gmail.com
This handout is only valid for October 2022 PLE batch. This will be rendered obsolete for the next batch since we update our handouts regularly.
TOPNOTCH MEDICAL BOARD PREP PEDIATRICS MAIN HANDOUT BY DRS. DE VERA AND PUNONGBAYAN
For inquiries visit www.topnotchboardprep.com.ph or https://www.facebook.com/topnotchmedicalboardprep/
This handout is only valid for October 2022 PLE batch. This will be rendered obsolete for the next batch since we update our handouts regularly.
PATIENTS PRESENTING WITH INFECTED WOUND DIPHTHERIA
Hx of shallow puncture wound
Pseudomonas HOW IS DIPHTHERIA DIFFERENT FROM STREP THROAT?
through tsinelas or rubber shoes
• Relative lack of fever
Surgical wound clean S. aureus
• Non-exudative throat
Surgical wounds dirty S. aureus
Trauma Clostridium DIAGNOSIS/TREATMENT OF DIPHTHERIA
• Culture from nose & throat obtained from beneath the
Pasteurella multocida
membrane including a portion of the membrane itself; negative
• Cats are the source of infection in 60 to 80 percent of human P.
multocida infections.
results do not rule it out
• Gram-negative nonmotile coccobacillus • Specific antitoxin – mainstay of therapy & given based on
• Commensals in the upper respiratory tracts of >90 percent of felines clinical dx; administered once at empirical dose based on the
and are the major pathogen causing infection as a result of a cat bite degree of toxicity, site & size; single empirical dose of 20,000-
or scratch 100,000 units
• Spreading edema, erythema, and tenderness at the site of bite or scratch
• First line drug: Amoxicillin/Clavulanic Acid
TREATMENT OF DIPHTHERIA:
• Why give antimicrobials?
Bartonella henselae
To halt toxin production, treat localized infection & prevent
• Etiologic agent of Cat Scratch Disease (CSD) can cause disease in both
normal and immunocompromised hosts
transmission to contacts
• 80 percent of cases occur in children with a peak incidence between • Procaine Penicillin G IM or IV (300,000 units/day for those
ages 2 and 14 years weighing 10 kgs or less and 600,000 units/day for those
• CSD – low-grade fever, papule or pustule at the site of scratch, tender weighing more than 10 kgs) for 14 days, OR;
enlarged lymph node that develop 1-3 weeks after exposure; 3-10 days • Erythromycin: eradicates nasopharyngeal carriage 40-50
incubation period mg/kg/day orally or IV every 6 hours for 14 days
• Gram-negative coccobacillus, aerobic, non-spore forming, facultatively
intracellular How do you determine treatment?
• Most cases of CSD are self-limited; may give Azithromycin • Elimination of the organism documented by at least 2
Dr. Punongbayan successive cultures from the nose & throat taken 24 hrs apart
PATIENTS WITH ENT INFECTION after completion of therapy
Red, bulging • Asymptomatic carrier: antimicrobial prophylaxis given for 7-
Acute Otitis Streptococcus.
tympanic 10 days and diphtheria toxoid
Media pneumoniae, Moraxella
membrane, fever
catarrhalis, ✔ GUIDE QUESTION
Low grade fever,
Haemophilus A 5-year-old male child presents with a 2-day history of sore throat
Sinusitis headache,
influenzae type b progressively getting worse, and fever of 38-39°C. PE revealed HR 108,
prolonged colds
RR 28, T 39°C. There is erythematous posterior pharyngeal wall,
Sneezing bilateral tonsils are enlarged and with yellow exudates, and with (+)
Common cold Scratching throat Rhinovirus tender enlarged anterior cervical lymph nodes. What is the most likely
Runny nose diagnosis?
S. pneumoniae is the most common organism causing Acute Otitis Media A. Retropharyngeal abscess
followed by Hib and Moraxella catarrhalis) B. peritonsillar abscess
Dr. Punongbayan C. diphtheria
✔ GUIDE QUESTIONS D. Epiglottitis
E. Streptococcal tonsillopharyngitis
Pharyngoconjunctival fever in children is most commonly caused by:
A. Rhinovirus C. Respiratory syncytial virus PATIENT PRESENTING WITH SORE THROAT
B. Coronavirus D. Adenovirus
Inflamed tonsils or pharynx Streptococcal
A 4-year-old female child presents with a 3-day history of
undocumented fever with sore throat. She is brought to the clinic today Purulent, Fever tonsillopharyngitis
due to fever of 38°C, hoarseness, worsening of sore throat, cough, and White papules with red base on
Herpangina
difficulty swallowing. The pharynx is erythematous with an adherent posterior palate & pharynx
gray membrane. She has also a bullneck appearance. What is the most Purulent tonsils Infectious
likely diagnosis? Fatigue, Lymphadenopathy mononucleosis
A. Diphtheria Low grade fever with gradual onset of
B. Streptococcal pharyngitis
C. viral pharyngitis
membranous nasopharyngitis Diphtheria
D. adenovirus related pharyngitis Bull neck from lymphadenopathy
In relation to the case above, the patient has a brother who recently had Finely popular, erythematous eruption
his 3rd dose of DTaP 2 weeks ago. What is your plan of management on producing a bright red discoloration of
the patient’s brother? the skin, which blanches on pressure,
A. Give a 4th dose as soon as possible more intense along the creases of the
B. No further interventions necessary Scarlet Fever
elbows, axillae, and groin. The skin has
C. Give IVIg a goose-pimple appearance & feels
D. Repeat the primary series and isolate the patient from the
brother
rough. After 4 days, the rash begins to
fade & is followed by desquamation
In this case, the patient is a 4-year-old girl whose brother had just
received the 3rd dose of DTaP vaccine 2 weeks prior. The brother then
is adequately protected and would need no further intervention at the
PATIENTS PRESENTING WITH INFECTIOUS ARTHRITIS
moment. • Pain • All ages except between
S. aureus
Dr. Punongbayan
• Redness 15-40
Diphtheria antitoxin, produced in horses, was used for treatment of • Low grade • 15-40 y/o
diphtheria in the US since the 1890s. It is not indicated for prophylaxis of Neisseria
fever • Sexually active
contacts of diphtheria patients. gonorrhoeae
• Tenderness • Promiscuous
The antitoxin will neutralize circulating (unbound) toxin and prevent joints • Prosthetic joint S. epidermidis
progression of disease. The patient must be tested for sensitivity before • Reduced • Following GIT infection Reiter’s
antitoxin is given.
ability few weeks ago syndrome
For close contacts, especially household contacts, a diphtheria booster,
appropriate for age, should be given. (Take note: single antigen Reiter’s syndrome (reactive arthritis):
diphtheria toxoid is not available → give DTaP or TDaP, whichever is Can’t see Conjunctivitis), Can’t pee (Urethritis),
appropriate) Can’t climb a tree (Arthritis)
• Develops in response to an infection and characterized by a triad of
Contacts should also receive antibiotics—benzathine Penicillin G arthritis, conjunctivitis, and non-specific urethritis
(600,000 units for persons younger than 6 years old and 1.2 million units
• Considered an autoimmune disease marked by inflammatory synovitis
for those 6 years old and older) or a 7- to 10-day course of oral
and erosion at the insertion sites of ligaments and tendons
erythromycin (40 mg/kg/day for children and 1 g/day for adults). Dr. Punongbayan
Dr. Punongbayan

TOPNOTCH MEDICAL BOARD PREP PEDIATRICS MAIN HANDOUT BY DRS. DE VERA AND PUNONGBAYAN Page 89 of 106
For inquiries visit www.topnotchboardprep.com.ph or email us at topnotchmedicalboardprep@gmail.com
This handout is only valid for October 2022 PLE batch. This will be rendered obsolete for the next batch since we update our handouts regularly.
TOPNOTCH MEDICAL BOARD PREP PEDIATRICS MAIN HANDOUT BY DRS. DE VERA AND PUNONGBAYAN
For inquiries visit www.topnotchboardprep.com.ph or https://www.facebook.com/topnotchmedicalboardprep/
This handout is only valid for October 2022 PLE batch. This will be rendered obsolete for the next batch since we update our handouts regularly.
PATIENTS PRESENTING WITH OSTEOMYELITIS ✔ GUIDE QUESTIONS
Adults, children, and In relation to the condition above, what is the causative organism?
infants w/o major A. Paramyxovirus D. HHV-6
FEVER Staphylococcus B. Coxsackie Virus E. Parvovirus B19
trauma or special
Bone pain with conditions C. Streptococcus pyogenes
swelling, Neonates, <1 mo old Staphylococcus
redness CHILD PRESENTING WITH MACULOPAPULAR RASHES
Salmonella Cough, coryza and conjunctivitis,
Associated Measles or Rubeola
Sickle cell anemia choleraesuis or and fever
cellulitis
enterica, Posterior cervical, cervical or German Measles or
Hx of trauma Pseudomonas auricular nodes Rubella
Salmonella enterica Slapped cheek looking then spread Erythema infectiosum
• rod-shaped, flagellate, facultatively aerobic, gram-negative bacteria to trunk with central clearing or 5th disease
• Salmonella osteomyelitis – rare sequelae of typhoid which may occur Abrupt onset of fever with URTI for
as a result of hematogenous seeding of the bone or joints due to a 3-4 days, fever disappears & rashes Roseola or 6th disease
bacteremic episode; symptoms may include pain in a specific bone with appears on trunk and spread
overlying redness, fever, and weakness Preceded by rapid onset of fever,
• Capillary occlusion secondary to intravascular sickling may devitalize myalgia, ocular pain, hypesthesia. Dengue Fever
and infarct the gut, permitting Salmonella invasion.
Dr. Punongbayan
Rashes blanch on pressure
PATIENTS PRESENTING WITH TOXIC LOOKING RASHES MOST CONFUSION BETWEEN RUBELLA, RUBEOLA & ROSEOLA
Generalized • TIPS: when maculopapular rash with fever presents, take note of
Neisseria the following:
Abrupt onset with
meningitidis 1. appearance of rash in relation to onset of fever
fever, chills, and 2. whether rash desquamates or not
(Meningococcemia)
shock 3. accompanying symptoms of fever and rash
Petechiae to
Fever, headache • Rubella – post-auricular lymphadenopathy + rash
purpuric
Rash that includes Rickettsia rickettsii • Rubeola/MEASLES – 3 C’s, fever disappears once rash has moved
lesions
palms and soles down to feet
Fever, headache • Roseola – Fever disappears then RASH
Rickettsia Dr. Punongbayan
Rash that spares the
prowazekii
palms and soles ✔ GUIDE QUESTIONS
Erythematous A 5-year-old male presents with fever and rashes. 6 days prior, he was
Trunk & neck and
maculo- noted to have a fever of 37.9°C, dry cough, runny nose and redness of
then extremities GABHS (Scarlet
papular rash, the eye with tearing. Then 3 days prior, the mother noted flat red spots
associated with sore fever)
Sand paper starting on the hairline, behind the ears and upper neck spreading to
throat and fever the entire face, neck, upper arms, and chest. Today, the rash spread
like
Trunk & neck and over the back, abdomen, and thighs which prompted them to seek
consult. PE: HR 110, RR 32 T 39.5°C, generalized maculopapular rash
then extremities
Diffuse with areas of confluence. What is the most likely diagnosis?
with desquamation A. Rubeola D. Kawasaki disease
erythematous S. aureus (Toxic
on palm and soles B. Rubella E. Scarlet Fever
macular Shock Syndrome)
Acute onset of fever, C. Roseola
sunburn-like
pharyngitis In relation to the case above, which among the ff interventions is said
rash
Diarrhea, to decrease mortality in the above condition?
hypotension A. Immediate antibiotic therapy
B. Ribavirin therapy
Rickettsia prowazekii – typhus or epidemic fever; C. Vitamin A supplementation
• human body louse vector; D. IV Ig
• reservoir – humans, flying squirrel;
• Central Asia and Africa, North and South America;
• poor communities and refugee populations; MEASLES
• pink macules on trunk and axillae sparing the face, palms, and soles SUPPLEMENT: QUICK SHEET
becoming purpuric MEASLES MNEMONIC
Rickettsia rickettsii – Rocky Mountain Spotted Fever; • "MEASLES COMP" (complications):
• tick vector o Myocarditis
• reservoir – rodents; o Encephalitis
• North, Central, and South America; pink macules on wrists, forearms, o Appendicitis
trunk, palms and soles becoming purpuric
o Subacute sclerosing panencephalitis
• DOC for rickettsia – Doxycycline
o Laryngitis
• Mnemonic for RMSF – R-R → Rocky mountain spotted fever, rickettsii
o Early death
TSST- toxic shock syndrome toxin
o pooh poohs (diarrhea)
• Staph aureus, Strep pyogenes;
o Corneal ulcer
• major criteria: fever 38.8 or greater, hypotension. rash
Dr. Punongbayan o Otis media
o Mesenteric lymphadenitis
✔ GUIDE QUESTIONS
o Pneumonia and related (bronchiolitis-bronchitis-croup)
An 8-year-old male presents to the clinic with a history of low-grade
fever, headache, and cough and colds for the past 3 days. He then
develops a reddish facial flushing which then spread into the trunk and What do you give for postexposure prophylaxis?
the upper arms. On close examination, the rash is erythematous • Measles Ig for prevention & attenuation within 6 days of
macular with central clearing and does not affect the palms and soles. exposure (0.25 mL/kg max. of 15 mL IM)
What is the most likely diagnosis? • Measles active vaccine can be given for susceptible children > 1
A. Rubeola D. Kawasaki Disease
year old within 72 hours
B. Rubella E. Erythema Infectiosum
C. Roseola o Infants <6 months old, pregnant & immunocompromised
A 1-year-old male came for consult for rashes. Four days prior, patient persons should receive Ig but not the active vaccine.
started having runny nose with clear nasal discharge with mild
conjunctival redness. Three days ago, patient had a fever of 39°C which ✔ GUIDE QUESTIONS
resolved on the 3rd day but a few hours after there was appearance of a A 5-year-old boy with measles lesions of five days duration was brought
rash starting on the trunk which eventually spread to the face and to the ER because of continuing high-grade fever and progressive
extremities. PE revealed small pinkish maculopapular lesions over the difficulty of breathing. Despite optimal supportive treatment, however,
trunk, face and extremities, HR: 120, RR: 33, T 37.2°C. What is the most the patient succumbed. On autopsy, one should look for:
likely diagnosis? A. Negri bodies C. Cowdry type A inclusions
A. Rubeola D. Kawasaki B. Warthin-Finkeldey cells D. Councilman bodies
B. Roseola E. Scarlet Fever

TOPNOTCH MEDICAL BOARD PREP PEDIATRICS MAIN HANDOUT BY DRS. DE VERA AND PUNONGBAYAN Page 90 of 106
For inquiries visit www.topnotchboardprep.com.ph or email us at topnotchmedicalboardprep@gmail.com
This handout is only valid for October 2022 PLE batch. This will be rendered obsolete for the next batch since we update our handouts regularly.
TOPNOTCH MEDICAL BOARD PREP PEDIATRICS MAIN HANDOUT BY DRS. DE VERA AND PUNONGBAYAN
For inquiries visit www.topnotchboardprep.com.ph or https://www.facebook.com/topnotchmedicalboardprep/
This handout is only valid for October 2022 PLE batch. This will be rendered obsolete for the next batch since we update our handouts regularly.
✔ GUIDE QUESTIONS SPOTS OF VIRAL EXANTHEMS
A 13-year-old female presents to the clinic due to generalized rash of 2 • Koplik Spot
days duration. It initially started in the face and spread rapidly up to o 2-3 days before rashes
the legs within 1 day. There was accompanying fever of 38°C, (+) joint
o grains of salt on a wet background
pains over the knees and ankles, persistence of symptoms prompted
consult. PE revealed: slightly hyperemic conjunctivae and posterior o Clustered white lesions of buccal mucosa, adjacent 2nd-3rd
pharynx, (+) generalized maculopapular rash over the whole body with lower molar
some evidence of clearing in the face, (+) tender enlarged posterior • Forchheimer Spot
auricular and cervical lymph nodes. What is the most likely diagnosis? o Petechiae on soft palate
A. Rubeola D. Kawasaki o Rubella
B. Rubella E. Scarlet Fever o 20% of cases only
C. Roseola o Not diagnostic of rubella
In relation to the condition above, what is its associated enanthem?
§ Can be seen in measles or other viral exanthem
A. Koplik spots C. Forchheimer spots
B. Nagayama spots D. Strawberry tongue • Nagayama Spot
Strawberry tongue may be seen in Kawasaki disease or in scarlet fever. o Roseola
Dr. Punongbayan o Reddish papules of soft palate
A 23-year-old pregnant woman decided to continue with her pregnancy
even when she tested positive for anti-rubella IgM on the 8th week of ✔ GUIDE QUESTIONS
her gestation. Which of the following will indicate congenital rubella if A 3-year-old male patient presents to the ER with fever 40°C and sore
the newborn was infected during pregnancy? throat. Upon inspection, there are multiple vesicles and ulcers over the
A. Anti-Rubella IgG in the mother tonsillar pillars and posterior pharyngeal wall, no rashes on the body
B. Anti-Rubella IgM in the mother or extremities. What is the most likely diagnosis?
C. Anti-Rubella IgG in the infant A. Herpes labialis
D. Anti-Rubella IgM in the infant B. Hand Foot and Mouth Disease
C. Herpangina
What to expect in rubella & what can you give? D. Streptococcal Pharyngitis
• Infection usually confers permanent immunity although re- In relation to the above case, what is the most likely causative agent?
infection may occur in 3-10% among those with a history of A. Human Herpes Virus
previous rubella B. Coxsackie A Virus
• Active vaccine can theoretically prevent illness if given within C. GABHS
72 hours of exposure D. Adenovirus
In relation to the above case, which among the ff. is the best treatment
• Use of immune globulin for post exposure prophylaxis is not for this condition?
routine but may be considered if termination of pregnancy is not A. Acyclovir
an option (0.55 mL/kg IM) B. Supportive
AGE IgM IgG interpretation C. Amoxicillin-Clavulanate
D. Ribavirin
Positive
Adult /
Positive or Recent infection
Child MUMPS
negative
Adult / Prior infection or What viral illness has these features?
Positive
Child vaccination; immune • Due to RNA virus of the family Paramyxoviridae and the genus
Recent postnatal or Rubulavirus; only 1 serotype is known
Newborn Positive --
congenital infection • Painful enlargement of the parotid glands: usual presenting sign
Mother has passed • Spread from human reservoir by direct contact, airborne
antibodies to baby droplets, fomites contaminated by saliva
Newborn -- Positive during pregnancy; this
FACTS TO REMEMBER ABOUT MUMPS
passive immunity may
• Incubation period: 16-18 days (usually 12-25 days)
last for 6-12 months
No current or prior • Period of communicability: 1-2 days before the onset of parotid
infection; not immune; swelling until 5 days after the onset of swelling
Any age Negative Negative • About 30-40% are subclinical
no or low immune
response • Initially unilateral but becomes bilateral in 70% of cases
DISTINCT FEATURES OF MUMPS
SUPPLEMENT: APPROACH TO RASH • Pain & swelling in one or both parotid glands (peaks in 1-3 days)
• Identify the lesion… primary lesion? secondary lesion? • Swollen glands push the ear lobe upward & outward & the
• Distribution angle of mandible is no longer visible
• Timing of appearance in relation to fever • Swelling subsides within 3-7 days
• Associated symptoms ALARMING MANIFESTATIONS OF MUMPS!!!
• Identify if there is absence or presence of desquamation • Edema of the homolateral pharynx & soft palate
• Incubation period and period of communicability accompanies the parotid swelling & displaces the tonsil medially
• Acute edema of the larynx
✔ GUIDE QUESTIONS • Edema over the manubrium & upper chest wall may be due
A 5-year-old male patient presents to the clinic with mild fever of 38°C, to lymphatic obstruction
sore throat and rash which started 2 days ago. On careful examination,
there were noted scattered vesicles over the buccal mucosa, palate and COMPLICATIONS OF MUMPS
posterior pharynx. There was also an erythematous maculopapular 1. Meningoencephalitis
and vesicular type rash over the hands, fingers, feet, buttocks, and groin o Most frequent;
areas. HR 90, RR 28, T 38.3°C. What is the most likely causative o enters the CNS via the choroid plexus and infect the choroidal
organism for this condition?
A. Paramyxovirus C. HHV-6
epithelium; occurs 5 days after the parotitis; M >F
B. Coxsackie A virus D. parvovirus B19 2. Orchitis & epididymitis
A 7 y/o male patient comes to the clinic with fever and rash. Two days o Occurs in 30-40% of males after puberty; extremely rare in
prior, patient had runny nose, headache, and a fever of 37.9°C. Then he prepubescent boys
started having an erythematous rash initially characterized as facial o Begins within days after onset of parotitis
flushing, which rapidly spread to the trunk and upper extremities o Infertility is rare even with bilateral orchitis
characterized as maculopapular with central clearing giving a reticular 3. Oophoritis
lacy appearance. There was also noted arthritis over the right knee and
o Pelvic pain & tenderness noted in 7% of post pubertal females;
ankle. PE: reveals characteristic rash as described, slightly
erythematous posterior pharynx, HR 98, RR 22 Temp: 37.8°C What is
fertility is unimpaired
the most likely causative agent in this case? 4. Pancreatitis
A. Paramyxovirus C. HHV-6 5. Arthritis
B. Coxsackie A virus D. Parvovirus B19 o occurs within 3 weeks of onset of parotitis; mild and self-
limiting
TOPNOTCH MEDICAL BOARD PREP PEDIATRICS MAIN HANDOUT BY DRS. DE VERA AND PUNONGBAYAN Page 91 of 106
For inquiries visit www.topnotchboardprep.com.ph or email us at topnotchmedicalboardprep@gmail.com
This handout is only valid for October 2022 PLE batch. This will be rendered obsolete for the next batch since we update our handouts regularly.
TOPNOTCH MEDICAL BOARD PREP PEDIATRICS MAIN HANDOUT BY DRS. DE VERA AND PUNONGBAYAN
For inquiries visit www.topnotchboardprep.com.ph or https://www.facebook.com/topnotchmedicalboardprep/
This handout is only valid for October 2022 PLE batch. This will be rendered obsolete for the next batch since we update our handouts regularly.
✔ GUIDE QUESTION ✔ GUIDE QUESTIONS
A 12-year-old male was brought in for fever, abdominal pain, and In relation to the above case, which test will you request for this patient
decreased urine output. Physical examination revealed jaundice, to document infection?
hepatomegaly, calf tenderness, and fever. BUN and creatinine levels are A. Dengue IgM
elevated. Upon further questioning there was a history of wading in B. Dengue IgG
flood waters 2 weeks ago. What is your drug of choice for this patient? C. Dengue NS1
A. Penicillin D. All of the above
B. Vancomycin
C. Meropenem
D. Azithromycin DENGUE FEVER
• 4 serotypes from the family Flaviviridae
LEPTOSPIROSIS • Vector: Aedes aegypti
• Incubation period: 4-6 days
PATHOGENESIS
• Virus replication → viremia → Ag-Ab complex → complement
• Leptospires enter humans through moist & abraded skin or cascade & activation of Hageman factor → ávascular
mucous membranes → circulate in the bloodstream → primary permeability → shock
lesion is damage to the endothelial lining of small blood
vessels with ischemic damage to the liver, kidneys, meninges & DENGUE FEVER PPS/DOH 2012 CLASSIFICATION
muscles
• PROBABLE DENGUE FEVER (Dengue without warning
• Incubation period: 7-12 days
signs):
o Lives in/travels to endemic areas
CLINICAL MANIFESTATIONS
o Fever and 2 of the ff criteria:
• Anicteric leptospirosis § Nausea, vomiting § Retroorbital pain
o Initial or septicemic phase is abrupt, with fever, chills, severe § Rash § Anorexia
headache, malaise, nausea, vomiting, severe muscular pain & § Headache § Diarrhea
tenderness (lower extremities) § Body malaise § Flushed skin
o Conjunctival suffusion with photophobia & orbital pain § Myalgia § (+) tourniquet test
without chemosis & purulent exudate § Arthralgia
o Hepatosplenomegaly, generalized lymphadenopathy AND
o Truncal red maculopapular rash § Lab test, at least CBC (leukopenia with or without
o Second or immune phase follows a brief asymptomatic thrombocytopenia);
interlude with recurrence of fever (biphasic) § And/or Dengue NS1 antigen test or dengue IgM antibody test
o Aseptic meningitis (optional tests)
• Icteric (Weil syndrome) • Dengue with warning signs:
o Severe form affecting <10% of children 1. Abdominal pain or tenderness
o Hemorrhage & cardiovascular collapse 2. Persistent vomiting
o RUQ pain, hepatomegaly, increased liver enzymes, 3. Clinical fluid accumulation
hyperbilirubinemia 4. Mucosal bleeding
o Azotemia → oliguria → anuria 5. Lethargy, restlessness
6. Liver enlargement >2 cms
COURSE OF LEPTOSPIROSIS: 7. Decreased or no urine within 6 hours
• initial or septicemic phase lasting 2-7 days, during which 8. LAB: increase in Hct with decrease in platelet
leptospires can be isolated from the blood, cerebrospinal fluid • Severe dengue:
(CSF), and other tissues. o Lives in or travels to an endemic area with fever of 2-7 days
• This phase may be followed by a brief period of well-being and any of the above clinical manifestations for dengue with or
before onset of a second symptomatic immune or without warning signs, plus ANY of the following:
Leptospiruric phase – associated with the appearance of 1. Severe plasma leakage
circulating IgM antibody, disappearance of organisms from the § Shock (DSS)
blood and CSF, and appearance of signs and symptoms § Fluid accumulation with respiratory distress
associated with localization of leptospires in the tissues. 2. Severe bleeding
§ As evaluated by clinician
DIAGNOSIS 3. Severe organ involvement
• Presumptive dx: symptomatic children with titers of 1:100 or § liver: AST or ALT >1,000
greater in 2 or more specimens & asymptomatic children with § CNS: impaired consciousness
evidence of exposure & seroconversion (fourfold rise in Ab § Heart and other organs
titer in specimens obtained 2 or more weeks apart)
• ELISA and dot-ELISA
• Microscopic slide-agglutination test using killed Ags – most
useful screening test
• Clinical evidence and serologic test establishes the diagnosis

TREATMENT
• Initiation of treatment before the 7th day probably shortens the
clinical course & decrease the severity of infection
• Penicillin or Tetracycline
• Parenteral Penicillin G 6-8 M U/m2/day in 6 divided doses for 1
week

✔ GUIDE QUESTIONS
A 7-year-old female was brought in for consult due to fever for 3 days,
headache, arthralgia, and myalgia. Patient was noted to have decreased
appetite, abdominal pain, and vomiting. Vital signs: HR 140 RR 30 BP
90/60. Pertinent physical examination finding shows flushed skin, with
fair pulses, and cold distal extremities. You will advise the following
EXCEPT?
A. This patient can be managed as outpatient WHO Dengue Guidelines for Diagnosis, Treatment, and Prevention Control 2009
B. Cornerstone in management is fluid hydration and CBC 2017 PPS-PIDSP Clinical Practice Guideline on Dengue Fever

monitoring
C. WBC and platelet counts are expected to be decreased
D. Hemoconcentration is expected

TOPNOTCH MEDICAL BOARD PREP PEDIATRICS MAIN HANDOUT BY DRS. DE VERA AND PUNONGBAYAN Page 92 of 106
For inquiries visit www.topnotchboardprep.com.ph or email us at topnotchmedicalboardprep@gmail.com
This handout is only valid for October 2022 PLE batch. This will be rendered obsolete for the next batch since we update our handouts regularly.
TOPNOTCH MEDICAL BOARD PREP PEDIATRICS MAIN HANDOUT BY DRS. DE VERA AND PUNONGBAYAN
For inquiries visit www.topnotchboardprep.com.ph or https://www.facebook.com/topnotchmedicalboardprep/
This handout is only valid for October 2022 PLE batch. This will be rendered obsolete for the next batch since we update our handouts regularly.
DIAGNOSTIC TESTS FOR DENGUE FEVER: o Primary infection: anti-dengue serum IgG is detectable in low
Confirmed dengue: titers at the end of 1st wk of illness → increases slowly after
• PCR → IgG detectable after several months
• Viral culture isolation o Secondary infection: IgG detected even in the acute phase &
• Detection of the virus persists from 10 months to life
• Detection of the viral nucleic acid
• Detection of antigens and antibodies
* Or a combination of these techniques

Dengue IgM and IgG:


• Primary infection: anti-dengue serum IgG is detectable in low
titers at the end of 1st wk of illness → increases slowly after →
IgG detectable after several months
• Secondary infection: IgG detected even in the acute phase &
persists from 10 months to life
• IgM is detected in 80% of patients by day 5 and 99% by day 10
• IgM peaks in 2 weeks after onset of symptoms and decline in 2-
3 months

Timing of diagnostic tests of dengue fever


• IgM detected in 80% of patients by day 5 and 99% by day 10
• IgM peaks in 2 wks after onset of symptoms & decline in 2-3
months
© Topnotch Medical Board Prep

APPROACH TO GENITAL ULCERS


SIGNS / SYMPTOMS SYPHILIS (PRIMARY) CHANCROID HERPES SIMPLEX TYPE 2
Well-demarcated indurated borders Unindurated borders and a Vesicles rupture to form shallow
Ulcers
and a clean base (chancre) purulent base ulcers
Pain Painless Painful Painful
Number of lesions Usually single Multiple Usually multiple
Inguinal Painful nodes >50%; inguinal Constitutional symptoms and
Usually mild and minimally tender
lymphadenopathy bubo nodes
• Early: chancre plus reactive
• Exclusion of other causes of
nontreponemal test and no history
ulcers in the presence of:
of syphilis
• Typical ulcers and
• 4-fold increase in quantitative • Typical lesions
lymphadenopathy
Clinical suspicion nontreponemal test with history of • Positive HSV-2 type-specific
• Typical Gram stain
syphilis serology test
• History of contact with high-risk
• (+) treponemal EIA with reactive
individual or living in an
nontreponemal test and no prior
endemic area
history of treatment
Identification of T. pallidum from a Detection of HSV by culture or
Detection of Haemophilus ducreyi
Definitive diagnosis chancre or lymph node aspirate on PCR from ulcer scraping or
by culture
dark field microscopy aspiration of vesicle fluid
DIAGNOSIS OF SYPHILIS: STATUS TREATMENT
• 2-step screening process with a nontreponemal test followed Aqueous crystalline penicillin G,
by a confirmatory treponemal test 200,000-300,000 U/kg/ day, IV,
Neurosyphilis
• Nontreponemal tests: VDRL and RPR – sensitive tests that every 4-6 hrs for 10-14 days, in
detect Abs against phospholipid Ags that cross react with doses not to exceed the adult dose
cardiolipin-lecithin-cholesterol Ags of damaged host cells
o Quantitative results useful in screening and in monitoring therapy OTHER SEXUALLY TRANSMITTED INFECTIONS:
o Usually become nonreactive within 1 year of adequate tx for CAUSATIVE
primary form and within 2 yrs for secondary form MANIFESTATIONS
AGENT
• Treponemal tests confirm diagnosis and measure specific T. • infection of the GUT mucous membranes and of
pallidum Abs the mucosa of the rectum, oropharynx, and
o T. pallidum hemagglutination assay (TPHA), particle conjunctiva
agglutination test, fluorescent treponemal Ab absorption test • postpubertal males: urethritis and epididymitis
o Do not correlate with disease activity; become (+) soon after • postpubertal females: endometritis,
initial infection Neisseria salpingitis, and peritonitis (PID)
gonorrhea • asymptomatic: about 80% of mature females
TREATMENT OF SYPHILIS: (non-motile, are asymptomatic; 10% in males
STATUS TREATMENT aerobic, gram- • localized: incubation period of 2-5 days in
negative men and 5-10 days in women
Aqueous crystalline penicillin G,
diplococcus) o urethritis: purulent discharge and dysuria
200,000-300,000 U/kg/ day, IV,
Congenital syphilis without urgency or frequency
administered as 50,000 U/kg,
o vulvovaginitis: purulent discharge with swollen,
every 4-6 hrs for 10 days
tender, and excoriated vulva; dysuria
Primary, secondary, Penicillin G benzathine, 50,000 • disseminated: asymmetric arthralgia,
and early latent syphilis U/kg, IM, up to the adult dose of 2.4 petechiae, tenosynovitis, suppurative arthritis
(ELI - acquired within the
million U in a single dose
preceding year) TREATMENT FOR GONORRHEA
Penicillin G benzathine, 50,000
• all patients with presumed or proven gonorrhea should be
U/kg, IM, up to the adult dose of 2.4
evaluated for concurrent syphilis, HIV, and C. trachomatis
Late latent syphilis
million U, administered as 3 single
infection (15-25% coinfection in males and 35-50% in females)
(beyond 1-year
doses at 1-wk interval (total
• 45 kgs or less: Ceftriaxone 50 mg/kg IV or IM once for
duration)
150,000 U/kg, up to the adult dose
pharyngeal, anorectal, urogenital infection, or conjunctivitis;
of 7.2 million U) DGI or septic arthritis for 7 days; meningitis 50 mg/kg IV or IM
every 12-24 hrs 10-14 days
TOPNOTCH MEDICAL BOARD PREP PEDIATRICS MAIN HANDOUT BY DRS. DE VERA AND PUNONGBAYAN Page 93 of 106
For inquiries visit www.topnotchboardprep.com.ph or email us at topnotchmedicalboardprep@gmail.com
This handout is only valid for October 2022 PLE batch. This will be rendered obsolete for the next batch since we update our handouts regularly.
TOPNOTCH MEDICAL BOARD PREP PEDIATRICS MAIN HANDOUT BY DRS. DE VERA AND PUNONGBAYAN
For inquiries visit www.topnotchboardprep.com.ph or https://www.facebook.com/topnotchmedicalboardprep/
This handout is only valid for October 2022 PLE batch. This will be rendered obsolete for the next batch since we update our handouts regularly.
• more than 45 kgs: Ceftriaxone 250 mg IM plus Azithromycin o 3 options for recurrent infections:
1 g PO once for pharyngeal, anorectal, and urogenital infection; § Option 1: no therapy
Ceftriaxone 1 g IM plus Azithromycin 1 g PO once for § Option 2: episodic tx with acyclovir 800 mg tid PO for 2 days
conjunctivitis or valacyclovir 500 mg bid PO for 3 days
o for DGI and septic arthritis: Ceftriaxone 1g IV or IM OD for 7 § Option 3: long-term suppressive tx with acyclovir 400 mg bid
days plus Azithromycin 1 g PO once PO; prevents outbreaks and improves quality of life
o meningitis: Ceftriaxone 1-2 g IV every 12-24 hrs 10-14 days • CNS infection: IV acyclovir 10 mg/kg every 8 hours given as 1
plus Azithromycin 1 g PO once hr infusion for 14-21 days

TREATMENT FOR CHANCROID SUPPLEMENT: SPIDERMAN DROPLET


• Most are resistant to penicillin and ampicillin due to plasmid- * Separate 3 feet between patients
mediated B-lactamase production * Wear regular mask / close curtain or varifold
• Easy to treat if recognized early • S – scarlet fever
• Azithromycin 1 g PO single dose OR Ceftriaxone 250 mg IM • P - Parvovirus B19 / Pneumonia / Pertussis
single dose • I – Influenza
• Alternative: erythromycin 500 mg PO tid 7 days and • D – Diphtheria
ciprofloxacin 500 mg bid 3 days for 18 years old and older • E – Epiglottitis
• Symptoms usually resolve in 3-7 days • R – Rubella
• Evaluate for other STIs as well as the sexual contacts • M – Mumps / Meningitis / Mycoplasma infection
• AN- Adenovirus
TREATMENT OF HERPES SIMPLEX INFECTION
DRUG Route of administration Bioavailability SUPPLEMENT: MRS. WEE CONTACT
Oral (the only one with Poorest; requires *private room
Acyclovir suspension form); more frequent *wear procedural gown and gloves
topical; IV form dosing • M – Multi-drug resistant organisms (MRSA)
Very good; once • R – Respiratory infections (RSV)
Valacyclovir oral • S- Skin infections (scabies)
or bid dosing
• W – Wound infection (open wounds)
Very good; once
Famciclovir oral • E – Eye infection (conjunctivitis)
or bid dosing
• early initiation of therapy results in maximal therapeutic
benefits SUPPLEMENT: METV AIRBORNE
• all 3 drugs have exceptional safety profiles *negative pressure room
*wear N95 mask
• doses should be modified in patients with renal impairment
• M – Measles
• Foscarnet and cidofovir used for acyclovir-resistant mutants
• E – emerging and re-emerging infections (SARS, MERS CoV, Ebola)
• First episode genital herpes reduces severity and duration of illness • T – Tuberculosis
o Acyclovir 400 mg tid PO for 7-10 days (for smaller children • V – Varicella
10-20 mg/kg/dose 4x a day not to exceed the adult dose),
Valacyclovir 1 g bid PO for 7-10 days, or Famciclovir 250 mg
tid PO for 7-10 days
OTHER SEXUALLY TRANSMITTED INFECTIONS:
CAUSATIVE AGENT MANIFESTATIONS TREATMENT
• major cause of epididymitis and a cause of • males and nonpregnant females: Azithromycin (1 g PO
23-55% of nongonococcal urethritis single dose) and Doxycycline (100 mg PO bid for 7
• sexually abused children usually asymptomatic days)
• perinatally acquired rectal and vaginal • pregnant: Azithromycin 1 g PO OR Amoxicillin 500 mg
Chlamydia trachomatis tid 7 days
infections can persist for 3 years or longer
(gram negative obligate
• urethritis, epididymitis, cervicitis, • treat sex partners of patients with nongonococcal
intracellular pathogen): 2
salpingitis, proctitis, PID urethritis if they had contact for the last 60 days
biovars LGV and trachoma
• autoinoculation from the genital tract to eye
• less acute symptoms with mucoid discharge
• diagnosis by NAAT like PCR (cervical,
vaginal, or urethral swab)
• moist, fleshy, papillomatous lesions on the • genital warts are benign and usually remit
perianal mucosa, labia, vaginal introitus, on • weekly application of 25% podophyllin in tincture of
the shaft, corona, and glans of the penis benzoin left on the warts for 4-6 hours
Human papilloma virus
• squamous intraepithelial lesions (cervical) • topical podofilox (0.5% solution or gel bid for 3 days
(genital; types 6 and 11) –
• occur after inoculation during birth, result followed by 4 days of no treatment repeated for up to 4
condyloma acuminata
of sexual abuse, or from incidental spread cycles) or imiquimod 5% cream applied at bedtime
from cutaneous warts 3x/week up to 16 wks
• laser surgery, surgical excision

COVID-19
INTERIM GUIDELINES ON THE SCREENING, CLASSIFICATION, AND MANAGEMENT
OF PEDIATRIC PATIENTS WITH SUSPECTED OR CONFIRMED CORONAVIRUS 19 DISEASE
VERSION 5, JANUARY 5, 2022
PHILIPPINE PEDIATRIC SOCIETY AND PEDIATRIC INFECTIOUS DISEASE SOCIETY OF THE PHILIPPINES

SEVERITY DEFINITION
Mild No Symptomatic patients meeting the case definition for COVID-19 without evidence of viral pneumonia or
disease pneumonia hypoxia
Child with clinical signs of non-severe pneumonia (cough or DOB + fast breathing and/or chest indrawing)
and no signs of severe pneumonia, including SpO2 ≥95% on room air
Tachypnea (breaths/min)
Moderate • 3 mos-12 mos old: ≥ 50 breaths/min
disease Pneumonia • 1-5 yrs old: ≥ 40 breaths/min
• 5-12 yrs old: ≥ 30 breaths/min
• >12 yrs old: ≥ 20 breaths/min
While the dx can be made clinically, chest imaging (radiograph, ultrasound, CT scan) may assist in diagnosis and identify /
exclude pulmonary complications

TOPNOTCH MEDICAL BOARD PREP PEDIATRICS MAIN HANDOUT BY DRS. DE VERA AND PUNONGBAYAN Page 94 of 106
For inquiries visit www.topnotchboardprep.com.ph or email us at topnotchmedicalboardprep@gmail.com
This handout is only valid for October 2022 PLE batch. This will be rendered obsolete for the next batch since we update our handouts regularly.
TOPNOTCH MEDICAL BOARD PREP PEDIATRICS MAIN HANDOUT BY DRS. DE VERA AND PUNONGBAYAN
For inquiries visit www.topnotchboardprep.com.ph or https://www.facebook.com/topnotchmedicalboardprep/
This handout is only valid for October 2022 PLE batch. This will be rendered obsolete for the next batch since we update our handouts regularly.
SEVERITY DEFINITION
Child with clinical signs of pneumonia (cough or DOB) + at least 1 of the ff:
Severe Severe • Central cyanosis of SpO2 <95%; severe respiratory distress (e.g.. fast breathing, grunting, very severe chest
disease pneumonia indrawing); general danger sign: inability to breastfeed or drink, lethargy or unconsciousness, or convulsions
• Tachypnea (see above rates per age)
• onset: within 1 week of a known clinical insult or new or worsening respiratory symptoms
• chest imaging: bilateral opacities, not fully explained by volume overload, lobar or lung collapse, or nodules
• origin of pulmonary infiltrates: respiratory failure not fully explained by cardiac failure or fluid overload; need
Acute
objective assessment like ECG to exclude hydrostatic cause of infiltrates / edema if no risk factor is present
respiratory
• oxygenation impairment in adolescents / adults:
distress
o Mild ARDS: 200 mmHg <PaO2/FiO2 ≤ 300 mmHg (with PEEP or CPAP ≥ 5 cmH2O)
syndrome
o Moderate ARDS: 100 mmHg <PaO2/FiO2 ≤ 200 mmHg (with PPEP ≥ 5 cmH2O)
(ARDS)
o Severe ARDS: PaO2/FiO2 ≤ 100 mmHg (with PEEP ≥ 5 cmH2O)
• oxygenation impairment in children: note OI and OSI, use OI when available. If PaO2 not available, wean
FiO2 to maintain SpO2 ≤ 97% to calculate OSI or SpO2/FiO2 ratio
• preliminary case definition: children and adolescents >3 days of fever AND 2 of the ff:
o Rash or bilateral non-purulent conjunctivitis
o Mucocutaneous inflammation signs
o Hypotension or shock
o Features of myocardial dysfunction, pericarditis, valvulitis, or coronary abnormalities (including echo findings
• preliminary case definition: children and adolescents >3 days of fever AND 2 of the ff:
o Rash or bilateral non-purulent conjunctivitis
o Mucocutaneous inflammation signs
o Hypotension or shock
MIS-C o Features of myocardial dysfunction, pericarditis, valvulitis, or coronary abnormalities (including echo
findings or elevated troponin/NT-proBNP)
Critical
o Evidence of coagulopathy (PT, PTT, D-dimer)
disease
o Acute GI problems (diarrhea, vomiting, abdominal pain)
o AND elevated markers of inflammation (ESR, CRP, procalcitonin)
• preliminary case definition: children and adolescents >3 days of fever AND 2 of the ff:
o AND no other obvious microbial cause of inflammation including bacterial sepsis, SSS or streptococcal
shock syndrome
o AND evidence of COVID-19 (RT-PCR, antigen test, or serology +)
o Or likely contact with patients with COVID-19
• Adolescents: persistent hypotension despite volume resuscitation, requiring vasopressors to maintain MAP
≥ 65 mmHg and serum lactate level >2 mmol/L
• Children: any hypotension (SBP <5th centile or >2 SD below normal for age) or 2 or 3 of the ff:
o altered mental status
o bradycardia or tachycardia (HR <90 bpm or >160 bpm in infants and HR <70 bpm and >150 bpm in
Septic children
Shock o prolonged CRT >2 secs or weak pulse
o Fast breathing
o Mottled or cool skin or petechial or purpuric rash
o High lactate
o Reduced urine output
o Hyperthermia or hypothermia
• Children: suspected or proven infection ≥ 2 age-based systemic inflammatory response syndrome (SIRS) criteria, of
Sepsis
which one must be abnormal temperature or white blood cell count
DIAGNOSTIC TESTING PHARMACOLOGIC INTERVENTIONS FOR CHILDREN:
SARS-CoV 2 TESTING • Corticosteroids (dexamethasone for severe and critical cases)
• Molecular-based assay (NAAT using RT-PCR) – done at or • Tocilizumab (plus systemic steroids; for patients showing rapid
shortly after the onset of illness for symptomatic patients or at least deterioration and/or requiring high doses of 02 and with
5-7 days after exposure for presumed asymptomatic close contacts elevated inflammatory markers)
• Antigen tests detect the presence of viral proteins expressed by • Experimental therapy for mild to moderate Covid-19 in children:
the virus; recommended for use under ALL these conditions in o Bamlanivimab + Etesivimab
patients suspected of Covid-19 infection: o Casirivimab + Imdevimab
o Symptomatic • Experimental therapy for severe and critical Covid-19 in children:
o At the early phase (</= 7 days) o Remdesivir (hospitalized and requires supplemental 02)
o Using specific brands that showed sensitivity >/= 80% and o Baricitinib in combination with Remdesivir (hospitalized in
specificity >/= 97-100% whom steroids cannot be used, aged 2 years old or more)
• Antibody serologic tests (CLIA, LFIA, ELISA) are utilized in o Convalescent plasma (clinical trial)
the ff situations: o Prophylactic anti-coagulation (in consultation with hematology)
o Provide evidence of previous infection with SARS-CoV 2
o As part of work up for MIS-C ACQUIRED IMMUNODEFICIENCY SYNDROME
o For studies of population seroprevalence to understand the
epidemiology of SAR-CoV 2 • Human immunodeficiency virus from the Retroviridae family
o **Antibody testing is NOT recommended: • Transmission of HIV occurs via sexual contact, parenteral
§ to determine immunity to Covid-19 following vaccination, exposure to blood, or vertical transmission from mother to child
§ to assess the need for vaccination • Vertical transmission can occur before (intrauterine), during
• ANCILLARY LABORATORY TESTS (intrapartum), or after (through breastfeeding) delivery
o CBC PATHOGENESIS OF AIDS
o Inflammatory markers (CRP, ESR, procalcitonin, IL-6, LDH, CK, • HIV in circulation → intense viremia → widespread seeding of
cardiac troponin I, D-dimer) virus to various organs → HIV selectively binds to cells
o Imaging studies (chest X ray, chest CT scan, chest ultrasound) expressing CD4+ molecules & cells of monocyte-macrophage
lineage → CD4+ cells migrate to lymph nodes
• CD8+ cells/suppressor cells play an important role in containing
the infection
TOPNOTCH MEDICAL BOARD PREP PEDIATRICS MAIN HANDOUT BY DRS. DE VERA AND PUNONGBAYAN Page 95 of 106
For inquiries visit www.topnotchboardprep.com.ph or email us at topnotchmedicalboardprep@gmail.com
This handout is only valid for October 2022 PLE batch. This will be rendered obsolete for the next batch since we update our handouts regularly.
TOPNOTCH MEDICAL BOARD PREP PEDIATRICS MAIN HANDOUT BY DRS. DE VERA AND PUNONGBAYAN
For inquiries visit www.topnotchboardprep.com.ph or https://www.facebook.com/topnotchmedicalboardprep/
This handout is only valid for October 2022 PLE batch. This will be rendered obsolete for the next batch since we update our handouts regularly.
3 DISTINCT PATTERNS OF AIDS IN CHILDREN • NRTIs: Didanosine, Abacavir, Stavudine, Lamivudine,
1. HIV-infected newborns 15-25% - rapid course; median Zidovudine
survival of 6-9 months if untreated • NNRTIs: Delavirdine, Efavirenz, Nevirapine
2. Perinatally infected newborns 60-80% - much slower • Protease inhibitors are potent agents that prevent packaging
progression with median survival time of 6 yrs. of infectious virions before they leave the infected cells;
3. Long-term survivors <5% of perinatally infected children – Indinavir, Amprenavir, Nelfinavir, Ritonavir, Saquinavir
minimal or no progression of disease with normal CD4+ • HIV-infected children with symptoms (clinical category A, B, C)
counts & very low viral loads for >8 yrs or with evidence of immune dysfunction (immune category 2 or
3) should be treated with antiretrovirals regardless of age or
CLINICAL MANIFESTATIONS viral load.
• Generalized lymphadenopathy • In general, the best single prognostic indicator is the plasma
• Hepatosplenomegaly viral load.
• Failure to thrive
• Chronic or recurrent diarrhea PLEASE READ MORE ON:
• Interstitial pneumonia • Polio
• Oral thrush • EBV/Infectious Mononucleosis
• Recurrent bacterial infections • Cytomegalovirus
• Lymphocytic interstitial pneumonitis (LIP) • Influenza
• Early onset of progressive neurologic deterioration
ENDOCRINOLOGY
HIV CLASSIFICATION SYSTEM TO CATEGORIZE
TIPS:
THE STAGE OF PEDIATRIC DISEASE:
CLINICAL STATUS • Are you dealing with deficiency or excess of hormone?
• Category A (mild symptoms) • Deficiency: challenge test
o At least 2 mild symptoms: • Excess hormone: deprivation test
1. Lymphadenopathy 4. Dermatitis • Treatment is straightforward
2. Parotitis 5. persistent or recurrent • Achieving balance and homeostasis
3. Hepatosplenomegaly sinusitis or otitis media
• Category B (moderate symptoms)
o Any of the ff:
1. oropharyngeal thrush persisting for >2 months
2. recurrent or chronic diarrhea
3. persistent fever for >1 month
4. hepatitis
5. recurrent herpetic gingivostomatitis
6. pneumonitis
7. disseminated varicella with visceral involvement
8. cardiomegaly
• Category C (severe symptoms):
o 2 serious bacterial infections:
1. Sepsis
2. Meningitis
3. pneumonia in a 2-year period
4. lower RT candidiasis
5. Cryptococcosis
6. Encephalopathy
7. Malignancies
8. disseminated mycobacterial infection
9. Pneumocystis carinii pneumonia © Topnotch Medical Board Prep
✔ GUIDE QUESTION
10. severe weight loss
A 15-year-old female was being examined in the clinic as part of her
wellness checkup. Her breast areolae and papilla were found to form
DIAGNOSIS secondary mounds and pubic hair was coarse and curly. What is the
• Any child >18 months: demonstration of IgG Ab to HIV by a most likely SMR stage of this patient?
repeatedly reactive enzyme immunoassay & confirmatory test A. 1
(Western immunoblot or IF assay) establishes the dx of HIV B. 2
infection C. 3
D. 4
• Some viral detection assays: HIV DNA or RNA by PCR, HIV E. 5
culture, HIV p24 Ag, immune complex-dissociated p24 Ag
Primary mound of breast – SMR 2 – thelarche
Secondary mound of breast – SMR 4
TIMING OF DIAGNOSTIC TESTS Dr. Punongbayan
• For infants born to HIV-infected mothers: viral diagnostic
testing should be done within the 1st 2 days of life, at 1-2 months SMR LANDMARKS
old & at 4-6 months old STAGE BOY GIRL
• CD4+ & CD8+ lymphocyte counts done at 1 & 3 months old & 1 Prepubertal Prepubertal
repeated every 3 months starting at 6 months of age 2 Testes enlarge Thelarche
Penis enlarge, pubic Enlarged breasts and
TREATMENT 3
hair begins areola
• Basis of initiating antiretroviral treatment for pediatric HIV- Curly hair and papilla
infected patients are based on: 4 Curly pubic hair
form secondary mound
1. the magnitude of viral replication Adult testes, pubic
2. CD4+ count & percentage Adult triangle. Medial
5 hair to medial surface
3. clinical condition surface of thigh
of thigh
• Antiretrovirals able to inhibit the HIV reverse transcriptase
or protease enzymes
• Reverse transcriptase inhibitors: nucleoside or non-
nucleoside RTIs

TOPNOTCH MEDICAL BOARD PREP PEDIATRICS MAIN HANDOUT BY DRS. DE VERA AND PUNONGBAYAN Page 96 of 106
For inquiries visit www.topnotchboardprep.com.ph or email us at topnotchmedicalboardprep@gmail.com
This handout is only valid for October 2022 PLE batch. This will be rendered obsolete for the next batch since we update our handouts regularly.
TOPNOTCH MEDICAL BOARD PREP PEDIATRICS MAIN HANDOUT BY DRS. DE VERA AND PUNONGBAYAN
For inquiries visit www.topnotchboardprep.com.ph or https://www.facebook.com/topnotchmedicalboardprep/
This handout is only valid for October 2022 PLE batch. This will be rendered obsolete for the next batch since we update our handouts regularly.
STAGING OF BREAST DEVELOPMENT SEXUAL DEVELOPMENT
• GIRLS: thelarche (10-11 years) → pubarche → growth spurt →
menarche (average age 12.5 years old, 9-15 years)
• BOYS: gonadarche (growth of testes ≥4 mL volume or 2.5cm
longest diameter, 11-12 years old) → pubarche → adrenarche →
growth spurt
Menstruation start 2-2.5 years after thelarche
Girls’ peak height velocity (PHV) coincides with SMR 2-3 while boys’ PHV
coincides with SMR 3-4
Dr. Punongbayan
VARIANTS:
• GIRLS: Precocious puberty → 8 years
Delayed puberty → 13 years
• BOYS: Precocious puberty → 9 years
Delayed puberty → 14 years
INCOMPLETE (PARTIAL) PRECOCIOUS PUBERTY
• Premature thelarche – breast development in the first 2 yrs of
life, regress after 2 yrs & rarely progressive
• Premature adrenarche – pubic hair; early maturational event of
adrenal androgen production
• Premature menarche
WHEN TO SUSPECT AN ABNORMALITY
• If 13: No thelarche
• If 14: no menses, hematocolpos, no vagina
• If 16: with menses but other secondary sex characteristics not
present (gonadal failure or pituitary)
• Precocious puberty
CHILD WITH SUSPECTED SEXUAL PRECOCITY:
© Topnotch Medical Board Prep INITIAL WORK UP: URGENT REASONS FOR REFERRAL:
STAGES OF MALE GENITALIA DEVELOPMENT • Bone age (X ray) • CNS abnormality like headache,
• Serum LH, FSH seizure, visual changes
• Testosterone and • Very high 17-OH progesterone level
estradiol serum • Rapidly progressing puberty
levels • Very young age
• DHEAS • Initial presentation is vaginal
• TSH bleeding
• Free T4 • Bone age >2 SD above CA
• Pelvic ultrasound • Signs of rapid virilization
CHILD WITH SUSPECTED DELAYED PUBERTY:
SUGGESTIVE
WHEN TO
HISTORY & PE WORK UP
REFER
FINDINGS
GIRLS: No breast • Bone age • If with CNS
enlargement by age • Serum LH and symptoms
13 years FSH • If with features
BOYS: No testicular • Testosterone and of Turner
enlargement by age estradiol levels syndrome
14 years (equal or • TSH, FT4
>4 mL or 2.5 cm in • prolactin
diameter)
BOTH: delay in
© Topnotch Medical Board Prep
adrenarche; look
for CNS symptoms,
family history

✔ GUIDE QUESTIONS
A 15-year-old female presents to the clinic with short stature and is
lagging behind in height amongst her peers. She was born via normal
vaginal delivery with normal weight and height. On further history
taking, she does not have menses yet but with some evidence of breast
development. Which among the ff. is the best next step in the
management?
A. Xray of the wrist
B. FSH
C. GnRH
D. LH
The case mentioned an adolescent who had features that appeared to
be consistent with a syndrome, most probably Turner syndrome
because of the following features: teenaged girl, short stature, with
primary amenorrhea, hypothyroidism, and liver function
abnormalities.
The most common clinical feature of Turner syndrome (45, X) is short
stature hence, bone aging through X ray of the wrist can be
prioritized. Prompt recognition and diagnosis is important to provide
optimal benefits from growth hormone treatment.
© Topnotch Medical Board Prep Primary hypogonadism is one of the common features and an elevated
serum FSH can be expected because of the ovarian failure.
Dr. Punongbayan

TOPNOTCH MEDICAL BOARD PREP PEDIATRICS MAIN HANDOUT BY DRS. DE VERA AND PUNONGBAYAN Page 97 of 106
For inquiries visit www.topnotchboardprep.com.ph or email us at topnotchmedicalboardprep@gmail.com
This handout is only valid for October 2022 PLE batch. This will be rendered obsolete for the next batch since we update our handouts regularly.
TOPNOTCH MEDICAL BOARD PREP PEDIATRICS MAIN HANDOUT BY DRS. DE VERA AND PUNONGBAYAN
For inquiries visit www.topnotchboardprep.com.ph or https://www.facebook.com/topnotchmedicalboardprep/
This handout is only valid for October 2022 PLE batch. This will be rendered obsolete for the next batch since we update our handouts regularly.
✔ GUIDE QUESTIONS Normal urine osmolality 500-800 mOsm/kg water
A 14-year-old female comes to the clinic for a complaint of short Random urine osmolality 300-900 mOsm/kg water
stature. Her mother expressed concern because the father was 5’10 in Serum osmolality <280 and serum sodium <135
height and the mother is 5’4 in height while the daughter was only 5 Random urine Na 40-220 mEqs/L/day
Dr. Punongbayan
feet tall. The daughter has not had her menarche as well. The mother
had her menarche at age 15. Physical examination revealed essentially
normal findings, however, a bone age X ray shows a bone age of 12 SIADH
years old. What is the most likely diagnosis?
A. Familial short stature DIAGNOSTIC CRITERIA OF SIADH
B. Constitutional growth delay • Absence of: renal, adrenal, or thyroid insufficiency, CHF,
C. Pathological growth delay nephrotic
D. XO (Turner syndrome) • syndrome, or cirrhosis, diuretic ingestion, dehydration
Delayed growth but final adult height is achieved; normal birth and • Urine Osmolality > 100 (usually>plasma)
growth history • Serum Osmolality < 280 and serum sodium < 135
Dr. Punongbayan
• Urine Na >25
GROWTH DELAY
• Hyponatremia and hypoosmolality resulting from inappropriate
• Short stature is defined as 2 or more standard deviations below continued secretion or action of vasopressin despite normal or
the mean height for children of that gender and chronological increased plasma volume → impaired water excretion
age. The height growth pattern over a period of time is more • Hyponatremia results from an excess of water
important than a single measurement at one point in time.
FAMILIAL SHORT STATURE SUPPLEMENT: QUICK SHEET
• Normal birth history and gestational weight SIADH Bartter-Schwartz criteria:
• Family history of short stature • Continued renal excretion of Na
• Short Stature With • Hyponatremia with hypo-osmolality
o Normal linear growth velocity for age • Urine less than maximally dilute
o Bone age consistent with chronological age • Absence of clinical evidence of volume depletion
o Normal age for onset of puberty • Absence of other causes of hyponatremia
o Absence of physical or psychological disease • Correction of hyponatremia by fluid restriction
CONSTITUTIONAL SHORT STATURE
• Delayed growth in one parent but average final stature Serum Serum Urine osmolality
Disorder
• Normal birth history and growth for first few months Na osmolality / urine Na
• Chronologic age is greater than bone age (CA>BA) SIADH low low high
Diabetes
high high low
insipidus

✔ GUIDE QUESTIONS
A 13-year-old male who had a recent head trauma 5 months ago from
an accident suddenly presented with polyuria and polydipsia. Serum
osmolality is 285 mOsm/kg and the urine osmolality is 100mOsm/kg,
urine specific gravity 1.005. What is the next best step for this patient?
A. Water Deprivation Test
B. Urine Sodium Excretion
C. Serum Vasopressin Levels
D. 24 Hour Urine Protein
Water Deprivation – fluid restriction for 12 hours results in >850
mOsm/kg
Normal Person = concentrate urine
DI = still have diluted urine
Normal urine osmolality is 300-900 mOsm/kg
Dr. Punongbayan
If after water deprivation test the urine osmolality is 200 mOsm/kg and
with administration of vasopressin, urine osmolality is still 200
mOsm/kg, what is the most likely diagnosis?
A. SIADH
B. Central Diabetes Insipidus
C. Nephrogenic Diabetes Insipidus
D. Psychogenic Polydipsia

DIABETES INSIPIDUS
• What are the cardinal features of diabetes insipidus?
MIDPARENTAL HEIGHT: o Polyuria and polydipsia
(𝑓𝑎𝑡ℎ𝑒𝑟 ! 𝑠 ℎ𝑒𝑖𝑔ℎ𝑡 − 13) + 𝑚𝑜𝑡ℎ𝑒𝑟 ! 𝑠 ℎ𝑒𝑖𝑔ℎ𝑡 • What are the two types of DI?
𝐺𝐼𝑅𝐿𝑆 = o 1. Vasopressin Deficiency (Central DI)
2
𝒇𝒂𝒕𝒉𝒆𝒓! 𝒔 𝒉𝒆𝒊𝒈𝒉𝒕 + (𝒎𝒐𝒕𝒉𝒆𝒓! 𝒔 𝒉𝒆𝒊𝒈𝒉𝒕 + 𝟏𝟑) o 2. Vasopressin Insensitivity (Nephrogenic DI)
𝑩𝑶𝒀𝑺 = • How will you distinguish between these conditions?
𝟐
• Parents height in cm o Water deprivation test and ADH administration
𝑇𝑎𝑟𝑔𝑒𝑡 ℎ𝑒𝑖𝑔ℎ𝑡 = 𝑚𝑖𝑑𝑝𝑎𝑟𝑒𝑛𝑡𝑎𝑙 ℎ𝑒𝑖𝑔ℎ𝑡 ± 2𝑆𝐷 (10𝑐𝑚)
FACTS ABOUT DIABETES INSIPIDUS
FAILURE TO THRIVE • What is the hormone involved in DI and what is its role?
• Persistent weight less than 5th percentile for age o Vasopressin – secreted from the posterior pituitary
• Growth curve: crossing 2 major percentile lines on growth chart o the principal regulator of tonicity
o has both antidiuretic & vascular pressor activity
✔ GUIDE QUESTIONS
o synthesized in the paraventricular & supraoptic nuclei of the
A 10-year-old male with pneumonia was found to have serum sodium
of 130, patient had moist mucous membranes, BP 90/60, HR 88, RR 20,
hypothalamus
T > 36.5°C, serum osmolality was 265 mOsm/kg, urinary osmolality Urine osmolality Urine osmolality
200 mOsm/kg, urinary sodium 40mmol/liter, low serum uric acid. (mOsm/kg) (mOsm/kg)
What is the most likely diagnosis in this case? Diagnosis
After After fluid
A. Diabetes Insipidus
B. Cerebral salt wasting
desmopressin deprivation
C. SIADH Central DI >750 <300
D. Psychogenic polydipsia Nephrogenic DI <300 <300
TOPNOTCH MEDICAL BOARD PREP PEDIATRICS MAIN HANDOUT BY DRS. DE VERA AND PUNONGBAYAN Page 98 of 106
For inquiries visit www.topnotchboardprep.com.ph or email us at topnotchmedicalboardprep@gmail.com
This handout is only valid for October 2022 PLE batch. This will be rendered obsolete for the next batch since we update our handouts regularly.
TOPNOTCH MEDICAL BOARD PREP PEDIATRICS MAIN HANDOUT BY DRS. DE VERA AND PUNONGBAYAN
For inquiries visit www.topnotchboardprep.com.ph or https://www.facebook.com/topnotchmedicalboardprep/
This handout is only valid for October 2022 PLE batch. This will be rendered obsolete for the next batch since we update our handouts regularly.
APPROACH TO POLYURIA AND POLYDIPSIA: ✔ GUIDE QUESTION
• Serum osmolality A 17-year-old female complains of anxiety attacks with palpitations and
• Serum Na, K, Ca, glucose sweating which started about 6 months ago but are increasing in
frequency. PE reveals marked proptosis and fine tremors were noted
• BUN, serum creatinine
as well. She reveals that her feelings of restlessness have begun to
• Urinalysis (SG and sugar) adversely affect her studies. Among the ff, which is expected?
• Urine osmolality (normal is 500-800 mOsm/kg of water) A. Increase TSH, Decreased T4 and T3
B. Decreased TSH, Decreased T4 and T3
DIAGNOSIS OF DI C. Decreased TSH, Increased T3 and T4
• DI is established if the serum osmolality is >300 mOsm/kg and D. Increased TSH, Increased T3 and T4
the urine osmolality is <300 mOsm/kg
• DI is unlikely if the serum osmolality is <270 mOsm/kg or the HYPERTHYROIDISM (GRAVES DISEASE)
urine osmolality is >600 mOsm/kg.
• What is the most likely diagnosis?
o Hyperthyroidism (Graves’ disease)
NEPHROGENIC DIABETES INSIPIDUS
• Who are affected?
• If the serum osmolality value is 290 mOsm/kg or higher with a o F > M; rarely occurs before adolescence
simultaneous urine osmolality value of <290 mOsm/kg, a formal • What is the pathophysiology of this condition?
water deprivation test is not necessary. o autoimmune induced (autoantibodies stimulate
• inability to respond to ADH (and thus the presence of NDI) hypersecretion of thyroid hormones)
should then be confirmed by the administration of vasopressin • What is thyroid storm?
(10-20 μg intranasally) followed by serial urine and serum o acute-life-threatening surge of thyroid hormone in the blood
osmolality measurements hourly for 4 hr. usually precipitated by surgery, trauma, infection, acute iodine
Type of DI Etiology Management load or long-standing hyperthyroidism
Congenital, trauma, o Tachycardia (>140/min), heart failure, fever, agitation,
tumors, autoimmune, Fluid therapy; long- delirium, psychosis, stupor, and/or coma
Central infection, drugs acting vasopressin o Treated with Methimazole
(ethanol, phenytoin, analog dDAVP
etc.) DIAGNOSTIC TESTS
Congenital (more • Increased T4, T3; low TSH
Treat underlying
severe), hypercalcemia, • (+) TRSAb; its disappearance predicts remission of the disease
disorder; thiazides
hypokalemia, renal
(decrease urine flow • Radionuclide study: palpable nodule and increased T3
Nephrogenic disease (PCKD, CRF),
to DCT, induce
drugs (lithium,
formation of TREATMENT FOR GRAVES DISEASE:
amphotericin,
functional receptors) • Can remit and recur
rifampicin, etc.)
• Definitive Tx: radioactive iodine ablation or thyroidectomy
CLINICAL PARAMETERS • PTU and Methimazole inhibit iodine organification and coupling
CLINICAL in the thyroid
SIADH CSW CENTRAL DI • Clinical response is apparent in 2-3 weeks
PARAMETER
Serum Na ↓ ↓ ↑ • Adequate control evident in 1-3 months
Urine Output N or ↓ ↑ ↑ ADVERSE
Urine Na ↑ ↑↑↑ ↓ DRUG PROPERTIES
EFFECT
Serum Uric acid ↓ N or ↑ ↑ • Inhibits extrathyroidal
Vasopressin conversion of T4-T3
↑ ↓ ↓
(ADH) Level PTU • Protein-bound --
Intravascular • Less able to cross
N or ↑ ↓ ↓
Volume status placenta
• 10x more potent than • Transient
Methimazole PTU • leukopenia
• Longer serum half-life • (asymptomatic)

HYPOTHYROIDISM
• Due to deficient production of hormone or a defect in
POLYURIA HYPONATREMIA
hormonal receptor activity
https://qrs.ly/zzdurd8 https://qrs.ly/i7durjo
• congenital or acquired
POLYURIA
Nephrogenic Primary • Etiologies of congenital hypothyroidism:
Central DI o Thyroid dysgenesis
DI Polydipsia
o Thyrotropin receptor-blocking antibody
Random
↑ ↑ ↓ o Defective synthesis of thyroxine
plasma osm
o Defect of iodide transport
Random
↓ ↓ ↓ o Thyroid peroxidase defects of organification & coupling
urine osm
• Birthweight & length are normal
Mild water
L UOsm L UOsm H UOsm • Prolonged physiologic jaundice
deprivation
• Feeding difficulties, sluggishness, lack of interest, somnolence
Plasma ADH ↓ N/↑ ↓
• Respiratory difficulties due to large tongue
UO after ADH ↓ NC NC
• Frequent constipation
HYPONATREMIA • Large abdomen; usually with umbilical hernia
Cerebral Salt • Hypothermic; cold & mottled skin; dry & scaly
SIADH • Edema of the genitals & extremities
Wasting
Serum Na ↓ ↓ • Retardation of physical & mental development progresses
UO ↓ ↑
DIAGNOSTICS & THERAPEUTICS:
Urine Na ↑ ↑↑↑
Serum UA ↓ N/↑ • Low serum T4 & T3; elevated serum TSH
Plasma ADH ↑ ↓ • X-ray: epiphyseal dysgenesis, deformity of T12 or L1-2; large
Intravascular volume fontanels & wide sutures; large Sella turcica; cardiomegaly
N/↑ ê • Tx: Sodium-L-thyroxine 10-15 ug/kg/day
status
• Monitor hormone levels & maintain
TOPNOTCH MEDICAL BOARD PREP PEDIATRICS MAIN HANDOUT BY DRS. DE VERA AND PUNONGBAYAN Page 99 of 106
For inquiries visit www.topnotchboardprep.com.ph or email us at topnotchmedicalboardprep@gmail.com
This handout is only valid for October 2022 PLE batch. This will be rendered obsolete for the next batch since we update our handouts regularly.
TOPNOTCH MEDICAL BOARD PREP PEDIATRICS MAIN HANDOUT BY DRS. DE VERA AND PUNONGBAYAN
For inquiries visit www.topnotchboardprep.com.ph or https://www.facebook.com/topnotchmedicalboardprep/
This handout is only valid for October 2022 PLE batch. This will be rendered obsolete for the next batch since we update our handouts regularly.

THYROIDITIS CONGENITAL ADRENAL HYPERPLASIA


Refer back to Newborn Screening (page 4) for a recap of CAH
• Lymphocytic / Hashimoto / Autoimmune
• AR disorders of adrenal steroidogenesis leading to a deficiency of
• Most common cause of thyroid disease in children & cortisol → increased secretion of corticotropin → adrenocortical
adolescents hyperplasia & overproduction of intermediary metabolites
• Etiology: organ-specific autoimmune disease is characterized • Deficiency of 21-hydroxylase accounts for 90% of affected patients
histologically by lymphocytic infiltration between the thyroid
• Lack of 21-hydroxylase (about 90% of cases) → excess
follicles
substrates which are shunted towards synthesis of sex
• HLA-DR4, HLA-DR5 associated with an increased risk of goiter hormones → decreased cortisol → loss of feedback inhibition →
& thyroiditis increased ACTH → further stimulation of the conversion of
• Thyroid anti-peroxidase antibodies (TPOAbs) seen in the sera cholesterol into sex hormone precursors (17-OH progesterone
of 90% of children; inhibit enzyme activity & stimulate natural and androstenedione)
killer cell cytotoxicity • low serum Na & Cl; increased K; low serum cortisol
• Inc. plasma renin; low serum aldosterone
MANIFESTATIONS, DIAGNOSTICS, AND TREATMENT
• 21-hydroxylase deficiency: increased serum 17-OHP
• Thyroid is diffusely enlarged, firm & nontender in most patients • Pelvic ultrasound to visualize presence of uterus in female
• Most of the affected children are euthyroid & asymptomatic pseudohermaphroditism
• Thyroid scan: 50% reveal irregular & patchy distribution of the • Glucocorticoids inhibit excessive production of androgens &
radioisotope prevents progressive virilization
• Thyroid UTZ: scattered hypoechogenicity • Tx: Hydrocortisone 10-20 mg/m2/day orally in 2-3 divided doses
• Tx: if with evidence of hypothyroidism, give sodium-L-thyroxine
(50-150 ug/day) TYPE MANIFESTATIONS
• May be self-limited; periodic re-evaluation • Normal at birth but signs of sexual precocity
• (+)nodules – biopsy (identify CA) Non- appear w/in 1st 6 mos of life
salt • Usually normal mental development
losing • Small testes & enlarged penis
ADRENAL GLANDS • Females: enlarged clitoris or labial fusion
• Adrenal gland: 2 endocrine systems: medullary gland & • Symptoms begin shortly after birth
cortical system • progressive weight loss, prominent vomiting,
• Adrenal cortex: Salt- dehydration
1. Zona glomerulosa – aldosterone losing • Females: more pronounced virilization of
2. Zona fasciculata – cortisol & androgens external genitals
3. Zona reticularis – androgens • Males: genitals appear normal
CAH DUE TO 21-OH DEFICIENCY
CLASSIC SIMPLE
CLASSIC SALT-WASTING NON-CLASSIC
VIRILIZING
Males Females Males Females Males Females
Birth to Birth to 1 2 to 4 Birth to 2
Age at diagnosis Child to adult
6 months month years years
Usually normal; may have
External Genitalia Normal Ambiguous Normal Ambiguous Normal
clitoromegaly
Aldosterone Low Normal Normal
Cortisol Low Low Normal
17-OHP Basal > 20,000 ng/dL Basal >10,000-20,000 ng/dL ACTH stimulated 1,500-10,000 ng/dL
% of normal 21-OH
0 1-2 20-50
activity
Adapted from Pediatrics Endocrinology, Mechanisms, Manifestations and Management, Ora H. Pescovitz, Erica A. Eugster, 2004 by Lippincott Williams & Wilkins

MANIFESTATIONS INDICATING ADRENAL CRISIS MANAGEMENT OF ADRENAL CRISIS


• Vomiting, diarrhea • Shock: NSS bolus 20 mL/kg
• Abdominal pain o Repeat up to a total of 60 mL/kg within 1 hour
• Unexplained fever • Hypoglycemia: initial bolus of D10W 0.5-1 g/kg IV
• Weight loss • Hyperkalemia: ECG (tall, peaked T waves with shortened QT
• anorexia interval, progressive lengthening of PR interval and QRS duration)
o Insulin- glucose solution
• Hypoglycemia
• Stress glucocorticoids
• Micropenis
o Given as IV bolus over several minutes started ASAP
• Bilateral cryptorchidism
• Hydrocortisone succinate sodium
• Hyperpigmentation of the skin
o 50 mg/m2 for children
o Age-based dosing may be used
When to consider AI crisis:
o 0-3 years old: 25 mg IV
• Neonates with atypical genitalia o 3-12 years old: 50 mg IV
• Electrolyte abnormalities o Older than 12 years: 100 mg IV
• Lethargy • Glucocorticoids inhibit excessive production of androgens
• Dehydration & prevents progressive virilization
• Hypotension or shock • Hydrocortisone 10-20 mg/m2/day orally in 2-3 divided doses
• Failure to thrive • Monitor growth & hormonal levels
• Critically ill patients with septic shock unresponsive to fluid • Fluorohydrocortisone (0.05-0.3 mg daily) & NaCl 1-3 grams
resuscitation and inotropes given to normalize plasma renin activity
• Hydrocortisone continued indefinitely in all patients with
LAB FINDINGS IN CAH classic forms of CAH
• low serum Na & Cl; inc. K; low serum cortisol
• Increased plasma renin; serum aldosterone;
• Decreased serum glucose
CUSHING SYNDROME
• 21-hydroxylase deficiency: increased serum 17-OHP • Characteristic pattern of obesity with associated hypertension
• Pelvic ultrasound to visualize presence of uterus in female which is the result of abnormally high blood levels of cortisol
pseudohermaphroditism resulting from hyperfunction of the adrenal cortex
• Etiology: functioning adrenocortical tumor (infants);
pituitary adenomas; hyperplasia of adrenals
TOPNOTCH MEDICAL BOARD PREP PEDIATRICS MAIN HANDOUT BY DRS. DE VERA AND PUNONGBAYAN Page 100 of 106
For inquiries visit www.topnotchboardprep.com.ph or email us at topnotchmedicalboardprep@gmail.com
This handout is only valid for October 2022 PLE batch. This will be rendered obsolete for the next batch since we update our handouts regularly.
TOPNOTCH MEDICAL BOARD PREP PEDIATRICS MAIN HANDOUT BY DRS. DE VERA AND PUNONGBAYAN
For inquiries visit www.topnotchboardprep.com.ph or https://www.facebook.com/topnotchmedicalboardprep/
This handout is only valid for October 2022 PLE batch. This will be rendered obsolete for the next batch since we update our handouts regularly.
• Rounded face, prominent cheeks, moon facies, buffalo hump, ✔ GUIDE QUESTION
generalized obesity, abnormal masculinization, impaired A 7-year-old girl was brought to her doctor because she was noted to
growth, hypertension, increased susceptibility to infection be less active and wetting her bed, which she had stopped doing 3 years
• Older children: purplish striae on hips, abdomen & thighs, prior. There was significant weight loss noted too. Her mother noticed
her to be eating and drinking more than usual. Few days ago, patient
delayed puberty, emotional lability, weakness, headache
developed diarrhea and vomiting. Patient had decreased oral intake
• Diagnostics and was then rushed to the ER. The patient was drowsy and weak
o Serum cortisol levels are normally elevated at 8 am & decrease looking. HR 140 RR 30 O2 sat 98% BP 100/70. CBG 450. What should
to <50% by 8pm → diurnal rhythm is lost be the initial management for this patient?
o Urinary excretion of free cortisol & 17-hydroxycorticosteroids A. Hydration
are increased B. Insulin Administration
• Management C. Biguanides
D. Intubation
o Unilateral adrenalectomy for benign cortical adenomas
o Bilateral tumors: subtotal adrenalectomy
o Trans-sphenoidal pituitary microsurgery for children DIABETES MELLITUS
o Adequate preoperative & postoperative replacement therapy • Syndrome of metabolic disease characterized by hyperglycemia
✔ GUIDE QUESTION due to deficiency of insulin secretion or insulin action or both
An adolescent female was referred to you due to episodes of headaches resulting in abnormal metabolism of CHO, CHON & fat
that comes and goes. Upon physical examination you noted the patient • Most common endocrine-metabolic disorder of childhood &
to be tachycardic, hypertensive with excessive sweating. Imaging was adolescence
requested by the referring physician which showed an abdominal mass. • Girls=boys; peaks at 5-7 yrs old & puberty
The following is expected for this condition EXCEPT?
• Basic cause of the initial clinical findings is the sharply
A. Management is primarily surgical
B. Urine VMA is expected to be elevated diminished insulin secretion
C. This is a catecholamine secreting tumor from chromaffin cells • Mechanisms that lead to failure of pancreatic ß-cell function
D. Most common site of the tumor is the adrenal cortex point to the likelihood of autoimmune destruction of pancreatic
islets in predisposed individuals
PHEOCHROMOCYTOMA • About 80-90% of newly diagnosed patients have islet-cell
• Catecholamine-secreting tumor arising from the chromaffin cells antibodies (ICAs) directed at cell surface.
• Most common site of origin is the adrenal medulla • Tissue damage of pancreatic 𝛽 cells is mediated by T lymphocytes
• Tumors may develop anywhere along the abdominal → produce cytokines → induce destruction of islet cells
sympathetic chain, likely to be located near the aorta at the level • Type I – absolute insulin deficiency
of the IMA or at its bifurcation. • Type II – insulin resistance and increased insulin levels
• Periadrenal area, urinary bladder, ureteral walls, thoracic cavity, • Both types can result in retinopathy, nephropathy, and
cervical region neuropathy.
EFFECTS OF INSULIN DEFICIENCY:
• With progressive deficiency → excessive glucose production &
impairment of its utilization → hyperglycemia with glucosuria
→ resultant osmotic diuresis produces polyuria, urinary losses
of electrolytes, dehydration, polydipsia → hypersecretion of
epinephrine, glucagon, cortisol, & GH which amplifies &
perpetuates metabolic derangements & accelerates metabolic
decompensation
• Combination of insulin deficiency & ↑ counterregulatory
hormones is responsible for accelerated lipolysis & impaired
lipid synthesis → ↑ plasma total lipids, cholesterol, TG, FFA →
ketone body formation which exceeds the capacity for
FACTS RE: PHEOCHROMOCYTOMA peripheral utilization & renal excretion → metabolic acidosis &
• Occur in children 6-14 yrs old rapid deep breathing
• Tumors found more often on the right side, about 1-10 cm in diameter CLINICAL MANIFESTATIONS:
• Bilateral in >20% affected children • Classic: polyuria, polydipsia, polyphagia, weight loss (often in
• Inherited as AD trait a less than a month)
• May be associated with other syndromes such as • Clue to polyuria: onset of enuresis in a previously toilet-trained
neurofibromatosis, part of MEN syndromes, tuberous sclerosis, child
Sturge-Weber syndrome, ataxia-telangiectasia • Pyogenic skin infections & monilial vaginitis in adolescent
CLINICAL MANIFESTATIONS females
• S/Sx result from excessive secretion of epinephrine & DIAGNOSTIC CRITERIA:
norepinephrine • Symptoms of diabetes plus a
• May be symptom-free in between attacks of hypertension 1. random plasma glucose >200 mg/dL, or;
• Headache, palpitations, abdominal pain, dizziness, pallor, 2. fasting plasma glucose >126 mg/dL
vomiting, sweating, convulsions • Polyuria, polydipsia, & unexplained weight loss with glucosuria
• Severe: precordial pain radiates into the arms, pulmonary & ketonuria
edema, cardio- & hepatomegaly
LAB FINDINGS DIABETIC KETOACIDOSIS (ISPAD) criteria:
• Dx: demonstration of elevated blood or urinary levels of • glucose >200
catecholamines & their metabolites • Venous pH < 7.3 or bicarbonate <15mmol/L
• Predominant catecholamine in children is norepinephrine • ketonemia (B-hydroxybutyrate 3 or more mmol/L) or
derived from the adrenal gland & adrenergic nerve endings moderate or large ketonuria
• Urinary excretion of vanillylmandelic acid (major metabolite • Precipitating factors: trauma, infections, vomiting, psychologic
of epi-, norepi- & metanephrine) is increased disturbances
• Ultrasound, CT scan, MRI: tumors • Clinical signs: tachycardia, tachypnea, dehydration, deep,
sighing (Kussmaul) respiration with acetone-like breath,
MANAGEMENT
nausea, vomiting, abdominal pain, confusion, drowsiness
• Surgical removal of tumors
• Preoperative 𝛼- & 𝛽-adrenergic blockers CLASSIFICATION OF DKA (ISPAD):
• Thorough transabdominal exploration of all the usual sites MILD MODERATE SEVERE
• Accurate indicators of malignancy – presence of metastatic disease HCO3 <15 <10 <5
or local invasiveness that precluded complete resection or both pH <7.3 <7.2 <7.1
TOPNOTCH MEDICAL BOARD PREP PEDIATRICS MAIN HANDOUT BY DRS. DE VERA AND PUNONGBAYAN Page 101 of 106
For inquiries visit www.topnotchboardprep.com.ph or email us at topnotchmedicalboardprep@gmail.com
This handout is only valid for October 2022 PLE batch. This will be rendered obsolete for the next batch since we update our handouts regularly.
TOPNOTCH MEDICAL BOARD PREP PEDIATRICS MAIN HANDOUT BY DRS. DE VERA AND PUNONGBAYAN
For inquiries visit www.topnotchboardprep.com.ph or https://www.facebook.com/topnotchmedicalboardprep/
This handout is only valid for October 2022 PLE batch. This will be rendered obsolete for the next batch since we update our handouts regularly.
CLASSIFICATION OF DKA: MONITORING
NORMAL MILD MODERATE SEVERE • Reliable index of long-term glycemic control – measure
HC03 20-28 16-20 10-15 <10 glycosylated Hgb
pH 7.35-7.45 7.25-7.35 7.15-7.25 <7.15 • Glycohemoglobin (HbA1c) represents the fraction of Hgb to
Kussmaul or which glucose has been nonenzymatically attached in the
Kussmaul
depressed
Oriented; respirations, bloodstream
respirations;
Clinical No change alert but oriented but
depressed • reflects the average blood glucose concentration of the
fatigued sleepy; preceding 2-3 months
sensorium to
arousable
coma • Glycated Hgb predicts risk of progression of diabetic
complications
MANAGEMENT
• Ketoacidosis: expansion of intravascular volume, correction of HbA1c
deficits in fluid, electrolyte & acid-base status; initiation of • The more consistently lower the level, the better the metabolic
insulin therapy control, the more likely it is that microvascular complications
• Initial hydrating fluid is isotonic saline (hypotonic relative to the will be less severe, delayed in appearance, or avoided.
patient’s serum osmolality) • In known diabetics:
• Administration of glucose (5% solution in 0.2 N saline) is o 7% indicates good diabetic control
initiated when blood glucose approaches 300 mg/dL to limit the o 10% indicates fair diabetic control
decline of serum osmolality & reduce cerebral edema o 13-20% indicates poor diabetic control
• Give potassium added after the initial 20 ml/kg if UO is adequate. PLEASE READ MORE ON:
• Bicarbonate only if pH <7.2 given slowly • Hypoparathyroidism
• Anticipate cerebral edema – limit rate of fluid to 4 L/m2/day or less • Hyperparathyroidism
• Insulin 0.1 U/kg of regular insulin followed by constant • Klinefelter syndrome
infusion of 0.1 U/kg/hr • Noonan syndrome
• Turner syndrome

IMMUNOLOGY/ALLERGOLOGY

TIPS: ✔ GUIDE QUESTION


• Take note of acuity and chronicity of symptoms A 9-year-old male was brought to you for chronic sneezing and cough
more prominent in the morning. Father and mother were known
• Effect on activities of daily living
asthmatics. Upon physical examination, patient had watery nasal
• Medications discharge with boggy turbinates. (+) allergic shiners. For the past 2
• Pertinent PE findings months, patient has been having almost daily attacks with difficulty
sleeping. What is the classification of this patient’s disease?
GENETIC BASIS OF ATOPY & COMMON MANIFESTATIONS A. Mild Persistent
• Atopic diseases have a strong familial predisposition with about B. Moderate-Severe Persistent
C. Mild Intermittent
60% heritability D. Moderate-Severe Intermittent
• The 5q23-35 region comprises several genes implicated in
allergic disease pathogenesis Based on ARIA guidelines, given the chronic history of sneezing and
coughing for 2 months with difficulty of sleeping, this patient falls
• The risk of allergic disease in a child approaches 50% when one under the clinical classification moderate-severe persistent AR. JUST
parent is allergic and 66% when both parents are allergic REMEMBER THE NUMBER 4! The cutoff duration to distinguish
• Allergic salute, nasal crease, intermittent from persistent is less than 4 weeks for the former while
allergic cluck, allergic shiners with more than 4 weeks for the latter.
Dennie-Morgan folds The cutoff frequency of symptoms to differentiate intermittent from
persistent is less than 4 days for the former while more than 4 days for
Nasal crease – a horizontal wrinkle the latter.
Dr. Punongbayan
over the nasal bridge
ALLERGIC RHINITIS
• The classification of Allergic Rhinitis takes into account the
following criteria:
o Based on allergen causing symptoms (etiological
classification)
o Duration of symptoms (clinical classification)
o Severity of clinical symptoms (depending on impact of disease
on quality-of-life measures)
o Disease pathophysiology (limited use)
TOPNOTCH MEDICAL BOARD PREP PEDIATRICS MAIN HANDOUT BY DRS. DE VERA AND PUNONGBAYAN Page 102 of 106
For inquiries visit www.topnotchboardprep.com.ph or email us at topnotchmedicalboardprep@gmail.com
This handout is only valid for October 2022 PLE batch. This will be rendered obsolete for the next batch since we update our handouts regularly.
TOPNOTCH MEDICAL BOARD PREP PEDIATRICS MAIN HANDOUT BY DRS. DE VERA AND PUNONGBAYAN
For inquiries visit www.topnotchboardprep.com.ph or https://www.facebook.com/topnotchmedicalboardprep/
This handout is only valid for October 2022 PLE batch. This will be rendered obsolete for the next batch since we update our handouts regularly.

I. BASED ON ALLERGEN CAUSING SYMPTOMS • Exposure of an atopic host to an allergen leads to specific IgE
(ETIOLOGICAL CLASSIFICATION) production
• Early-phase reaction: degranulation of mast cells & release of
• Seasonal – occurs only during specific periods of the year; e.g.
inflammatory mediators
pollination, mold sporulation
• Late-phase reaction: arises 4-8 hours following allergen
• Perennial – triggered by allergens formed in the patient’s
exposure
environment at concentrations sufficient to induce symptoms all
year round; dust mites, pet fur, cockroach • Sneezing, rhinorrhea, nasal obstruction, itching of the nose,
palate, pharynx & ears, itching, redness & tearing of the eyes
• Episodic – exposure to a specific airborne allergen on a sporadic
• Pale mucosa, clear mucoid nasal discharge, allergic salute, rabbit
and short-term basis
nose
• Avoidance of exposure to suspected allergens and irritants
II. DURATION OF SYMPTOMS (CLINICAL
CLASSIFICATION)
• Intermittent – less than 4 days/week or less than a month/year
• Persistent – equal or more than 4 days/week or equal or more
than a month/year

III. SEVERITY OF CLINICAL SYMPTOMS


(DEPENDING ON IMPACT OF DISEASE ON QUALITY-OF-
LIFE MEASURES: DAILY ACTIVITIES, SCHOOL OR WORK
ATTENDANCE, SLEEP, NEED OF THERAPY)
• Mild
• Moderate / Severe – either 1, 2, or 3 out of 4 elements or all 4
elements affected

✔ GUIDE QUESTION
In line with the previous case, what is the drug of choice for this patient?
A. Intranasal steroids
B. Oral antihistamine
C. Topical antihistamine
D. Oral steroids
The 9-year-old boy in the aforementioned case had moderate-severe
persistent allergic rhinitis for which he would need intranasal steroids
ARIA at-a-Glance Pocket Reference 2007 to control his symptoms.
Dr. Punongbayan

IV. DISEASE PATHOPHYSIOLOGY (LIMITED USE) ALLERGIC RHINITIS


• Immunotherapy for those who cannot avoid inhalant allergens
• IgE-mediated – about >90%
and drug therapy
• Non-IgE-mediated – IgG, T lymphocytes, eosinophils
• Oral antihistamines; inhaled budesonide, fluticasone,
• Diagnosis generally established by 6 yrs old mometasone
• 20% are seasonal, 40% perennial, 40% mixed

Adapted from ARIA at-a-Glance Pocket Reference 2007


✔ GUIDE QUESTION A. Selective IgA Deficiency
A 3-year-old male patient comes to the clinic for right ear pain without B. Bruton Agammaglobulinemia
discharge and fever that started 1 day ago which prompted consult. C. Common Variable Immunodeficiency
Patient had cough and colds 3 days ago. The mother says that the D. Severe Combined Immunodeficiency
patient has been having repeated ear infections and colds since infancy. This is a case of selective IgA deficiency based on the cluster of
He was also hospitalized twice for pneumonia at ages 9 months and 18 repeated symptoms involving the respiratory and urinary tract
months with concomitant urinary tract infection. He is also known to systems with a less than normal IgA level. This deficiency has
have allergies to certain milk products. Further work up revealed IgA characteristically recurrent R-U-G infections!!! (respiratory –
<10mg/dL, IgG, IgM and IgE within normal limits, what is the most urogenital – gastrointestinal)
likely condition? Dr. Punongbayan

TOPNOTCH MEDICAL BOARD PREP PEDIATRICS MAIN HANDOUT BY DRS. DE VERA AND PUNONGBAYAN Page 103 of 106
For inquiries visit www.topnotchboardprep.com.ph or email us at topnotchmedicalboardprep@gmail.com
This handout is only valid for October 2022 PLE batch. This will be rendered obsolete for the next batch since we update our handouts regularly.
TOPNOTCH MEDICAL BOARD PREP PEDIATRICS MAIN HANDOUT BY DRS. DE VERA AND PUNONGBAYAN
For inquiries visit www.topnotchboardprep.com.ph or https://www.facebook.com/topnotchmedicalboardprep/
This handout is only valid for October 2022 PLE batch. This will be rendered obsolete for the next batch since we update our handouts regularly.

IMMUNODEFICIENCY This is a case of SCID – severe combined immunodeficiency. The main


features that were mentioned in the case are the following: presence
• Diagnosis: suspected in every patient, regardless of age, who has of a fungal infection (oral thrush), much earlier onset of
recurrent, persistent, and unusual or opportunistic symptoms, in this case, at 4 months of age, and the low levels of
infections immunoglobulins.
• Persistent infection in the context of proper antibiotic The other choice was CVID – presence of low, but not absent, T cell
therapy is significant! function; recurrent or chronic pulmonary infections, FTT, oral or
cutaneous candidiasis, chronic diarrhea, recurrent skin infections,
• Infections in more than one organ system and severe
UTIs, gram (-) bacterial sepsis, severe varicella in infancy; neutrophils
• Consanguinity of parents, early deaths of relatives, unusual and eosinophilia
reaction to vaccines Dr. Punongbayan

10 WARNING SIGNS OF PRIMARY IMMUNODEFICIENCY COMBINED IMMUNODEFICIENCY DISORDER:


Primary immunodeficiency [PI] causes children and adults to have infections SEVERE CID (SCID)
that come back frequently or are unusually hard to cure. 1:500 persons are
affected by one of the known PIs. • Profound system immunodeficiency
1. Four or more new ear infections within 1 year • Selective defect in cell-mediated immunity with susceptibility to
2. Two or more serious sinus infections within 1 year chronic candida infection of the skin and & mucous membranes
3. Two or more months on antibiotics with little effect w/ or w/o endocrinopathy
4. Two or more pneumonias within 1 year • 50% are due to deficiency of adenosine deaminase due to
5. Failure of an infant to gain weight or grow normally deletion or mutation of the encoding gene → deoxyadenosine &
6. Recurrent, deep skin or organ abscesses deoxy-ATP accumulate inside lymphocytes → cell death
7. Persistent thrush in mouth or fungal infection on skin • Infants usually succumb within the first 2 years of life
8. Need for intravenous antibiotics to clear infections • Failure to thrive, chronic otitis media, chronic diarrhea,
9. Two or more deep-seated infections including septicemia oropharyngeal candidiasis, recurrent pneumonia
10. A family history of PI • Varicella, Candida, Pneumocystis carinii, CMV
TIPS IN APPROACHING A CASE OF PROBABLE IMMUNODEFICIENCY: • Profound lymphopenia, unable to reject transplant, short-
1. Consider the age of onset of symptoms limbed dwarfism
o for less than 6 months of age in onset: consider a severe form of • Bone marrow transplantation
immunodeficiency
o for more than 6 months of age in onset: consider a B-cell
T-CELL IMMUNODEFICIENCY DISORDER
immunodeficiency as it is known to exhibit its manifestations
beyond 6 months old when protective maternal IgG levels • DiGeorge syndrome - Congenital Thymic Aplasia
transferred to a newborn start to go down • Increased susceptibility to viral, fungal & protozoal infections
2. Identify the systems involved and whether the infections are • Aberrant thymus or its absence occurs in the 12th week of
recurrent, persistent, or unusual gestation
3. Ask for the response to usual / conventional treatment
Dr. Punongbayan • Symptoms at birth: abnormal facies, hypothyroidism, congenital
heart disease
✔ GUIDE QUESTION
A 1-year-old male patient comes to the clinic due to cough, fast
• Low lymphocyte count
breathing, and fever. It started 1 day ago which prompted consult. • Fetal thymus transplant as early as possible results in
Patient had cough and colds 3 days ago. There was history of repeated reconstitution of T-cell immunity
ear infections at 8 months and 11 months. He was also admitted for
pneumonia at 8 months of age. At present, HR 120, RR 55, T 38.5°C, (+)
crackles bilateral lung fields with decreased breath sounds over the
right lung, (-) cervical lymphadenopathies, (-) tonsillar tissue. After
antibiotic treatment, further workup was done which revealed
depressed levels of IgG, IgA, IgM, and IgE below that of appropriately
aged matched controls. What is the most likely diagnosis in this case?
A. Selective IgA Deficiency
B. Bruton Agammaglobulinemia
C. Common Variable Immunodeficiency
D. Severe Combined Immunodeficiency ✔ GUIDE QUESTION
A 10-month-old male patient is brought to the clinic by his mother
B-CELL IMMUNODEFICIENCY DISORDER because of a rash on his face. This started 1 week ago and he was seen
by his mother to be scratching or touching the facial area. She also
• Bruton disease – X-linked agammaglobulinemia noticed rashes in the area over the upper extremities. There was no use
• A block in the development of pre-B cell to B-cell due to a defect of new soaps/shampoos, no substance was applied to the skin; mother
in heavy chain gene rearrangement is a known asthmatic. On close examination, there is an eczematous
• Healthy infants until 6 months old → recurrent ear infections, rash over the face particularly the cheeks and the extensor surfaces of
bronchitis or pneumococcal pneumonia, and/or dermatitis the upper extremities. What is the most likely diagnosis?
A. Irritant Contact Dermatitis D. Atopic Dermatitis
• Low or absent Igs; few or absent B-cells
B. Allergic Contact Dermatitis E. Infantile SLE
• Total Ig < 250 mg/dL C. Photosensitive Dermatitis
• Gamma globulin IM once a month at interval of 2-3 weeks
• Maintenance of trough level > 400 mg/dL
Remember B-B-B!!!
WISKOTT-ALDRICH SYNDROME
(Bruton disease, B-cell deficiency, bacterial infections are recurrent) • Immunodeficiency with thrombocytopenia and eczema
Dr. Punongbayan • X-linked recessive syndrome
✔ GUIDE QUESTION • With normal appearing megakaryocytes but small defective
A 6-month-old male infant was seen for persistent diarrhea for the past platelets
2 weeks. The patient has visible wasting, underweight, and appears • Often have prolonged bleeding from the circumcision site or
weak and irritable. The mother said that the patient has been having bloody diarrhea during infancy
repeated bouts of diarrhea since 4 months of age. He also was admitted
for pneumonia at 3 months of age. The infant on examination has oral
• Treatment: nutrition, routine IVIG, use of killed vaccines, platelet
thrush, no visible tonsillar tissue, no palpable lymphadenopathies. transfusion, splenectomy
Previous X ray at 3 months of age show no thymic shadow. Further • Treatment of choice: bone marrow or cord blood
workup was done which revealed depressed levels of IgG, IgA, IgM and transplantation
IgE below that of appropriately aged matched controls. What is the
most likely diagnosis?
A. DiGeorge Syndrome
B. Bruton Agammaglobulinemia
C. Common Variable Immunodeficiency
D. Severe Combined Immunodeficiency

TOPNOTCH MEDICAL BOARD PREP PEDIATRICS MAIN HANDOUT BY DRS. DE VERA AND PUNONGBAYAN Page 104 of 106
For inquiries visit www.topnotchboardprep.com.ph or email us at topnotchmedicalboardprep@gmail.com
This handout is only valid for October 2022 PLE batch. This will be rendered obsolete for the next batch since we update our handouts regularly.
TOPNOTCH MEDICAL BOARD PREP PEDIATRICS MAIN HANDOUT BY DRS. DE VERA AND PUNONGBAYAN
For inquiries visit www.topnotchboardprep.com.ph or https://www.facebook.com/topnotchmedicalboardprep/
This handout is only valid for October 2022 PLE batch. This will be rendered obsolete for the next batch since we update our handouts regularly.
✔ GUIDE QUESTION TREATMENT OF B CELL DEFECTS
Which of the following is the gold standard in the diagnosis of food • Stem cell transplant recommended for XLP and Hyper IgM type 3
allergy? • Judicious use of antibiotics for infections and regular IVIG (or
A. Patch Test subcutaneous IG) are the only effective treatment for primary
B. Skin Prick Test
B cell defects
C. RAST (radioallergosorbent test)
D. Double Blind Placebo Controlled Food Challenge • Dose of 400 mg/kg per month
PRIMARY DEFECTS OF ANTIBODY PRODUCTION
CHROMOSOME FUNCTIONAL
DISORDER MAIN CLINICAL FEATURES
& REGION DEFICIENCY
No antibody production • Bacterial infections (resp. tract) and esp. enteroviruses;
XLA or Bruton
-- (total Ig <100 mg/dL) • small to absent tonsils;
agammaglobulinemia
lack of B cells • no palpable lymph nodes
• Later age of onset of infections;
Low IgG with normal • echovirus meningitis is rare;
CVID --
B cells • normal nodes & tonsils; cancer;
• ↑ risk of granuloma
Selective IgA Low or absent IgA • respiratory, GI (giardiasis),
6p21.3
deficiency (<10 mg/dL) • urogenital tract infections
• Become symptomatic during the 1st or 2nd year of life with
Failure to produce IgG,
12p13, recurrent pyogenic infections;
Hyper IgM syndrome IgA, IgE antibodies with
Xq26 • profoundly neutropenic;
normal or ↑ IgM
• P. jiroveci pneumonia
X-linked low Ig; • Inadequate immune response to EBV;
lymphoproliferative lack of anti-EBNA and • affected males are usually healthy until they acquire EBV infection;
Xq25
disease or Duncan long-lived T cell • fatal infectious mononucleosis, lymphomas, acquired
disease immunity hypogammaglobulinemia

PRIMARY DEFECTS OF CELLULAR IMMUNITY


CHROMOSOME
DISORDER FUNCTIONAL DEFICIENCY MAIN CLINICAL FEATURES
& REGION
Thymic hypoplasia or DiGeorge • Partial or complete
• Low number of T cells and
syndrome – hypoplastic or -- • CATCH 22 syndrome
impaired T cell function
aplastic thymus and parathyroid • susceptible to fungi, viruses, and P. jiroveci
• chronic and severe Candida skin & mucous
• poor response to Candida
Chronic mucocutaneous membrane infections in the 1st mo or until 2nd
21q22.3 antigen;
candidiasis decade of life;
• autoimmune responses
• APECED

TREATMENT OF T CELL IMMUNODEFICIENCY PLEASE READ MORE ON:


• Thymic tissue transplant
• Organs of immune system
• Some have been given nonirradiated unfractionated BM or
• 4 types of immunopathologic Reactions
peripheral blood transplant from HLA-identical sibling
• Treatment of choice for fatal combined T- and B-cell defects:
transplantation of MHC-compatible sibling or T-cell depleted PRIMARY
parental hematopoietic stem cells IMMUNE
• For SCID: bone marrow transplant (92% have survived after T- DEFICIENCIES
cell-depleted parental marrow is given soon after birth without https://qrs.ly/btdurlf
pretransplant chemotherapy)

PRIMARY COMBINED ANTIBODY AND CELLULAR IMMUNODEFICIENCIES


CHROMOSOME FUNCTIONAL
DISORDER MAIN CLINICAL FEATURES
& REGION DEFICIENCY
5p13, • present w/in the 1st few months of life with recurrent or
SCID 10p13, Absence of T- and B- persistent diarrhea, pneumonia, OM, sepsis, skin infections;
(most severe
immunodeficiency)
11p13, cell functions • persistent infections with opportunistic agents;
1q31-q32 • unable to reject foreign tissue
• Immunodeficiency with thrombo-cytopenia and eczema;
T cell deficiency; poor Ab
Wiskott-Aldrich • prolonged bleeding from circumcision site or bloody diarrhea
Xp11.4-11.21 production to
syndrome during infancy;
polysaccharides
• survival beyond the teens is rare
Coarse facies, coronary
Hyper IgE syndrome • Recurrent bacterial infections (S. aureus, pneumococcus, H.
artery aneurysm,
(AD or sporadic or Job 17q21.3 influenzae),
mortality in adulthood;
syndrome) • Aspergillus, mucocutaneous candidiasis
pathologic fractures
Higher incidence of
Hyper IgE syndrome • Abscesses, HPV, HSV, VZV, MCV, mucocutaneous candidiasis;
21q22.3 lymphomas; skin cancer;
(AR) asthma, allergies / anaphylaxis
mortality in childhood

TREATMENT:
• Treatment of choice for fatal combined T- and B-cell defects: transplantation of MHC-compatible sibling or T-cell depleted parental
hematopoietic stem cells
• For SCID: bone marrow transplant (92% have survived after T-cell-depleted parental marrow is given soon after birth without pre-
transplant chemotherapy)

TOPNOTCH MEDICAL BOARD PREP PEDIATRICS MAIN HANDOUT BY DRS. DE VERA AND PUNONGBAYAN Page 105 of 106
For inquiries visit www.topnotchboardprep.com.ph or email us at topnotchmedicalboardprep@gmail.com
This handout is only valid for October 2022 PLE batch. This will be rendered obsolete for the next batch since we update our handouts regularly.
TOPNOTCH MEDICAL BOARD PREP PEDIATRICS MAIN HANDOUT BY DRS. DE VERA AND PUNONGBAYAN
For inquiries visit www.topnotchboardprep.com.ph or https://www.facebook.com/topnotchmedicalboardprep/
This handout is only valid for October 2022 PLE batch. This will be rendered obsolete for the next batch since we update our handouts regularly.
DISORDERS OF NEUTROPHIL FUNCTION
DISORDER ETIOLOGY IMPAIRED FUNCTION CLINICAL CONSEQUENCE
• Recurrent pyogenic infections;
• neutropenia;
• hepatosplenomegaly;
• mild bleeding diathesis;
Degranulation AR; disordered • Decreased chemotaxis,
• partial oculo-cutaneous albinism
abnormality: coalescence of degranulation and
• progressive peripheral neuropathy;
Chediak-Higashi lysosomal bactericidal activity;
• light skin and silvery hair;
syndrome granules • impaired NK function
• photophobia;
• rotary nystagmus;
• Gram +, Gram -, fungi;
• prolonged BT with normal platelet
• Neutrophilia;
Adhesion • Impaired neutrophil
• recurrent bacterial infection associated with a lack of pus formation;
abnormality adhesion and platelet
• bleeding tendency; delayed umbilical cord separation with significant
Leukocyte AR activation;
omphalitis;
adhesion • decreased binding of
• slow healing ulcers;
deficiency (1-3) C3b to neutrophils
• S. aureus, E. coli, Candida, Aspergillus
X-linked and • Recurrent pyogenic infections with catalase-positive microorganisms;
Microbicidal AR; due to a • Failure to activate • onset in early infancy;
activity defect in the neutrophil respiratory • S. aureus, Serratia, B. cepacia, Candida; pneumonia,
Chronic generation of burst leading to failure • osteomyelitis,
granulomatous microbicidal to kill catalase-positive • skin infections,
disease oxygen microbes • lymphadenitis;
metabolites • hallmark is granuloma formation and inflammatory process
PLEASE READ MORE ON:
• Organs of immune system
• 4 type of immunopathologic reactions

END OF PEDIATRICS MAIN HANDOUT

IMPORTANT LEGAL INFORMATION


The handouts, videos and other review materials, provided by Topnotch Medical Board Preparation Incorporated are duly protected by RA 8293 otherwise known as the Intellectual Property Code
of the Philippines, and shall only be for the sole use of the person: a) whose name appear on the handout or review material, b) person subscribed to Topnotch Medical Board Preparation
Incorporated Program or c) is the recipient of this electronic communication. No part of the handout, video or other review material may be reproduced, shared, sold and distributed through any
printed form, audio or video recording, electronic medium or machine-readable form, in whole or in part without the written consent of Topnotch Medical Board Preparation Incorporated. Any
violation and or infringement, whether intended or otherwise shall be subject to legal action and prosecution to the full extent guaranteed by law.

DISCLOSURE
The handouts/review materials must be treated with utmost confidentiality. It shall be the responsibility of the person, whose name appears therein, that the handouts/review materials are not
photocopied or in any way reproduced, shared or lent to any person or disposed in any manner. Any handout/review material found in the possession of another person whose name does not
appear therein shall be prima facie evidence of violation of RA 8293. Topnotch review materials are updated every six (6) months based on the current trends and feedback. Please buy all
recommended review books and other materials listed below.
THIS HANDOUT IS NOT FOR SALE!

LOST a motor vehicle accident patient in my Ophthalmology rotation. I did the eye
by David Wagoner exam and noted my findings on the chart, since the patient was unable to
ambulate and there was no available stretcher in the crowded Emergency
"Stand still. The trees ahead and bushes beside you Room, I used some printed material instead of the Snellen chart.
Are not lost. Wherever you are is called Here, I notified the resident physician and endorsed my findings. This particular
And you must treat it as a powerful stranger, resident was an intern’s monitor and projected himself as “pro-student” and
Must ask permission to know it and be known. having oriented us during the beginning of the rotation, I thought he, of all
The forest breathes. Listen. It answers, people would be more tolerant.
I have made this place around you. “Ano ‘to?” (What’s this?) He said as he looked at my chart entry. I explained
If you leave it, you may come back again, saying Here. how I did my physical exam, and since I did my clerkship in another institution,
No two trees are the same to Raven. the order of my eye exam was not the standard one in the current institution.
No two branches are the same to Wren. I apologized and explained that I was confused with the order of the eye
If what a tree of a bush does is lost on you, exam, but I thought as long as it was complete, it would not be that of an issue.
You are surely lost. Stand still. The forest know I believed that we settled the matter then, but when he learned I used a fine
Where you are. You must let it find you." print instead of the Snellen chart to check the visual acuity, he launched into a
full tirade.
DEAR TOPNOTCH FRIENDS: “Saang school ka ba graduate?” (What school did you graduate from?) He
asked loudly, while we were in front of the patient and his family. He crushed
Change the System. If you get bullied as a junior doctor, tanggap lang at konting out my entries.
tiis. But do not become the bully when you become a senior doctor. Be part of the I was aware of the particular culture in certain institutions that medical
generation who will break the tradition of abuse. Stop bullying in the medical school of origin was a big deal, such that interns are judged based on where they
profession. Be kind to everyone. came from. It would have not been a problem if I came from one of the top
schools like University of Santo Tomas or Cebu Institute of Medicine, but I came
Here’s an article written by a Topnotch MD regarding this issue. Read and reflect. from a modest medical school Remedios Trinidad Romualdez Medical
Foundation in Tacloban. I have gotten used to people replying with, “Saan yun?”
Your teacher, (Where’s that?). And then I had to explain where my Alma Mater was. I thought
Enrico Paolo Banzuela, MD it was offensive to me why I needed to explain where I came from, I mean it is
not my fault I was born in Leyte and I chose to go to school there, in fact I am
proud that I am Waraynon. It was certainly nobody’s business to embarrass me
How the Culture of Bullying in the Hospital Hurts Patients by looking down on my credentials while in front of the patient and his family.
Essay first published in Health and Lifestyle Magazine The family member looked away, sensing the tension.
by Topnotch MD Thad Hinunangan, MD “Mas magaling pa sa iyo ang mga third year dito!” (Third year students here
are better than you!) He said.
I remember the first time I was fighting back tears while examining a I was trying to hold my composure, and still tried to assist him while he was
patient as a medical intern. It was early morning when I received a referral for examining the patient. My vision was starting to cloud. I have very shallow

TOPNOTCH MEDICAL BOARD PREP PEDIATRICS MAIN HANDOUT BY DRS. DE VERA AND PUNONGBAYAN Page 106 of 106
For inquiries visit www.topnotchboardprep.com.ph or email us at topnotchmedicalboardprep@gmail.com
This handout is only valid for October 2022 PLE batch. This will be rendered obsolete for the next batch since we update our handouts regularly.
TOPNOTCH MEDICAL BOARD PREP PEDIATRICS MAIN HANDOUT BY DRS. DE VERA AND PUNONGBAYAN
For inquiries visit www.topnotchboardprep.com.ph or https://www.facebook.com/topnotchmedicalboardprep/
This handout is only valid for October 2022 PLE batch. This will be rendered obsolete for the next batch since we update our handouts regularly.
tears, I tried not to blink because I was afraid my tears would fall directly to the Patients also become the unwitting ultimate victims. The quality of care
patient’s face. becomes compromised as clerks, interns, and residents focus more on avoiding
Before we were done, I curiously checked the findings he scribbled on the the wrath of their seniors, rather than providing the utmost care. I remember a
chart particularly the visual acuity. It turned out my findings which he crushed certain Emergency Medicine resident who gets severely agitated and rude
out were exactly the ones he wrote on the chart. It was one of those instances when there are many patients for admission, that the interns would spend time
when I experienced bullying in the medical setting as a novice. It would not be convincing the relatives of patients to just transfer to another hospital. Sadly,
the last. some of these patients had nowhere else to go because of poverty, and in the
end, some do not make it at all.
Different forms As a Pathology resident now, I was surprised how other specialties also
Abuse comes in different forms. It could be verbal, negligence, or unjust bully their way in the laboratory. Some choose not to follow the policy of 24
treatment or unreasonably long hours. It is as old as the medical profession hours’ notice for frozen sections, which require Pathology consultants to read
itself. People have gotten so used to it that it sometimes feels like the “norm.” the slides and thus need to be properly scheduled. Some just operate on their
Some of my friends who took their internship in different institutions in patient and decide at a moment’s notice, which requires a written explanation
Manila experienced being “runners” and personal servants of some of their by policy but such is a rule that some surgeons choose to ignore. Several times,
seniors. Some interns were assigned to make coffee, some clerks designated as residents of other specialties would barge in the cutting room while we are
the “takeout person” or ones assigned to bring the bag/s of their senior while dissecting to demand that we prioritize their particular case as if we were their
the senior strutted like a fashion model. employees or that other cases deserved any less of our full attention. Newsflash:
There is a fine line between doing a few favors and being a compete we are all Medical Officers- same position, different department.
personal slave. I don’t mind holding the door open or making coffee for Even the menial task of providing complete information on the Surgical
someone now and then, but when the tasks are not training related or nothing Pathology form has a lot of benefit for the patient, which unfortunately some
remotely geared towards improving one’s medical skills, then we have a fail to give. I once received a modified radical mastectomy specimen which had
problem. only one phrase written on the history. I processed the specimen as usual but
As a clerk, a male surgeon once refused to be assisted by me and wanted a when I examined it microscopically, there were only fibrous tissue and very
“pretty female clerk” (his words) who could do surgery with him all day in the scant foci of tumor cells. My consultant had me repeat the sections of the breast
Operating Room. At the end of the day, we felt sorry for our petite female thinking that I just missed the lesion but the other possibility was that the
colleague who had to retract all day at three nephrectomies just because she patient probably had chemotherapy beforehand which could help explain the
was a pretty female. The jokes and vague sexual innuendos are another thing. altered morphology.
Hospitals have advocacies on gender sensitivity and sexual harassment, but I tried contacting the surgeon to no avail. I visited the General Surgery office
somehow the practice is still thriving. with a written letter inquiring about the patient’s history, but they provided no
response. I went to the ward to try to see the patient and the chart but she was
When Tables Turn already discharged. Finally, I personally went to the Medical Records and dug
In the private institution where I had my clerkship, we clerks did majority through the bulk of files to retrieve the chart and found what I was looking for,
of the work because interns have protected time of five hours for their board the line that said: “History of Invasive Ductal Carcinoma, status post 6 cycles of
review, and when we get patients in the Emergency Room, we clerks do the chemotherapy (2016).” It was all I needed and was able to sign-out the case.
history and primary intervention first before we endorse to our interns. The morphology was altered because she had already undergone treatment
Since I had my internship in a large public teaching hospital, I never had and had responded well- there was only microscopic tumor foci left and all the
the chance to experience being a “senior” because no such distinction existed margins were clear. Just like that, our final diagnosis hit the mark.
between roles of clerks and interns. Only during the time when I became a I was disappointed that because of the lack of information given, the
resident did I experience being in a position of authority. patient had to wait for a longer period before we could sign out the case. Had I
I remember that incident in the Emergency Room of my Ophtha rotation, not investigated further; the case would have dragged on. Perhaps the surgeon
and I realized every resident physician who work closely with interns and thought it was only their job that mattered, when it is the pathologist who will
clerks actually have a choice: they can continue to perpetuate that culture or give the final anatomic diagnosis on what the lesion is and ascertain if they had
they can choose to stop the culture of bullying. adequately treated the patient.
Being in a training institution meant we had an obligation to teach, rather No doubt doctors are intelligent people, but what escapes me is how we
than berate juniors when they make mistakes which is expected in the learning never realize how much of a disservice we are doing to patients by bullying
curve. juniors or fellow doctors, thinking that one specialty is more superior than
another.
Far-Reaching Consequences The lack of respect ultimately results in delay of patient diagnosis or
The consequences are not limited to the aspiring doctors who, sadly take inadequate treatment, something all of us should avoid at all costs. It also goes
the brunt of the abuse. These would-be doctors experience depression, feelings against the principle of teamwork of all members of the health care team. Lastly,
of inferiority, and possibly lead to more serious consequences like quitting the let me echo the words of one of our mentors: “We all stand on the shoulders of
training program and even suicides in severe cases. those who came before us.” Let that be a reminder of the responsibility we have
to train the younger generation of doctors and uplift the medical profession.

TOPNOTCH MEDICAL BOARD PREP PEDIATRICS MAIN HANDOUT BY DRS. DE VERA AND PUNONGBAYAN Appendix
For inquiries visit www.topnotchboardprep.com.ph or email us at topnotchmedicalboardprep@gmail.com
This handout is only valid for the March 2022 PLE batch. This will be rendered obsolete for the next batch since we update our handouts regularly.

You might also like